You are on page 1of 1619

WIN A CASH AWARD OF ` 500.

00

It has come to our notice that some booksellers are fraudulently selling fake/duplicate copies
of some of our fast selling titles.
In our sincere efforts to provide you with our genuine books and to protect you against these
counterfeit books, Laxmi Publications (LP) has put a Hologram on the cover of some of its fast
selling titles. The Hologram displays a unique 3D Litho, multi-level, multi-colour effect from
different angles. It has the following seven levels of flat graphics merged together. The background
artwork seems to be ‘under’ or ‘behind’ the Hologram and gives the illusion of depth unlike the fake
Hologram on the fake/duplicate books.
DOTMATRIX /
FOREGROUND FLIP : 1 FLIP : 2
BACKGROUND
LAYER
LAYER

LAXMI
7
PUBLICATIONS
P L

ANIMATION LENS / MOVING REVOLVING EARTH


LAYER STAR LAYER LAYER

Presently, only some titles have got the Holograms. In this case, Comprehensive Physics
Volume (I and II) for Class XI has got the Hologram.
If you or any of your friends finds anywhere in India/abroad any book of this Edition without
the LP Hologram, he/she is requested to write to us at M/s LAXMI PUBLICATIONS PVT. LTD.,
113, Golden House, Ansari Road, Daryaganj, New Delhi-110002, giving the name and address
of the bookseller from where he/she purchased this book, together with the photocopy of the cover
and the 4th page on which the name of the printer is printed, he/she will be sent a cash award of
` 500.00.
How to decide if the book is genuine or fake ?
1. The above information may or may not be printed.
2. The counterfeit edition of the book may have no LP Hologram or if it has, it will be without
the illusionary depth as described above.
What is the harm in purchasing duplicate books ?
— Poor quality of paper and printing which affect your eyes.
— No royalty to authors who are scholars and have put their hard labour in writing the book,
thus depriving them of their intellectual rights.

Warning : Selling or buying pirated books is an offence. Legal action shall be taken against the
bookseller(s) and student(s) or whoever found guilty of such an offence in any way.
By
NARINDER KUMAR
M.Sc. PES(I)
Formerly, Senior Lecturer
Department of Physics
S.D. Govt. College, Ludhiana
Punjab

LAXMI PUBLICATIONS (P) LTD


(An ISO 9001:2008 Company)
BENGALURU • CHENNAI • COCHIN • GUWAHATI • HYDERABAD
JALANDHAR • KOLKATA • LUCKNOW • MUMBAI • RANCHI • NEW DELHI
BOSTON (USA) • ACCRA (GHANA) • NAIROBI (KENYA)
Comprehensive PHYSICS XI (Vol-I)

© by Laxmi Publications (P) Ltd.


All rights reserved including those of translation into other languages. In accordance with the Copyright (Amendment) Act, 2012, no part of this publication
may be reproduced, stored in a retrieval system, or transmitted in any form or by any means, electronic, mechanical, photocopying, recording or otherwise.
Any such act or scanning, uploading, and or electronic sharing of any part of this book without the permission of the publisher constitutes unlawful piracy and
theft of the copyright holder’s intellectual property. If you would like to use material from the book (other than for review purposes), prior written permission
must be obtained from the publishers.

Typeset at Kalyani Computer Services, Delhi


New Edition
ISBN 978-81-318-0196-3

Limits of Liability/Disclaimer of Warranty: The publisher and the author make no representation or warranties with respect to the accuracy or completeness
of the contents of this work and specifically disclaim all warranties. The advice, strategies, and activities contained herein may not be suitable for every
situation. In performing activities adult supervision must be sought. Likewise, common sense and care are essential to the conduct of any and all activities,
whether described in this book or otherwise. Neither the publisher nor the author shall be liable or assumes any responsibility for any injuries or damages
arising herefrom. The fact that an organization or Website if referred to in this work as a citation and/or a potential source of further information does not
mean that the author or the publisher endorses the information the organization or Website may provide or recommendations it may make. Further, readers
must be aware that the Internet Websites listed in this work may have changed or disappeared between when this work was written and when it is read.
All trademarks, logos or any other mark such as Vibgyor, USP, Amanda, Golden Bells, Firewall Media, Mercury, Trinity, Laxmi appearing in this work are
trademarks and intellectual property owned by or licensed to Laxmi Publications, its subsidiaries or affiliates. Notwithstanding this disclaimer, all other
names and marks mentioned in this work are the trade names, trademarks or service marks of their respective owners.

& Bengaluru 080-26 75 69 30


& Chennai 044-24 34 47 26, 24 35 95 07
& Cochin 0484-237 70 04, 405 13 03
& Guwahati 0361-254 36 69, 251 38 81
Branches

& Hyderabad 040-27 55 53 83, 27 55 53 93


& Jalandhar 0181-222 12 72
& Kolkata 033-22 27 43 84

Published in India by & Lucknow 0522-220 99 16


& Mumbai 022-24 91 54 15, 24 92 78 69
& Ranchi 0651-220 44 64
Laxmi Publications (P) Ltd.
(An ISO 9001:2008 Company)
113, GOLDEN HOUSE, DARYAGANJ,
NEW DELHI - 110002, INDIA
Telephone : 91-11-4353 2500, 4353 2501
Fax : 91-11-2325 2572, 4353 2528 C—
www.laxmipublications.com info@laxmipublications.com Printed at:
Chapter Page

Syllabus ... (xix)–(xxi)

1. PHYSICAL WORLD ... 1.3–1.22

2(a). SYSTEMS OF UNITS ... 2.1–2.11

(v)
(vi)

2(b). LENGTH, MASS AND TIME MEASUREMENTS ... 2.12–2.29

2(c). SIGNIFICANT FIGURES AND ERROR ANALYSIS ... 2.30–2.49

2(d). DIMENSIONAL ANALYSIS ... 2.50–2.78


(vii)

3(a). MATHEMATICAL TOOLS ... 3.3–3.24

3(b). UNIFORM MOTION ... 3.25–3.50


(viii)

3(c). UNIFORMLY ACCELERATED MOTION ... 3.51–3.106

4(a). VECTORS ... 4.1–4.52


(ix)

4(b). MOTION IN A PLANE ... 4.53–4.99


(x)

5(a). NEWTON’S LAWS OF MOTION ... 5.3–5.58


(xi)

5(b). FRICTION ... 5.59–5.77

5(c). DYNAMICS OF UNIFORM CIRCULAR MOTION ... 5.78–5.116

 
(xii)

6. WORK, ENERGY AND POWER ... 6.3–6.93


(xiii)

7(a). CENTRE OF MASS ... 7.3–7.25


(xiv)

7(b). ROTATIONAL MECHANICS AND MOMENT OF INERTIA ... 7.26–7.120


(xv)
s
(xvii)
(xviii)

(xviii)
(xix)

(xix)
(xx)
(xxi)
(xxii)
UNIT—I
10 Periods
PHYSICAL WORLD AND
MEASUREMENT
Chapter–1: Physical World

Chapter–2: Units and Measurements

SOLVED NUMERICALS 52 PLUS

UNSOLVED NUMERICALS 51 PLUS

VERY SHORT, SHORT & LONG ANSWER QUESTIONS 165 PLUS

QUESTIONS & PROBLEMS BASED ON HOTS 13 PLUS

NCERT EXERCISES, EXAMPLES & EXEMPLAR PROBLEMS 91 PLUS

MULTIPLE CHOICE QUESTIONS 81 PLUS


SOME MATHEMATICAL SYMBOLS

 

 



x

 f / x

THE GREEK ALPHABET

  
   
   
   
    
   
   
     
1.3
1.4 PHYSICS (Vol. I)—XI
PHYSICAL WORLD 1.5
1.6 PHYSICS (Vol. I)—XI
PHYSICAL WORLD 1.7
1.8 PHYSICS (Vol. I)—XI
PHYSICAL WORLD 1.9
1.10 PHYSICS (Vol. I)—XI
PHYSICAL WORLD 1.11
1.12 PHYSICS (Vol. I)—XI

?
PHYSICAL WORLD 1.13
1.14 PHYSICS (Vol. I)—XI
PHYSICAL WORLD 1.15


1.16 PHYSICS (Vol. I)—XI
PHYSICAL WORLD 1.17


1.18 PHYSICS (Vol. I)—XI

VERY SHORT ANSWER QUESTIONS—Each Question Carries 1 Mark Only

SHOR SHORT ANSWER QUESTIONS—Each Question Carries 2 Marks


PHYSICAL WORLD 1.19
1.20 PHYSICS (Vol. I)—XI

LONG ANSWER QUESTIONS

NCERT EXERCISES (With Solutions)


NCERT EXERCISES (UNIT I) 1.21
1.22 PHYSICS (Vol. I)—XI

    

     

 

2.1
2.2 PHYSICS (Vol. I)—XI
SYSTEMS OF UNITS 2.3


2.4 PHYSICS (Vol. I)—XI




SYSTEMS OF UNITS 2.5

 


  
OP
  
Q
2.6 PHYSICS (Vol. I)—XI

 

  


   

  

  

  

 

    

  

  

  
SYSTEMS OF UNITS 2.7

  

 
 
    
  

Sun


1 AU

l 1°
Earth
1 parsec

  Fig. 2(a).1. Parsec
p
  
 
   

p





2.8 PHYSICS (Vol. I)—XI
SYSTEMS OF UNITS 2.9

VERY SHORT ANSWER QUESTIONS—Each Question Carries 1 Mark Only

 



 

SHORT ANSWER QUESTIONS—Each Question Carries 2 Marks


2.10 PHYSICS (Vol. I)—XI



 


  
 

 
 

  



SYSTEMS OF UNITS 2.11

 

      

   


LONG ANSWER QUESTIONS


2.12
LENGTH, MASS AND TIME MEASUREMENTS 2.13


 

 

  

 
2.14 PHYSICS (Vol. I)—XI

 

= = =

   =

é ù
ê  ú
ë  û

 
 
  

 
 
  


LENGTH, MASS AND TIME MEASUREMENTS 2.15



  

  
 = 

Fig. 2(b).1. Determination of


angular distance between stars

 d
A B

D

  
E

Fig. 2(b).2. Determination of


 diameter of moon

  
 
 

  
    
 

2.16 PHYSICS (Vol. I)—XI


 
   

   
   
  
 




 
 
   
   

Fig. 2(b).3. Parallax

 =



  

 

 


Fig. 2(b).4. Determination of distance
of moon from Earth
LENGTH, MASS AND TIME MEASUREMENTS 2.17

 
FG IJ
 
H 
K
Fig. 2(b).5. Determination of distance
of near star [Suppose A represents
  the position of Earth on Dec. 22 and B
represents the position of Earth on
   June 22.]
 

     



 

  
  
 
   

  
Fig. 2(b).6. Determination of distance
  of an inferior planet
2.18 PHYSICS (Vol. I)—XI

  

F I
GG JJ FG IJ
=
GH JK
=
H K
=
FG IJ
=
H K

  

  

=

   

Fig. 2(b).7

( a / ae )3

=
LENGTH, MASS AND TIME MEASUREMENTS 2.19

  Fig. 2(b).8. Determination of


height of accessible pole

 

 
Fig. 2(b).9. Determination of
height of an accessible tree

 

 

 

Fig. 2(b).10. Determination of height of


 inaccessible mountain
 

 
  
 
2.20 PHYSICS (Vol. I)—XI


 

   

Fig. 2(b).12



Fig. 2(b).11   

 

 

  Fig. 2(b).13. Determination


of width of river
LENGTH, MASS AND TIME MEASUREMENTS 2.21

 transmitted
radar pulse
reflected
radar pulse

Fig. 2(b).14. Radar ranging: the time of flight of the radar


pulses is measured and converted to kilometres
by air traffic control

Fig. 2(b).15. Measurement of


thickness of a piece of matter
2.22 PHYSICS (Vol. I)—XI

 
 

FG IJ FG  I
JK
H K H  
LENGTH, MASS AND TIME MEASUREMENTS 2.23

Fig. 2(b).16

Fig. 2(b).17

Fig. 2(b).18. Inertial Balance


2.24 PHYSICS (Vol. I)—XI




Fig. 2(b).19. Common Balance

Fig. 2(b).20. Spring Balance


LENGTH, MASS AND TIME MEASUREMENTS 2.25
2.26 PHYSICS (Vol. I)—XI
LENGTH, MASS AND TIME MEASUREMENTS 2.27

VERY SHORT ANSWER QUESTIONS—Each Question Carries 1 Mark Only

 
2.28 PHYSICS (Vol. I)—XI

LONG ANSWER QUESTIONS


LENGTH, MASS AND TIME MEASUREMENTS 2.29

VALUE BASED QUESTION


2.30
SIGNIFICANT FIGURES AND ERROR ANALYSIS 2.31
 
 


2.32 PHYSICS (Vol. I)—XI

 

  

 
SIGNIFICANT FIGURES AND ERROR ANALYSIS 2.33


 

2.34 PHYSICS (Vol. I)—XI

 

   

  
SIGNIFICANT FIGURES AND ERROR ANALYSIS 2.35

  

 

.  .
 .
 

 
2.36 PHYSICS (Vol. I)—XI

   



SIGNIFICANT FIGURES AND ERROR ANALYSIS 2.37


     
 

   

 


2.38 PHYSICS (Vol. I)—XI

   


   

 


  

  
SIGNIFICANT FIGURES AND ERROR ANALYSIS 2.39
      
  
      
 
  

 
 

  
   
   
    
   

   

 


 


 

 

  
      
2.40 PHYSICS (Vol. I)—XI

      

    
   

 

 

  
   

FG    IJ FG    IJ FG    IJ FG    IJ
H 
K H 
K H 
K H 
K
  

  

  
  

 


 

  
   
SIGNIFICANT FIGURES AND ERROR ANALYSIS 2.41

  




FG  IJ FG  IJ 
  
H 
K H 
K
FG  IJ FG   IJ
H 
KH  
K
 

FG   IJ FG  IJ
 
H KH 
K
LM       OP

N 
Q
  LM 

 OP

N 
Q
     
     

FG    IJ FG    IJ FG    IJ FG    IJ
H 
K H 
K H 
K H 
K
  

  
  

 
2.42 PHYSICS (Vol. I)—XI

  

FG  IJ FG  IJ

H 
K H 
K
FG  IJ FG  IJ

H 
K H 
K

FG  IJ FG  IJ

H 
KH 
K
 FG  IJ FG  IJ

H 
KH 
K
      
     
 
    
  

  
  

  
 

 

  
  

  
 
SIGNIFICANT FIGURES AND ERROR ANALYSIS 2.43

  

   
   

   
  

   
      

   
   

 

    
   
  

 



LM OP 
N  
Q

   
  

 
 

  

2.44 PHYSICS (Vol. I)—XI

  
     


    


    


       

 


 


 
 


      
     
     

 
 
 

     
     
     

   
 

 

SIGNIFICANT FIGURES AND ERROR ANALYSIS 2.45

   
    
   
   



 
   

VERY SHORT & SHORT ANSWER QUESTIONS—Each Question Carries 1 Mark Only


 

 


2.46 PHYSICS (Vol. I)—XI

LONG ANSWER QUESTIONS

 

(      
  

  
 

 

 

 

   


 
SIGNIFICANT FIGURES AND ERROR ANALYSIS 2.47

  

  


  

 

  
 
  
   
 
 
 



  
2.48 PHYSICS (Vol. I)—XI

   

   

LM   OP 

N 
Q   

LM OP 
N 
Q 
  

 



   

 
 


  



    


  
  


LM   OP
N 
Q 
LM OP 
N 
Q 

         
 
 
SIGNIFICANT FIGURES AND ERROR ANALYSIS 2.49


     

   





2.50
DIMENSIONAL ANALYSIS 2.51

 


2.52 PHYSICS (Vol. I)—XI





DIMENSIONAL ANALYSIS 2.53

 

LM   OP
MN PQ


2.54 PHYSICS (Vol. I)—XI


 






DIMENSIONAL ANALYSIS 2.55










2.56 PHYSICS (Vol. I)—XI

  

 

 

 


DIMENSIONAL ANALYSIS 2.57

LM OP LM OP LM OP
N Q N Q N Q

LM OP LM

OP

N Q N Q


LM OP LM OP LM OP
N Q N Q N Q LM OP LM OP LM OP
LM OP
LM OP LM OP  N Q N Q N Q

N Q N Q N Q LM OP LM OP L O

N Q N Q MN PQ
2.58 PHYSICS (Vol. I)—XI

FG IJ FG IJ 

H KH K 

  LM OP LM OP LM OP
N Q N Q N Q
LM OP L OP LM OP 

N Q MN Q N Q
LM OP LM OP LM OP 

N Q N Q N Q


 

 

 
 
DIMENSIONAL ANALYSIS 2.59








 

  


   

  

FG IJ l
H K g

 
2.60 PHYSICS (Vol. I)—XI




  



 

   G

h

m


 

LM OP 

N 
Q 

 
DIMENSIONAL ANALYSIS 2.61

VERY SHORT ANSWER QUESTIONS—Each Question Carries 1 Mark Only

 





  
 
 

 


2.62 PHYSICS (Vol. I)—XI

SHORT ANSWER QUESTIONS—Each Question Carries 2 Marks

FG IJ
H 
K

SHORT ANSWER QUESTIONS—Each Question Carries 3 Marks

 
 


   

 

 

DIMENSIONAL ANALYSIS 2.63

 
   

  
 

  

LONG ANSWER QUESTIONS


LM OP LM OP LM OP
N Q N Q N Q 

  

 
2.64 PHYSICS (Vol. I)—XI

 
   
 


  

  
  
 





   
    
    

LM OP LM OP LM OP LM OP
N QN Q N Q   
N Q
LM OP LM OP LM OP

LM OP LM OP 

N QN Q N Q N QN Q
   

NCERT EXERCISES (UNIT I) 2.65

LM OP LM OP LM OP 
LM OP L O LM OP
N QN Q N Q N Q MN PQ  
N Q
     LM OP LM OP
 


 
N Q N Q
 
  
LM OP LM OP L O 

N Q N Q MN PQ 

LM OP LM OP L O 

N Q N Q MN PQ

NCERT EXERCISES (With Solutions)

 
 
 



   

     


LM OP LM OP LM OP
 
N Q N Q N Q
LM OP LM OP LM OP 

N Q N Q N Q
  

 
2.66 PHYSICS (Vol. I)—XI

 
   



NCERT EXERCISES (UNIT I) 2.67

    

   

P 4
   
P 100

   
 
 

LM  OP
N Q 

  
 



 
2.68 PHYSICS (Vol. I)—XI

 


 
   
 

 
  
 





 
   
 

 
 
  
 
 
 
Fig. 2(d).1
 





    


NCERT EXERCISES (UNIT I) 2.69

   

   



 


 


 Fig. 2(d).2
2.70 PHYSICS (Vol. I)—XI




 
 
  




 
 


 

 


 

  

        


 


 
NCERT EXERCISES (UNIT I) 2.71






 







  



2.72 PHYSICS (Vol. I)—XI

 
  



      


      
 

 
 
  



 
   
  

 

Fig. 2(d).3
NCERT EXERCISES (UNIT I) 2.73

 
 
   
 
  
   
   
   
   


      
    
   

NCERT EXEMPLAR PROBLEMS (With Answers)


[Based on Higher Order Thinking Skills].
MCQ I (Only one option is correct.)

 



 
 

 

2.74 Comprehensive PHYSICS (Vol. I)—XI

12. If momentum (P), area (A) and time (T) are taken to be 18. Which of the following are not a unit of time ?
fundamental quantities, then energy has the dimensional (a) Second (b) Parsec
formula
(c) Year (d) Light year.
(a) (P1 A–1 T1) (b) (P2 A1 T1)
Ans. (b), (d).
(c) (P1 A–1/2 T1) (d) (P1 A1/2 T–1).
Ans. (d) VSA
MCQ II (Any number of options may be 19. Why do we have different units for the same physical
quantity ?
correct.) Ans. Because, bodies differ in order of magnitude significantly
13. On the basis of dimensions, decide which of the following in respect to the same physical quantity. For example,
relations for the displacement of a particle undergoing interatomic distances are of the order of angstroms,
simple harmonic motion is not correct : inter-city distances are of the order of km, and
interstellar distances are of the order of light year.
2πt 20. The radius of atom is of the order of 1 Å and radius of
(a) y = a sin
T nucleus is of the order of fermi. How many magnitudes
(b) y = a sin vt higher is the volume of atom as compared to the volume of
nucleus ?
a t Ans. RA  1Å = 10–10 m
(c) y = sin  
T  a R N  1 fermi = 10–15 m

 2πt 2πt  
(d) y = a 2  sin − cos . 8# p4# Ê 
4 ˆ
 T T  =  =Á #˜
80 
p40 Ë 4 0 ¯
Ans. (b), (c) 
14. If P, Q, R are physical quantities, having different 
dimensions, which of the following combinations can never Ê - ˆ
= Á - ˜ = 
be a meaningful quantity ? Ë  ¯
(a) (P – Q)/R (b) PQ – R 21. Name the device used for measuring the mass of atoms
(c) PQ/R (d) (PR – Q2)/R and molecules.
(e) (R + Q)/P. Ans. Mass spectrograph.
Ans. (a), (e). 22. Express unified atomic mass unit in kg.
15. Photon is quantum of radiation with energy E = hν where Ans. 1 u = 1.67 × 10–27 kg.
ν is frequency and h is Planck’s constant. The dimensions 23. A function f(θ) is defined as :
of h are the same as those of
(a) Linear impulse θ2 θ3 θ4
f(θ) = 1 – θ + – + ...
(b) Angular impulse 2! 3! 4!
(c) Linear momentum Why is it necessary for θ to be a dimensionless quantity ?
(d) Angular Momentum. Ans. Since f(θ) is a sum of different powers of θ, therefore θ
Ans. (b), (d) has to be dimensionless.
16. If Planck’s constant (h) and speed of light in vacuum (c) 24. Why length, mass and time are chosen as base quantities
are taken as two fundamental quantities, which one of the in mechanics ?
following can, in addition, be taken to express length, mass Ans. Because all other quantities of mechanics can be
and time in terms of the three chosen fundamental expressed in terms of length, mass and time through
quantities ?
simple relations. Moreover length, mass and time
(a) Mass of electron (me) cannot be derived from one another.
(b) Universal gravitational constant (G) SA
(c) Charge of electron (e)
25. (a) The earth-moon distance is about 60 earth radius.
(d) Mass of proton (mp). What will be the diameter of the earth (approximately
Ans. (a), (b) and (d). in degrees) as seen from the moon ?
17. Which of the following ratios express pressure ? (b) Moon is seen to be of (1/2)° diameter from the earth.
What must be the relative size compared to the earth ?
(a) Force/Area (b) Energy/Volume
(c) From parallax measurement, the sun is found to be at
(c) Energy/Area (d) Force/Volume.
a distance of about 400 times the earth-moon distance.
Ans. (a), (b). Estimate the ratio of sun-earth diameters.
NCERT EXEMPLAR PROBLEMS (UNIT I) 2.75

Fig. 2(d).4

   
    
   
   


Fig. 2(d).5

=  ´
´

   
2.76 Comprehensive PHYSICS (Vol. I)—XI
33. Calculate the solid angle subtended by the periphery of an where P is the pressure difference between the two ends
area of 1 cm2 at a point situated symmetrically at a of the pipe and η is coefficient of viscosity of the liquid
distance of 5 cm from the area. having dimensional formula ML–1T–1.
Check whether the equation is dimensionally correct.
ds 1 cm2
Ans. dω = = = 4 × 10–2 steradian È L3 ˘
r2 (5 cm)2
Ans. [v] = Í ˙ = [M0L3T–1]
ÎÍ T ˚˙
34. The displacement of a progressive wave is represented by
y = A sin (ωt – kx), where x is distance and t is time. Write È p P r4 ˘ –1 –2 4
the dimensional formula of (i) ω and (ii) k. Í ˙ = [ML T ] [L ] = [M0L3T–1]
ÍÎ 8 hl ˙˚ [ML–1 T –1 ] [L]
Ans. Dimensional formula of ω = T–1
The dimensions of both the terms of the given equation
Dimensional formula of k = L–1.
are same. So, according to principle of homogeneity of
35. Time for 20 oscillations of a pendulum is measured as dimensions, the given formula is dimensionally correct.
t1 = 39.6 s; t2 = 39.9 s; t3 = 39.5 s. What is the precision 38. A physical quantity X is related to four measurable
in the measurements? What is the accuracy of the quantities a, b, c and d as follows :
measurement?
X = a2 b3 c5/2 d–2.
Ans. Precision is given by the least count of the instrument.
The percentage errors in the measurement of a, b, c and d
For 20 oscillations, precision = 0.1 s are 1%, 2%, 3% and 4% respectively. What is the percentage
For 1 oscillation precision = 0.005 s error in the quantity X ? If the value of X calculated on the
Mean value of time for 20 oscillations, basis of the above relation is 2.763, to what value should
you round off the result ?
t1 + t2 + t3 39.6 + 39.9 + 39.5
t= = s = 39.7 s È Da Db Dc Dd ˘
3 3 Ans. DX ¥ 100 = Í2 +3 + 2.5 +2 ¥ 100
∆t1 = t – t1 = 39.7 – 39.6 = 0.1 s X Î a b c d ˙˚
∆t2 = t – t2 = 39.7 – 39.9 = – 0.2 s = 2(1%) + 3(2%) + 2.5(3%) + 2(4%)
∆t3 = t – t3 = 39.7 – 39.5 = 0.2 s = 23.5% = 24%
Mean absolute error Since the error is in first decimal, hence the result
should be rounded off as 2.8.
= ^DV^ + ^DV^ + ^DV^ =  +  +  =  39. In the expression P = El2 m–5 G–2, E, m, l and G denote
  energy, mass, angular momentum and gravitational
Rounding off to one decimal place, accuracy of constant respectively. Show that P is a dimensionless
measurement = ± 0.2 s quantity.
Ans. [E] = [ML2T–2], [l] = [ML2T–1], [G] = [M–1 L3 T–2]
LA [P] = [ML2T–2] [ML2T–1]2 [M–5] [M–1 L3 T–2]–2
36. A new system of units is proposed in which unit of mass is
= [M0 L0 T0]
α kg, unit of length β m and unit of time γ s. How much
will 5 J measure in this new system? Thus, P is dimensionless.
40. If velocity of light c, Planck’s constant h and gravitational
Ans. M1 = 1 kg, L1 = 1 m, T1 = 1 s
constant G are taken as fundamental quantities, then
M2 = α kg, L2 = β m, T2 = γ s express mass,, length and time in terms of dimensions of
n1 = 5, n2 = ? these quantities.
[energy] = [ML2 T–2] Ans. [c] = [LT–1], [h] = [ML2T–1], [G] = [M–1 L3 T–2]
  - Let m = cx hy Gz ...(i)
Ê/ ˆ Ê. ˆ Ê6 ˆ
n2 = P Á  ˜ Á  ˜ Á  ˜ [ML0 T0] = [LT–1]x [ML2 T–1]y [M–1 L3 T–2]z
Ë / ¯ Ë . ¯ Ë 6 ¯
or [ML0 T0] = [My – z Lx + 2y + 3z T–x – y – 2z]
  - Comparing powers of M, L and T,
 Ê ˆ Ê ˆ
n2 = n1 ÊÁ ˆ˜ y – z = 1, x + 2y + 3z = 0, – x – y – 2z = 0
Ë a ¯ ÁË b ˜¯ ÁË g ˜¯
1 1 1
2
On solving, x = ,y= ,z=-
2 2 2
= 5g new units of energy.
ab2 ch
From (i), m = c1/2 h1/2 G–1/2 =
37. The volume of a liquid flowing out per second of a pipe of G
length l and radius r is written by a student as Proceeding as above, we can prove that

Pr 4 hG hG
v= L= and T =
8 l c3 c5
NCERT EXEMPLAR PROBLEMS (UNIT I) 2.77

´ =

 

 
 
é + + ù
ê  ú
ê ú
ë û

+ + =  = =

 + + = =

 


´

 ´

 °
 

 ´ =
2.78 PHYSICS (Vol. I)—XI
°

°
 ´ =

°
´ °=

°
´ °= = ¢

-
´ ´
-
=
´

 

  

 

2.79
2.80 PHYSICS (Vol. I)—XI

  
    
 

    
  
 

 
 


 

     
     
PHYSICAL WORLD AND MEASUREMENT (UNIT I) 2.81

  

 
 










2.82 PHYSICS (Vol. I)—XI

  
  
 

 
  
 
PHYSICAL WORLD AND MEASUREMENT (UNIT I) 2.83

 

  
  
 

 


2.84 PHYSICS (Vol. I)—XI

 

 

    
     

 
  
  


PHYSICAL WORLD AND MEASUREMENT (UNIT I) 2.85



 
 


2.86 PHYSICS (Vol. I)—XI

 
    
PHYSICAL WORLD AND MEASUREMENT (UNIT I) 2.87

µ µ

µ µ

=
= p
2.88 PHYSICS (Vol. I)—XI




 

 

  
 
   
 

      

     
    
   
 

  
  
  

  
  
 


   

 


 

PHYSICAL WORLD AND MEASUREMENT (UNIT I) 2.89

  

 



   
  
 

 



  



 
 
 
2.90 PHYSICS (Vol. I)—XI

  


     
         
           


     
    
 

 
 


    
  
 

  

  
 

  
 


  

 


  
    

 


 
 

 
PHYSICAL WORLD AND MEASUREMENT (UNIT I) 2.91

 


  
µ

 


¢

 

 
 

  
  




 
  

   

  µ

  
    

 
2.92 PHYSICS (Vol. I)—XI
UNIT—II
KINEMATICS 24 Periods

Chapter–3: Motion in a Straight Line

Chapter–4: Motion in a Plane

SOLVED NUMERICALS 110 PLUS

UNSOLVED NUMERICALS 150 PLUS

VERY SHORT, SHORT & LONG ANSWER QUESTIONS 245 PLUS

QUESTIONS & PROBLEMS BASED ON HOTS 90 PLUS

NCERT EXERCISES, EXAMPLES & EXEMPLAR PROBLEMS 105 PLUS

MULTIPLE CHOICE QUESTIONS 225 PLUS


Galileo Galilei (1564–1642)

3.3
3.4 PHYSICS (Vol. I)—XI

Fig. 3(a).1

Fig. 3(a).2


MATHEMATICAL TOOLS 3.5


 

 


  

 ´

  

     
3.6 PHYSICS (Vol. I)—XI

  

´ ´ ´ ´ ´ ´



XOP = 


  

 

  Fig. 3(a).3. Trigonometrical functions

 

   ´

 
MATHEMATICAL TOOLS 3.7

  
  
 
   
 
   

 Fig. 3(a).4. Signs of T-ratios



 


 

 
 
 
 

°


sin (   )   sin  cos ( )  cos  tan (   )   tan 



     

     

     

     

     

     

     
3.8 PHYSICS (Vol. I)—XI

     

     

     

     

     

     

  
      

  
     

     

     

 

 
 


 °   

 

    

 ¥    ¥




MATHEMATICAL TOOLS 3.9

 

 

 

 

(c)
Fig. 3(a).5


3.10 PHYSICS (Vol. I)—XI




MATHEMATICAL TOOLS 3.11


3.12 PHYSICS (Vol. I)—XI

 

 

  
  
 
y
x

 æ   ö
  çè   ÷ø
  


MATHEMATICAL TOOLS 3.13


 


 

Fig. 3(a).6. Geometrical interpretation
     of derivative

  



 
  

 

 
3.14 PHYSICS (Vol. I)—XI

 




   

 LM OP

N 
Q
  LM OP  

N 
Q

     

 

  

FG IJ 
H K

 

 

  
MATHEMATICAL TOOLS 3.15

 

 

 



 


LM  OP
N 
Q

LM  OP
N 
Q
3.16 PHYSICS (Vol. I)—XI

LM OP  LM OP
F I

N 
Q 
N 
Q
GH JK 

    
 

 

LM  OP     
MN  PQ LM OP
    
N  
Q
LM OP LM OP
 
N 
Q 
N 
Q
LM OP     

N 
Q   
   

LM OP
    
N 
Q

LM    OP
     MN    PQ

LM       OP
        MN   

   PQ
 
LM     
OP


MN    PQ
LM  OP  

N  Q LM OP
N  
Q
    
LM  OP
 N   
Q
LM OP  LM OP é
ê 
ù
ú

N Q 
N Q ê
ë  ú
û

   
 

F I
GH 
 JK
    


MATHEMATICAL TOOLS 3.17

LM OP
 N 
Q
LM OP
N   
Q
LM OP
 N 
Q
LM OP
N  
Q
LM  OP


N 
Q

  

LM OP

N  
Q
 LM OP


N 
Q
 
  

FG IJ
H K   

 

 
 

LM OP
N 
 


Q 

LM OP
   

N 



Q
 
 

LM  OP 
 

N 
Q 

3.18 PHYSICS (Vol. I)—XI

 

       

  LM  OP

 N       
Q
 

   

LM OP
 
MN 
 

PQ
  é  ù
ê  ú
    ê   úû
ë
   
   

LM OP LM OP
N 
Q
MN  PQ
      

LM OP 

N    
Q é ù
  êë  úû

 




  
MATHEMATICAL TOOLS 3.19

z 


  

z 
z 


z  z 

z 
z 

z   

z 

z  

z  

z     u dx     

z 

z  



z 




3.20 PHYSICS (Vol. I)—XI

 


z LM
N

OP
Q
 


z LM OP LM OP
 
  

MN   PQ N Q
 
LM  OP

N Q 


z F
GH 
I
JK z   z  z  

 

 


  
  
  

  

z 

z 

z
 
FG IJ 

H  
K z 

z z z z FGH 
I
JK
 
MATHEMATICAL TOOLS 3.21

 
  

z

z  


z LM OP LM OP
 
 

z z


MN   PQ N Q  

LM  OP
N Q z

z 

  LM  OP 
LM OP 

z N Q N Q




LM  OP
 N 
Q  

 

z
 
LM OP
z z

LM OP


N Q MN PQ

z 







 z  

e x

é  ù
êë  úû


LM  OP
MN  
PQ
LM  
 
OP 

MN PQ
é æ  öù
 ê çè ÷ø ú
ë û

LM OP   
 
N 
Q   

LM OP LM F I OP
MM GG JJ P
 MMN 
PPQ N H

 K PQ
LM OP LM OP

N 
Q  N Q
3.22 PHYSICS (Vol. I)—XI

 LM 
OP FG IJ
 MN  PQ H K
LM OP 

 MN  PQ

LM OP  
N Q

 
 
FG IJ LM OP
H K N 
Q LM OP
N  
Q

z 
LM 

  
OP

MN  PQ
LM 
 OP
 
MN  PQ  
  
 
LM   
OP
MN PQ
é
ê 
ù
ú LM OP
ë û
N  
Q
z LM
N
OP
Q

 FG IJ
H 
K
LM    
OP
MN PQ

z 
LM
N
OP
Q

z
MATHEMATICAL TOOLS 3.23



z


z  LM
N
 OP
Q z 

z



z 
 

z  LM
N
OP
Q z LM
N 
OP
Q

    

   

LM OP
N  
Q
dy LM OP  
dx N Q 
LM  OP
MN  PQ
F I F I  
LM  
OP
 GH JK  GH JK MN PQ
 


   


FG   IJ FG   IJ 
 
H  KH  K

LM OP 
 

N  Q  

  


FG IJ
z z 

H K   
LM
N z z  OP
Q
 
3.24 PHYSICS (Vol. I)—XI

z FG
H   
IJ
K LM 
N
OP
Q




FG   IJ
   
 
    

H K

 
    LM  FG   IJ OP
H KQ
z

N 

z z LMN z 
OP
Q FG
H


IJ
K
z 1
2


z z z


 
FG   IJ
H 
K
 z 
 

LM
N z 

z  OP
Q 
FG  IJ
LM  OP

H 
K
z

MN  PQ
LM   OP 
N 
Q
3.25
3.26 PHYSICS (Vol. I)—XI

Fig. 3(b).1. One-dimensional


motion
UNIFORM MOTION 3.27

Fig. 3(b).2. Two-dimensional


motion

Fig. 3(b).3. Three-dimensional


motion

Fig. 3(b).4
3.28 PHYSICS (Vol. I)—XI

Fig. 3(b).5
UNIFORM MOTION 3.29

Fig. 3(b).6

Fig. 3(b).7

Fig. 3(b).8. Position-time graph


of a stationary object
3.30 PHYSICS (Vol. I)—XI

Fig. 3(b).9. Positive displacement


 

   

  Fig. 3(b).10. Zero displacement



Fig. 3(b).11. Negative displacement


 Fig. 3(b).12. Displacement is
  independent of origin

  

Fig. 3(b).13
UNIFORM MOTION 3.31





 

 


3.32 PHYSICS (Vol. I)—XI
 

  

   
   
   
   

   
   







UNIFORM MOTION 3.33

 

 





 
 
 
  






Fig. 3(b).14. Displacement-time graph

Fig. 3(b).15. Displacement-time


graph in the case of variable velocity
3.34 PHYSICS (Vol. I)—XI

Fig. 3(b).16. Displacement-time graph


in the case of constant velocity

  Fig. 3(b).17. Velocity-time graph in


uniform motion
UNIFORM MOTION 3.35

Fig. 3(b).18 Fig. 3(b).19




Fig. 3(b).20. Position-time
 graph for uniform motion



 
  Fig. 3(b).21





Fig. 3(b).22. Position-time


graph

 
3.36 PHYSICS (Vol. I)—XI

Fig. 3(b).23

Fig. 3(b).27

Fig. 3(b).24

Fig. 3(b).28

Fig. 3(b).25

Fig. 3(b).29
Fig. 3(b).26
UNIFORM MOTION 3.37

Fig. 3(b).30

FG IJ
H K
FG IJ
H K
 




 

 




3.38 PHYSICS (Vol. I)—XI

x
80

Fig. 3(b).31


Fig. 3(b).32. Uniform one-dimensional


motion of two bodies

Fig. 3(b).33. Uniform one-dimensional


motion of two bodies
UNIFORM MOTION 3.39

Fig. 3(b).34

Fig. 3(b).35

Fig. 3(b).36
3.40 PHYSICS (Vol. I)—XI

Fig. 3(b).37

Fig. 3(b).38

Fig. 3(b).39

Fig. 3(b).40

Fig. 3(b).41. Position-time graphs of


two cars moving with equal velocities
UNIFORM MOTION 3.41

Fig. 3(b).42. Position-time graphs of two


cars moving with unequal velocities in
the same direction




3.42 PHYSICS (Vol. I)—XI

VERY SHORT ANSWER QUESTIONS—Each Question Carries 1 Mark Only

Fig. 3(b).46 Fig. 3(b).47

Fig. 3(b).43 Fig. 3(b).44

Fig. 3(b).45

SHORT ANSWER QUESTIONS—Each Question Carries 2 Marks

 
UNIFORM MOTION 3.43

SHORT ANSWER QUESTIONS—Each Question Carries 3 Marks

Fig. 3(b).49

Fig. 3(b).48


3.44 PHYSICS (Vol. I)—XI

 


 

LONG ANSWER QUESTIONS


UNIFORM MOTION 3.45

LM  OP
N  Q
3.46 PHYSICS (Vol. I)—XI

    
 

 

 


 


UNIFORM MOTION 3.47


 

 

LM   OP
 N  Q  

   
  
3.48 PHYSICS (Vol. I)—XI

Fig. 3(b).50

 

Fig. 3(b).52

Fig. 3(b).51




UNIFORM MOTION 3.49


 FG IJ

H K

   FG IJ
H 
K

FG IJ
H 
K

 

FG IJ
H K
FG IJ
H K 
3.50 PHYSICS (Vol. I)—XI

 
  

VALUE BASED QUESTION


1 2

O A B L

Fig. 3(c).1. Uniformly accelerated motion


in one-dimension

 
 

 

t

3.51
3.52 PHYSICS (Vol. I)—XI

 

 

Fig. 3(c).2


 
 

  


UNIFORMLY ACCELERATED MOTION 3.53

Fig. 3(c).3

 
 
 

  
3.54 PHYSICS (Vol. I)—XI

   
 
 

     
   
   

  

 
 

   
       
 
UNIFORMLY ACCELERATED MOTION 3.55

CHECKPOINT

  

d FG IJ d
dt H K dt

 

   
 
 
3.56 PHYSICS (Vol. I)—XI

   
       
 

ò ò
UNIFORMLY ACCELERATED MOTION 3.57



ò ò  

 
 
 

ò ò 

ò ò

 
 
 
3.58 PHYSICS (Vol. I)—XI

ò ò ò

ò ò



  


UNIFORMLY ACCELERATED MOTION 3.59

ò ò

 
 

ò 
ò  ò 

  

  
  

 

     

ò 
ò  ò 

  
3.60 PHYSICS (Vol. I)—XI

Fig. 3(c).4. Position-time graph of


uniformly accelerated motion

Fig. 3(c).5. Position-time graph of Fig. 3(c).6. Position-time graph of particle


uniformly retarded motion. moving with zero acceleration.

 

Fig. 3(c).7
UNIFORMLY ACCELERATED MOTION 3.61

0


0 C
 A

Fig. 3(c).8. Velocity-time graph of


uniformly accelerated motion

F B

E A
K
Velocity

 

C D
O t t
Time

    Fig. 3(c).9. Velocity-time graph of


uniformly accelerated motion

  


  

  
 
3.62 PHYSICS (Vol. I)—XI
 

 


 

Fig. 3(c).10 Fig. 3(c).11

Fig. 3(c).12 Fig. 3(c).13

Fig. 3(c).14 Fig. 3(c).15

Fig. 3(c).16 Fig. 3(c).17

Fig. 3(c).18
UNIFORMLY ACCELERATED MOTION 3.63

Fig. 3(c).19. Velocity-time graph of Fig. 3(c).20. Velocity-time graph of


uniformly accelerated motion uniformly retarded motion

Fig. 3(c).21

 
 
 



3.64 PHYSICS (Vol. I)—XI

 
  

 
 

Fig. 3(c).22
UNIFORMLY ACCELERATED MOTION 3.65

Fig. 3(c).23. Velocity-time


graph of uniformly accelerated
motion


3.66 PHYSICS (Vol. I)—XI

  
 
 

Fig. 3(c).24
UNIFORMLY ACCELERATED MOTION 3.67

 

 

  


3.68 PHYSICS (Vol. I)—XI

1  
2 

 
 
 

  

 

  
  
 

   


Fig. 3(c).25
   

 


 


  
UNIFORMLY ACCELERATED MOTION 3.69

  

2
0
2a  

 

Fig. 3(c).26



  


3.70 PHYSICS (Vol. I)—XI

 

FG IJ
H K

Fig. 3(c).28

  

Fig. 3(c).27

 
UNIFORMLY ACCELERATED MOTION 3.71

  

  

Fig. 3(c).30
Fig. 3(c).29

 
   
     
   

  


Fig. 3(c).31
3.72 PHYSICS (Vol. I)—XI

  

Fig. 3(c).32

Fig. 3(c).33


Fig. 3(c).34
UNIFORMLY ACCELERATED MOTION 3.73

   

Fig. 3(c).36

Fig. 3(c).35

 



3.74 PHYSICS (Vol. I)—XI

FG IJ
H K 

FG IJ FG IJ
H K H K

FG IJ FG IJ
 

H K H K

Fig. 3(c).37


 


 

FG  IJ
H  K



   
 
   
   
 


 
FG IJ y FG IJ
H K h H K
UNIFORMLY ACCELERATED MOTION 3.75

Fig. 3(c).38
Fig. 3(c).39

 

FG  IJ
H 
 K
 

 
 

FG IJ
H 


 K Fig. 3(c).40

FG IJ 


H 


K  

FG  IJ 

 

H  K 

  
3.76 PHYSICS (Vol. I)—XI

 
 

VERY SHORT ANSWER QUESTIONS—Each Question Carries 1 Mark Only

  
UNIFORMLY ACCELERATED MOTION 3.77


SHORT ANSWER QUESTIONS—Each Question Carries 2 Marks


3.78 PHYSICS (Vol. I)—XI

Fig. 3(c).41

   
UNIFORMLY ACCELERATED MOTION 3.79


SHORT ANSWER QUESTIONS—Each Question Carries 3 Marks

  

  

  


3.80 PHYSICS (Vol. I)—XI

Fig. 3(c).43

Fig. 3(c).42

  

LM OP  

N 
Q 
 

Fig. 3(c).44

 
UNIFORMLY ACCELERATED MOTION 3.81

Fig. 3(c).45

Fig. 3(c).46
3.82 PHYSICS (Vol. I)—XI

Fig. 3(c).47
Fig. 3(c).49


Fig. 3(c).48

Fig. 3(c).50

 

LONG ANSWER QUESTIONS


UNIFORMLY ACCELERATED MOTION 3.83

LM OP
N Q
3.84 PHYSICS (Vol. I)—XI
UNIFORMLY ACCELERATED MOTION 3.85

 
 
   

      

FG IJ

H K
   


   
   

  

  

     



  

    

   
3.86 PHYSICS (Vol. I)—XI

 
  

 

 

 

   

  
 


  
 

   

 

 
 

   
 
UNIFORMLY ACCELERATED MOTION 3.87

 
3.88 PHYSICS (Vol. I)—XI

LM 
OP Fig. 3(c).51

MM 
PP 
MM 
PP
N Q

 

 

   
UNIFORMLY ACCELERATED MOTION 3.89

 

 
  
 

Fig. 3(c).52

   

Fig. 3(c).54
Fig. 3(c).53
3.90 PHYSICS (Vol. I)—XI

LM OP
N  
Q 

  

  


Fig. 3(c).55

 LM   OP
 N  Q
 a 
 


   
   


UNIFORMLY ACCELERATED MOTION 3.91

   

 
  

  
FG   IJ



H K
  FG   IJ FG   IJ  
 H K H K

FG   IJ 
 H K 

  

     

 

FG IJ
H 
K



Fig. 3(c).56

 


 

 
 
3.92 PHYSICS (Vol. I)—XI

 

  

 
  

Fig. 3(c).57
 

 

 



  


 

NCERT EXERCISES (With Solutions)


NCERT EXERCISES (UNIT II) 3.93

Fig. 3(c).58

Fig. 3(c).60

Fig. 3(c).59

Fig. 3(c).61
3.94 PHYSICS (Vol. I)—XI

   
 


   


Fig. 3(c).63

  

Fig. 3(c).62
NCERT EXERCISES (UNIT II) 3.95


 

   

  



  
3.96 PHYSICS (Vol. I)—XI

Fig. 3(c).65

Fig. 3(c).64


NCERT EXERCISES (UNIT II) 3.97

Fig. 3(c).66


 
  

 Fig. 3(c).67
  
3.98 PHYSICS (Vol. I)—XI

FG IJ

H 
K 1 2 t

FG IJ
H 
K
Fig. 3(c).70

Fig. 3(c).68

Fig. 3(c).71

Fig. 3(c).69
NCERT EXERCISES (UNIT II) 3.99

Fig. 3(c).73

Fig. 3(c).72

 
3.100 PHYSICS (Vol. I)—XI

Fig. 3(c).75

 

Fig. 3(c).74

  

    
NCERT EXERCISES (UNIT II) 3.101

Fig. 3(c).76

NCERT EXEMPLAR PROBLEMS (With Answers)


[Based on Higher Order Thinking Skills].
MCQ I (Only one option is correct.)

Fig. 3(c).77
3.102 PHYSICS (Vol. I)—XI

MCQ II (Any number of options may be


=
correct.)
é ù
+ ê ú
ë û

=
é ù
ê ú
ë û

= - = = - Fig. 3(c).79

Fig. 3(c).78
 

´ ´ + ´ ´

Fig. 3(c).80

+
-

+ = +

+
NCERT EXEMPLAR PROBLEMS (UNIT II) 3.103

VSA

 

=-

Fig. 3(c).81

Fig. 3(c).82
3.104 PHYSICS (Vol. I)—XI

 Fig. 3(c).84 Fig. 3(c).85



 
 


  
 
 
 





SA

t
Fig. 3(c).83

=
NCERT EXEMPLAR PROBLEMS (UNIT II) 3.105

9 m s–1

9m - +

=- +

10 m é ù
- ê- + ú
ë û

Fig. 3(c).86 -

Fig. 3(c).89

LA

Fig. 3(c).87
-
 =

= ´ ´

-

´ ´

- -

Fig. 3(c).88
3.106 PHYSICS (Vol. I)—XI

 r ´




-
 ´ -
= = ´

-
´
= -
 ´


= ´ ´ = - =

-
= æ ö æ öæ ö
´ ´ + çè - ÷ø - çè ÷ø çè - ÷ø

æ ö æ ö
çè ÷ø - ´ ´ç ÷ =
è ø
   
  

4.1
4.2 PHYSICS (Vol. I)—XI


Fig. 4(a).1. Representation of a
vector

Y
B


F
 A

 O X
Fig. 4(a).2

  

 
b

 Fig. 4(a).3. Parallel vectors

 
Fig. 4(a).4. Antiparallel vectors
 
VECTORS 4.3

  

 

  

 

   
Fig. 4(a).5. Orthogonal triad of unit
vectors

Fig. 4(a).6. Co-initial vectors

   

  

 
Fig. 4(a).7. Co-terminus vectors
   

 

Fig. 4(a).8. Negative vector


  

Y
Fig. 4(a).9 P(x, y)


r


O X

 Fig. 4(a).10. Position vector in two


dimensions
4.4 PHYSICS (Vol. I)—XI

     


       

 
 
Fig. 4(a).11. Position vector in three
dimensions


  Y

Q
 
 


 P
 

O X
Fig. 4(a).12. Displacement vector

     
 
VECTORS 4.5

CHECKPOINT
( x1 , y1 ) ( x2 , y2 )
    

 
   
      
 

     
 

 

 Fig. 4.(a).13

 Fig. 4.(a).15
 
  

      
   

     
      

    
     
  


 d2
d1

 + d2
d1

O x
Fig. 4.(a).14


  
   
 
 
 

4.6 PHYSICS (Vol. I)—XI
 

 

 
Fig. 4(a).16. Scalar multiplication
  of a vector

 

 

  Fig. 4(a).17. Scalar


multiplication of a vector


 


 

  
c

  
Fig. 4(a).18. Equal vectors

  

  
    
VECTORS 4.7

 

 

 

Fig. 4(a).19. Displacement


of a vector parallel to itself

 
   
 
       


0

 

  

 

  
4.8 PHYSICS (Vol. I)—XI

 
 0

 
0

B

 
 
C

 
 

   A

Fig. 4(a).20. Triangle law of vectors

 

 
(a) (b)
   Fig. 4(a).21. Triangle law of vectors
(Graphical method for addition of
  vectors)

  
VECTORS 4.9
  

  

   

    Fig. 4(a).22

 

  
C


R
  
 Q  Q
 
O  A N
P

Fig. 4(a).23. Addition of vectors by


triangle law of vectors
  

 

 
  
  

 

  

    
  

     
   
4.10 PHYSICS (Vol. I)—XI

 

    


 

FG  IJ

H   K
S

F  I B
 GG  JJ C

H K
  

P
Q
 A

Fig. 4(a).24

  
  

 

  
 
 

     

(a) (b)
Fig. 4(a).25. Parallelogram law of vectors

 
Q

   
Q

Fig. 4(a).26. Addition of vectors



by parallelogram law of vectors
VECTORS 4.11

 

 

  

 

 
  
  
  
      

  

 
Q 

 

 d 



 

FG  IJ

H  K

 
P Q

   

 
Fig. 4(a).27

  
4.12 PHYSICS (Vol. I)—XI



 

 

 
P Q


    
 R
Q

   
 
P
 Fig. 4(a).28

 

FG IJ

H K

 

 
P Q

   

 



  

   



 


 
 
Fig. 4(a).29

VECTORS 4.13
 


Fig. 4(a).30  
 
  
CHECKPOINT
 

a B C
D 
 
Q R
Q

 
O 
P A N

Fig. 4(a).32

 

 


  

Fig. 4(a).31  

   


 

W1 W2

F1
F2
R1 R R2

Fig. 4(a).33

Fig. 4(a).34
4.14 PHYSICS (Vol. I)—XI

   

  
P
  

 

  

   

Fig. 4(a).35. Polygon law of vectors

   


R
    

  
AC

     
AD AC CD P Fig. 4(a).36. Proof of Polygon
law of vectors
       
AE AD DE

        
VECTORS 4.15

 

  

  

Fig. 4(a).37. Commutative


  
property of vector addition
  

   

   
    

     

    

     

   
PT
   

     

 Fig. 4(a).38. Associative


  
PT law of vector addition
   

     
4.16 PHYSICS (Vol. I)—XI



 
 

    
 
   

c2  


 

 
 




   

    


 
 

  



   D C

  

  

A B

Fig. 4(a).39 Fig. 4(a).40


VECTORS 4.17
  

   

  
 
CD AB
     

 
 


    


Fig. 4(a).43
Fig. 4(a).41

  

 
 

  

  

    

    

     

Fig. 4(a).42
4.18 PHYSICS (Vol. I)—XI

 

 

 

   

B
B
B A A
A–
–B B –B
A–
A B

(a) (b) (c)


Fig. 4(a).44. Subtraction of vectors

 

 

Fig. 4(a).46(a)
   
 ¹ 

     
  ¹  
VECTORS 4.19


  

 
 C

Fig. 4(a).45
    

 

Fig. 4(a).46(b)

Fig. 4(a).46
 

 

   

Fig. 4(a).47(a)
2
 


+ (– 1) – 1
2

or 2 – 1

Fig. 4(a).47(b)

 

 

Fig. 4(a).48
4.20 PHYSICS (Vol. I)—XI

  

 
 
 

 
 PC
 
 
  
 
  


 

 

  
a b

    
   
 
    
 

  

 
b


   
 
 
  Fig. 4(a).49. Resolution of vector

M P

A
Ay Ay

 
O M X
Ax

Fig. 4(a).50. Resolution of a vector
 into two rectangular components
VECTORS 4.21

  

  

  

 

 

  

  

  

 

 

 

 
  

 

 
  

j
  


 Fig. 4(a).51. Resolution of position
vector
4.22 PHYSICS (Vol. I)—XI

®

  ®
= + +
®
| | + = + +

V R

F
Fig. 4(a).52. Walk of a man

®

® ® ®  

 

Fig. 4(a).53
®

® ®

®
VECTORS 4.23

® ®

® ® ®

®
A
® ® ® S

® ® ® ®
Ay P
® ® ® ® ®

A ®
® ® O Ax Q

® ® ® ® ®
 Az

® ® ® ® ® ® ® ® T K

Fig. 4(a).54. Resolution of a vector


®    into three rectangular components

+ +

®
Y
® ® ®

®
A


  O
X
  
  
Z
+ +
Fig. 4(a).55. Direction cosines
of a vector
4.24 PHYSICS (Vol. I)—XI


  

®   

®   

Fig. 4(a).56
+ +


+ + 

+ +
 ®   
 + 

  

Fig. 4(a).57

 + 

®   ®  

® ®    

  

® ®
+ + +

+ +

Fig. 4(a).58
VECTORS 4.25

  

®   
A
®   
A

+ + + +

+ +

  

 

Fig. 4(a).59
 Fig. 4(a).60

 
  

 

  


  
 

 
  

4.26 PHYSICS (Vol. I)—XI

 

 

Fig. 4(a).61

® ®


®

® ® ®

 Fig. 4(a).62. Relative velocity of


  two inclined velocities

 +  



+ 



 
®

® ® ®

 Fig. 4(a).63. Relative velocity of two


inclined velocities
+  

 

+   

 

+  


VECTORS 4.27

+  

+ +

® ® ® ® ® ®

Fig. 4(a).64. Rain and Man





  Fig. 4(a).65

® ® ®

+ 

+ Fig. 4(a).66
=
+
4.28 PHYSICS (Vol. I)—XI

® ®

Fig. 4(a).67
 

® ®

® ® ®

Fig. 4(a).68

Fig. 4(a).69
®
® ® ®
VECTORS 4.29

Fig. 4(a).70 Fig. 4(a).71

 

® ®
 Fig. 4(a).72. Scalar product of
vectors
® ®

® ®

® ®

® ®
® ® ® ®
®

® ® ® ®
 

           

® ®

® ® Fig. 4(a).73
 ®

®
4.30 PHYSICS (Vol. I)—XI


     
 

   
 
     

 

   

   

  Fig. 4(a).75

     

     Fig. 4(a).74

 


    


     
 × l =l ×
 l
 

 

   

     
VECTORS 4.31
® ®
CHECKPOINT
® ® ® ® ® ®

® ® ® ®

® ®
®    ®   
® ®
® ®      

       

   

      
k ® ®

® ®      
 B 
        ® ®

   

® ®

® ® ® ®

+ +

® ®

® ®

® ®
 ×
® ®
| || |

+ +
+ + + +

® ®

® ®
×
4.32 PHYSICS (Vol. I)—XI

® ®



® ®

® ®

®
  
Fig. 4(a).76. Dot product of vectors

® 

® ®   ®  

® ® ® ®

® ® ® ®
®   
® ® ® ® ® ® ® ®
®   
® ® ® ® ® ® ® ®


® ®
® ®
 

® ®     

® ®

     

     
® ®

® ® ® ®

® ®


® ® ® ®

® ® ® ®
VECTORS 4.33
          

           
 
i i
       
  
i
   
     
 

       
     
     
  
           


     
   

       
   

 
       
 
   
       

         

            
i
     

 
 
    

  
  

    

    


     

 

 
 

Fig. 4(a).77. Vector product of vectors

   
 
4.34 PHYSICS (Vol. I)—XI

® ®

® ®

® ®  ®
 
® ® 

® ®

Fig. 4(a).78. Right Fig. 4(a).79. Right


® ® hand thumb rule for hand screw rule for
finding the direction finding the direction
® ® of vector product of vector product

® ® ®

® ® ®

Fig. 4(a).80 Fig. 4(a).81

® ® ® ®
  
® ® 
 n
® ®  
  n  
 ® ® ® ® ® ® ®
     
® ® ®

           
     
® ® ® ®

® ® 

VECTORS 4.35
® ®   
LM  OP

N =
Q
        
  
        
  

Fig. 4(a).82. Aid to memory


® ® ® ® ® ® ®
  

® ® ® ® ® ® ® ® ® ® ® ®
+ + +
  

 
  
® ® 
 

® ®
  =

® ® ®
 =
     
  
S T

®
B h

® ® 
B
P N ® Q
A

® ®  Fig. 4(a).83. Area of parallelogram


 n
® ®
   

® ® ® ®
4.36 PHYSICS (Vol. I)—XI
 

 
  

   

  Fig. 4(a).84. Area of triangle

 
 =

 
       
 

     
  
   

             
   
        
k i
     

 
 =
  
   

  
   
 = 
 
 
 

   
 = 
 

=  

   

   
  
   

VECTORS 4.37
® ® ® ® ®
  
®

® ® ® ® ®
  
®

® ® ® ® ®

®

® ® ® ® ®
  
®

® ® ® ®
 +
® ® ®

® ® ® ® ® ® ® ® ®
 × =  × = ×  =

® ® ® ® ® ® ® ® ®
  = ×  ×
® ® ® ® ® ® ® ® ®
  = ×  ×

  

  

®    ®   
     
    + +
  
® ®      
 
      

  
  

® ®
 ®   

 
®   ® ® ® ®

  
  
® ® ®
    
 
® ®  
  

  
4.38 PHYSICS (Vol. I)—XI
  

             
j
 

 
      

    

   
   
 
     

 

VERY SHORT ANSWER QUESTIONS—Each Question Carries 1 Mark Only

   

 

   

 
 

 

 
  
 


    
 
i j

  
VECTORS 4.39

 
A A

     
   

     

     
j j

   
   

   

 
 

 
 
 

 

   

    
 
 

 

       
 
 
 

   

   

   

 

   
4.40 PHYSICS (Vol. I)—XI
    

 

   
 
 
      
  

    
 

SHORT ANSWER QUESTIONS—Each Question Carries 2 Marks

  

 


   

Fig. 4(a).85
VECTORS 4.41

  
    

 

 


 

  Fig. 4(a).86
 

 


  

  

Fig. 4(a).87

 

 
Fig. 4(a).88


    
i j k
 
   
i j k

  

       

i j k i j k


 
 

4.42 PHYSICS (Vol. I)—XI

SHORT ANSWER QUESTIONS—Each Question Carries 3 Marks

 

      
  

Fig. 4(a).91
Fig. 4(a).89

      

  

    

  

  

 
 

 Fig. 4(a).90
Fig. 4(a).92


 

  

   
  
s r

 

VECTORS 4.43

Fig. 4(a).93
Fig. 4(a).94
 
 
 

  

 
Q. 69. The magnitudes of displacements and
  
 
are 3 m and 4 m respectively, and = + . Considering
 
various orientations of and , what is (a) the maximum    


possible magnitude for and (b) the minimum possible   
magnitude ? 
Answer. (a) 7 m (b) 1 m 

       
   
 
 
          
    

    
  

   

 
 
4.44 PHYSICS (Vol. I)—XI

Fig. 4(a).96


Fig. 4(a).95

Fig. 4(a).97

LONG ANSWER QUESTIONS NS

 


VECTORS 4.45


   

 

   

  


  


Fig. 4(a).98

   
4.46 PHYSICS (Vol. I)—XI

 

 

      
 
  

    
  

       
   
 

       
    
      
  
       
    

 
 
  
VECTORS 4.47

       
  B     
 
   

     
 B       
     
A
  
 

  

        
n
LM 



 OP
        N Q
    
      
   

 
 

 



  
 
 


 
   



  
 
  



 
4.48 PHYSICS (Vol. I)—XI

     



     
 
 


 


 
  

 

   
 
     

     
   

 
 
 
    

        
    
        

  
 


   





VECTORS 4.49

         
 i   
      

 
    

 
  
      
    
 
   
     
i j k
 
  
 
i j k
  

    

 F   I  

GG JJ
 
   
 
FG  IJ H K 
 
H  
K   
          
    
     
          
   
     
  

Fig. 4(a).99

 
4.50 PHYSICS (Vol. I)—XI


 

Fig. 4(a).101

z 

Fig. 4(a).100
z 

  

LM  OP 








N Q
  

 





 

 

  
 
      
 
       

     
  
   

    

     F I
GG JJ
  

LM    OP H K


MM  PP LM OP
N Q  
    
 

N Q
 

 
 

 
 
 
 
 
VECTORS 4.51

     Solution. The unit vector parallel to


  
  

       
     
 

 
  

       
   

           
 
 

   

  

  

 
   

  
   
Fig. 4(a).102

P. 8. Prove that the vectors


  
       

   
 

  Solution. (a) The given vectors will constitute a triangle
 
only if one of the given vectors is equal to vector sum of the
remaining two vectors.
       
  

 

        
k
  
     
P. 7. Determine the unit vector parallel to the cross
product of the vectors        
   
     
   
4.52 PHYSICS (Vol. I)—XI
       

     

 

 

P. 9. Prove that the diameter of a circle subtends a


right angle at any point on the circumference of the circle.
Solution. (a) As is clear from Fig. 4(a).103
Fig. 4(a).103
     
     
 
  
   
 
 

 
    

  

 Fig. 4(b).1. Position vector of a


particle in a plane

Y 

on of .
 Directi
P
    P
  
r
 
r

r
    
   X
O 
 
  Fig. 4(b).2. Displacement and
average velocity of a particle
   
 


   
 

  
      
   
  

4.53
4.54 PHYSICS (Vol. I)—XI


 


 
 
  

  
  
  
     Fig. 4(b).3. The components and

    of velocity and the angle  it makes
      with x-axis. Note that = cos ,
= sin 
    



 
   
 

 
 



Fig. 4(b).4. Uniform motion in two


  dimensions
MOTION IN A PLANE 4.55

  

  
 

  

  
,



   

     

     

 


  
       
   
  




    
 
    
  
 
  
     
     
4.56 PHYSICS (Vol. I)—XI

  

 

   
  

  

Fig. 4(b).5. Uniformly accelerated


motion in two dimensions

 

 
MOTION IN A PLANE 4.57

 

         
      
   
   
   
 

   

  

 

Fig. 4(b).6. Determination of position


vector in two dimensions


 
  

   
  
4.58 PHYSICS (Vol. I)—XI

    

 

 

 

  

  
  


 
  

  
MOTION IN A PLANE 4.59



O
X

Fig. 4(b).7. Trajectory of horizontal


projectile
4.60 PHYSICS (Vol. I)—XI


æ ö
çè ÷ø

æ ö
çè = ÷ø

Fig. 4(b).8

 


Fig. 4(b).9. Resultant velocity of
horizontal projectile at any time


MOTION IN A PLANE 4.61


O

 

 
     Fig. 4(b).10. Velocity at different points

Fig. 4(b).12

Fig. 4(b).11

  

  

  

 


4.62 PHYSICS (Vol. I)—XI


Fig. 4(b).13

 

 Fig. 4(b).14. Trajectory of an


 oblique projectile


MOTION IN A PLANE 4.63
 2
Substituting values, y = u sin θ t – gt


Remember
Ê Z ˆ 1 Ê Z ˆ
Using equation (1), y = u sin θ Á – g Á If the projectile is thrown at an angle θ
˜
Ë W EQU q ¯ 2 Ë W EQU q ˜¯
Z
IZ with the vertical, then t =
W UKP q
or y = x tan θ – ...(2)
W EQU q
Ê Z ˆ
This is a first degree equation in y and a second degree equation in x. y = u cos θ Á
This is the equation of a *parabola. So, the path followed by the projectile, i.e., Ë W UKP q ˜¯
the trajectory of the projectile is parabolic. 
1 Ê Z ˆ
It is clear from equation (2) that the trajectory is completely known if u – g
2 ÁË W UKP q ˜¯
and θ are known. It should be kept in mind that equation (2) is valid only if θ
lies between 0 and π/2.
IZ 
or y = x cot θ –
W UKP  q

4(b).07. RESULTANT VELOCITY OF OBLIQUE PROJECTILE

Suppose the projectile is at P at time t. Let v be the resultant velocity of


the projectile at time t. This velocity is along the tangent to the trajectory at
point P.
Since the horizontal motion of the projectile is uniform motion therefore
the horizontal component of velocity will remain unchanged.
v
∴ vx = u cos θ ...(1) Y vy β vy
Let us now calculate the vertical component vy of the velocity at P. For P vx
this, we shall consider vertical motion of the projectile.

u sin θ
uy = u sin θ, ay = – g, ‘t’ = t, vy = ? u

We know that vy = uy + ayt θ


∴ vy = u sin θ – gt ...(2) O u cos θ X

Applying parallelogram law of vectors, we find that Fig. 4(b).15. Resultant velocity of
oblique projectile at any time
v2 = XZ + X[
v2 = (u cos θ)2 + (u sin θ – gt)2
[From equations (1) and (2)]
or v = u cos θ + u sin2 θ + g2t2 – 2ug t sin θ
2 2 2 2

or v2 = u2 (cos2 θ + sin2 θ) + g2t2 – 2ug t sin θ


But cos2 θ + sin2 θ = 1
∴ v2 = u2 + g2 t2 – 2u gt sin θ

or v= W + I  V - WIV UKP q

This equation gives the magnitude of the resultant velocity of the


projectile at time t.
→ Remember
If β is the angle which the resultant velocity v makes with the
X[ W UKP q - IV If the projectile is thrown at an angle θ
horizontal, then tan β = or tan β = with the vertical, then
XZ W EQU q
v= W I  V  Ō W IV EQU q
Ê IV ˆ
or β = tan–1 Á VCP q - Ê IV ˆ
Ë W EQU q ˜¯ and β = tan–1 Á EQV q -
Ë W UKP q ˜¯
This equation gives the direction of the resultant velocity.

*The general equation of a parabola which passes through the origin is y = ax – bx2.
4.64 PHYSICS (Vol. I)—XI

Fig. 4.(b).16. Maximum height of


an oblique projectile

 



 æ ö
 çè ÷ø

  
-


 

 





MOTION IN A PLANE 4.65

4(b).10. HORIZONTAL RANGE Y

u
It is the total horizontal distance from the point of projection to the point
where the projectile comes back to the plane of projection. It is denoted by R.
θ
In order to calculate horizontal range R, we shall consider horizontal X
R
motion of the projectile. The horizontal motion is uniform motion. It takes
place with constant velocity u cos θ. Fig. 4(b).17. Horizontal range
of an oblique projectile
 W UKP q
∴ R = u cos θ × time of flight = u cos θ ×
I

W
 UKP q EQU q
or R=
I If θ is with vertical, even then

W UKP q
W UKP q R=
or R= (... 2 sin θ cos θ = sin 2θ) I
I

4(b).11. MAXIMUM HORIZONTAL RANGE

For a given velocity of projection and at a given place, the value of R will
be maximum when the value of sin 2θ is maximum i.e., 1.
For R to be maximum, sin 2θ = 1 (maximum value)
or sin 2θ = sin 90° or θ = 45°
So, for a given velocity, the angle of projection for maximum range is
π
45°, i.e., .
4
Fig. 4(b).18. Two angles of
W projection for the same range
Maximum horizontal range, Rmax. =
I

4(b).12. TWO ANGLES OF PROJECTION FOR THE SAME RANGE


Y
W UKP q W UKP
∞ - q θ = 75°
Again, R= =
I I θ = 60°
[... sin (180° – 2θ) = sin 2θ] θ = 45°
Height

 
W UKP 
q W UKP 
∞ - q θ = 30°
or R= =
I I
θ = 15°
This shows that there are two angles of projection for the same horizontal Rmax.
range i.e., θ and (90° – θ) with the horizontal. The projectile will cover the same X
Range
horizontal range whether it is thrown at an angle θ or (90° – θ) with the horizontal.
This means that the horizontal range is the same whether the projectile is Fig. 4(b).19. R is same for θ = 15° and 75°.
Again, R is same for θ = 30° and 60°.
thrown at an angle θ with the horizontal or at an angle θ with the vertical.
R is maximum for θ = 45°
4.66 PHYSICS (Vol. I)—XI

CHECKPOINT


Fig. 4(b).21

 
 

    
 - -  
  


   

 
i j

 
i j


Fig. 4(b).20

|| ^

Fig. 4(b).22

 
 
MOTION IN A PLANE 4.67

 WHEN VELOCITY OF PROJECTION OF AN  WHEN HORIZONTAL RANGE IS n TIMES THE


OBLIQUE PROJECTILE IS INCREASED n TIMES MAXIMUM HEIGHT

1. Time of ascent becomes n times. W UKP q W UKP q


=n
2. Time of descent becomes n times. I I
3. Time of flight becomes n times.
Ê UKP  q ˆ
4. M ax i mu m he i ght i s i nc r ea s ed b y a facto r 2 sin θ cos θ = n Á ˜ or 4 = n tan θ
Ë  ¯
of n 2 .
5. Horizontal range is increased by a factor of n2. 4
or tan θ =
n

 
Solved Problems Maximum height, H = W UKP q
I

Example 5. A projectile is thrown at an angle θ with H W UKP q I


Now, = ×
the horizontal with kinetic energy E. Calculate the R I 
W UKP q EQU q
potential energy at the topmost point of the trajectory.
H 1 4H
Solution. Potential energy at the topmost point of the or = tan θ or tan θ =
R 4 R
trajectory = mg hmax.
  4 × 10 4
tan θ = = 0.8 or θ = 38.66°
= mg W UKP q
or =
50 5
I
Example 8. A cricket ball is thrown at a
Ê ˆ speed of 28 m s –1 in a direction 30° above the
= Á OW ˜ sin2 θ = E sin2 θ
Ë ¯ horizontal. Calculate (a) the maximum height, (b) the
time taken by the ball to return to the same level, and
Example 6. A projectile is thrown with an initial (c) the distance from the thrower to the point where
∧ ∧ the ball returns to the same level.
velocity of x K + y L . The range of the projectile is twice the [NCERT Solved Example]
Solution. (a) The maximum height is given by
[
maximum height of the projectile. Calculate .
Z
W UKP q  (28 sin 30 ° )2
hmax. = = m
I 2( 9. 8)
W UKP q W UKP q
Solution. =2
I I 14 × 14
= = 10.0 m
2 × 9. 8
or 2u2 sin θ cos θ = u2 sin2 θ
(b) The time taken to return to the same level is
or 2(u sin θ) (u cos θ) = (u sin θ) (u sin θ)
= (2u sin θ)/g = (2 × 28 × sin 30°)/9.8
But u sin θ = y and u cos θ = x (given)
= 28/9.8 s = 2.9 s
y (c) The distance from the thrower to the point where
∴ 2yx = y2 or 2x = y or =2
x the ball returns to the same level is

Example 7. A projectile has a range of 50 m and W UKP  q 28 × 28 × sin 60 °


reaches a maximum height of 10 m. What is the elevation = = = 69.3 m
I 9. 8
of the projectile ?
Solution. We know that Example 9. From the same point, two balls A and B
horizontal range, are thrown simultaneously. A is thrown vertically up with
a velocity of 20 m s–1. B is thrown with a velocity of 20 m s–1
W UKP q W UKP q EQU q at an angle of 60° with the vertical. Determine the
R= =
I I separation between the two balls at t = 1 second.
4.68 PHYSICS (Vol. I)—XI

 

Fig. 4(b).23
LM OP LM OP
Q N Q
  
N   

 

 

  

  

  g


Fig. 4(b).24
 
 

MOTION IN A PLANE 4.69

 
 


- 



   
  - 
 

 

Fig. 4(b).25. Velocity and acceleration of an object in uniform circular motion. The time
interval t decreases from (a) to (c) where it is zero. The acceleration is directed, at
each point of the path, towards the centre of the circle

   
 

  
 

    
  
4.70 PHYSICS (Vol. I)—XI

 
    
   
  
 

  
 

 

 

 


 
  

 
 
 

 
  
 

  
   

  
   
        

  

  
    

MOTION IN A PLANE 4.71

  






  

 
 
 


  
 




 




 
  
 
4.72 PHYSICS (Vol. I)—XI


 


 
 
 


  

 

  

  

 

   


Fig. 4(b).26. Resolving vectors at a
 point on a general path

 


MOTION IN A PLANE 4.73

 

   


     

  

  

   

   

   
a

Fig. 4(b).27. Position vector in


three dimensions
4.74 PHYSICS (Vol. I)—XI


 
a
   


  


 
  
  

  
  
  

 

VERY SHORT ANSWER QUESTIONS—Each Question Carries 1 Mark Only


MOTION IN A PLANE 4.75

 



 

 



4.76 PHYSICS (Vol. I)—XI

SHORT ANSWER QUESTIONS—Each Question Carries 2 Marks

 

 

 

 


MOTION IN A PLANE 4.77

 



    

SHORT ANSWER QUESTIONS—Each Question Carries 3 Marks

 
 

 

      

  °

°- 

 


4.78 PHYSICS (Vol. I)—XI


 

°
°- °    

   


Fig. 4(b).28
 

f 


 



 

 
°


°  


Fig. 4(b).29

QUESTONS BASED ON HIGHER ORDER THINKING SKILLS


MOTION IN A PLANE 4.79

    
 
 





 

   

Fig. 4(b).31
 

 
 


    

 

 

Fig. 4(b).30
4.80 PHYSICS (Vol. I)—XI

 

 

  

   
r
  
r

LONG ANSWER QUESTIONS


MOTION IN A PLANE 4.81

 

 
4.82 PHYSICS (Vol. I)—XI


  

     

  


    


 °
 
    °

 °
 

 
°


  °

 
°-



MOTION IN A PLANE 4.83

°- 
 

 


 -
  
°- 
 °
°

 - 

  °
 


 

  ° 
FG - IJ
H     
K

 

TYPICAL PROBLEMS BASED ON HIGHER ORDER THINKING SKILLS

P. 1. A body falling freely from a given height H hits


an inclined plane in its path at a height h. As a result of
this impact, the direction of the velocity of the body be-

comes horizontal. For what value of , the body will take


-
maximum time to reach the ground ?
Solution. Let us first calculate the time t1 taken by the
given body to fall through a height (H – h) by using the relation

Fig. 4(b).32
4.84 PHYSICS (Vol. I)—XI

Solution. Let 1 be the vertical velocity at the point A on


the lower level. Let 2 be the vertical velocity at the point A of
the upper level.
LM - 
OP

MN PQ

-  

- 
 

LM OP

N  -
Q
 


LM - FG - IJ FG - IJ OP
N H K H K
- -
- - 
Q 

LM - - 
OP
MN - PQ

LM 
OP
MN - PQ

Fig. 4(b).34

P. 2. At the National Physical Laboratory in


England, a measurement of the value of acceleration due
to gravity was made by throwing a glass ball straight up  
in an evacuated tube and letting it return as shown in
Fig. 4(b).33. If the time interval between the two passages

across the lower level is t1 and the time interval between
the two passages across the upper level is t2 , then show

that g = , where H represents the distance


   

between the two levels.


 

 -
P. 3. A stone is projected at a speed of 36.576 m s–1
directed 60° above the horizontal, at a cliff of height h as
shown in Fig. 4(b).35. The stone strikes the ground at A,
5.5 s after launching. Calculate the height h of the cliff
and the speed of the stone just before impact at A. Also
calculate the maximum height reached above the ground.
Given : g = 9.75 m s–2.
Fig. 4(b).33
MOTION IN A PLANE 4.85
Solution. Sound travels from Q to P in 3 s with velocity
of 335 m s–1.
 

Fig. 4(b).35


Fig. 4(b).36

 



  °
 


 


 

  
FG IJ

H K
 

P. 5. A ball is thrown horizontally from the top of a


 building 19.6 m high. How long will the ball take to strike
the ground ? What must be the initial velocity of the ball if
the line joining the point of projection to the point where
 it hits the ground is 45° with the horizontal ? Also calculate
the vertical velocity with which the ball hits the ground ?
Solution. Let us first calculate the time t taken by the
 ball to hit the ground.

P. 4. A shell fired from P is observed to explode at 


Q after an interval of 10 second. The sound of the explo-
sion reaches the gun after a further interval of 3 s.
Calculate 0 and .  
4.86 PHYSICS (Vol. I)—XI

-  - 
-  - 

 - 
 - 

Fig. 4(b).37

 
P. 7. A terrorist sitting in a stationary car shoots
normally on the compartment of a moving train. The hole

on the farther wall of the compartment is cm below

and cm behind the hole on the nearest wall of the


 compartment. Calculate the speeds of the bullet and the
P. 6. Prove that the motion of one projectile as viewed train if the width of the compartment is 4 m ? Given : the
from another projectile will be a straight line motion. bullet does not lose any kinetic energy on piercing the nearer
wall of the compartment. Take g = 10 m s–2.
Solution. Let us assume that two projectiles are simul-
  [Ans. 2160 km h–1, 90 km h–1]
taneously projected from a point O with velocities and at
angles 1 and 2 respectively with X-axis. Hint. =0+  10t2 ; = t


P. 8. A hunter aims his gun and fires a bullet
directly at a monkey in a tree. At the instant, the bullet
leaves the barrel, the monkey drops. Will the bullet hit
the monkey ? [Ans. Yes]
Hint. Equation of trajectory of bullet is


Fig. 4(b).38


 
Fig. 4(b).39
 
MOTION IN A PLANE 4.87

g Ê Z ˆ 1 2
or y=h– Hint. (i) 80 sin θ t – gt
2 ÁË W EQU q ˜¯ 2

1 2
1 2 = (100 cos 60°)t – gt
or y=h– gt 2
2
P. 9. Prove that there are two values of time for the (ii) For first cadet,
same height in the case of an oblique projectile. Also prove x + 50 = 100 sin 60° t ;
that the sum of the two values of time at which these heights
For second cadet, x = 80 cos θ t.
are attained is equal to the time of flight of the projectile.
P. 13. A projectile is thrown on an inclined plane.
1 2 Determine (i) time of flight (ii) range on the inclined plane
Hint. y = u sin θ t – gt
2 (iii) maximum range on the inclined plane (iv) maximum
height of the projectile above the inclined plane.
1 2
or gt – u sin θ t + y = 0 Solution. (i) Consider an inclined plane which makes
2
an angle α with the horizontal. Let a projectile be projected with
Since y is given to be constant therefore the above a velocity v making an angle θ with the horizontal. Let us choose
equation is a quadratic in time t.
X-axis along the inclined plane and Y-axis perpendicular to the
W UKP q inclined plane. g cos α and g sin α are the two rectangular com-
t1 + t2 = =T
I ponents of g.
P. 10. If R is the horizontal range for θ inclination Now, vx = v cos (θ – α) and vy = v sin (θ – α).
and h is the maximum height reached by the projectile,
Let us now calculate the time of flight T of the projectile.
4 The displacement of the projectile perpendicular to the
show that maximum range is given by + 2h.
J inclined plane is clearly zero.

W UKP  q W UKP q
Hint. h = ,R=
I I
R2
Put these values in + 2h.
8h
P. 11. A football is kicked with a velocity of 20 m s–1
at a projection angle of 45°. A receiver on the goal line 25 m
away in the direction of the kick runs the same instant to
meet the ball. What must be his speed if he has to catch the
ball before it hits the ground ?
Fig. 4(b).40
1 1
Solution. T = 2 × 20 × × s = 2.89 s
2 9. 8 1 2
Using S = ut + at for motion along y-axis, we get
20 × 20 2
R= m = 40.82 m
9. 8
1
Distance to be covered 0 = v sin (θ – α) T – g cos α T2
2
= (40.82 – 25) m = 15.82 m ;
2v sin (θ − α)
15. 82 ⇒ T=
v= m s–1 = 5.5 m s–1 g cos α
2. 89
(ii) Let us now calculate the range on the inclined plane.
P. 12. Two NCC cadets aim their guns simultane-
For this, we shall first calculate OC.
ously at a target which is placed on the top of the tower.
The first cadet fired the bullet with a speed of 100 m s–1 at 2v sin (θ − α)
OC = v cos θ × T = v cos θ × g cos α
an angle of 60° with the vertical. The second cadet is ahead
of the first cadet by a distance of 50 m. He fires the bullet with
a speed of 80 m s–1. 2v 2 sin (θ − α) cos θ
=
g cos α
(i) How must the second cadet aim his fire so that
both the bullets hit the target simultaneously ? Range on the inclined plane,

(ii) Calculate the approximate distance of foot of OC 2v 2 sin (θ − α) cos θ


R= = g cos2 α
the tower from the first cadet. cos α
[Ans. (i) 38.68° with the horizontal (ii) 180 m] (iii) Let us now calculate the maximum range on the
inclined plane.
4.88 PHYSICS (Vol. I)—XI

  

  
  
 -

    

   

-   

 -   - 

- 
  -   

NCERT EXERCISES (With Solutions)


NCERT EXERCISES (UNIT II) 4.89
   

 
       
     
   
          
     

   
  

    
   
   

   

 

 

  
Fig. 4(b).41
    
 

  
   
  

   
   
  

   
   

 

   
  

   
  

   
  
 

   
  
   
  

Fig. 4(b).42
4.90 PHYSICS (Vol. I)—XI

  

  

F I
GH   JK 

Fig. 4(b).43

   

FG IJ 
H 
K 

Fig. 4(b).44
NCERT EXERCISES (UNIT II) 4.91

  

Fig. 4(b).45

  


Fig. 4(b).47


  

  

 



 

FG IJ
Fig. 4(b).46 H K
 
FG IJ
   
H K



4.92 PHYSICS (Vol. I)—XI

 
q q

q  

   


 

   

  


 


q
  Fig. 4(b).48
q

 

     
   

  
 

 
NCERT EXERCISES (UNIT II) 4.93

       
  
  
   

 
   
 
   

    

  
   
  
      
   

    

     

q  q

 

 

 

    

 

   
 
  
 Fig. 4(b).49
 
q
q
 

     
 
 Fig. 4(b).50

4.94 PHYSICS (Vol. I)—XI



 

 




 

FG IJ

H K
Fig. 4(b).51

  

  

FG IJ

H 
K  

 


NCERT EXERCISES (UNIT II) 4.95

Fig. 4(b).53
Fig. 4(b).52


q


 F  I
q  q  q  q 
GH JK

q
 
q  
 
q
q

  q
q    
  q

q q

 q q

 FG IJ
q q
H K

NCERT EXEMPLAR PROBLEMS (With Answers)


[Based on Higher Order Thinking Skills].
MCQ I (Only one option is correct.)
     


 
×
q 

     
4.96 PHYSICS (Vol. I)—XI

Fig. 4(b).54


+

+ MCQ II (Any number of options may be

correct.)

 

  


 
 
  

A
P

0

C

– 2 – 1 B – 0
( = 0)

Fig. 4(b).55
NCERT EXEMPLAR PROBLEMS (UNIT II) 4.97

VSA

  

 
 

Fig. 4(b).56
  

 
 

Fig. 4(b).57
     

 

 

   

 

 

 
| + | + + = +

 
|  | +  = +
4.98 PHYSICS (Vol. I)—XI

SA

æ ö æ ö

èç ø÷ ç
è
÷
ø


   

 

Fig. 4(b).58

  

  

  

(a) (b)
Fig. 4(b).59

  

Fig. 4(b).61

 

 

Fig. 4(b).60
NCERT EXEMPLAR PROBLEMS (UNIT II) 4.99

LA
 

 

 

 



+

=
  
=  = +  =  +

 


  =

 
 
 =  = +  =  +
 

 

=  | | = +  =
40
Displacement

30
20
10

0 2 4 6 8 10
Time (s)

4.100
KINEMATICS (UNIT II) 4.101



   

  
4.102 PHYSICS (Vol. I)—XI

   
  

 

   

   
    

 

   

       

  

       

       

   
KINEMATICS (UNIT II) 4.103

  

     

 
 

     

 
  
 

 


   
 
4.104 PHYSICS (Vol. I)—XI

   


 

  

  
 

 

  

 
 

 

 

O 0
KINEMATICS (UNIT II) 4.105

 

   

 


Y
Q

O P X

 

 





4.106 PHYSICS (Vol. I)—XI

( ) ( )

speed

( ) ( )

   
 
      
 
   
   

 

   

 
KINEMATICS (UNIT II) 4.107




  
 
 
4.108 PHYSICS (Vol. I)—XI

  D

Distance
C
S
   

B
    A
 Time

   

   
KINEMATICS (UNIT II) 4.109

   



   
   
   

   
   
   



4.110 PHYSICS (Vol. I)—XI

  

O 
    

   
   
   

 
 
 
KINEMATICS (UNIT II) 4.111

 
   
     
   

     
   
   

    
   
  

  

 

 

   


4.112 PHYSICS (Vol. I)—XI


    
 
      
KINEMATICS (UNIT II) 4.113

 

   


  

 
      


 
   
 

 
4.114 PHYSICS (Vol. I)—XI

     

 

     

     

2
= 1.2t

   

  

 
KINEMATICS (UNIT II) 4.115
 

 

 

  
 
4.116 PHYSICS (Vol. I)—XI

   


a

  

 
KINEMATICS (UNIT II) 4.117

 
   
 

   

b

  

  

 

 
 
 
4.118 PHYSICS (Vol. I)—XI

 

   
 
 
KINEMATICS (UNIT II) 4.119

     

 


4.120 PHYSICS (Vol. I)—XI

    

    

   
   
   
   
   
   

 
   
 
 

 
KINEMATICS (UNIT II) 4.121

  

 

  

 

 

 
 
   

 
4.122 PHYSICS (Vol. I)—XI

 

    


KINEMATICS (UNIT II) 4.123

     

   
   
   

  
 
 
4.124 PHYSICS (Vol. I)—XI

= =

 

 

   
y (m s–1)

30    
20
10
0 1 2 3 4 Time (s)
–10


   
   

 Y



  B
A X
 
KINEMATICS (UNIT II) 4.125

 

 

= 

 
=  = 
 
=  = 
 
=  = 
 
=  = 


4.126 PHYSICS (Vol. I)—XI
 


 

(y 2 – y 1) m (y 2 – y 1) m
240 240

t(s) t(s)
8 12 8 12
(y2 – y1 ) m
(y 2 – y 1) m 240
240

t(s)
t(s) 12
8 12
KINEMATICS (UNIT II) 4.127

Velocity B

40°

25°
Time
Displacement in m

30
s –1

20
m
20

10

10 20 30 40 50 B
Time in second
60°
X
A 10 m s–1
4.128 PHYSICS (Vol. I)—XI

 
 
   
   

  
v  
 


 

 

 
 

 
   
       
 
      



 






 

 

   
     
   
   
     




   
KINEMATICS (UNIT II) 4.129

     

        
  
 
   

  
   
  
   

   
  
 

    


 

 
 
  
 

   
 
  




 
 
 
 
   

       
   
   

    
      
    
4.130 PHYSICS (Vol. I)—XI

   
  
 
 
 
 
 
 
  
     
 
  
 
    
 
  
 
  
   
 
  
   
     
  
   
 
 
 
   
 
      
  
   

     
 

 
  
    
   
   
  
   
 
 

 
  
  

 

  
KINEMATICS (UNIT II) 4.131

 

 


 

 

  

  

 

 

 


4.132 PHYSICS (Vol. I)—XI

 

  


 
 
  
 
     
     
     
   

 
  
   

 

 
  
      




 

 

  
KINEMATICS (UNIT II) 4.133

  

 
 
 
  



 
 

  

 





  


4.134 PHYSICS (Vol. I)—XI

 

 




 

 


 

 
    
   
  
 

   
 
  

 

KINEMATICS (UNIT II) 4.135

  
 
 
 
  

  

  

  
=

  

   

 

  

  
 
 
 

  

 


 

 

 


= =

4.136 PHYSICS (Vol. I)—XI

     

 

 
 
= = 
= 


=  =   


=
=  
  
= =
 

 =

= 
   = =
  
 

    = =  
  =
 
=
=
     
 
  
  =  
 
 

= 

  = 


  =  =    
 =      

     
 =        
   
=
   =

 =     =
= =  
 


=
KINEMATICS (UNIT II) 4.137


=
  
 
=
 

  
=
 
  =  
=

= 
= 

= 

B N =    =

(100 – ) m

100 m
A = 10 km h–1 O
W E
25 m s –1 m
45°
P 90° 100 km
A
= =
B = 10 km h–1 

S


= = =
= =

= =  =

   =
= =

=  

  =  =    
 
=     
 
  = =
= 
  
  = =  = 
4.138 PHYSICS (Vol. I)—XI


UNIT—III
LAWS OF MOTION 14 Periods

Chapter–5: Laws of Motion

SOLVED NUMERICALS 55 PLUS

UNSOLVED NUMERICALS 85 PLUS

VERY SHORT, SHORT & LONG ANSWER QUESTIONS 150 PLUS

QUESTIONS & PROBLEMS BASED ON HOTS 58 PLUS

NCERT EXERCISES, EXAMPLES & EXEMPLAR PROBLEMS 80 PLUS

MULTIPLE CHOICE QUESTIONS 174 PLUS


Sir Isaac Newton (1642–1727)
5.3
5.4 PHYSICS (Vol. I)—XI

®
NEWTON’S LAWS OF MOTION 5.5

® ® ®

®
®

® ®

® ®


m = constant
= constant

p p = constant
p

m m

Fig. 5(a).1 Fig. 5(a).2 Fig. 5(a).3


5.6 PHYSICS (Vol. I)—XI

® ® ®

® ®
® ®

® ®
® ® ®
® ® ®
   ®

®
®
 
NEWTON’S LAWS OF MOTION 5.7
® ® ®

® ®

® ®

®
® 
® ® 

® ®

®

®
®
®
®

® ®

® ® ®
Fig. 5(a).4. Simultaneous
® ® action of two forces

® ® ® ®

® ® ® ® ®

® ® ® ® ® Fig. 5(a).5. Resultant


acceleration

® ® ®

® ® ® ® ®   

® ® ® ®

® ® ® ® ®   
5.8 PHYSICS (Vol. I)—XI

     




 
  

 
 


NEWTON’S LAWS OF MOTION 5.9

 

 


5.10 PHYSICS (Vol. I)—XI

 Fig. 5(a).6
®  

   

®    

®
  
 
 

 

 
    
 

®  

 

 

 
NEWTON’S LAWS OF MOTION 5.11

® ®

®
z ®

® ®
z ® LM OP
NQ
®

® ®
 
®


® ®

®
®
5.12 PHYSICS (Vol. I)—XI
 

 




z 

LM OP
N Q z   
z 

z   


z    LM OP
NQ
 

     

Fig. 5(a).7. Impulse of constant force


NEWTON’S LAWS OF MOTION 5.13

Fig. 5(a).8. Impulse of variable force

 

 

 

 

 
5.14 PHYSICS (Vol. I)—XI

Fig. 5(a).10

    

    
 

Fig. 5(a).9
 


   
  
NEWTON’S LAWS OF MOTION 5.15

  
 

LM OP   

N   
Q

 
Fig. 5(a).12

   
    
AB

 

Fig. 5(a).11

 

Fig. 5(a).13

 


5.16 PHYSICS (Vol. I)—XI

 

 
Fig. 5(a).14. Newton’s third law
of motion
NEWTON’S LAWS OF MOTION 5.17

   

Fig. 5(a).15. Body resting on a


horizontal surface

 
V R

Fig. 5(a).16. Walk of man

Fig. 5(a).17. Lawn sprinkler


5.18 PHYSICS (Vol. I)—XI

 Fig. 5(a).18. Horse and cart problem

 

 

  


Fig. 5(a).19

Fig. 5(a).20. Elevator at rest


NEWTON’S LAWS OF MOTION 5.19


Fig. 5(a).21. Elevator moving up with
uniform velocity

 


Fig. 5(a).22. Elevator moving down with
uniform velocity

Fig. 5(a).23. Elevator moving up with


uniform acceleration

 


Fig. 5(a).24. Elevator moving down with
uniform acceleration
5.20 PHYSICS (Vol. I)—XI


 

Fig. 5(a).25. Elevator falling freely

CHECKPOINT

Fig. 5(a).26

 

 




NEWTON’S LAWS OF MOTION 5.21


 
 

 
   

 

   



 Fig. 5(a).27. Connected motion
5.22 PHYSICS (Vol. I)—XI


 
 

Fig. 5(a).28



NEWTON’S LAWS OF MOTION 5.23

Fig. 5(a).29

Fig. 5(a).32

 
 
 
Fig. 5(a).30

Fig. 5(a).33

Fig. 5(a).31

 

LM 
  OP
N  Q Fig. 5(a).34

 
 
 
 
5.24 PHYSICS (Vol. I)—XI

Fig. 5(a).35

  

Fig. 5(a).36 Fig. 5(a).37

Fig. 5(a).38 Fig. 5(a).39



= = =  

 


 
NEWTON’S LAWS OF MOTION 5.25

 


 
5.26 PHYSICS (Vol. I)—XI
 

 

Fig. 5(a).40. One-dimensional collision

 

 

 

 

 

 

 

 

     

 

   

   
1f
NEWTON’S LAWS OF MOTION 5.27


Fig. 5(a).41. Recoil of gun

 

   

 


Fig. 5(a).42. Machine gun


firing bullets

 
5.28 PHYSICS (Vol. I)—XI

    

         


     
 0

Fig. 5(a).43. Explosion of a bomb

   

    

R|     U|

S|    
V|
T    
W
   
    
   
NEWTON’S LAWS OF MOTION 5.29

 


 


 


  

FG IJ
H 
K
FG IJ
H 
K
5.30 PHYSICS (Vol. I)—XI

FG IJ 
H 
K


V

 

 

 





   

 
NEWTON’S LAWS OF MOTION 5.31
  
 
  

       

 

 

    

    

  LM 

OP
N Q

FG IJ
 H K
Fig. 5(a).44
5.32 PHYSICS (Vol. I)—XI




 
  
   

Fig. 5(a).45. Rocket propulsion



  
    
   
 

  
NEWTON’S LAWS OF MOTION 5.33

    

 


 

 


5.34 PHYSICS (Vol. I)—XI

 
–      
   
  
    
   
 
 

   
   




 

  


NEWTON’S LAWS OF MOTION 5.35

 
  


z z z


LM  OP
N 
 Q

 
 

FG IJ
 H 


K

FG IJ
H 
K
FG  IJ
H K   

 
 




 
   

5.36 PHYSICS (Vol. I)—XI

  

 
  
  
  
 
   

  

Fig. 5(a).46. Equilibrium of


concurrent forces



  
  

  

 Fig. 5(a).47. Concurrent forces on a


 
rigid body
 

  

   

   
NEWTON’S LAWS OF MOTION 5.37

  

  
   
 
 

Fig. 5(a).48. Concurrent forces
in equilibrium

Fig. 5(a).50
Fig. 5(a).49

   

 


 

 

  
 

 
    


 

5.38 PHYSICS (Vol. I)—XI
NEWTON’S LAWS OF MOTION 5.39

Fig. 5(a).51. Cartesian system of


co-ordinates


5.40 PHYSICS (Vol. I)—XI

Fig. 5(a).52 Fig. 5(a).53


Fig. 5(a).54 Fig. 5(a).55

 

F cos 

Fig. 5(a).57


Fig. 5(a).56





NEWTON’S LAWS OF MOTION 5.41

Fig. 5(a).61 Fig. 5(a).62

F I
  GH 
JK
F I
 GH 
 JK 

Fig. 5(a).58


 

Fig. 5(a).59 Fig. 5(a).60

F I
GH  JK
LM 
OP
N  Q 
Fig. 5(a).63 Fig. 5(a).64

 

5.42 PHYSICS (Vol. I)—XI


 


   

 
Fig. 5(a).67

Fig. 5(a).68 Fig. 5(a).69


Fig. 5(a).65 Fig. 5(a).66

 
 

F I
GH  
JK 

 


NEWTON’S LAWS OF MOTION 5.43

 


   
Fig. 5(a).70 Fig. 5(a).71

VERY SHORT ANSWER QUESTIONS—Each Question Carries 1 Mark Only

Fig. 5(a).72
5.44 PHYSICS (Vol. I)—XI


  


NEWTON’S LAWS OF MOTION 5.45

SHORT ANSWER QUESTIONS—Each Question Carries 2 Marks

Fig. 5(a).73

Fig. 5(a).74
5.46 PHYSICS (Vol. I)—XI

z
z
Fig. 5(a).75

SHORT ANSWER QUESTIONS—Each Question Carries 3 Marks

 

Fig. 5(a).77
Fig. 5(a).76
NEWTON’S LAWS OF MOTION 5.47

FG IJ
H K
 

Q. 69. Three masses connected by massless string as


shown in Fig. 5(a).78 are placed on a horizontal
frictionless surface. Find tensions T1 and T2 .

Fig. 5(a).78

Answer. Acceleration of system =


 

= m s–2 = 1 m s–2
 

Now, T1 = m1a = 10 × 1 N = 10 N
and T2 – T1 = m2a or T2 = T1 + m2a
or T2 = 10 + 20 × 1 = 30 N.

Q. 72. Fig. 5(a).80 shows overhead views of four situ-


ations in which two forces accelerate the same block across
a frictionless floor. Rank the situations according to the
magnitudes of (a) the net force on the block and (b) the
acceleration of the block, greatest first.

Fig. 5(a).79
5.48 PHYSICS (Vol. I)—XI


Q. 74. With what accele-
ration ‘a’ should a box descend
so that a block of mass M placed

in it exerts a force on the

floor of the box ?


Answer. The block is at Fig. 5(a).82
Fig. 5(a).80
rest with respect to the box which
Answer. (a) and (b) 1, 4, 3, 2. is accelerated with respect to the ground. So, the acceleration of
the block w.r.t. the ground is ‘a’ downward.
  

 

FG  IJ
H K
  
 

Q. 73. The body that is suspended by a rope in Fig.
5(a).81 has a weight of 75 N. Is T equal to, greater than, or
less than 75 N when the body is moving upward (a) at
constant speed, (b) at increasing speed and (c) at
decreasing speed ? Fig. 5(a).83

Fig. 5(a).81
Answer. (a) Equal ; (b) Greater ; (c) Less


NEWTON’S LAWS OF MOTION 5.49

Q. 82. A spring weighing machine inside a station-


ary lift reads 50 kg when a man stands on it. What would
happen to the scale reading if the lift is moving up-
wards with (i) constant velocity, and (ii) constant accel-
eration a ? [IIT 1972]
Answer. (i) In this case, acceleration = 0

LM 
OP LM 
OP
N Q N Q
Q. 83. A cricket ball of mass 150 g is moving with a
velocity of 12 m s–1 and is hit by a bat so that the ball is
turned back with a velocity of 20 m s–1. The force of blow
acts for 0.01 second on the ball. Calculate the average force
exerted by the bat on the ball. [IIT 1974]
Answer. | J | = 150  10–3 (12 + 20) N s = 4.8 N s

Fig. 5(a).84

Fig. 5(a).86

   

  

Fig. 5(a).87

Fig. 5(a).85
5.50 PHYSICS (Vol. I)—XI


     

 

LONG ANSWER QUESTIONS

  

  

Fig. 5(a).88
   



NEWTON’S LAWS OF MOTION 5.51

Fig. 5(a).89
5.52 PHYSICS (Vol. I)—XI

Fig. 5(a).90

LM OP
N  Q Fig. 5(a).91
NEWTON’S LAWS OF MOTION 5.53

Fig. 5(a).94

LM OP
N   Q

Fig. 5(a).92

Fig. 5(a).93
5.54 PHYSICS (Vol. I)—XI







NEWTON’S LAWS OF MOTION 5.55

FG IJ
H 
K

 

 

LM OP 
 N 
Q

Fig. 5(a).95
Fig. 5(a).96
5.56 PHYSICS (Vol. I)—XI

  
 
 

  
 
  Fig. 5(a).98



 
  
 

FG  IJ  
H K 


Fig. 5(a).97



NEWTON’S LAWS OF MOTION 5.57

Fig. 5(a).99  


Fig. 5(a).100


5.58 PHYSICS (Vol. I)—XI

  


  


 

LM     
OP
MN PQ


 
 


Fig. 5(a).101 

 Fig. 5(a).102


Friction

Fig. 5(b).1

Fig. 5(b).2

Fig. 5(b).3

Fig. 5(b).4

Fig. 5(b).5

Fig. 5(b).6

5.59
5.60 PHYSICS (Vol. I)—XI

Fig. 5(b).7. Irregular projection of two


surfaces

Fig. 5(b).8. Static friction

 

 
FRICTION 5.61

Fig. 5(b).9. Experimental verification


of the laws of limiting friction
5.62 PHYSICS (Vol. I)—XI


Fig. 5(b).10. Variation of frictional force


with the applied force

 

 

 
FRICTION 5.63

  

    


   

   

 

 


 

 


Fig. 5(b).11. Angle of friction
 


5.64 PHYSICS (Vol. I)—XI

  

 
Fig. 5(b).12. Angle of repose

 



  

     

  
  

    

 

 


Fig. 5(b).13. Body accelerating
     down an inclined plane
FRICTION 5.65

   

  


 
Fig. 5(b).14. Body sliding over a rough
  horizontal surface


 


    
    Fig. 5(b).15. Body moving up a rough
inclined surface

   

  

fk

 θ
sin θ
W

P W co

θ W
 

  Fig. 5(b).16. Body moving down an


inclined plane
5.66 PHYSICS (Vol. I)—XI
    
   
  

  

Fig. 5(b).17. Cause of rolling friction

  

 


FRICTION 5.67

Fig. 5(b).18. Ball-bearings

 

 Fig. 5(b).19. Pulling a block


 

 
  
  

Fig. 5(b).20. Pushing a block

 

  
5.68 PHYSICS (Vol. I)—XI

 
 
 
 


   




   

  



Fig. 5(b).21
   
FG IJ Fig. 5(b).23
  
H  
K

Fig. 5(b).24

   

 
 


Fig. 5(b).22
FRICTION 5.69

Fig. 5(b).26

P
    
m
  

F I
GH  JK

Fig. 5(b).25

  

   
 
  

  

Fig. 5(b).27

   

   
 

 
5.70 PHYSICS (Vol. I)—XI



VERY SHORT ANSWER QUESTIONS—Each Question Carries 1 Mark Only

S SHORT ANSWER QUESTIONS—Each Question Carries 2 or 3 Marks

 
FRICTION 5.71

 




R sin 

 Fig. 5(b).28
 

  

Fig. 5(b).29



Fig. 5(b).30

Fig. 5(b).31
5.72 PHYSICS (Vol. I)—XI

   

Fig. 5(b).33

   

Fig. 5(b).32


Fig. 5(b).34



 



FRICTION 5.73

 

LONG ANSWER QUESTIONS

  
 

  


5.74 PHYSICS (Vol. I)—XI


 Fig. 5(b).35

F  I
   
GH  JK
  

   
    

    

 

 

  

 

 
 
 

   
FRICTION 5.75

 

  




 



  

     

  

  
 
  
5.76 PHYSICS (Vol. I)—XI



 
  
 
    
 

Fig. 5(b).36

Fig. 5(b).38

 

 






 

  

  
  
Fig. 5(b).39

 
Fig. 5(b).37

 
 

  
FRICTION 5.77


 

é  ù
ê 
ú
ê  ú Fig. 5(b).40
ë û
   

    

 

VALUE BASED QUESTION


5.78 PHYSICS (Vol. I)—XI

Dynamics of Uniform Circular Motion

 Fig. 5(c).1

 

 

 Fig. 5(c).2

 

     

   
    

5.78
DYNAMICS OF UNIFORM CIRCULAR MOTION 5.79

Fig. 5(c).3

Fig. 5(c).4

Fig. 5(c).5. (a) From its initial position, at the top, the
book is given two successive 90° rotations, first about
the (horizontal) x-axis and then about the (vertical)
y-axis. (b) The book is given the same rotations,
but in the reverse order.
5.80 PHYSICS (Vol. I)—XI

 

     
 

  
Fig. 5(c).6
 


 
 Fig. 5(c).7


  

 

  

 

  

 

 
 

 
 


Fig. 5(c).8
DYNAMICS OF UNIFORM CIRCULAR MOTION 5.81




Fig. 5(c).9

 

 

 
 
 

 
    
 

LM   OP

N Q

 

 
5.82 PHYSICS (Vol. I)—XI

 

 

  

 



 

      

  

 

  




  
  
  
 
 
DYNAMICS OF UNIFORM CIRCULAR MOTION 5.83



 


  


 LM  OP
 
N 
Q

Fig. 5(c).10. Uniform circular


motion in horizontal plane


 

  Fig. 5(c).11. Expression for


centripetal force
5.84 PHYSICS (Vol. I)—XI


  

  

  

     
 


 






  
 

 



  





DYNAMICS OF UNIFORM CIRCULAR MOTION 5.85

FG  IJ CHECKPOINT

H K

  


 
    

Fig. 5(c).12

 

5.86 PHYSICS (Vol. I)—XI

FG  IJ

 
H K
FG  IJ


H  K


  

      

FG IJ
  
H  
K



Fig. 5(c).13. Bending of cyclist


DYNAMICS OF UNIFORM CIRCULAR MOTION 5.87

 

 



F I
  GH JK


 


   


 
  
 
  

 

5.88 PHYSICS (Vol. I)—XI

 
 

 

  

   

 
Fig. 5(c).14. Motion of car
on circular level road


  

 Fig. 5(c).15. Motion of car on


 
 banked circular track
DYNAMICS OF UNIFORM CIRCULAR MOTION 5.89

  



Fig. 5(c).16. Resolution of

 

   

 

 
 
Fig. 5(c).17. Motion of car on
     rough banked circular track

   s
   
 

   

  

 
 
5.90 PHYSICS (Vol. I)—XI

 
 

  
    


 

 





 


F I
   GH   

JK

VERY SHORT ANSWER QUESTIONS—Each Question Carries 1 Mark Only


DYNAMICS OF UNIFORM CIRCULAR MOTION 5.91

SHORT ANSWER QUESTIONS—Each Question Carries 2 or 3 Marks


5.92 PHYSICS (Vol. I)—XI

FG    IJ
H K 

  


 

  Fig. 5(c).18

   


  
   
 

 
 
DYNAMICS OF UNIFORM CIRCULAR MOTION 5.93

FG IJ
 
H 
K

  

   

  

 

Fig. 5(c).19

Fig. 5(c).20
5.94 PHYSICS (Vol. I)—XI

Fig. 5(c).22


Fig. 5(c).21 

  
  

Fig. 5(c).23 Fig. 5(c).24

x y

LONG ANSWER QUESTIONS


DYNAMICS OF UNIFORM CIRCULAR MOTION 5.95




 

 


 

Fig. 5(c).25 

LM  
OP
MN  PQ
5.96 PHYSICS (Vol. I)—XI

  

F I /

GH   

JK

  

 
    
   
   

     


 
  
T cos 
T

  

  

T sin LM OP

  
NQ 


mg  


 
DYNAMICS OF UNIFORM CIRCULAR MOTION 5.97

 
 

 

    

 

   

 
 

  
     
 
 
 


  
  

 
 
   



 
FG IJ

  
H 
K
 
 
   


 

   


Fig. 5(c).26
5.98 PHYSICS (Vol. I)—XI


Fig. 5(c).27


    Fig. 5(c).29
  


  



 


Fig. 5(c).30


 

 
 

 


 
 

 
 
 
Fig. 5(c).28

  
NCERT EXERCISES (UNIT III) 5.99

NCERT EXERCISES (With Solutions)


Fig. 5(c).31
5.100 PHYSICS (Vol. I)—XI


 

  

 

  

 




 
 

 

 
   

 




NCERT EXERCISES (UNIT III) 5.101

(m)
A
Fig. 5(c).32 3

2

 
1

0 1 2 3 4 5 t(s)

Fig. 5(c).33

3m


O t
4s

Fig. 5(c).34

 
5.102 PHYSICS (Vol. I)—XI



Fig. 5(c).35

 
 
 
 

 

p
   

 

 

 Fig. 5(c).36

 


NCERT EXERCISES (UNIT III) 5.103

 

 

  

  
Fig. 5(c).37


 


5.104 PHYSICS (Vol. I)—XI

Fig. 5(c).38

 Fig. 5(c).39

 


     
  

Fig. 5(c).40

 

 


NCERT EXERCISES (UNIT III) 5.105



 


Fig. 5(c).41
 


Fig. 5(c).42
5.106 PHYSICS (Vol. I)—XI

 Fig. 5(c).44

   

Fig. 5(c).43


 

  

 
NCERT EXERCISES (UNIT III) 5.107

  

 
  
 
 

Fig. 5(c).45


 


 

LM OP
N  
Q

   

 
    
 
1
3

 
5.108 PHYSICS (Vol. I)—XI

 


 
 

   

  


 

       
  

  


  







FG IJ
H K
  Fig. 5(c).46
NCERT EXEMPLAR PROBLEMS (UNIT III) 5.109

NCERT EXEMPLAR PROBLEMS (With Answers)

[Based on Higher Order Thinking Skills].

MCQ I (Only one option is correct.)

  ®   ®

 
 

®  
   

® ® ®
D - = é- + - + ù
ë û
®
®
- + =- + = +

-
D +
® 
5.110 PHYSICS (Vol. I)—XI

æ ö
çè ÷ø

æ ö
çè + ÷ø

® ®
+ ´ =

® ®
=

MCQ II (Any number of options may be


correct.) Fig. 5(c).48


 
 
 
 

  

Fig. 5(c).49

  
Fig. 5(c).47
  

  
NCERT EXEMPLAR PROBLEMS (UNIT III) 5.111

  - 


     
  

R
m2
θ

θ
in
s
gs θ co
m1 m 2g
g
f
1
m

θ m 1g

Fig. 5(c).50



  
  

 

æ ö
ç ÷
è ø

æ ö
ç ÷
è ø

æ ö
Fig. 5(c).51 ç ÷
è ø

æ ö
ç ÷
è ø
 - 


Fig. 5(c).53

=
Fig. 5(c).52
 
 =
   
5.112 Comprehensive PHYSICS (Vol. I)—XI
 account of inertia of motion. The only retarding force
θ1 = VCP - ÊÁ ˆ˜ that acts on the driver, if he is not using a seat belt,
Ë ¯
comes from the friction exerted by the seat. This is not
  enough to prevent the driver from moving forward.
tan θ2 = =
  20. The velocity of a body of mass 2 kg as a function of t is
or θ2 = VCP - ÊÁ  ˆ˜ Æ
given by X
V = V Kł + V łL Find the momentum and the
Ë ¯
force acting on it, at time t = 2 s.
VSA
Æ
16. A girl riding a bicycle along a straight road with a speed Ans. At t = 2 s, X = Kł +  Lł
of 5 m s–1 throws a stone of mass 0.5 kg which has a speed
Æ Æ
of 15 m s–1 with respect to the ground along her direction R = O X = 
Kł +  łL = Kł +  łL
of motion. The mass of the girl and bicycle is 50 kg. Does Æ F
the speed of the bicycle change after the stone is thrown ? C = È VKł + V łL ˘ = Kł + VLł
FV Î ˚
What is the change in speed, if so ?
Æ
Ans. Initial momentum = 50.5 × 5 kg m s–1 At t = 2 s, C = Kł +  łL
Final momentum = (50 v + 0.5 × 15) kg m s–1 Æ Æ

Applying the principle of conservation of momentum, ( = O C = Kł +  łL


21. A block placed on a rough horizontal surface is pulled by
50 v + 0.5 × 15 = 50.5 × 5
a horizontal force F. Let f be the force applied by the rough
v = 4.9 m s–1, change in speed = 0.1 m s–1 surface on the block. Plot a graph of f versus F.
17. A person of mass 50 kg stands on a weighing scale on a
Ans. f = F until the block is stationary.
lift. If the lift is descending with a downward acceleration
f remains constant if F increases beyond point A and
of 9 m s–2, what would be the reading of the weighing
the block starts moving.
scale ? (g = 10 m s–2)
Ans. R = m(g – a) = 50 (10 – 9) = 50 N
The weighing scale shall show 5 kg.
18. The position time graph of a body of mass 2 kg is as given
in Fig. 5(c).54. What is the impulse on the body at t = 0 s
and t = 4 s?
Fig. 5(c).55
22. Why are porcelain objects wrapped in paper or straw before
packing for transportation ?
Ans. In transportation, the vehicle say a truck, may need to
halt suddenly. To bring a fragile material, like porcelain
object to a sudden halt means applying a large force
and this is likely to damage the object. If it is wrapped
up in straw, the object can travel some distance, as the
Fig. 5(c).54 straw is soft, before coming to a halt. The force needed
to achieve this is less, thus reducing the possibility of
Ans. At t = 0, the body is at rest. So, impulse of body is zero.
damage.
From t = 0 to t = 4 s, the body moves with uniform
velocity u. 23. Why does a child feel more pain when she falls down on a
hard cement floor, than when she falls on the soft muddy
 ground in the garden ?
Slope, u = m s–1

Ans. The body of the child is brought to a sudden halt when
Beyond t = 4 s, the body comes to rest.
she/he falls on a cement floor. The mud floor yields and
  the body travels some distance before it comes to rest,
Impulse at t = 4 s = m(v – u) =  ÊÁ  - ˆ˜ = - kg m s–1
Ë ¯  which takes some time. This means the force which
brings the child to rest is less for the fall on a mud floor,
19. A person driving a car suddenly applies the brakes on
as the change in momentum is brought about over a
seeing a child on the road ahead. If he is not wearing seat
belt, he falls forward and hits his head against the steering longer period.
wheel. Why ? 24. A woman throws an object of mass 500 g with a speed of
Ans. When the car is brought to a sudden halt, the upper 25 m s–1.
part of driver’s body continues to move forward on (a) What is the impulse imparted to the object ?
NCERT EXEMPLAR PROBLEMS (UNIT III) 5.113

   

Fig. 5(c).58

SA

 

Fig. 5(c).56 

Fig. 5(c).59

=
Fig. 5(c).57
5.114 PHYSICS (Vol. I)—XI

u + =

LA
® ® ®

h = 500 m

= 400 m

Fig. 5(c).60
® ® ® ® ® ®
+ + =
® ®

® ®
x y +
(m) ® ® ® ®
4 = - +
3
2m
2
1m
1 ® ® ® ®
´ + +
O 1s 2s 3s t
O 1s 2s 3s t

(a) (b)
Fig. 5(c).61

Fig. 5(c).62

+
NCERT EXEMPLAR PROBLEMS (UNIT III) 5.115

-
  

 

-
+

   
-

 = ´ ´ ´
 - 

´
–1 –1  = 
(m s ) (m s )

 = ´ ´
2 2

1 1 

O 1s 2s t O 1s 2s 3s t

(a) (b)
Fig. 5(c).63


®
 

®  
® ®
 =- 
 

 ®
  +  
®
®
  -  

® ®
-  +  = -
® ® ®
=- 

Fig. 5(c).64
5.116 PHYSICS (Vol. I)—XI

= - =

+ = + + =


H

 

 

Fig. 5(c).65

 +

  
 +  = + +
   
   
   
  
  
  

2N
1N
45° 45°
45°
®
90° F1
®
F2

Fig. 5(c).66

5.117
5.118 PHYSICS (Vol. I)—XI


 
   
  

   
 
  

 

 
LAWS OF MOTION (UNIT III) 5.119

 
 
  


 
5.120 PHYSICS (Vol. I)—XI

 

 

LAWS OF MOTION (UNIT III) 5.121

  

  

 

     
     

 


5.122 PHYSICS (Vol. I)—XI

 


 



LAWS OF MOTION (UNIT III) 5.123


 

 


5.124 PHYSICS (Vol. I)—XI

4N 1N

2N

  

     
 

  
  

  
LAWS OF MOTION (UNIT III) 5.125

 
 

 


5.126 PHYSICS (Vol. I)—XI

 


LAWS OF MOTION (UNIT III) 5.127
   

   

 

 

 


5.128 PHYSICS (Vol. I)—XI





 

 
 

 
 

 
 

 

 
 

 
 
LAWS OF MOTION (UNIT III) 5.129



 

 

 




   
5.130 PHYSICS (Vol. I)—XI

 
  
 

   
  
   

  
   
 

 
  
 
LAWS OF MOTION (UNIT III) 5.131

   
   
   



 


5.132 PHYSICS (Vol. I)—XI

  
LAWS OF MOTION (UNIT III) 5.133

 

 
5.134 PHYSICS (Vol. I)—XI



LAWS OF MOTION (UNIT III) 5.135

 
 
5.136 PHYSICS (Vol. I)—XI

 

1m
2.9 kg
1m
1.9 kg

   
   
   

   
   
   


LAWS OF MOTION (UNIT III) 5.137

 
   

= =  
 

2m

3m

m2 m3
P

 m1

  

   

3
F(N)

0
2 4 6 8
–3
t (s)
5.138 PHYSICS (Vol. I)—XI

F = 120 N

4 kg

A B C

2 kg

 
 

 
 

 


LAWS OF MOTION (UNIT III) 5.139

F
A B





   
5.140 PHYSICS (Vol. I)—XI



 


  
    
 

 

  
 
 
 
  


 







   


 


 

  

 
   
 

    
   
  

  
 
   
LAWS OF MOTION (UNIT III) 5.141


 

 
   
 



   

 
  


 


 
 
 
   

   
  
 
 
   
 

 

 

 

 

 


 
  
 
5.142 PHYSICS (Vol. I)—XI

 
 
 
 
 


 





   


 


 


 

  

   
   
 

  
 
  
LAWS OF MOTION (UNIT III) 5.143

 

  

  



 



 
 
  
 


 
  
 

  

 
5.144 PHYSICS (Vol. I)—XI

  
 a 



  
a 
  
 
  
 

  


 



 





 

  
 
 

 
LAWS OF MOTION (UNIT III) 5.145

  
     
 
     


 
  
 
 
 

   
 
 

     
      
     

 

 
 




    
  

  
   

 
 



5.146 PHYSICS (Vol. I)—XI


  



  
  
 

 
  
 

 
    
 

  

   

 
     
    

  


  

 


 
 


LAWS OF MOTION (UNIT III) 5.147


 =


=  =


= =

4 kg

 F F

  

=
  

 = =

L/2
o ot h 
Sm

L/2 h
ug 
Ro


 

    
= 
 
  T
 
2 kg a


  2g
= 
 
 
 

 

   
    
 
=  
 
  = 


=

  
=

5.148 PHYSICS (Vol. I)—XI

 

 
UNIT—IV
WORK, ENERGY AND POWER 12 Periods

Chapter–6: Work, Energy and Power

SOLVED NUMERICALS 60 PLUS

UNSOLVED NUMERICALS 70 PLUS

VERY SHORT, SHORT & LONG ANSWER QUESTIONS 125 PLUS

QUESTIONS & PROBLEMS BASED ON HOTS 36 PLUS

NCERT EXERCISES, EXAMPLES & EXEMPLAR PROBLEMS 75 PLUS

MULTIPLE CHOICE QUESTIONS 130 PLUS


James Watt (1736–1819)


Fig. 6.1

  
Fig. 6.2. Definition of work

6.3
6.4 PHYSICS (Vol. I)—XI

 


 

 
Fig. 6.3. 

Fig. 6.4. Work done by centripetal force


T
 = 90°


S
Fig. 6.5. Work done by tension

 


Fig. 6.6. Positive work
WORK, ENERGY AND POWER 6.5


  F
(> 90°)


S
Fig. 6.7. Negative work


   

         
    

 
 
6.6 PHYSICS (Vol. I)—XI

   

  


 

 
WORK, ENERGY AND POWER 6.7


  

 

Fig. 6.8 


 


 
   

 


  

  
6.8 PHYSICS (Vol. I)—XI
   
  

  

 

       
   
       
   

       

         

           

       

 b
F



ds

 

  a

Fig. 6.9. Work done by variable force

 z 
 
 

z 
WORK, ENERGY AND POWER 6.9


 
 
 
 
  
   

   
   

Fig. 6.10. Graphical method for


measurement of work done by
variable force

LM  OP
N Q
 

 

z 
z 

z 

z 



LM
N z z 
OP
Q
LM
MN

OP
PQ

z z
 
 

 
 

Fig. 6.11

6.10 PHYSICS (Vol. I)—XI

  

z  
z  

 





WORK, ENERGY AND POWER 6.11

u=0 , v ,= v

F

Fig. 6.12. Kinetic energy of a moving


body

 

 

FG IJ
H K
6.12 PHYSICS (Vol. I)—XI

  

F I
z z LM OP   

N Q GH  JK

 
 

 
  



WORK, ENERGY AND POWER 6.13



  

 

 

  

 
6.14 PHYSICS (Vol. I)—XI


 



 

1
2

  

    
 

   

LM OP
N   
Q

FG IJ
H  
K
    

 
WORK, ENERGY AND POWER 6.15

e
in
Ek tl
gh m constant
r ai
St

m constant p
Ek
Fig. 6.14. Variation of with p
Parabola
m constant

p
Ek Hy
Fig. 6.13. Variation of E k with p pe
rb
ola

1
p
 Fig. 6.15. Variation of with

p constant

Hy
Ek pe
rb
ola

Fig. 6.16. Variation of Ek with m



Ek constant

p ne
h t li
 aig
r
St

m
Fig. 6.17. Variation of p with


 
6.16 PHYSICS (Vol. I)—XI



 
 



 
LM 
 OP
MMN 
PPQ

 

 


CHECKPOINT
 P

 


 

  
  
  
  
WORK, ENERGY AND POWER 6.17


  


   


6.18 PHYSICS (Vol. I)—XI

 




  


  
 


    

 

 

 

FG   IJ
H K   
  


WORK, ENERGY AND POWER 6.19

R
 F

mg
 
 sin f
mg

co s
 mg


Fig. 6.18
 

FG IJ

H       
K
  Fig. 6.19
 

   

  

 

Fig. 6.20. Potential energy due to


configuration
6.20 PHYSICS (Vol. I)—XI

Fig. 6.21. Gravitational potential energy

Fig. 6.22. Variation of potential


energy with height


 

  

 

 


  

(a) (b) (c)


Fig. 6.23. Vertical motion of a body in
gravitational field
WORK, ENERGY AND POWER 6.21

   


6.22 PHYSICS (Vol. I)—XI

SUMMARISING

 

 

  
WORK, ENERGY AND POWER 6.23

   
 

 

B
h vB

C
Reference
level
vC

Fig. 6.24. Freely falling body


6.24 PHYSICS (Vol. I)—XI


mgh (= K.E. + P.E.)

Energy
 
P.E. K.E.

h=0 Height h
(Above reference level)

Fig. 6.25. Variation of kinetic energy and


potential energy with height


u=0
A
l
h1

h
h2

B
C

 Fig. 6.26. Body sliding down a smooth


inclined plane
  


R

  
m


g

sin
co


g
s

m

mg

Fig. 6.27
WORK, ENERGY AND POWER 6.25

   

E1
E2
h

M

Fig. 6.28. Vibrating simple pendulum


6.26 PHYSICS (Vol. I)—XI

=0
F=0
F=–k m

=+ 0

F=–k m

=– 0

=0
F=0
=– 0 =+ 0
=0

Fig. 6.29. Oscillating block

z 


z  z z
LM OP
N Q
Force


Extension
Fig. 6.30. Force-extension graph

 z z LM OP
N Q

WORK, ENERGY AND POWER 6.27

  

Fig. 6.31

Energy
E = Ep + Ek

Ep Ek
E
2

=– 0
– 0 =0 0 =+ 0
2 2

Fig. 6.32. Variation of Ek and Ep

   


Fig. 6.33

6.28 PHYSICS (Vol. I)—XI

 

 
    

 



   
  

   

 


5 kg

 3 kg

Fig. 6.34
WORK, ENERGY AND POWER 6.29

   

 
  
      

  

   

 
 

Fig. 6.36

N
+ kx

+  mg

mg
Fig. 6.35


 ± 

 


6.30 PHYSICS (Vol. I)—XI
B
R
P

   i n
gs

m
g
m 

co
fk

s
  
A
 mg

   Fig. 6.37

F I
 GH   JK

m
10 5m



  5 3m

   Fig. 6.38
   
 

FG IJ
H K

 
 

kg
10
M
m 8 kg
 
30°
Fig. 6.39

 

R T

a
°
30
sin
30°

0
10
100 cos 30° 80 N
30°

Fig. 6.40
WORK, ENERGY AND POWER 6.31

Fig. 6.41

Fig. 6.42

  
    
  
    
  


6.32 PHYSICS (Vol. I)—XI


WORK, ENERGY AND POWER 6.33

 

FG IJ

H 
K

F I
GH JK




z z

F I 
F I
GH  JK GH  JK
6.34 PHYSICS (Vol. I)—XI

  

 




  
   
 
 
 


   



    

 
  
 


WORK, ENERGY AND POWER 6.35
 
  

     


 

Fig. 6.43. Motion in vertical circle

 

 LM  OP Fig. 6.44. Tension in the string

N 
Q
6.36 PHYSICS (Vol. I)—XI


   

 

 

   

  

  

Fig. 6.45


WORK, ENERGY AND POWER 6.37



H
mg
TH TM
O M
TL
mg

mg
Fig. 6.46

R
mg

r
R

mg

Fig. 6.47


6.38 PHYSICS (Vol. I)—XI

Fig. 6.48

 

  

h
r

Fig. 6.49


WORK, ENERGY AND POWER 6.39


 

 Fig. 6.50
 





6.40 PHYSICS (Vol. I)—XI

Fig. 6.51

A B A B A B

m1 m2 m1 m2 m1 m2
   
1i 2i 1f 2f

 
BEFORE COLLISION DURING COLLISION AFTER COLLISION
  Fig. 6.52. One-dimensional elastic collision
WORK, ENERGY AND POWER 6.41

 


  
   
  
  

 

  


 



 

 

 
 
6.42 PHYSICS (Vol. I)—XI


 


 

A B A B

m2 m2
m1 m1
 (= m1 )   (= m1) 
1i 2i 1f 2f
 
( 2i) ( 1i)

Fig. 6.53. Colliding bodies are of same mass


 1f  1i
WORK, ENERGY AND POWER 6.43

 

A A
B B
m1 m2 m1 m2
  
1i  1f 2f
2i =0  
( 1i) ( 2 1i)

Fig. 6.54. Moving massive body colliding against a stationary light body


 

Wall

1i
m1

m1

– 1i

m2

Fig. 6.55. Light body colliding with


a stationary massive body


A B A B

m2
m1 m1 m2
 (= m1 ) 
1i 2f
    
2i =0 1f =0 (= 1i)

BEFORE COLLISION AFTER COLLISION

Fig. 6.56. Moving body colliding with a stationary body of


equal mass
6.44 PHYSICS (Vol. I)—XI


 

FG   IJ
H 
 


K

 

FG   IJ
 H 
 


K
 
 
FG   IJ
H 
 


K

 
 
FG   IJ
H 
 


K


2f
 
WORK, ENERGY AND POWER 6.45


 
  

  

 

FG IJ
   
H  K 

  

Fig. 6.57

 
 

  



 


 



 FG  IJ
 


H  K
FG  IJ

H  K
6.46 PHYSICS (Vol. I)—XI

 FG  IJ
H  K 


  



 







   

 
 
Fig. 6.58. Oblique collision

 

   


WORK, ENERGY AND POWER 6.47

 

   

    

     
  
     
×  × 
         
×  
 
  ×
 
   
 

 
 ^

 

   

 

   

  
  
 
     

     
6.48 PHYSICS (Vol. I)—XI

N V V


V
2m
W V  E
m

S
Fig. 6.59


     

      
    

 

 

 

 


WORK, ENERGY AND POWER 6.49

Fig. 6.60. Ball bouncing back



  

 

 

 

 



6.50 PHYSICS (Vol. I)—XI

 
 

 
 

 
   

 
 

   
 

 
 

   

 
 

 



WORK, ENERGY AND POWER 6.51

   
      
   







6.52 PHYSICS (Vol. I)—XI


 
   
 

    
    
    

     

 

  
 
 

  
 
  


 

FG IJ

H  K
LM 
OP
N  Q
 
 
WORK, ENERGY AND POWER 6.53

   

 

 

 

 
 
 
 

  

 


 

Fig. 6.61
6.54 PHYSICS (Vol. I)—XI

 

 

 
  Fig. 6.62
 

M+m
m
M h

Fig. 6.63


WORK, ENERGY AND POWER 6.55

VERY SHORT ANSWER QUESTIONS—Each Question Carries 1 Mark Only

  
6.56 PHYSICS (Vol. I)—XI

Fig. 6.64
WORK, ENERGY AND POWER 6.57

 

SHORT ANSWER QUESTIONS—Each Question Carries 2 Marks

 




6.58 PHYSICS (Vol. I)—XI

  

  


  

 

 = 

  2



WORK, ENERGY AND POWER 6.59


 

A
P
Q

B C

Fig. 6.65



 

SHORT ANSWER QUESTIONS—Each Question Carries 3 Marks


  
6.60 PHYSICS (Vol. I)—XI

  
+
  
+ æ ö
  ç + ÷ø
è
+ æ ö
 ç + ÷ø
è

Fig. 6.67

Fig. 6.66


100 N

Fig. 6.68


WORK, ENERGY AND POWER 6.61

 

     

 
Fig. 6.69

 

¶ ¶ æ ö æ ö
  ÷ø 
¶ ¶ çè çè ÷ø

 

 

 

+
 +
6.62 PHYSICS (Vol. I)—XI




z
 z

 z


æ
ç
è
ö
÷
ø
LM  + OP
MN  + PQ

0

Fig. 6.70

 

æ ö æ ö
ç ÷ +ç ÷
è ø è ø

+
O X

 Fig. 6.71


WORK, ENERGY AND POWER 6.63

Fig. 6.73

 
– 60 J
 
a
60 J  

60 J b  

Fig. 6.72

  

     

 


 

  

 
 
6.64 PHYSICS (Vol. I)—XI

Fig. 6.74

Fig. 6.76


 

 

Fig. 6.75
WORK, ENERGY AND POWER 6.65

Fig. 6.77

 

Fig. 6.78

  

LONG ANSWER QUESTIONS


6.66 PHYSICS (Vol. I)—XI

   

   
WORK, ENERGY AND POWER 6.67

   

     

 

  

  

  

   

  
10 N

Force

25 m 50 m
Distance
Fig. 6.79
6.68 PHYSICS (Vol. I)—XI

Fig. 6.80

 

Fig. 6.81
WORK, ENERGY AND POWER 6.69

 
 

Fig. 6.83


 


Fig. 6.82 



6.70 PHYSICS (Vol. I)—XI


Fig. 6.84

  
 +
+ +


+ +

+
WORK, ENERGY AND POWER 6.71

Fig. 6.85

 
 
 

 + 
 
  

   

 
 

 +    

 
  
 +
  
 

   


6.72 PHYSICS (Vol. I)—XI

 

 
    


 
 

  


 LM  +
OP +
 MN PQ

+

   

  
  
 

  
LM OP
 N + Q





 

 FG IJ
H K
 
+ +





WORK, ENERGY AND POWER 6.73


  
+
+ +

 
 +   
+ + 

+ 
  

+ + F I
 GG  JJ

 +


 H K
+ +


 
 +
+ +

 
 

LM OP

 +


MN + 
PQ
+ +

  

   

+ 
   

+

 æ ö
 çè ÷ø

  


  


6.74 PHYSICS (Vol. I)—XI

P. 1. A woman pushes a trunk on a railway plat-


form which has a rough surface. She applies a force of
100 N over a distance of 10 m. Thereafter she gets pro-
gressively tired and her applied force reduces linearly
with distance to 50 N. The total distance by which the
trunk has been moved is 20 m. Plot the force applied by
the woman and the frictional force, which is 50 N. Cal-
culate the works done by the two forces over 20 m.
[NCERT Solved Example]
Solution. Work done by woman, WF = Area of rectan-
gle ABCD + Area of trapezium CDEI

Fig. 6.88

B F C  
100


50 E
Force (in N)

A D I
0
10 m 20 m
Distance
 
–50 
G Frictional force H
f
Fig. 6.86

Fig. 6.89
2


 

Fig. 6.87
WORK, ENERGY AND POWER 6.75

P. 4. A bob of mass m is suspended by a light string 


of length L. It is imparted a horizontal velocity v0 at the
lowest point A such that it completes a semi-circular 
trajectory in the vertical plane with the string becoming
slack only on reaching the topmost point C. This is
shown in Fig. 6.90. Obtain an expression for (i) v 0 ;
(ii) the speeds at points B and C ; (iii) the ratio of
the kinetic energies (KB /KC) at B and C. Comment on the
nature of the trajectory of the bob after it reaches the
point C. [NCERT Solved Example]
C


TC

L TA
 
0
A

Fig. 6.90

Solution. (i) Following two external forces act on the


bob :
P. 5. A force F = 30 N acts parallel to the inclined
plane as it accelerates a block of mass m = 2 kg up the
30° incline, with a coefficient of kinetic friction k = 0.3
[Fig. 6.91].

Fig. 6.91

Solution. Total work done = Increase in kinetic energy +


increase in gravitational potential energy + increase in potential
energy of spring + work done against friction.
6.76 PHYSICS (Vol. I)—XI

    

      
æ ö
ç ÷
è ø
P. 6. A 1 kg mass on a floor 
is connected to a 2 kg mass by a
string passing over a pulley as æ ö
ç ÷ 
shown [Fig. 6.92]. Obtain the speed è ø
of the masses (after they are 
released) when 2 kg mass just
touches the floor. Establish that æ ö
  ç ÷
the gain in kinetic energy of the è ø
system equals the loss in its
potential energy. The 2 kg mass is
initially at a height of 3 m above
the ground. (Take g = 9.8 m s–2) P. 8. A block of mass m, initially at rest, is dropped
Solution. For 2 kg mass, Fig. 6.92 from a height h onto a spring whose force constant is k.
Find the maximum distance x through which the spring
2  9.8 – T = 2a
will be compressed.
 
Solution. In this case, the principle of conservation of
mechanical energy holds. At the moment of release, the kinetic
energy is zero [Fig. 6.95(a)]. At the moment when maximum
compression occurs, there is again no kinetic energy [Fig.6.95(b)].
 

 h
h+

 
 

P. 7. A simple pendulum is l
suspended from a peg on a
vertical wall. The pendulum is
pulled away from the wall to a Fig. 6.95
horizontal position [see Fig. 6.93]
and released. The ball hits the
wall, the coefficient of restitution
being 2/ . What is the minimum Fig. 6.93
number of collisions after which
the amplitude of oscillation
becomes less than 60° ?

Solution. vn = en ; m = mgh
 
  
A  
O
P. 9. A perfectly elastic oblique collision takes place
n
between a moving particle and a stationary particle of
equal masses. Show that after the collision, the two
C particles move at right angles to each other.
h Solution. [Refer to Fig. 6.58]
B
m1 1i = m1 1f cos 1 + m2 2f cos 2 ...(1)
Fig. 6.94
WORK, ENERGY AND POWER 6.77
 

z   





 
zL 

OP
 MM 
PQ  
N
 
P. 11. A body of mass m
  was slowly hauled up the hill
by a force F which at each point
was directed along a tangent
 
of the trajectory. Find the work
  performed by this force, if the
height of the hill is h, the length
of its base l and the coefficient
    of friction k.

Fig. 6.96
Solution. Since is given
    
    to be parallel to .

 +  + 
 + 
  dx

 

  

 
   
  

P. 10. A cylindrical cork of length l and radius r is


slowly extracted from the neck of a bottle. If the normal Fig. 6.97
pressure per unit of area between the bottle and unextracted

z
part of the cork at any instant be constant and equal to P,
prove that the work done in extracting it is r Pl2, where
 is coefficient of friction.
Solution. Suppose, at any instant, the length x of the
cork has been extracted.
z 

P. 12. A 105 kg engine is moving up a slope of


 gradient 5° at a speed of 100 m h–1. The coefficient of
friction between the engine and the rails is 0.1. If the engine
 has an efficiency of 4% for converting heat into work, find
  the amount of coal the engine has to burn up in one hour.
Given : 1 kg of coal, on burning, yields 5  104 J.
Solution. Total force applied by the engine
  +
6.78 PHYSICS (Vol. I)—XI

+ 
+  

+  

Fig. 6.99


 
Fig. 6.98
= +
+

 +  
 
   
 
P. 14. An alpha particle suffers one-dimensional

elastic collision with a nucleus of unknown mass. After
  the collision, the alpha particle is scattered directly
backwards and loses 75 per cent of its energy. Calculate
 the mass of the nucleus in terms of the mass of th e
-particle.
 FG IJ
 Solution.  =
H K or 

P. 13. A bullet of mass m moving with a horizontal


velocity strikes a stationary block of mass M suspended FG IJ
by a string of length L. The bullet gets embedded in the H K
block. What is the maximum angle made by the string after
impact ?
Solution. Let V be the velocity of the block after the
bullet gets embedded into it. Applying law of conservation of 
momentum, we get

NCERT EXERCISES (With Solutions)


NCERT EXERCISES (UNIT IV) 6.79

V( )
V4 V( )
V3 V0

V2
E
V1
V0
a b
E –b –a 2 2
2 2 –V1
O a b c d X

Fig. 6.100

  
  

  



 

V( )

V( )
V( )
V0

V0
E
E b
O a X –2 m –1 m = 0 +1 m +2 m
O a X –V 1
Fig. 6.101
6.80 PHYSICS (Vol. I)—XI

  

Fig. 6.102

F I
E GH JK
NCERT EXERCISES (UNIT IV) 6.81

 

 

   


 

   


  

  
  



     

   

  

   

   

30° 30°
 

     

  Fig. 6.103
6.82 PHYSICS (Vol. I)—XI

30°

A
 m
 m B

Fig. 6.105

  

   

FG IJ
1 2 3

v
H    
K O
1.5 m A

1 2 3  

v (i )
v=0 2  

   B
Fig. 6.106
1 2 3

v (ii )
v=0

1 2 3

v (iii )

Fig. 6.104
NCERT EXERCISES (UNIT IV) 6.83


  

 
 

 

 

   

     

1 2
  B C
Fig. 6.107

 




6.84 PHYSICS (Vol. I)—XI

 
 



 
–1
K = 100 N m

1
kg
37°

Fig. 6.108

  

  
f
R


mg cos 

   mg sin 
mg

  
Fig. 6.109
V(r) V(r)
 V(r)


2R r 2R r 2R r

V(r) V(r) V(r)

2R r 2R r 2R r

 

Fig. 6.110
NCERT EXERCISES (UNIT IV) 6.85


 


per unit energy interval


No. of -particles

Kinetic energy of  
-particles emitted   
 
Fig. 6.111
 
   



NCERT EXEMPLAR PROBLEMS (With Answers)


[Based on Higher Order Thinking Skills].
MCQ I (Only one option is correct.)
6.86 PHYSICS (Vol. I)—XI


V( )

x
– m m

Fig. 6.113

1 2 3

A
V
I Fig. 6.114
II
h

1 2 1 1 2 3
B
C

Fig. 6.112
V=0 V/2 V=0 V
(a) (b)

1 2 3 1 2 3

V/3 V/1 V/2 V/3

(c) (d)

Fig. 6.115
NCERT EXEMPLAR PROBLEMS (UNIT IV) 6.87

K.E. K.E.

t t
(a) (b)

K.E. K.E.

t
 
t
(c) (d)

Fig. 6.117

ò 
ò
E
E

 t
t
(a) (b)

E E

d d

t t

t t (c) (d)
(a) (b)
Fig. 6.118
d d

t t
(c) (d)
 
Fig. 6.116

  

 
       


6.88 PHYSICS (Vol. I)—XI

h h PE
PE K.E. K.E.

h/4
KE
KE depth depth
t (a) (b)
t
(a) (b)

h
PE
K.E. K.E.
h KE

PE
KE
depth depth
t t (c) (d)
(c) (d)

Fig. 6.120
Fig. 6.119

   

  

  

NCERT EXEMPLAR PROBLEMS (UNIT IV) 6.89

MCQ II (Any number of options may be


correct.)

VSA

æ ö ¢
¢ 
èç ø÷

M1 = m M2 = m

Fig. 6.121
6.90 PHYSICS (Vol. I)—XI

SA

V( )
A F

Eo
B

C D

  Fig. 6.123
 


 

K.E.
C D
Eo

B
Eo

X
A F

Fig. 6.124

C
m
Velocity C D

X B

B A x
F
B
L
C D

A Fig. 6.125
Fig. 6.122
NCERT EXEMPLAR PROBLEMS (UNIT IV) 6.91

Ans. (a) Applying conservation of momentum, 37. The bob A of a pendulum released from horizontal to the
m(2v0) = mv1 + mv2 vertical hits another bob B of the same mass at rest on a
table as shown in Fig. 6.127.
or 2v0 = v1 + v2 ...(i)
If the length of the pendulum is 1m, calculate
v - v1
By definition, e = 2 (a) the height to which bob A will rise after collision.
2v0
(b) the speed with which bob B starts moving.
or v2 = v1 + 2v0e ...(ii) Neglect the size of the bobs and assume the collision
From (i), 2v0 = v1 + (v1 + 2v0e) = 2(v1 + v0e) to be elastic.
or v1 = v0(1 – e) A
m
e < 1. v1 has same sign as v0.
Thus, the ball moves forward after collision.
1m
From (ii), v2 = v0(1 – e) + 2v0e = v0(1 + e)
e < 1. v2 has the same sign as v0. So, the other ball also B m
moves forward after collision.

p1 Fig. 6.127
m
Ans. (a) The collision is elastic and two balls have same
θ
mass. So, ball A transfers its entire momentum to the
(b)
ball on the table and does not rise at all.
m 2V0 m
(b) Speed with which B starts moving is equal to the
Before collision m →
p2 speed with which A hits B.
After collision
2 gh = 2 ¥ 9.8 ¥ 1 = 4.42 m s
–1
v=
Fig. 6.126
38. A raindrop of mass 1.00 g falling from a height of 1 km
Applying principle of conservation of momentum,
hits the ground with a speed of 50 m s–1. Calculate
Æ Æ Æ
p = p1 + p 2 (a) the loss of P.E. of the drop.
(b) the gain in K.E. of the drop.
In a general collision, some K.E. is lost.
(c) Is the gain in K.E. equal to loss of P.E.? If not why?
∴ p2 Ê p12 ˆ Ê p22 ˆ Take g = 10 m s–2.
>Á ˜ +Á ˜
2m Ë 2m ¯ Ë 2m ¯
Ans. (a) Loss of P.E. = mgh = 1 × 10–3 × 10 × 103 = 10 J
or p2 > p12 + p22 (b) Gain in K.E. = 1 mv2 = 1 × 10–3 × 2500 = 1.25 J
This is possible only when θ < 90°. 2 2
36. Consider a one-dimensional motion of a particle with (c) No, because a part of P.E. is used up in doing work
total energy E. There are four regions A, B, C and D in against the viscous drag of air.
which the relation between potential energy V, kinetic 39. Two pendulums with identical bobs and lengths are
energy K and total energy E is as given below: suspended from a common support such that in rest
Region A : V > E position the two bobs are in contact (Fig. 6.128). One of
Region B : V < E the bobs is released after being displaced by 10° so that it
collides elastically head-on with the other bob.
Region C : K > E
Region D : V > K
State with reason in each case whether a particle can be
found in the given region or not.
Ans. Region A : No, as K.E. will become negative.
Region B : Yes, total energy can be greater than P.E. for
non-zero K.E.
Region C : Yes, K.E. can be greater than total energy if A B
P.E. is negative.
Region D : Yes, as P.E. can be greater than K.E. Fig. 6.128
6.92 PHYSICS (Vol. I)—XI

 
 

E1 E2
E0 E0

t t
T/4 3T/4 5T/4 7T/4 T/4 3T/4 5T/4 7T/4
Fig. 6.129 Fig. 6.130

LA


F
m

30°

Fig. 6.131

 
NCERT EXEMPLAR PROBLEMS (UNIT IV) 6.93

   


 

     

  
    
æ ö
ç       ÷ø
è

   
   
   

     

A C

D
B 

Fig. 6.132 

  


   

 

 
 
 


 
 
 
 
  
 

6.94
WORK, ENERGY AND POWER (UNIT IV) 6.95

   

  
6.96 PHYSICS (Vol. I)—XI
WORK, ENERGY AND POWER (UNIT IV) 6.97

in (m)

 
 
 
6.98 PHYSICS (Vol. I)—XI

 

 

 
 
WORK, ENERGY AND POWER (UNIT IV) 6.99

     
 
    
 
      
 

      
 

   

   

   
6.100 PHYSICS (Vol. I)—XI

 

 
 
 
 

 
  
 
WORK, ENERGY AND POWER (UNIT IV) 6.101

  
6.102 PHYSICS (Vol. I)—XI

 

2N
WORK, ENERGY AND POWER (UNIT IV) 6.103

   

   

 



6.104 PHYSICS (Vol. I)—XI

   
   
    

    
    
    
WORK, ENERGY AND POWER (UNIT IV) 6.105

  
 
 

 
  
 

  
     
     
     
  
     
     
     
  
     
     
     
  
     
     
     
6.106 PHYSICS (Vol. I)—XI


WORK, ENERGY AND POWER (UNIT IV) 6.107


é ù
ê  ú
ë   û

p p

p

 

   

 p p


 p p


y
R P
30°

R
Q

O x
6.108 PHYSICS (Vol. I)—XI

  
   
   
 

 

 
WORK, ENERGY AND POWER (UNIT IV) 6.109
141. A particle is acted upon by a constant power. Then, which 145. When a rubber-band is stretched by a distance x, it exerts
of the following physical quantity remains constant ? a restoring force of magnitude F = ax + bx2 where a and b
(a) speed are constants. The work done in stretching the unstretched
(b) rate of change of acceleration rubber-band by L is :
(c) kinetic energy Ê  ˆ
(d) rate of change of kinetic energy [WBJEE 2013] (a)  Á C. + D. ˜ (b) aL2 + bL3
Ë   ¯
142. A body of mass (4m) is lying in x-y plane at rest. It suddenly
explodes into three pieces. Two pieces, each of mass (m)
 C. D.
move perpendicular to each other with equal speeds v. The (c)
C. + D. (d) +
total kinetic energy generated due to explosion is   

 [JEE Main 2014]


(a) mv2 (b) OX
 146. In elastic collision
(c) 2mv2 (d) 4mv2 [AIPMT 2014] (a) both momentum and kinetic energy are conserved.
143. In an elastic collision, a neutron collides with carbon. (b) neither momentum nor kinetic energy is conserved.
How much energy (in percentage) of neutron is transferred
(c) only momentum is conserved.
to carbon ?
(a) 90% (b) 45% (d) only kinetic energy is conserved.
(c) 28% (d) 12% (e) forces involved in the interaction are
[AMU (Med.) 2013] non-conservative.
144. A tennis ball is dropped on a horizontal smooth surface. It [Kerala PET 2014]
bounces back to its original position after hitting the 147. If two persons A and B take 2 second and 4 second
surface. The force on the ball during the collision is respectively to lift an object to the same height h, then the
proportional to the length of compression of the ball. Which
ratio of their powers is
one of the following sketches describes the variation of its
kinetic energy K with time t most appropriately ? The (a) 1 : 2 (b) 1 : 1
figures are only illustrative and not to the scale. (c) 2 : 1 (d) 1 : 3
(e) 3 : 1 [Kerala PMT 2014]

(a) K 148. If a machine gun fires n bullets per second each with kinetic
energy K, then the power of the machine gun is
t -
(a) nK2 (b)
P

(b) K (c) n2K (d) nK

P
t (e) [Kerala PMT 2014]
-
149. A stationary object explodes into masses m1 and m2. They
move in opposite directions with velocities V1 and V2. The
(c) K
ratio of kinetic energy E1 to kinetic energy E2 is
t O
O
(a) (b)
O O

(d) K
O O
(c) (d)
O O
t
[JEE Advanced 2014] [Maharashtra CET 2014]

KEY

141. (d) 142. (b) 143. (c) 144. (b) 145. (d) 146. (a) 147. (c)
148. (d) 149. (a)
6.110 PHYSICS (Vol. I)—XI

 

 

Force of slope
Force
on barrel

Barrel
Slope
1.0 × 103 N
30°
WORK, ENERGY AND POWER (UNIT IV) 6.111


é ù
ê    ú
ë û


  

    


     

  

    
 

é   ù
ê    ú  
êë    úû  
  
   é    ù
   ê    ú
   êë    úû

 
   
  





é ù  
ê ú
ë û
6.112 PHYSICS (Vol. I)—XI

 
 

é  ù
ê ú
ë û

  
 
 

 

 

 


 
 

é  ù
ê   ú
ë  û é  ù
ê ú
é  ù ë  û
ê  ú
ë  û

 
WORK, ENERGY AND POWER (UNIT IV) 6.113



  
 
 
  

     
 

6.114 PHYSICS (Vol. I)—XI


  


 
 
 

 
   
 


 


  

  

  

 


   
  

 
WORK, ENERGY AND POWER (UNIT IV) 6.115

 

 
  
 

 

 

  

 
   
  

   
   
   

  
    
 

 
6.116 PHYSICS (Vol. I)—XI





 
 

 
   
 

  

   

     

 
 

 

 

 

 é     ù
ê   ú
ë    û

é   ù
 êp   ú
p ëê   ûú
WORK, ENERGY AND POWER (UNIT IV) 6.117


 

  
=    
 
 
 
 

 
 ×
 
  


 
é ù 
 ê



ú
 ëê ûú
  



 


  ï     ï
   
    
  ï     ï

   
  
   

    

   R
30°
f 60°
é ù é ù
N
êë  úû  ê  ú Q R
   ë û
mg cos

60°


60°

mg mg sin 60°

B
(0, a)
 
a 0 cos 
0 H
A 0
O (a , 0) 
m m

 é ù    
ê  ú
ë   û 
6.118 Comprehensive PHYSICS (Vol. I)—XI

i n t s a nd Tips
H

132. Let v be the speed of the block Æ For carbon, m2 = 12 m1


at Q. 138. T =
Kł +  łL - Mł -
Kł + Mł
∴ f= O
O
Applying work-energy theorem,
= Kł +  łL - Mł
O + O 
4 
OI -  = OX Æ Æ = 0.284 = 28.4%
  W = (  T =
Kł + łL ◊
Kł +  łL - Mł
  144. K =  OX =  O
I V
 ¥  ¥ -  = ¥  ¥ X =6+3=9J  
 
140. VA + KA = VE + KE K∝t 2
v = 10 m s–1
Relation between K and t is
133. Maximum energy loss VA = VE and KA = 0
parabolic.
 O/  ∴ KE = 0
During collision, velocity of the ball
= X
/+O F8 falls sharply to zero as it is
F= =0 compressed and regains maximum
OW FZ
134. v=– velocity in short time interval.
/ Slope is zero at points B, C and
. .
  D. 
K = OW  - ¢ = /X
 
145. W= Ú (FZ = Ú
CZ + DZ FZ
Æ  
142. Let X ¢ be the velocity of third piece

- ¢ / X / Ê O ˆ O of mass 2m. È Z Z ˘
.
= = Á ˜ = = ÍC +D ˙
- O W O Ë / ¯ /
Applying conservation of Î   ˚
As m < < M, ∴ K′ < K momentum,
 
Æ = C. + D.
135. a =  -  =  OXKł + OXLł + O X ¢ =   
O O
Æ Xł Xł OIJ OIJ
Now, v2 = u2 + 2aS X¢=- K- L 147. 2# =  2$ =
  V# V$
v= C5 (∵ u = 0)
2# V$  
Ê Xˆ

Ê Xˆ
 = = =
  X¢ = Á - ˜ + Á - ˜ 2$ V#  
K = OX = O
C5 Ë ¯ Ë ¯
 
149. m2V2 + m1V1 = 0
 X
= O ¥  =  , = O
O or V2 = – 8
 O
136. At equilibrium position,
Total kinetic energy generated due
 
'  O8
to explosion 
F  O ÊO ˆ

Z -  Z =  = =  Á ˜
FZ '  O 8  O Ë O ¯
=  OX +  OX + 
O X¢   

or 2x – 3 = 0 or x = 1.5 m   
FO O
137. m= =  OX +  OX +  OX =
FZ O
  
F F Ê ˆ F9

- = Á OX ˜¯  = M F9 = MFV
FV FV Ë  = OX 150.
FV

=  FO X W = kt
 FV 143. Fractional kinetic energy loss by
W =  OX -  O
 
neutron,
 FO FZ 
= X  
 FZ FV  O O
f=
=  OX
O + O  
kt = OX
 
WORK, ENERGY AND POWER (UNIT IV) 6.119


 ×

  


 

 

  

   

 
   
 





 


6.120 PHYSICS (Vol. I)—XI
UNIT—V
MOTION OF SYSTEM OF
PARTICLES AND RIGID BODY
18 Periods

Chapter–7: System of Particles and Rotational Motion

SOLVED NUMERICALS 40 PLUS

UNSOLVED NUMERICALS 52 PLUS

VERY SHORT, SHORT & LONG ANSWER QUESTIONS 139 PLUS

QUESTIONS & PROBLEMS BASED ON HOTS 53 PLUS

NCERT EXERCISES, EXAMPLES & EXEMPLAR PROBLEMS 56 PLUS

MULTIPLE CHOICE QUESTIONS 140 PLUS


Satyendranath Bose (1894–1974)
A

P2
P1


B C

Fig. 7(a).1. Translational (sliding)


motion of a block down an inclined
plane.
(Any point like P 1 or P2 of the block
moves with the same velocity at any
instant of time.)

7.3
7.4 PHYSICS (Vol. I)—XI

r2
C2 P2

P1
(a) C1 r1

P3

Fig. 7(a).3. A rigid body rotation about


(b) the z-axis (Each point of the body such
Fig. 7(a).2. Rotation about a fixed axis as P1 or P2 describes a circle with its
centre (C1 or C2) on the axis. The
(a) A ceiling fan
radius of the circle (r1 or r2) is the
(b) A potter’s wheel perpendicular distance of the point (P1
or P2) from the axis. A point on the axis
like P3 remains stationary.)

Fig. 7(a).4. (a) A spinning top (The


point of contact of the top with the
ground, its tip O, is fixed)

Fig. 7(a).4. (b) An oscillating table fan.


The pivot of the fan, point O, is fixed
CENTRE OF MASS 7.5

A
P4
P2 P
1

P3


B C

Fig. 7(a).5. Rolling motion of a cylinder.


It is not pure translational motion.
Points P1, P2, P3 and P4 have different
velocities (shown by arrows) at any
instant of time. In-fact, the velocity of
the point of contact P 3 is zero at any
instant, if the cylinder rolls without
slipping

Y
m1
Centre
of mass
)
r1 (t

 (t)
R
 (t) m2
r2

O X

  Z

Fig. 7(a).6. Centre of mass of a two-
particle system
7.6 PHYSICS (Vol. I)—XI

  

  

 

 

   

   

     

 

   

  

  

 

 
 
CENTRE OF MASS 7.7

 
 

 

 

  

F   I

GG  JJ
H K
 


  
 



   
  
=

 


 
-
7.8 PHYSICS (Vol. I)—XI

 

  
   
  



=


=


=



=

       

     
 
=

  
= = =

  
  

  
  

  
  
CENTRE OF MASS 7.9



   

     
  

 
  
  
  

  

 

 

 

  
7.10 PHYSICS (Vol. I)—XI

dm O dm
X-axis

Fig. 7(a).7. Determining the CM of a


thin rod

   
 

     
       
  

 LM 


 
 OP
N Q
   
CENTRE OF MASS 7.11



V

  
  

  

   

 

=

   

   
V

 

     

  

   

   


7.12 PHYSICS (Vol. I)—XI

 

Centre of mass
=

m1 m2

r1 r2   

 
r
Fig. 7(a).8
 


CENTRE OF MASS 7.13

  

      

   

 

 

 





7.14 PHYSICS (Vol. I)—XI

He He

CM
Ra
Ra Rn
Rn

(a) (b)
Fig. 7(a).9. (a) A heavy nucleus (Ra) splits into a
lighter nucleus (Rn) and an alpha particle (He).
The CM of the system is in uniform motion
(b) The same splitting of the heavy nucleus (Ra)
with the centre of mass at rest. The two product
particles fly back to back

Parabolic path
of the projectile Explosion

Path of the CM
of fragments

O x1 X

Fig. 7(a).10. The centre of mass of the


fragments of the projectile continues along
the same parabolic path which it would have
followed if there were no explosion

M
Centre
of mass

E
Sun

Fig. 7(a).11. Centre of mass of Earth-


Moon system
CENTRE OF MASS 7.15
Y

Centre
of mass

Fig. 7(a).13. Exploding missile

O X

Fig. 7(a).12. Tossing of axe

S1
S1
C
C
S2
S2

(a) (b)
Fig. 7(a).14. (a) Trajectories of two stars, S1 (dotted line)
and S2 (solid line) forming a binary system with their
centre of mass C in uniform motion
(b) The same binary system, with the centre of mass C
at rest
7.16 PHYSICS (Vol. I)—XI

Fig. 7(a).15. Centre of mass of a


triangular body

3h/4
h

Fig. 7(a).16. Centre of mass of a right


circular solid cone

     
i j k i j k

L
  

      
R Q
G
   

    M P N
Fig. 7(a).17
CENTRE OF MASS 7.17
  

  
y(m)

  
8.0 kg

2

4.0 kg
1 
 
3.0 kg 
x(m) 
0 1 2 3

Fig. 7(a).18

    
Y
 

     l

  (3l/2, 2l)

( l/2, 2 l) 2l (0, l) (2l, l ) 2l

2l (0, l)
( l, 0)
(l /2, 0)
O X
O 2l X
l

Fig. 7(a).19 Fig. 7(a).21

      
       
 
 

    
        

FG IJ






H K
A B
D

P
 2
F
a a

C
 m2 Fig. 7(a).22
m1

Fig. 7(a).20
  
7.18 PHYSICS (Vol. I)—XI

  

 
 

A B
A 
 + r C
rA

Y rA

2m
F(0, 2) E(1, 2) B C
B  C
rC
C3

D(1, 1) B(2, 1) Fig. 7(a).24 Fig. 7(a).25

 
1m  
C1 C2

O(0, 0) A(2, 0) X   
 

Fig. 7(a).23

 
 

 
 

Fig. 7(a).26
CENTRE OF MASS 7.19

   
 

VERY SHORT ANSWER QUESTIONS—Each Question Carries 1 Mark Only

1 C.M. 2

1 2

Fig. 7(a).27

 

 

SHORT ANSWER QUESTIONS—Each Question Carries 2 Marks

5
6
4

3 1
2

Fig. 7(a).28
7.20 PHYSICS (Vol. I)—XI

1 2
II I
II
O X

III IV
IV
4 3

Fig. 7(a).29 Fig. 7(a).30

Y Y

X X

(a) (b)
Y Y

X X

(c) (d)
Fig. 7(a).32

B A C
O

Fig. 7(a).31
CENTRE OF MASS 7.21

LONG ANSWER QUESTIONS

  



A B
  

E
O

LM OP D F C

 7 3  MN PQ Fig. 7(a).33
  ,  
  12 4  


LM OP

N Q
7.22 PHYSICS (Vol. I)—XI

(0, 0)

40 kg 120 kg 60 kg
(a)
8m
O

2m
(0, 0) 60 kg 120 kg 40 kg
(b)
3m 3m
Fig. 7(a).34

6m

Y
2m
2m

Fig. 7(a).36
1m 1m

O
m 1m m X
Fig. 7(a).35


   
 1
CM 2
 
 1 2

    
 
 1 2

    2 – 
1 –
   

FG  IJ  FG IJ
   H K H K   
 

     
 
CENTRE OF MASS 7.23

Y
   



X
  
 
2
     mi xi
 i 1
2

 mi
i 1

 


 
  

P. 1. From a disc of mass m and radius r, another P. 2. If four uniform and identical metre sticks were
disc of radius r/3 is cut as shown in Fig. 7(a).37. Calculate stacked on a table with 10 cm, 15 cm, 30 cm and 50 cm
shift in the centre of mass i.e., OO2 . segments over the edge as shown in Fig. 7(a).38, would the
[AIPMT Mains 2006] metre sticks remain on the table or topple down ?
Solution. Mass of small disc
C4
2 50 cm
m r m C3 70 cm
  C2 85 cm
r 2 3 9 C1 90 cm
O2 O O1 O

m 8m
9 9
Fig. 7(a).37

Fig. 7(a).38

[Physics Olympiad]
Solution. For the metre sticks to remain on the table, it
8m m  2r  r
  is necessary that the centre of mass of the system is not beyond
9 9  3  12
the edge O of the table.
r
Since the metre sticks are identical therefore the centre
12
of mass of each stick is located at its midpoint.
7.24 PHYSICS (Vol. I)—XI

Let the edge O of the table be taken as the origin. Start- P. 4. Determine the centre of mass of a homogene-
ous, semi-circular plate of radius r. [Fig. 7(a).40]
ing from the lowermost stick, the distances of the centre of mass
of the sticks are given as [Ans. r from the centre]
x1 = – 40 cm, x2 = – 35 cm, x3 = – 20 cm and x4 = 0
If X be the distance of the centre of mass of the system r
from O and m is the mass of each stick, then

  
X=
  
d
Fig. 7(a).40
   
= cm = – 23.75 cm
Hint. dm =  x dx = x dx

Clearly, the centre of mass of the system is not beyond

z LM OP
the edge O of the table. So, the metre sticks will remain on the
table.
MN PQ
P. 3. A thin uniform flat plate of mass 4.5 kg is in
the form of a L-shaped lamina. Its dimensions are as shown P. 5. A lone canoeist sitting in the stern of a canoe
in Fig. 7(a).39. Find the position of its centre of mass. unknowingly drops the only paddle in the water. By the
time the canoeist notices the problem, the paddle is 0.50 m
Y
E in front of the bow. The canoeist walks to the bow to retrieve
F (0, 4 m)
the paddle. After reaching the bow, how far is the canoeist
C3
D B from the paddle ? Assume the canoeist has a mass of 50.0 kg,
the canoe a mass of 30.0 kg, that the canoe is 4.00 m long,
C1 C2 and that the canoeist starts from the stern 1.50 m from the
O A (4 m, 0) X centre of the canoe.
Solution. We take a co-ordinate system fixed relative
Fig. 7(a).39
to the water with x = 0.00 cm at the position of the paddle as
shown in figure 7(a).41(a). We assume that the water exerts no
Solution. The given plate can be regarded as a combi- horizontal force during the movement of the canoeist. Because
nation of three square plates of equal masses. no unbalanced external force acts, the centre of mass is at the
same location before and after the move. The position of the
Mass of each square plate, m = = 1.5 kg centre of mass before the canoeist moves is calculated as usual.
The co-ordinates of C1, C2 and C3 are (1, 1), (3, 1) and mA xA  mB xB
(1, 3) respectively. If (X, Y) are the co-ordinates of the centre of
mA  mB
mass of the L-shaped lamina, then

      
X= = m
   

= m= m

      
Y= = m
   
 

= m= m.

kg
CENTRE OF MASS 7.25

0.5 m 1.5 m
 

x
 
CM

4.0 m
P. 7. Fig. 7(a).43 shows a simple model of the
structure of a water molecule. The separation between
atoms is d = 9.57 × 10–11 m. Each hydrogen atom has mass
1.0 u and the oxygen atom has mass 16.0 u. Find the
position of the centre of mass.

x Hydrogen
CM
Oxygen
d
Fig. 7(a).41 cm 105°
X
O d

Hydrogen

Fig. 7(a).43

Solution. Nearly the whole of the mass of each atom is


concentrated in its nucleus, which is only about 10 –5 times the
overall radius of the atom. So, we can safely represent each
P. 6. A circular plate of atom as a point.
uniform thickness has a diameter
of 56 cm. A circular portion of 28cm
r2 r1 21cm
diameter 42 cm is removed from O
C2 C C1
one edge of the plate as shown in
Fig. 7(a).42. Find the position of
the centre of mass of the    
remaining portion.     
Fig. 7(a).42  
Solution.  
2
  2
M =    = (28) ,
 
   
   
        
   

 
 
z


v
P

C r P

O y
 
 
    
 
 Fig. 7(b).1. Rotation about a fixed
axis. (A particle (P) of the rigid body

rotating about the fixed (z) axis
moves in a circle with centre (C) on
the axis)
 


Fig. 7(b).2. (a) If the head of a right handed
screw rotates with the body, the screw
 advances in the direction of the angular
velocity . If the sense (clockwise or
anticlockwise) of rotation of the body
 
 changes, so does the direction of 

7.26
ROTATIONAL MECHANICS AND MOMENT OF INERTIA 7.27

Z
 
  
v

r
C
P

 
 r

O


  Fig. 7(b).2. (b) The angular velocity vector

 is directed along the fixed axis as
shown. The linear velocity of the particle
  
at P is =  × . It is perpendicular to
       both  and r and is directed along the
   tangent to the circle described by the
    particle
 
   
  
  
 

    
  
   
 
 

     

  
     
 
  
 

  
   

 




7.28 PHYSICS (Vol. I)—XI


 

 
 


  

  
  



 


    
  

   
 
 

    
 

  
 
 

 

 

  

ROTATIONAL MECHANICS AND MOMENT OF INERTIA 7.29

  
 
 

    

 

   Fig. 7(b).3


 
 
     
  

 

Fig. 7(b).4
7.30 PHYSICS (Vol. I)—XI

  

  


  

Fig. 7(b).5
  

d
 
r
  
   

 
  
Fig. 7(b).6

  


Fig. 7(b).7. Rotational motion of a single


  particle


 
 

      

 
ROTATIONAL MECHANICS AND MOMENT OF INERTIA 7.31

    

    
 
   

W 

t t


 
   
  
    

   
  

Fig. 7(b).8. Torque. The wiggly line



through F shows that we have


replaced F by its components

      
7.32 PHYSICS (Vol. I)—XI

Fig. 7(b).9. A mobile crane


  
 

r
 
     

 
     
  
    
       
 
        
       
 
    
 

ROTATIONAL MECHANICS AND MOMENT OF INERTIA 7.33

       
r
  
 r 
      
  
  

   
   

     
  
  

  

  100 N
i j
30°

k C

   50 N
i j k
Fig. 7(b).10

       
r
  
i j k
  
 r 1 1 1 100 sin 30°

7 3 5 0.5 m 0.5 m

 C
 
50 N
  
i j k Fig. 7(b).11
7.34 PHYSICS (Vol. I)—XI


 Fig. 7(b).12. The Earth’s magnetic
field exerts equal and opposite
forces on the poles of a compass
needle. These two forces form a
  couple
  Y


2
     
  
    O
  

X
 r2 F
r1

   P 
r
Q 
  1 r2

Z 
r1
   
  –F

  Fig. 7(b).13. Moment of couple


        
      
   

     
+ = – =

  
 = 

  Fig. 7(b).14. Our fingers apply a


couple to turn the lid


r2

 
F F

r1
Fig. 7(b).15
ROTATIONAL MECHANICS AND MOMENT OF INERTIA 7.35

       
r1 F r2 F r2 F r1 F
  
r2 r1 F
     
r1 r2 r2 r1

 
F

Fig. 7(b).16

L

2

L
 
2

 


 
 
7.36 PHYSICS (Vol. I)—XI

  
Fs F

 

P
  
 r

  d
ds 90° – 
O
   
r P


   
Fig. 7(b).17. Work done in rotational
   motion
    
   
 F 
 
  
 
 
   
    

z z 
 



z 
   



  

  
 

Fig. 7(b).18

  


ROTATIONAL MECHANICS AND MOMENT OF INERTIA 7.37

Fig. 7(b).19

 

  


 Fig. 7(b).20

  

d
 
r
 
  

  Fig. 7(b).21

 
7.38 PHYSICS (Vol. I)—XI

 





 



 
  
   
 
Fig. 7(b).22. Angular momentum-radial
  and angular components. The wiggly
  
line through p shows that we have
  
replaced p by its components



ROTATIONAL MECHANICS AND MOMENT OF INERTIA 7.39

 

  

 

     

    

       

  

       

      

   
     
     

     
 
        


  

  

L= 
 




 

Fig. 7(b).23 
7.40 PHYSICS (Vol. I)—XI

 


h     
2π 
   

  



 

  
 
     
 



  Fig. 7(b).24. Geometrical meaning
 of angular momentum
 
 



 
  
 

   
   
      
 
  

 
   

 
  


ROTATIONAL MECHANICS AND MOMENT OF INERTIA 7.41


 


 Fig. 7(b).25. Kepler’s

second law


  

  
   
   

 
  
 
 
      
   

 
  
  
7.42 PHYSICS (Vol. I)—XI

  

    

  

 

   
 
 

´

 

   

 


  

 

 
    
 

 ´   

ROTATIONAL MECHANICS AND MOMENT OF INERTIA 7.43

 
   
  
  

   
  


  
   


 



7.44 PHYSICS (Vol. I)—XI

 


 

       
  



 


 

 


  

        
  


ROTATIONAL MECHANICS AND MOMENT OF INERTIA 7.45

Fig. 7(b).26(a)

 

Fig. 7(b).26(b)

Fig. 7(b).27
7.46 PHYSICS (Vol. I)—XI


Fig. 7(b).28



 
Fig. 7(b).29. Balancing a cardboard on
the tip of a pencil. The point of support
G is the centre of gravity

 


ROTATIONAL MECHANICS AND MOMENT OF INERTIA 7.47

  

   
  

 

Fig. 7(b).30. Stable equilibrium

Fig. 7(b).31. Unstable equilibrium


7.48 PHYSICS (Vol. I)—XI

Fig. 7(b).32. Neutral equilibrium

Fig. 7(b).33

 
Fig. 7(b).34. Its wide wheelbase and low centre of
gravity make a racing car very stable
ROTATIONAL MECHANICS AND MOMENT OF INERTIA 7.49

 



Fig. 7(b).35. Swinging pendulum
 


 
  

 
   

 LM    OP
N Q

 


 

  

  
7.50 PHYSICS (Vol. I)—XI

  
 
  


  

z z LM OP
z LM OP


   
 N Q  NQ

     

  


  

z z

 

z z 
   z z   
z  z
LM  OP 
é
 ê
ù
ú
é
 ê
ù
ú   
CHECKPOINT

N Q ë û ëê ûú


   


  

  

 

     

z z L O
   
MP
  é ù

    ê ú
ëê úû  N Q
 
     
ROTATIONAL MECHANICS AND MOMENT OF INERTIA 7.51

   

   

  

   

   


  

  

 

   

  


  


 


 

300 
 
60
 

    

 
  
  
 
7.52 Comprehensive PHYSICS (Vol. I)—XI
Example 13. You have just finished watching a  2
movie on DVD and the disc is slowing to a stop. The (b) θ = ω0t + αt

angular velocity of the disc at t = 0 is 27.5 rad s–1 and its
angular retardation 10.0 rad s–2 is a constant. A line PQ on 1
= ÊÁ 27.5 ¥ 0.3 - ¥ 10 ¥ 0.3 ¥ 0.3ˆ˜ rad
the surface of the disc Ë 2 ¯
lies along the + x-axis at
t = 0. (a) What is the Ê 1 rev ˆ
= 7.8 rad = 7.8 rad Á = 1.24 rev.
disc’s angular velocity Ë 2p rad ˜¯
at t = 0.3 s ? (b) What
angle does the line PQ The DVD has turned through one complete revolution
make with the + x-axis plus an additional 0.24 rev—that is, through an additional angle
at this time ? (c) At of
what time t does the Ê 360 ∞ ˆ
(0.24 rev) Á = 86.4°.
DVD come to rest ? (d) Ë rev ˜¯
Through how many So, the line PQ is at an angle of 86.4° with the
revolutions does it + x-axis.
rotate between t = 0 and Fig. 7(b).36 ω − ω  − 
(c) t = = = 2.75 s
coming to rest ? α − 
Solution. (a) ω = ω0 + αt = (27.5 – 10 × 0.3) rad s–1  
(d) θ = (ω + ω0)t = (0 + 27.5) 2.75
= 24.5 rad s–1  
Ê 1 rev ˆ
= 37.8 rad = 37.8 rad Á = 6.01 rev.
Ë 2p rad ˜¯

7(b).22. MOMENT OF INERTIA

(i) Moment of inertia of a rigid body about a fixed axis is defined as the
sum of the products of the masses of all the particles constituting the body and
the squares of their respective distances from the axis of rotation. It is a scalar
quantity.
Let YY′ be the axis about which the rigid body is rotating [Fig. 7(b).37].
Let the body be composed of n particles of masses m1, m2, ......, mn. Let r1, r2,
......, rn be their respective distances from the axis of rotation. The moment of
inertia of the rigid body about the given axis YY′ is given by
n
I = m1r12 + m2r22 + ...... + mnrn2 = Σ mi ri2
i=1

(ii) In cgs system, the unit of moment of inertia is g cm2. In SI, moment Fig. 7(b).37. Moment of inertia
of inertia is measured in kg m2. of a rigid body
(iii) Moment of inertia depends on the following factors:
1. Mass of the body.
2. Position of the axis of rotation.
3. Distribution of mass about the axis of rotation.

7(b).23. PHYSICAL SIGNIFICANCE OF MOMENT OF INERTIA

It follows from Newton’s first law of motion that a body cannot change its
state of rest or of uniform motion unless some external force acts on it. This
property of the body is known as inertia. The greater the mass of a body, greater
ROTATIONAL MECHANICS AND MOMENT OF INERTIA 7.53
is the force required to produce a given (linear) acceleration in it. So, in linear
motion, the mass of a body is a measure of its inertia.
In rotational motion, a torque is required to produce angular acceleration
in a body. In other words, a body cannot change by itself its state of rotational For a continuous distribution of mass,
motion. This inability or property of the body is called rotational inertia or
I= ∫ T  FO
moment of inertia. It plays the same role in rotational motion as is played by
mass in linear motion. where r is the distance of mass
Larger the value of moment of inertia of a body, more is the rotational element dm from the axis.
inertia of the body. This property has been put to a great practical use. The
machines such as steam engine and the automobile engine, etc. that produce
rotational motion have a disc with a large moment of inertia, called a flywheel.
Because of its large moment of inertia, the flywheel resists the sudden increase Add to Your Knowledge
or decrease of the speed of the vehicle. It allows a gradual change in the speed
Moment of inertia is NOT a scalar
and prevents jerky motions. This ensures a smooth ride for the passengers on
because it has different values in
the vehicle. different directions. Moment of inertia
is also NOT a vector. This is because its
value about a given axis remains the
7(b).24. RADIUS OF GYRATION same whether the direction of rotation
is clockwise or anticlockwise. In-fact,
It is defined as the distance from the axis of rotation at which if whole moment of inertia is a tensor.
mass of the body were supposed to be concentrated, the moment of inertia would
be the same as with the actual distribution of mass. It is denoted by K.
The moment of inertia of a body of mass M and radius of gyration K is
given by
I = MK2 ...(1)
Also, I = m1r12 + m2r22 + ...... + mnrn2
If m1 = m2 = ...... = mn = m (say), then
I = m(r12 + r22 + ...... + rn2)
Ê r 2 + r22 + ... + rn2 ˆ
or I = mn Á 1 ˜
Ë n ¯
But mn = M (mass of body)

Ê r 2 + r22 + ... + rn2 ˆ


∴ I=M Á 1 ˜ ...(2)
Ë n ¯

Comparing equations (1) and (2), we get

Ê r 2 + r22 + ... + rn2 ˆ 2 2 2


r1 + r2 + ... + rn
MK2 = M Á 1 ˜ or K2 =
Ë n ¯ n

r12 + r22 + ... + rn2


or K = = root mean square distance of the particles from
n
the axis of rotation.
The radius of gyration of a body about an axis is equal to the root mean
square distance, of the various particles constituting the body, from the axis of
rotation.
The radius of gyration is measured in metre or cm. Its dimensional formula
is [M0LT0].
The radius of gyration depends upon (i) shape and size of the body (ii)
position of the axis of rotation and (iii) distribution of mass about the axis. Fig. 7(b).38. Axis of rotation
7.54 PHYSICS (Vol. I)—XI

Fig. 7(b).39

Fig. 7(b).41

Fig. 7(b).40

  

FG IJ FG IJ

H K H K

Fig. 7(b).42. Theorem of perpendicular


axis
ROTATIONAL MECHANICS AND MOMENT OF INERTIA 7.55



   

 

  Fig. 7(b).43. Theorem of parallel axes

 

 
  
  

 
 

   
 
7.56 PHYSICS (Vol. I)—XI

 

FG IJ Fig. 7(b).44. M.I. of ring about central axis
H  K 

z z LM OP 
 

   N Q
 

Fig. 7(b).45. M.I. of ring about diameter

Fig. 7(b).46. M.I. of ring about


tangent in the plane of the ring

Fig. 7(b).47. M.I. of ring about tangent


perpendicular to the plane of the ring

 
ROTATIONAL MECHANICS AND MOMENT OF INERTIA 7.57


FG IJ
H K 

  Fig. 7(b).48. M.I. of disc about


central axis

z  z LM OP
N Q
 

Fig. 7(b).49. M.I. of disc about


diameter

 Fig. 7(b).50. M.I. of disc about tangent


in the plane of the disc
7.58 PHYSICS (Vol. I)—XI

  

Fig. 7(b).51. M.I. of disc about tangent


perpendicular to the plane of disc

 

Fig. 7(b).52. M.I. of thin uniform rod


  é ù about central axis
   ê ú
ëê ûú 

éæ ö æ ö ù é ù
êç ÷  ç  ÷ø ú ê  ú 
è ø è
ëê ûú ëê ûú


ROTATIONAL MECHANICS AND MOMENT OF INERTIA 7.59

æ ö
 ç ÷
è ø

Fig. 7(b).53
7.60 PHYSICS (Vol. I)—XI


é ù
ê  ú 
ë û

é ù
ê  ú 
ë û
ROTATIONAL MECHANICS AND MOMENT OF INERTIA 7.61


Fig. 7(b).56(a) Fig. 7(b).56(b)

  

Fig. 7(b).54
 

 

 

Fig. 7(b).55

 

 

Fig. 7(b).57
7.62 PHYSICS (Vol. I)—XI

   
   
   

  

  

Z
  

  m3
1
3 m1
3
r1

  m2 r2

O
  Y
X

Fig. 7(b).58. Rigid body


rotating about z-axis.

   

  

 

 
ROTATIONAL MECHANICS AND MOMENT OF INERTIA 7.63

 
 



  

  
   

 

  

  

  
      
 

Fig. 7(b).59
    

 
  

Fig. 7(b).60
7.64 PHYSICS (Vol. I)—XI

 
   
    

  
 
 

 
=

 
 





 
  

  

   

 

F I
GH  JK 


  

Fig. 7(b).61
ROTATIONAL MECHANICS AND MOMENT OF INERTIA 7.65



  
   
  
 Axis of rotation
 
Fig. 7(b).62. Angular momentum

 

 

 

 

 

   
 

 

Fig. 7(b).63. For rotation about an axis of


 
symmetry,  and L are parallel and
along the axis. The directions of both
vectors are given by the right-hand rule

  

 

= = 
7.66 PHYSICS (Vol. I)—XI

 

   

 
 

 

  

    

   

  
L   


 
 


ROTATIONAL MECHANICS AND MOMENT OF INERTIA 7.67

 

Fig. 7(b).64

Fig. 7(b).65. Diver performing


 somersaults
7.68 PHYSICS (Vol. I)—XI

Fig. 7(b).66. Ballet dancer making use of law


of conservation of angular momentum

 

   

   
Fig. 7(b).67
ROTATIONAL MECHANICS AND MOMENT OF INERTIA 7.69

Analogous Parameters in Translational and Rotational Motions

Translational Rotational Translational Rotational


Motion Motion Motion Motion

dp dL
Mass (m) Moment of Inertia (I) F= τ=
dt dt
(translational inertia) (rotational inertia) Impulse = Ft Angular impulse = τt
Linear displacement (S) Angular displacement (θ) Work done = FS Work done = τθ
Linear velocity (v) Angular velocity (ω) Power = Fv Power = τω
1 1 2
Linear acceleration (a) Angular acceleration (α) Translational K.E. = mv2 Rotational K.E. = Iω
2 2
Linear momentum (p) Angular momentum (L) Equations of translational Equations of rotational
If no external force acts, then If no external torque motion motion
linear momentum is conserved. acts, then angular momen- (i) v = u + at (i) ω = ω0 + αt
tum is conserved. 1 2 1 2
(ii) S = ut + at (ii) θ = ω0t + αt
2 2
Force (F) Torque (τ) (iii) v2 – u2 = 2aS (iii) ω2 – ω02 = 2αθ
F = ma τ = Iα

7(b).37. LAWS OF ROTATIONAL MOTION

I. Unless an external torque is applied to it, a body in a state of rest or


uniform rotational motion about its fixed axis of rotation remains unchanged.
II. The rate of change of angular momentum of a body about a fixed
axis of rotation is directly proportional to the torque applied and takes place in
the direction of the torque.
III. When a torque is applied by one body on another, an equal and
opposite torque is applied by the latter on the former about the same axis of
rotation.

Ê/  ˆ /
Solved Problems or ω2 = Á ¥ ω1 = ω
Ë  / + O˜¯ / O

Example 28. What will be the duration of the day, if


Example 27. The initial angular velocity of a circular
disc of mass M is ω1. Then two small spheres of mass m are 
attached gently to two diametrically opposite points on the Earth suddenly shrinks to of its original volume,

the edge of the disc. What is the final angular velocity of
the disc ? [AIEEE 2002] mass remaining unchanged ? Moment of inertia of sphere

 
Solution. L1 = I1ω1 = MR2ω1 = × mass × (radius)2.
 

Solution. Let T1 and ω1 be the period of revolution and


È ˘
L2 = Í /4 + O4 + O4 ˙ w angular velocity respectively of the Earth before contraction.
Î ˚
Let T2 and ω2 be the corresponding quantities after contraction.
No external torque acts. ∴ L2 = L1 Let I1 and I2 be the moments of inertia of the Earth before
È ˘ 2 contraction and after contraction respectively.

Í  / + O˙ R ω2 = /4 ω


Î ˚  Applying the law of conservation of angular momentum,
I1ω1 = I2ω2
7.70 PHYSICS (Vol. I)—XI

FG IJ FG  IJ FG IJ FG  IJ LM OP
 H KH K H KH K 
N Q 

F I
 GH JK ´  


´ ´ ´

FG IJ
  H 
K    ´ 

´ FG IJ FG IJ
 H K 
H K

FG IJ  FG IJ
 
H K H K 

LM OP FG IJ

N Q

H K  

 

  Fig. 7(b).68. Mass point on a string


wound over a cylinder

é ù
êë + úû
ROTATIONAL MECHANICS AND MOMENT OF INERTIA 7.71

+ +

 
+

LM OP
+
MM 
+
PP
N Q

+ +


+ +


+ + + +


7.72 PHYSICS (Vol. I)—XI

P1
1
r 2

P2 cm
r C 
cm

 P0

Fig. 7(b).69. The rolling motion (without


slipping) of a disc on a level surface.

 
 

  


 



+ 
ROTATIONAL MECHANICS AND MOMENT OF INERTIA 7.73
vcm = Rω and I = mK2
2
∴ K = 1 mv2 + 1 m K 2 Ê vcm ˆ
2 cm
2 ÁË R ˜¯

Ê 2ˆ
or K = 1 mv2 1 + K
cm Á ˜
2 Ë R2 ¯

Special Cases

(i) Solid sphere. I = 2 mR 2


5

2
K = 1 mv2 + 1 × 2 mR 2 × v
2 2 5 R2

= 1 mv2 + 1 mv2 = 7 mv2


2 5 10

2
(ii) Hollow sphere. I = mR 2
3

1 1 2 v2
K= mv2 + × mR 2 × 2
2 2 3 R

1 1 5
= mv2 + mv2 = mv2
2 3 6

1
(iii) Solid cylinder. I = mR2
2

1 1 1 v2
K= mv2 + × mR 2 × 2
2 2 2 R

= 1 mv2 + 1 mv2 = 3 mv2


2 4 4

7(b).41. ACCELERATION OF A BODY ROLLING DOWN AN


INCLINED PLANE (From Energy Considerations)

Consider a body of mass M and radius R rolling down (without slipping)


an inclined plane of inclination θ. The gravitational potential energy of the body
(resting at the top of the inclined plane) is converted into kinetic energy of
translation as well as rotation.

1 1 1 v2LM OP
1
∴ Mgh =
2
Iω2 + Mv2
2
or Mgl sin θ =
2
MK2
R N Q
2 +
2
Mv2

LM
2gl sin θ = v2 1 +
K2 OP v2 =
2 gl sin θ
N Q
or or ...(1)
R2 K2
1+ 2 Fig. 7(b).70
R
7.74 PHYSICS (Vol. I)—XI

 
+ +

Fig. 7(b).71. Cylinder rolling without


slipping on an inclined plane

 LM   OP
N Q
 

LM OP 
N +
Q 
+
ROTATIONAL MECHANICS AND MOMENT OF INERTIA 7.75


 
+

  

  

 

  

   

  


æ ö 
ç ÷
è ø
   

 

+
+

   
7.76 PHYSICS (Vol. I)—XI

      

æ ö 
ç ÷
è ø

æ ö
   ç  ÷
è ø


LM +
OP
N Q

  



+ 


 +  +

+ 

+
+ +

Fig. 7(b).72

LM +
OP
N Q +
ROTATIONAL MECHANICS AND MOMENT OF INERTIA 7.77


 Fig. 7(b).73

 ´

+ +

LM +
OP
N Q






Fig. 7(b).74
7.78 PHYSICS (Vol. I)—XI

  

Fig. 7(b).75


æ ö
çç ÷÷
æ ö è ø
ç ÷
è ø

Fig. 7(b).77


+


+
Fig. 7(b).76


    +

´
 ´


ROTATIONAL MECHANICS AND MOMENT OF INERTIA 7.79

+ + +


+

  Fig. 7(b).78. Binary star system


 

´

+
7.80 PHYSICS (Vol. I)—XI

  a3
 ´
+ G

Fig. 7(b).79. Centre of mass of


Earth-Moon system

Fig. 7(b).80. Diatomic molecule

é ù é ù
ê ú ê ú
ë + û ë + û

+ + +

  
+
ROTATIONAL MECHANICS AND MOMENT OF INERTIA 7.81



 


  + +
+  
+
+ 
 

 

Fig. 7(b).81


´ 
+ +

+

 Fig. 7(b).82

´
7.82 PHYSICS (Vol. I)—XI

Fig. 7(b).83

LM +
OP  

MN PQ
 

LM 
OP  
MN PQ



 
 

 Fig. 7(b).84

VERY SHORT ANSWER QUESTIONS—Each Question Carries 1 Mark Only


ROTATIONAL MECHANICS AND MOMENT OF INERTIA 7.83

 

 

+

  

   



 

+

´ ´ 


7.84 PHYSICS (Vol. I)—XI

æ ö
  
  èç ø÷

FG IJ
 H 
K

 

 


ROTATIONAL MECHANICS AND MOMENT OF INERTIA 7.85

 
 

 Fig. 7(b).85
+


  


7.86 PHYSICS (Vol. I)—XI

SHORT ANSWER QUESTIONS—Each Question Carries 2 Marks


 


´ ´
 Fig. 7(b).87
Fig. 7(b).86

  
 ´


  

FG  IJ
H 
 K

 

  
ROTATIONAL MECHANICS AND MOMENT OF INERTIA 7.87

æ ö
´ çè + ÷ø  

I ´ 2
 
T


  
7.88 PHYSICS (Vol. I)—XI

SHORT ANSWER QUESTIONS—Each Question Carries 3 Marks

 

  

 

´ ´ ´

 

 Fig. 7(b).88

  

 
 

 Fig. 7(b).90

 

Fig. 7(b).89




ROTATIONAL MECHANICS AND MOMENT OF INERTIA 7.89
  

 +

 
é ù
ê  ú
ë û
 

é ù
ê  ú
ë û


  
 ´

 

 

 

Fig. 7(b).91

 


+

 

Fig. 7(b).92
7.90 PHYSICS (Vol. I)—XI

  Fig. 7(b).93




´
´ ´
´ ´
´

´ ´ ´ ´ ´
´
ROTATIONAL MECHANICS AND MOMENT OF INERTIA 7.91


Fig. 7(b).94
 

 

  


    

Fig. 7(b).95

Fig. 7(b).96
7.92 PHYSICS (Vol. I)—XI

 
 
  

  

 

 

  

 

Fig. 7(b).97

 

 

Fig. 7(b).98 Fig. 7(b).99

 
 
 
ROTATIONAL MECHANICS AND MOMENT OF INERTIA 7.93



 
 
Fig. 7(b).100

  

  

  

Fig. 7(b).102

 
 

Fig. 7(b).103



Fig. 7(b).101

7.94 Comprehensive PHYSICS (Vol. I)—XI
Q. 119. An object of
uniform mass density rolls
up a curved surface with an
initial velocity v. With
reference to the initial
position, the object reaches

3v 2
a maximum height of . Fig. 7(b).104
4g
Identify the given object.
Answer. Using conservation of energy, Fig. 7(b).106

Answer. (a), then (b) and (c) tie, then (e), (d) being zero.
1 1
mv2 + Iω2 = mgh Explanation. Let ab = bc = cd = da = x
2 2

Ov Z
1 1 v2 Ê 3v 2 ˆ (a) 2 mvx (b) mvx (c) mvx (d) zero and (e) .
mv2 + I 2 = mg Á 
2 2 r ˜
Ë 4g ¯
Q. 122. In Fig. 7(b).107, a block slides down a
frictionless ramp and a sphere rolls without sliding down
2r 2 È 3 mgv2 1 ˘ a ramp of the same angle θ. The block and sphere have the
or I= Í - mv2 ˙
2 4g 2
v ÎÍ ˚˙ same mass, start from rest at point A, and descend through
point B. (a) In that descent, is the work done by the block’s
1 weight on the block greater than, less than, or the same as
= mr2 the work done by the sphere’s weight on the sphere ? At B,
2
which object has more (b) translational kinetic energy and
Clearly, the object is a circular disc or a solid cylinder. (c) speed down the ramp ?

Q. 120. Fig. 7(b).105


shows a horizontal block
that is suspended by two
wires, A and B, which are
identical except for their
original lengths. The Fig. 7(b).107
centre of mass of the Answer. (a) same (b) block (c) block.
Fig. 7(b).105
block is closer to wire B
Explanation. (a) In both the cases, loss of gravitational
than to wire A. Measuring torques about the block’s centre
potential energy is the same.
of mass, state whether the torque due to wire A is greater
(b) and (c). In the case of the block, the potential energy
than, less than, or equal to the torque due to wire B. has been completely transformed into translational kinetic
Answer. Equal. energy. On the other hand, in the case of sphere, the potential
energy is partly transformed into rotational kinetic energy and
System is in equilibrium. So, the two torques must be
partly into translational kinetic energy. Larger the translational
equal in magnitude and opposite in direction.
kinetic energy, more is the speed down the ramp.
Q. 121. Fig. 7(b).106 shows three particles of the same
mass and the same constant speed moving as indicated by Q. 123. Why a helicopter must necessarily have two
the velocity vectors. Points a, b, c and d form a square, propellers ?
with point e at the centre. Rank the points according to Answer. If there were only one propeller in the
the magnitude of the net angular momentum of the three- helicopter, then the helicopter would have to turn in the opposite
direction to conserve angular momentum.
particle system when measured about the points, greatest
first.
ROTATIONAL MECHANICS AND MOMENT OF INERTIA 7.95

Fig. 7(b).108

   
 +   + 


r

Fig. 7(b).109
Fig. 7(b).110
 

  

 
 


7.96 PHYSICS (Vol. I)—XI

LONG ANSWER QUESTIONS

Fig. 7(b).112

Fig. 7(b).111 
 
ROTATIONAL MECHANICS AND MOMENT OF INERTIA 7.97




  

  


 

 


Fig. 7(b).113


 

 -


7.98 PHYSICS (Vol. I)—XI

Fig. 7(b).115

Fig. 7(b).114

 

é  ù
ê ú
êë úû

é  ù
ê ú
êë úû


 


ROTATIONAL MECHANICS AND MOMENT OF INERTIA 7.99

Fig. 7(b).116


Fig. 7(b).118 Fig. 7(b).119

Fig. 7(b).117

LM OP
N Q

 

Fig. 7(b).120




é ù
ê ³ ú
ë û
7.100 PHYSICS (Vol. I)—XI

 é ù
ê ú
ë û


+

 
+

 


 


  


ROTATIONAL MECHANICS AND MOMENT OF INERTIA 7.101

 
LM FG IJ OP  FG IJ 

=
MN +
H K PQ H K


LM OP  
 N Q

  
   

  
 

 


 
  

FG IJ 2

 H K R2



FG IJ 

H K
LM OP
N Q 
  
LM OP 
N Q   

 
 

 

   


LM OP  FG IJ
MN
+ 
PQ H K 

LM +
OP     

MN PQ 

  
 
7.102 PHYSICS (Vol. I)—XI


   

P. 1. A rod OA is hinged at its end O to the edge of a P. 2. The radius of a pulley is


table. Initially the rod is kept horizontal and released 20 cm and its moment of inertia is
0.32 kg m2. A block of mass 2 kg is
from rest. Determine the acceleration of the centre of mass
s u sp e n de d f ro m t h e p ul l e y a s
of the rod. [AIPMT Mains 2007] shown in Fig. 7(b).122. If the block
Solution. Let m and l be the mass and length respec- is released from rest, then deter-
mine the acceleration of the block.
tively of the rod. Torque, about O, due to the weight mg of the rod Take g = 10 m s–2. [AIPMT Mains 2007]
 
= mg   ...(1)
 

ma = mg – T ...(1)
Torque on pulley due to tension,

 = TR = I =
Fig. 7(b).122
where R is the radius of the pulley.
Fig. 7(b).121
T=
Let I be the moment of inertia of the rod about an axis
passing through O and perpendicular to the length of the rod.
Let  be the angular acceleration of the rod. From eq. (1), ma = mg –

Then,  = I = × ...(2)
 
or a   = mg
 
From (2) and (1), =

or a=
or = 
When the rod is released, the centre of mass moves along
I = 0.32 kg m2, m = 2 kg, R = 0.2 m
a circle of radius .
 a= m s–2 = 2 m s–2.

Acceleration of centre of mass, a = r = = .  
ROTATIONAL MECHANICS AND MOMENT OF INERTIA 7.103
P. 3. A painter weighing 50 kg paints the wall of a The total weight of the bees acts at C or L.
house, while standing on a scafford (a criss-cross stand
LN is horizontal.
made of steel bars and a long wooden board placed on it)
as shown in Fig. 7(b).123. If the mass of the board is 20 kg,  0.3 × 25 = m3 × 15 or m3 = 0.5 kg
how close to its end can be painter stand without risking Now, m1 + m2 + m3 = (0.1 + 0.2 + 0.5) kg = 0.8 kg
the board to topple over ?
Weight 0.8 g acts at R.
PR is horizontal.
 0.8 × 15 = m4 × 30 or m4 = 0.4 kg.
P. 5. There is a rectan-
gular plate of mass M kg of
dimensions (a × b). The plate
is held in horizontal position
by striking n small balls each
of mass m per unit area per
unit time. These are striking Fig. 7(b).125
in the shaded half region of
the plate. The balls are colliding elastically with velocity
v. What is v ? It is given n = 100, M = 3 kg, m = 0.01 kg ; b = 2
Fig. 7(b).123
m ; a = 1 m, g = 10 m s–2.
Solution. Torque about hinge side
Solution. Torque, about O, due to the weight Mg of the
painter = Mg(1.5 – x), where x = AP
Torque, about O, due to the weight mg of the wooden

board = mg × = 1.25 mg

For equilibrium, Mg(1.5 – x) = 1.25 mg



M = 50 kg, m = 20 kg
 
 50 g (1.5 – x) = 1.25 × 20 g or x = 1 m.
P. 4. An artist constructs a birds-and-bees mobile
toy as shown in Fig. 7(b).124.

Fig. 7(b).126

Solution. For equilibrium, net force in vertical direction


should be zero.

æ ö
Fig. 7(b).124  çè + ÷ø

If the bee on the lower left has a mass m1 = 0.1 kg


O
and each vertical support string has a length of 30 cm,
find the mass (m2) of the other bee and the masses (m3 and
m4) of the two birds. The masses of the bars AB, LN and PR
N mg N
may be neglected. [Physics Olympiad]
Solution. The bar AB is horizontal. Clearly, the moments Mg
Mg
of the weights of the two bees about C are equal and opposite.  

 0.1 × 40 = m2 × 20 or m2 = 0.2 kg A f f B

Now, m1 + m2 = 0.3 kg Fig. 7(b).127


7.104 Comprehensive PHYSICS (Vol. I)—XI
For rotation equilibrium, net torque about O is zero. P. 9. A uniform cir-
Therefore, cular disc has radius R and
mass m. A particle, also of
N
f l sin θ – Nl cos θ + Mg cos θ = 0 ...(2) mass m, is fixed at a point A

on the edge of the disc as
From equations (1) and (2), we get shown in Fig. 7(b).130. The
disc can rotate freely about

f= (M + m)g cot θ a fixed horizontal chord PQ Fig. 7(b).130

4
P. 7. A solid cylinder rolls without slipping on an that is at a distance from

inclined plane inclined at an angle θ . Find linear
the centre C of the disc. The line AC is perpendicular to
acceleration of the cylinder. Mass of the cylinder is M.
PQ. Initially the disc is held vertical with the point A at
Solution. For translational motion its highest position. It is then allowed to fall so that it
Mg sin θ – f = Ma ...(1) starts rotating about PQ. Find the linear speed of the par-
For rotation motion ticle as it reaches its lowest position.
Solution. Decrease in potential energy of system
 
f R =  /4  α ...(2)
  4 4
= mg   × +  ×  = 3mgR
  

Fig. 7(b).128
For no slipping, a = αR ...(3)
Fig. 7(b).131
From equations (1), (2) and (3), we get


a= g sin θ  
 O4 4  4 
I= + m  + m 
 
    
P. 8. A rod of mass M and
length L is suspended by a 
= mR2
frictionless hinge at point O as 
shown in Fig. 7(b).129. A bullet  2 
of mass m moving with velocity Gain in kinetic energy = Iω = mR2ω2
 
v in horizontal direction strikes
the end of the rod and gets em-   I
Equating, mR2ω2 = 3mgR or ω=
bedded in it. Find the angular Fig. 7(b).129  4
velocity acquired by the rod just  4 
after the collision. Now, v=  ω
  
Solution. Applying conservation of angular momentum
about hinge O, 4  I
= =  I4
 4
Ê 2 ML2 ˆ
mvL = Á m L + ˜ω P. 10. A homogeneous
Ë 3 ¯
rod AB of length L = 1.8 m
and mass M is pivoted at the
OX .
⇒ ω= centre O in such a way that

O + / .
it can rotate freely in the
vertical plane. The rod is
OX
⇒ ω= initially in the horizontal

O / .
position. An insect S of the Fig. 7(b).132
same mass M falls vertically
ROTATIONAL MECHANICS AND MOMENT OF INERTIA 7.105
with speed v on the point C, mid-way between the points O
and B. Immediately after falling, the insect moves towards
the end B, such that the rod rotates with a constant angu-
P. 12. A carpet of mass M made of inextensible mate-
lar velocity . Determine . [Ans. 0.42v]
rial is rolled along its length in the form of cylinder of
é radius R and is kept on a rough floor. The carpet starts
  ù
Hint. Mv × = ê +   ú  unrolling without sliding on a floor when a negligibly
ëê ûú small push is given to it. Calculate the horizontal velocity
P. 11. A uniform bar of length of 6a and mass 8 m of the axis of the cylindrical part of the carpet when its
lies on a smooth horizontal table. Two point masses m
and 2m moving in the same horizontal plane with speeds radius reduces to .
2 and respectively, strike the bar as shown and stick
to the bar after collision [Fig. 7(b).133].
Solution. When the radius is reduced to , the mass of

the rolled carpet

 LM OP
 N Q
 
 

Fig. 7(b).133   é   ù é ù
  ê   ú ê ú
êë úû ë û

Solution. Angular momentum before the collision


 

é ù
    ê ú
ëê ûú

P. 13. A ring of mass 0.3 kg and radius 0.1 m and a


solid cylinder of mass 0.4 kg and of the same radius are
given the same kinetic energy and released simultaneously
 OP on a flat horizontal surface such that they begin to roll as

PQ soon as released towards a wall which is at the same


distance from the ring and the cylinder. The rolling friction
is negligible in both cases. Which of the two will reach the
wall first ?
 Solution. Kinetic energy in rolling motion,

Er = m 2 + I2

 
  + 

  

+
FG IJ

H K
7.106 PHYSICS (Vol. I)—XI
P. 15. A solid cylinder of mass 20 kg and radius
0.12 m rotating with initial angular speed of 125 s–1 is placed
 lightly (i.e., without any translational push) on a horizontal
table with coefficient of kinetic friction k = 0.15, between
cylinder and table.


+

P. 14. A cylinder of mass 5 kg and radius 30 cm, and


free to rotate about its axis, receives an angular impulse
of 3 kg m2 s–1 initially followed by a similar impulse after
every 4 s. What is the angular speed of the cylinder 30 s after
the impulse ? The cylinder is at rest initially.

Solution. M = 5 kg, R = 30 cm = 0.3 m

 

LM OP
N   
Q
 


 


 -   
 =  

  
  

LM OP
N   
Q


ROTATIONAL MECHANICS AND MOMENT OF INERTIA 7.107
(iii) Total final energy = 250 J + 125 J = 375 J
CN DN N Ê al 3 ˆ
(d) No, loss of energy = 750 J dissipated against friction = + = Á + b˜ .
   Ë 2 ¯
before rolling motion begins.
(e) Work done by friction for translational motion P. 17. Calculate the moment of inertia of a uniform
= + 250 J
annular disc of internal radius r and external radius R
Work done by friction against rotational motion
and mass M about an axis passing through the centre and
= – µk mg Rθ = – 1000 J
perpendicular to the plane.
LMHere θ = ω t − 1 αt , t = Rω , α = µ
2 0 k mg R OP Solution. Mass of elementary ring of radius x and
N 0
2 3µ g 0 I Q thickness dx
Thus net work done by friction on the body
= 250 J – 1000 J = – 750 J M
= (2πx dx)
This accounts for the loss of energy in (d). π (R2 − r2 )
No need to proceed as follows
(i) calculate α (ii) calculate ω (iii) Use v = Rω 2Mx 3 dx
dI =
Explanation. R2 − r2
F I
z
2µ k g ω 0 R
v = Rω = R(ω0 – αt) = R ω 0 − GH R
×
3µ k g JK R 2M 2M R
I= 2 2 x3 dx = 2 2 Ú x 3 dx
FG IJ
2 1
r R −r R −r r
= Rω0 1 −
H 3K= Rω0
3
/
4 + T 
4 − T 
ω R 1 =
Again, v = µk gt = µk g × 0 = Rω0 4 − T
 
3 µk g 3
P. 16. A rod of length l rotates about an axis passing 
= M(R2 + r2).
through one of its ends and perpendicular to its length. The 
linear mass density of the rod varies as ρ = (ar3 + b) kg m–1.
Determine the moment of inertia of the rod about the given P. 18. A circular cylinder has an inextensible string
axis. wrapped around it as shown in Fig. 7(b).135. What is the
Solution. Consider a small ele- linear acceleration of the cylinder when
ment of length dr at a distance r from the released ?
given axis. È 2 ˘
Mass of element = ρdr Í Ans. 3 g ˙
Î ˚
Moment of inertia of rod about Hint. mg – F = ma,
given axis rF = Iα

l l +α Ê1 ˆ α
= Á mr 2 ˜
= Ú 0
(r dr) r 2 = Ú 0
( ar 3 + b) r 2 dr
Fig. 7(b).134
or F=
T Ë2 ¯ T
Fig. 7(b).135
l l
 
Ú Ú
F = mrα = ma
= ar 5 dr + br 2 dr  
0 0

NCERT EXERCISES (With Solutions)

Q. 1. Give the location of the centre of mass of a Q. 2. In the HCl molecule, the separation between
(i) sphere, (ii) cylinder, (iii) ring, and (iv) cube, each of the nuclei of the two atoms is about 1.27 Å (1 Å = 10–10 m).
uniform mass density. Does the centre of mass of a body Find the approximate location of the centre of mass of
necessarily lie inside the body ? the molecule. Given : chlorine atom is about 35.5 times as
Ans. The geometrical centre of each. No, the centre of massive as a hydrogen atom and nearly all the mass of an
mass may lie outside the body as in the case of ring, a hollow atom is concentrated in its nucleus.
sphere, a hollow cylinder, a hollow cube etc. In these examples, Ans. Let us choose the nucleus of the hydrogen atom as
there is no mass at the centre of mass. the origin for measuring distance.

☞ The centre of mass may or may not lie inside the body. Mass of hydrogen atom, m1 = 1 unit (say)
7.108 PHYSICS (Vol. I)—XI


 

Fig. 7(b).136


    
 

Fig. 7(b).138

  

 

 

   Fig. 7(b).139

  

  


 
  

  


  Fig. 7(b).137

Fig. 7(b).140
NCERT EXERCISES (UNIT V) 7.109

 

 

 
 

 


 

 
Fig. 7(b).141
  

 


  

7.110 PHYSICS (Vol. I)—XI

      


       

 
  


æ ö æ ö
 çè ÷ø çè  ÷ø 

é  ù
ê = =  =  ú
 ë  û

é ù
 ê=  ú
 ë û

é ù
ê=  ú
ë û

 


  
 


  
NCERT EXERCISES (UNIT V) 7.111
The linear acceleration of the rope,
a = R ∝ = 0.4 × 25 m s–2 = 10 m s–2.
Q. 15. To maintain a rotor at a uniform angular
speed of 200 rad s–1, an engine needs to transmit a torque
of 180 N m. What is the power required by the engine ?
Assume that the engine is 100% efficient. (Note : Uniform
angular velocity in the absence of friction implies zero
torque. In practice, applied torque is needed to counter Fig. 7(b).143
frictional torque.) Assume that the engine is 100%
10 g (45 – 12) = mg (50 – 45)
efficient.
10 g × 33 = 5 mg
Ans. ω = 200 rad s–1, τ = 180 N m, P = ?
or m = 66 gram.
Power = Torque × angular speed
Q. 18. A solid sphere rolls down two different
= 180 × 200 W = 36000 W = 36 kW.
inclined planes of the same heights but different angles
Q. 16. From a uniform disc of radius R, a circular of inclination. (a) Will it reach the bottom with the same
section of radius R/2 is cut out. The centre of the hole is at speed in each case ? (b) Will it take longer to roll down
R/2 from the centre of the original disc. Locate the centre one plane than the other ? (c) If so, which one and why ?
of mass of the resulting flat body.
Ans. (a) Using law of conservation of energy,
Ans. Mass of original disc = M (say)
 
OCUU / mv2 + Iω2 = mgh
=  
CTGC π4 

  Ê2 ˆ X
Mass of portion removed from the disc or mv2 + Á mR 2 ˜ = mgh
  Ë5 ¯ 4
2
/ Ê Rˆ /
= 
πÁ ˜ =  2  IJ
π4 Ë 2¯  or v = gh or v =
 
While the centre Since h is same for both the inclined planes therefore v
of mass of M is at O, the is the same.
/
centre of mass of is at Ê ˆ

 Á g sin q ˜ 2 g sin q 2  I UKP θ 2
O′. To locate the centre of (b) l = Á ˜ t = t = t
 Á 2
K ˜ Ê 2ˆ 
mass of the resulting flat ÁË 1 + 2 ˜¯ 2 ÁË 1 + ˜
R 5¯
body, we have to find the
centre of mass of mass M
Fig. 7(b).142 N
Ê Mˆ or t=
 I UKP θ
at O and mass Á – ˜ at
Ë 4¯
O′. If x be the distance of centre of mass from O, then J J
Now, sin θ = or l =
N UKP θ
/ 4
/×− ×
x=   = – /4 ×  = – 4  J
/  /  ∴ t = UKP θ
/− I

Lesser the value of θ, more will be t.
The negative sign indicates that the centre of mass is
(c) Clearly, the solid sphere will take longer to roll down
to the left of O.
the plane with smaller inclination.
Q. 17. A metre stick is balanced on a knife edge at
Q. 19. A hoop of radius 2 m weighs 100 kg. It rolls
its centre. When two coins, each of mass 5 g, are put one on
along a horizontal floor so that its centre of mass has a
top of the other at the 12.0 cm mark, the stick is found to
speed of 20 cm/s. How much work has to be done to stop it ?
be balanced at 45.0 cm. What is the mass of the metre stick ?
Ans. Energy of loop
Ans. Using rotational equilibrium about the 45 cm
mark,  
= mv2 + Iω2
 
7.112 PHYSICS (Vol. I)—XI

  

 

 

 




æ ö Fig. 7(b).144
 çè ÷ø
Fig. 7(b).145

 


Fig. 7(b).146

æ ö
çè ÷ø

 
 

   

 
 
NCERT EXERCISES (UNIT V) 7.113

9  I OXT  ×  × − ×  × 


or FB – FC = = = 10 g ...(2) or ω= = rad s–1
  /.  ×  × 
From (1) and (2), 2FB = 50 g or FB = 25 g = 25 × 9.8 N = 0.625 rad s–1.
= 245 N Q. 25. Two discs of moments of inertia I1 and I2
about their respective axes (normal to the disc and
FC = FB – 10 g = 25 g – 10 g = 15 g
passing through the centre), and rotating with angular
= 15 × 9.8 N = 147 N speeds ω1 and ω2 are brought into contact face to face
with their axes of rotation coincident. (a) What is the
(% ×  EQU θ angular speed of the two-disc system ? (b) Show that the
Again, T =  UKP θ kinetic energy of the combined system is less than the sum
of the initial kinetic energies of the two discs. How do you
= 147 × 2 × cot θ account for this loss in energy ? Take ω1 ≠ ω2 .
Fig. 7(b).147
 Ans. (a) Let I1 and I2 be the moments of inertia of two
T = 147 × 2 ×

N = 98 N discs having angular speeds ω1 and ω2 respectively. When they
are brought in contact, the moment of inertia of the two-disc
Q. 23. A man stands on a rotating platform, with system will be I1 + I2. Let the system now have an angular
his arms stretched horizontally holding a 5 kg weight in speed ω. From the law of conservation of angular momentum,
each hand. The angular speed of the platform is 30 we know that
revolutions per minute. The man then brings his arms I1ω1 + I2ω2 = (I1 + I2)ω
close to his body with the distance of each weight from ∴ The angular speed of the two-disc system,
the axis changing from 90 cm to 20 cm. The moment of
inertia of the man together with the platform may be taken +ω + + ω
ω= .
to be constant and equal to 7.6 kg m2. + + 
(a) What is his new angular speed ? (Neglect (b) The sum of kinetic energies of the two discs before
friction.) coming in contact,
(b) Is kinetic energy conserved in the process ? If 1 1
I1ω12 + I2ω22
k1 =
not, from where does the change come about ? 2 2
Ans. (a) I1 = (7.6 + 2 × 5 × 0.9 × 0.9) kg m2 The final kinetic energy of the two-disc system,
= 15.7 kg m2, ω1 = 30 rpm, 1
k2 = (I1 + I2) ω2
I2 = (7.6 + 2 × 5 × 0.2 × 0.2) kg m2 2
2
= 8 kg m2, ω2 = ? 1 Ê I1w1 + I 2w2 ˆ 1
+ω + +ω

= (I + I ) × ÁË I + I ˜¯ =
Using conservation of angular momentum, 2 1 2 1 2 2 + +

+ω 1  1
+ω + +ω 
I2ω2 = I1ω1 or ω2 = + Now k1 – k2 = I1ω12 + 
I2ω22 –
 2 2 + +

 ×  = × [(I1ω12 + I2ω22)
= rpm = 58.875 rpm 
+ +

(I1 + I2) – (I12ω12 + I22ω22 + 2I1I2ω1ω2)]
(b) Kinetic energy is not conserved. As moment of inertia

decreases, rotational kinetic energy increases. This change = × [I12ω12 + I22ω22 + I1I2ω12

+ +
comes about as work is done by the man in bringing his arms
close to his body. + I1I2ω22 – I12ω12 – I22ω22 – 2I1I2ω1ω2]

Q. 24. A bullet of mass 10 g and speed 500 m/s is = [I I (ω 2 + ω22 – 2ω1ω2)]
fired into a door and gets embedded exactly at the centre 
+ + 1 2 1
of the door. The door is 1.0 m wide and weighs 12 kg. It is
++ 
hinged at one end and rotates about a vertical axis = (ω – ω2)2

+ + 1
practically without friction. Find the angular speed of
the door just after the bullet embeds into it. Now (ω1 – ω2)2 will be positive whether ω1 is greater or
The moment of inertia of the door about the vertical smaller than ω2.
axis at one end is ML2/3. Also, I1I2/2(I1 + I2) is also positive because I1 and I2 ore
 positive.
/.
Ans. L = Iω ; mvr = ω Thus, k1 – k2 is a positive quantity.

7.114 PHYSICS (Vol. I)—XI





 

Fig. 7(b).148 
 

   

  

é ù 
ê ú    
ê ú
ê  ú
ê ú 
ë û     
 
  é  ù
  ê  ú
ë û


     


 
 

 é  ù
  ê  ú
ê ú
ë û
Fig. 7(b).149
NCERT EXERCISES (UNIT V) 7.115
    
   

 


NCERT EXEMPLAR PROBLEMS (With Answers)


[Based on Higher Order Thinking Skills].
MCQ I (Only one option is correct.)

Hollow
sphere

Air

A
B
C
Fig. 7(b).151

D
Sand

Fig. 7(b).150
7.116 PHYSICS (Vol. I)—XI

 
= = 


D =  =

  
D = D =    =  

MCQ II (Any number of options may be


correct.)

 

y y

 
hole
Q 
x x

Fig. 7(b).152


 

 
 

 
 
 Fig. 7(b).153
 


NCERT EXEMPLAR PROBLEMS (UNIT V) 7.117

 



  
 

Z z

H
Ä G
 z

Ä
D C
O
E F
Y

 = 
A a B
 X
= Ä
   Fig. 7(b).155
=  =  Ä

 = 

 


æ ö
  =  çè ÷ø = 


P

VSA

Fig. 7(b).154
 

 
 

 

  
7.118 PHYSICS (Vol. I)—XI

<


t1 t2 t3
O t

Fig. 7(b).156 Fig. 7(b).158

Fig. 7(b).157
 æ ö
=ç ÷ø
è

   

  

 = 

 
NCERT EXEMPLAR PROBLEMS (UNIT V) 7.119

SA
  
 = 


å 


 
å 
 
=
  

       LA
å   å    å
 

  
  


     
 =
 

(    )
D  

y


w

z
x

Fig. 7(b).159
7.120 Comprehensive PHYSICS (Vol. I)—XI
(c) What happens to the linear speed of the centre of m mgR
m k gt
mass when disc is placed in contact with the table ? = w0 + k t
R I
(d) Which force is responsible for the effects in (b) and
(c) ? or t= Rw0
(e) What condition should be satisfied for rolling to begin ? Ê mR 2 ˆ
m k g Á1 - ˜
(f) Calculate the time taken for the rolling to begin. Ë I ¯
Ans. (a) Before the disc is brought in contact with table, it is 25. A uniform disc of radius R, is resting on a table on its rim.
rotating about horizontal axis. The coefficient of friction between disc and table is µ
So, vcm = 0 (Fig.7(b).160). Now the disc is pulled with a force F as
(b) Linear velocity of a point on the rim will decrease, shown in Fig. 7(b).160 . What is the maximum value of F
when the disc is placed in contact with the table. for which the disc rolls without slipping ?
(c) The centre of mass acquires some velocity. So, velocity
of centre of mass increases. F

(d) In (b), force of friction decreases linear velocity of a


point on the rim of the disc. In (c), the disc rolls over the
Fig. 7(b).160
table because of friction. So, velocity of centre of mass
increases due to friction. Ans. If f is force of friction and a is acceleration of centre of
(e) vcm = Rω mass, then
(f) Acceleration produced in centre of mass due to friction, Ma = F – f ...(i)

a
acm = F = m kmg = m k g If there is no slipping, α =
m m R

Angular acceleration produced by torque due to friction, Ê1 ˆ a MaR


Now, f × R = Iα = Á MR 2 ˜ = or Ma = 2f
Ë2 ¯ R 2
α = t = m kmgR
I I
F
Now, vcm = ucm + acmt From (i), 2f = F – f or 3f = F or f =
3
vcm = 0 + µk gt = µk gt
Because there is no sliding,
m mgR
Again, ω = ω0 + αt = ω0 + k t F
I ∴ f ≤ µMg or ≤ µMg
3
For rolling without slipping,
or Fmax. = 3µMg
vcm
= w 0 + m kmgR t
R I




 
 


 

7.121
7.122 PHYSICS (Vol. I)—XI

 

   

 

  



MOTION OF SYSTEM OF PARTICLES AND RIGID BODY (UNIT V) 7.123

A solid sphere is rolling on a frictionless surface, shown


in figure, with a translational velocity v m s–1. If it is to
climb the inclined surface, then v should be

 
 
 
7.124 PHYSICS (Vol. I)—XI

 

 

 

 


 


 



MOTION OF SYSTEM OF PARTICLES AND RIGID BODY (UNIT V) 7.125

 
 

 
 
7.126 PHYSICS (Vol. I)—XI

 

 

 

 

 
 
 
MOTION OF SYSTEM OF PARTICLES AND RIGID BODY (UNIT V) 7.127

 

   
 
   

   

   
 

     
 

      

     

 

 
7.128 PHYSICS (Vol. I)—XI


MOTION OF SYSTEM OF PARTICLES AND RIGID BODY (UNIT V) 7.129

 
 


 
 

 
 

 
 

  


7.130 PHYSICS (Vol. I)—XI

  

  

   
     
   

   
    
   

  
 

  

   
MOTION OF SYSTEM OF PARTICLES AND RIGID BODY (UNIT V) 7.131

 

 
  
   

 
  
   

 
  
   
7.132 PHYSICS (Vol. I)—XI


MOTION OF SYSTEM OF PARTICLES AND RIGID BODY (UNIT V) 7.133


 


 

    
 


    
 

      
  
7.134 PHYSICS (Vol. I)—XI

 

 

 


MOTION OF SYSTEM OF PARTICLES AND RIGID BODY (UNIT V) 7.135






7.136 PHYSICS (Vol. I)—XI



 

 

 

 

 

 
 
MOTION OF SYSTEM OF PARTICLES AND RIGID BODY (UNIT V) 7.137

   
   
   

   
   
   

   
   
   

   
   
   

 
 
 


7.138 PHYSICS (Vol. I)—XI


P Q
L


 
MOTION OF SYSTEM OF PARTICLES AND RIGID BODY (UNIT V) 7.139
S tic k

R
m

 

 

   

    

M1 M2

O =  

=  

   

   

7.140 PHYSICS (Vol. I)—XI


=  

=  

     

   

 

 

 

E  
B C

 
G H

A D
F
MOTION OF SYSTEM OF PARTICLES AND RIGID BODY (UNIT V) 7.141

   
   

   
   

X

 

R0

7.142 PHYSICS (Vol. I)—XI

  

  

6.5 kg 2.2 m s–1


A
3.6 m s–1
1.5 m

B
O 2.8 m 3.1 kg


  
   

  
  


    
 

 
 
 
MOTION OF SYSTEM OF PARTICLES AND RIGID BODY (UNIT V) 7.143

   

  

   
 

 
  
 


    

   

   


    



  

  

 

    
   
   

 

   
 
 
7.144 PHYSICS (Vol. I)—XI

 
 
    
    
  

  
 
   

  

 

 
   
 



 
     
 

     
  
   
   
   
 



  

    
     
    
  
    
    
       
  
    
   
 
  
 


 
 
    


 
 


 
MOTION OF SYSTEM OF PARTICLES AND RIGID BODY (UNIT V) 7.145

   
   
   



 


 

  
 
  

   
   
   

 

 


   

 



 





 


   
  
 

 
7.146 PHYSICS (Vol. I)—XI

 


  
  
 
I


 
  
 
   

  

 


 
  
   
 
   

 

 
   
 


 
  
  

   
 
   

  

 
   
   
    
 
MOTION OF SYSTEM OF PARTICLES AND RIGID BODY (UNIT V) 7.147

     
      
     

 

 

 
   
 

 
    
 
    


    
    
    
 
 
  
  


    
   
   

 

   
7.148 PHYSICS (Vol. I)—XI





  

 



 

 

 
  
  
   
     
   
 
  
    
    
   

 
 

 
 
 

 
  

  
   
  

     
  =      
   

 
  

MOTION OF SYSTEM OF PARTICLES AND RIGID BODY (UNIT V) 7.149

nd Tips
Hints a
132. Iring = MR2, mr2ω0 = mvr + mr2ω
 X 
X + = /X
= OXT + OT ÊÁ ˆ˜  4 

 . 
Irod = ML2 + M  4  Ë T¯
  
= 2mvr I =  /4
I = Iring + Irod. 
133. Iω0 = (mv0)h, or v = Tw So, the object is a disc.

149. α = 2 rev s–2 = 2 × 2π rad s–2
 X J 143. L = Iω = MK2ω
mr2ω0 = mv0h or ω0 =  = 4π rad s–2
 T 144.
t 64
α= =
+  /4
m1 m2
O 
Z Z /4a
T=

137. For toppling, 0 = O
- Z + O Z
O + O 150. arolling = I UKP q = I UKP q
. - 
Mg × = F1 × h or m1x1 = m2x2 +  +
 4 


m1 d O d′ m2 arolling = I UKP q

Z – d Z – d′ aslipping = g sin θ
151. m1 = 2 kg, m2 = 6 kg
- O
Z - F + O
Z - F ¢ x1 = 1 m, x2 = 3.5 m
For sliding, 0=
O + O
µMg = F2
0 = m1(d – x1) + m2(x2 – d′) Xcm = O Z + O Z
Condition for sliding to occur O + O
0 = m1d – m1x1 + m2x2 – m2d′
first F1 > F2 153. a = Rα
0 = m1d – m2d′
/I. m2d′ = m1d mg – T = ma ...(i)
> µMg or L > 2µh.
J T × R = Iα
d′ = O F T × R = mR2α
 IJ O
138. v = . T = mRα
- 146. When the string is cut, the rod will T = ma
+ 
4 rotate about P. From eqn. (i)
 mg = 2ma
I UKP θ τ = Iα = /. a
139. a = .  a= I
-
+  . 
4 Also, τ = /I
 Æ Æ Æ
154. t = T ¥(
   . /. 
140. Kr = Iω2 = ω(Iω) = ωL /I = a
     Kł Lł Mł
- T - I =   
L= = T . or α=
ω πν .   
 
τ (4 147. /X + +w  = /IJ   
141. α = = .   157. +w = OX
+ /4  

  Ê Xˆ Ê X ˆ
142. Applying law of conservation at / X  + + Á ˜ = /I Á ˜
Æ Æ Æ
  Ë 4¯ Ë I ¯ 158. .= T ¥ R
the point of contact,
7.150 PHYSICS (Vol. I)—XI

   

    
  =
 
  

 
 = = 

   
   =
  =
 
   

 
   =
= =

=
= = =

 = =

 =
 
=  

  =


  =   =
   =

= =
 

   
 
 =
 
A B
d– 

N1 N2

B W 
r

O  d
= =
 
r
  
A  =
 
MOTION OF SYSTEM OF PARTICLES AND RIGID BODY (UNIT V) 7.151

 
= =
       
      
     = = =   

    
   
= = »
7.152 PHYSICS (Vol. I)—XI
WIN A CASH AWARD OF ` 500.00

It has come to our notice that some booksellers are fraudulently selling fake/duplicate copies
of some of our fast selling titles.
In our sincere efforts to provide you with our genuine books and to protect you against these
counterfeit books, Laxmi Publications (LP) has put a Hologram on the cover of some of its fast
selling titles. The Hologram displays a unique 3D Litho, multi-level, multi-colour effect from
different angles. It has the following seven levels of flat graphics merged together. The background
artwork seems to be ‘under’ or ‘behind’ the Hologram and gives the illusion of depth unlike the fake
Hologram on the fake/duplicate books.
DOTMATRIX /
FOREGROUND FLIP : 1 FLIP : 2
BACKGROUND
LAYER
LAYER

LAXMI
7
PUBLICATIONS
P L

ANIMATION LENS / MOVING REVOLVING EARTH


LAYER STAR LAYER LAYER

Presently, only some titles have got the Holograms. In this case, Comprehensive Physics
Volume (I and II) for Class XI has got the Hologram.
If you or any of your friends finds anywhere in India/abroad any book of this Edition without
the LP Hologram, he/she is requested to write to us at M/s LAXMI PUBLICATIONS PVT. LTD.,
113, Golden House, Ansari Road, Daryaganj, New Delhi-110002, giving the name and address
of the bookseller from where he/she purchased this book, together with the photocopy of the cover
and the 4th page on which the name of the printer is printed, he/she will be sent a cash award of
` 500.00.
How to decide if the book is genuine or fake ?
1. The above information may or may not be printed.
2. The counterfeit edition of the book may have no LP Hologram or if it has, it will be without
the illusionary depth as described above.
What is the harm in purchasing duplicate books ?
— Poor quality of paper and printing which affect your eyes.
— No royalty to authors who are scholars and have put their hard labour in writing the book,
thus depriving them of their intellectual rights.

Warning : Selling or buying pirated books is an offence. Legal action shall be taken against the
bookseller(s) and student(s) or whoever found guilty of such an offence in any way.
By
NARINDER KUMAR
M.Sc. PES(I)
Formerly, Senior Lecturer
Department of Physics
S.D. Govt. College, Ludhiana
Punjab

LAXMI PUBLICATIONS (P) LTD


(An ISO 9001:2008 Company)
BENGALURU • CHENNAI • COCHIN • GUWAHATI • HYDERABAD
JALANDHAR • KOLKATA • LUCKNOW • MUMBAI • RANCHI • NEW DELHI
BOSTON (USA) • ACCRA (GHANA) • NAIROBI (KENYA)
Comprehensive PHYSICS XI (Vol-II)

© by Laxmi Publications (P) Ltd.


All rights reserved including those of translation into other languages. In accordance with the Copyright (Amendment) Act, 2012, no part of this publication
may be reproduced, stored in a retrieval system, or transmitted in any form or by any means, electronic, mechanical, photocopying, recording or otherwise.
Any such act or scanning, uploading, and or electronic sharing of any part of this book without the permission of the publisher constitutes unlawful piracy and
theft of the copyright holder’s intellectual property. If you would like to use material from the book (other than for review purposes), prior written permission
must be obtained from the publishers.

Printed and bound in India


Typeset at Kalyani Computer Services, Delhi
New Edition
ISBN 978-81-318-0196-3

Limits of Liability/Disclaimer of Warranty: The publisher and the author make no representation or warranties with respect to the accuracy or completeness
of the contents of this work and specifically disclaim all warranties. The advice, strategies, and activities contained herein may not be suitable for every
situation. In performing activities adult supervision must be sought. Likewise, common sense and care are essential to the conduct of any and all activities,
whether described in this book or otherwise. Neither the publisher nor the author shall be liable or assumes any responsibility for any injuries or damages
arising herefrom. The fact that an organization or Website if referred to in this work as a citation and/or a potential source of further information does not
mean that the author or the publisher endorses the information the organization or Website may provide or recommendations it may make. Further, readers
must be aware that the Internet Websites listed in this work may have changed or disappeared between when this work was written and when it is read.
All trademarks, logos or any other mark such as Vibgyor, USP, Amanda, Golden Bells, Firewall Media, Mercury, Trinity, Laxmi appearing in this work are
trademarks and intellectual property owned by or licensed to Laxmi Publications, its subsidiaries or affiliates. Notwithstanding this disclaimer, all other
names and marks mentioned in this work are the trade names, trademarks or service marks of their respective owners.

& Bengaluru 080-26 75 69 30


& Chennai 044-24 34 47 26, 24 35 95 07
& Cochin 0484-237 70 04, 405 13 03
& Guwahati 0361-254 36 69, 251 38 81
Branches

& Hyderabad 040-27 55 53 83, 27 55 53 93


& Jalandhar 0181-222 12 72
& Kolkata 033-22 27 43 84

Published in India by & Lucknow 0522-220 99 16


& Mumbai 022-24 91 54 15, 24 92 78 69
& Ranchi 0651-220 44 64
Laxmi Publications (P) Ltd.
(An ISO 9001:2008 Company)
113, GOLDEN HOUSE, DARYAGANJ,
NEW DELHI - 110002, INDIA
Telephone : 91-11-4353 2500, 4353 2501
Fax : 91-11-2325 2572, 4353 2528 C—
www.laxmipublications.com info@laxmipublications.com Printed at:
Chapter Page

8. GRAVITATION ... 8.3–8.73

(v)
(vi)

9. MECHANICAL PROPERTIES OF SOLIDS ... 9.3–9.42


(vii)

10(a). FLUIDS AT REST [Mechanical Properties of Fluids (I) ] ... 10.1–10.30

10(b). FLUIDS IN MOTION [Mechanical Properties of Fluids (II) ] ... 10.31–10.70


(viii)

10(c). SURFACE TENSION [Mechanical Properties of Fluids (III) ] ...10.71–10.116


(ix)

11(a). THERMAL PROPERTIES OF MATTER ... 11.1–11.38


(x)

11(b). HEAT TRANSFER ... 11.39–11.75


(xi)

12(a). FIRST LAW OF THERMODYNAMICS ... 12.3–12.40

12(b). SECOND LAW OF THERMODYNAMICS ... 12.41–12.70


(xii)

13. KINETIC THEORY ... 13.3–13.60


(xiii)

14. OSCILLATIONS ... 14.3–14.85


(xiv)

15. WAVES ... 15.1–15.102


(xv)
(xvi)
UNIT—VI
GRAVITATION 12 Periods

Chapter–8: Gravitation

SOLVED NUMERICALS 55 PLUS

UNSOLVED NUMERICALS 90 PLUS

VERY SHORT, SHORT & LONG ANSWER QUESTIONS 100 PLUS

QUESTIONS & PROBLEMS BASED ON HOTS 22 PLUS

NCERT EXERCISES, EXAMPLES & EXEMPLAR PROBLEMS 42 PLUS

MULTIPLE CHOICE QUESTIONS 108 PLUS


F1 F2

A 2b
P S
S

C
2a

Fig. 8.1. Ellipse traced out by a planet


around the Sun
8.3
8.4 PHYSICS (Vol. II)—XI

P F1 O F2 A

Fig. 8.2. Drawing an ellipse. A string has its ends fixed


at F1 and F2. The tip of a pencil holds the string
taut and is moved around.

S2

A2

A1 A3 S3
S1
SUN

Fig. 8.3. Law of areas

 

 
     
 
r 

A
 r
 

 
 
  
   
GRAVITATION 8.5

B A
O

D
Fig. 8.4. Ellipse

 

é pù
    ê ú
ë û
p é p ù
 ê ú
ëê úû

+

-
8.6 PHYSICS (Vol. II)—XI

p
  
p
  

æ pö p
 çè ÷ø 
2
T


3
r

p p Fig. 8.5
  

LM p OP

MN PQ

 

    


A 2b
P S
S

C
æ ö 2a
 ç ÷ 
è ø
Fig. 8.6
GRAVITATION 8.7



 ´ ´ ´
  ´  
´ ´ ´

´ ´ ´
 
´ ´ ´


æ ö
ç ÷ø  ´
è

é ù é ù
ê ú ê ú
ë ´ ´ û ë û

 

´ ´
 
´
8.8 PHYSICS (Vol. II)—XI

p

p é ù
ê ú
p ëê p ûú
 

  
 -
    

 m2
 –F
m1 F
  r

 Fig. 8.7. Law of gravitation


GRAVITATION 8.9


8.10 PHYSICS (Vol. II)—XI

 

 

r12
A B
r12
r21
  A B
r21 = – r12
A B
 F12 F21

(– F21 )
 
Fig. 8.8. Vector form of law of gravitation

 

 

  

 
  

 

-
    
-
   
GRAVITATION 8.11

-
 -
  

-
   

-
   

 

-
   -

-
  -
 

8.12 PHYSICS (Vol. II)—XI

  

    
 + + + 

F1

F3 F2

  -

Fig. 8.9 M1 m M2

 d
Fig. 8.10


-

æ - ö - 
 çè ÷ø  
é ù
+ ê ú
ë û + æ ö
    ç ÷
è ø
GRAVITATION 8.13

r
y

3r 2r
A m

2m 0 x
30
Fig. 8.11 G
B C
m m
Fig. 8.13
dm
r

    dF cos 
dF
3r  
2r
 
  
M   
   
Fig. 8.12

  
   


 

   
  
   
    
 
  

   

8.14 PHYSICS (Vol. II)—XI

mg R

Fig. 8.14

   
 -

GRAVITATION 8.15

 p 

    p  p  
p

 
  -
    

  

-
   

 
 -

  
 -
 

 p 
p

 p   p  p


p

 

  p
 

 é ù
  ê   ú
 ë û
GRAVITATION 8.17

 
     
 

 

 

 
   

 
     


   

 
 



 
   

   

  
GRAVITATION 8.17

g (in m s
æ ö
= - = ç - ÷
è ø

= -

 

æ ö
= ç - ÷ø
è

æ ö
= çè - ÷ø = -

-
- = - =

9.8 m s
æ ö
=ç ÷ø
è

=
=
æ ö
 =ç ÷ø =
è

= = - =
=
=  
8.18 PHYSICS (Vol. II)—XI

=
+


+  +

r d

R-d

  r R
= =

=  r Fig. 8.16. Variation of g with depth


 =
-

=   - r =  - r
-

 - r 
-
= =
 r

æ ö
= - = çè - ÷ø
g

- =- - = R
O
Distance r from centre of Earth.
-
Fig. 8.17. Variation of g with r


r= = = 

=  -    - r
GRAVITATION 8.19

 - 

æ ö
= çè - ÷ø

æ ö
 = çè - ÷ø =

æ ö æ ö
çè - ÷ø çè - ÷ø

= - = -  - =-

= =  g
+
2
r

g  1/r
g

g/4
- g/9
= =  g/16
O R 2R 3R 4R r
Fig. 8.18. Variation of g with
distance r from the centre of Earth

=
8.20 PHYSICS (Vol. II)—XI

æ ö æ ö =   =
çè - ÷ø çè - ÷ø


= =

AXIS OF
ROTATION
OF EARTH
EQUATORIAL
NORTH
PLANE
P


O

SOUTH

Fig. 8.19. Latitude

Rp

Re O

Fig. 8.20
 

®
GRAVITATION 8.21


N

2
P 
C   A
 0 °–
g 18


m

mg



O
O
   
   

é æ ö ù
ê +  
ú S

ê ç ÷ - ú
è ø
ë û Fig. 8.21. Effect of rotation on g

é æ  ö  ù r
  ê + - ú
ê ç ÷ ú 
è ø
ë û
R r
cos  = —
R

Fig. 8.22




   
 

  

é  ù
  ê - ú
ëê ûú

é  ù é  ù
 ê - ú = ê - ú
êë úû êë úû

  

  

  
 
8.22 PHYSICS (Vol. II)—XI


=



æ  ö
 ç - ÷
è ø

  

æ  ö
 ç - ÷= - 
è ø
=   
 

   
=
   

 
= =
   


=
  


   

æ  ö
 ç - ÷
è ø

 

  

æ  ö
ç - ÷
è ø


=
GRAVITATION 8.23

M
F m

P
r
® ®
Fig. 8.23. Gravitational field strength

®
®

® ®

®
® ® ®
 = =
8.24 PHYSICS (Vol. II)—XI

 
 
  

 


b g
 

P
 
m  
0.

20
15

0.
m


A B         
800 kg 0.25 m 600 kg

 
Fig. 8.24     
GRAVITATION 8.25

M
A B

Fig. 8.25. Relation between V and

 =-

O B A 
P
r

Fig. 8.26. Gravitational potential



=
8.26 PHYSICS (Vol. II)—XI

= =

= z z =

= z

LM OP
= z 
-
=
MN -
- +

+ PQ 

é ù é ù
=- ê ú =- ê - ú
ë û ë û
LM =
OP
=-
N  Q

 =-

=-

M
m
O 
P B A
r

Fig. 8.27. Gravitational potential


= energy

= =

= z

= z 
= 
-
=
é - + ù
ê ú
ëê - + úû 
é ù é ù
- ê ú =- ê - ú =-
ë û ë û
GRAVITATION 8.27

æ ö
=- = ç- ÷ø 
è

M R
=-
+

-
Fig. 8.28. Gravitational potential
æ ö energy at a height h above the
 =- =- ç + ÷ surface of Earth
æ ö è ø
ç + ÷
è ø



æ ö
=- ç - ÷ - +
è ø

=  =- +

- =

=-

=-
+
8.28 PHYSICS (Vol. II)—XI

 
LM OP  LM  OP  
N  Q N Q 


LM 
OP  
LM OP
N  Q b  g b  g N

Q
 
 

 

 
     
  

    

   

   
      
 

Moon P

3R 

Fig. 8.29





 

 
   
GRAVITATION 8.29


      



m m

2r
r

 2l l

   m
r

 
     m m
é ù é ù
Fig. 8.30  ê ú  ê ú Fig. 8.32
ë û ë û

é ù
 ê  ú
ë û


é ù
 ê  ú
ë û

é ù
ê ú
ë û

 


A
5m 6m
3m

3m

1
4m 3 3m 
O
2 3

30°
 B C
Fig. 8.31 l
2
Fig. 8.33
 
 

 
é ù
 ê ú    
ë û
8.30 PHYSICS (Vol. II)—XI

 
é ù
   ê  ú
ë û

 
  

 

 
  
 


R
 M R m
4M
O N C

r
6R

Fig. 8.34
  


é  ù
   ê ú
ëê  úû

é ù é  ù
 ê  ú ê ú
ë û ë û

  
é ù
  ê  ú
ë û


 

  
GRAVITATION 8.31

   

   
     
  

dx

Fig. 8.35. Escape velocity

 


 


 



é   ù
    ê ú
ëê   úû

 
é  ù é ù é ù é ù
 ê ú  ê ú
 ê 
ë
ú
û ê   ú
ëê  úû ë û ë û
8.32 PHYSICS (Vol. II)—XI

  

 

  

   

 
 
GRAVITATION 8.33



 

  

 



 

   

   
8.34 PHYSICS (Vol. II)—XI

   

 
  
 


   
 


  


    
 

  


  
 

   


    
 

   
    


GRAVITATION 8.35

é ù  
  ê  ú   
ëê ûú


   
   
  
  


  
8.36 PHYSICS (Vol. II)—XI

A1
A2
A3

ORBIT

Fig. 8.36. Launching of artificial


satellite
GRAVITATION 8.37

R
r

Fig. 8.37. Orbital velocity

  

 

 

 

    

  
8.38 PHYSICS (Vol. II)—XI



é ù
  ê  ú
êë úû

 

é ù
 ê  ú
ë û

 

  

é ù é ù
ê ú ê ú 
êë  úû êë  úû
GRAVITATION 8.39

 

  


    
  
  


m
    

 
2R

m
 Fig. 8.38


   
 


 


é ù
ê ú 
ëê  ûú

  
 
é  ù
   [
 ê ú
êë  ûú

   
 

8.40 PHYSICS (Vol. II)—XI

36,000 km

Fig. 8.39. Geostationary satellite orbiting


the Earth

CHECKPOINT

é ù
ê ú
ëê  ûú

é  ù
ê  ú
ëê  ûú 


GRAVITATION 8.41




8.42 PHYSICS (Vol. II)—XI

Energy

K(r)

0 r
    E(r)
U(r)

 
E=K+U

Fig. 8.40. Variation of kinetic energy K,


potential energy U and total energy E
 with distance r for a satellite moving in
a circular orbit about a massive central
body. For any value of r, the values of U
and E are negative, the value of K is
positive, and E = – K. As r , all the
three energy curves approach a value
é ù of zero
  ê ú  
ë û

 

CHECKPOINT

2
1

P
r

Fig. 8.41
GRAVITATION 8.43

é ù é ù
 ê ú  ê ú
ë û ë û

  é ù
       
 ê  ú
ë û

é ù é ù
  ê ú  ê ú  
ë û ë û

é ù é ù
 ê  ú   ê ú
ë û ë û



é ùé ù
  ê úê ú  
ë ûë û

 
 
 

 
  
   

é ù é ù
 ê ú  ê ú  
ë û ë û

      

 
  





 


 
    
  
   
 

  
   
8.44 PHYSICS (Vol. II)—XI


 
   
   




 
  
 


GRAVITATION 8.45

VERY SHORT ANSWER QUESTIONS—Each Question Carries 1 Mark Only

 


8.46 PHYSICS (Vol. II)—XI

 


GRAVITATION 8.47


SHORT ANSWER QUESTIONS—Each Question Carries 2 Marks

 
8.48 PHYSICS (Vol. II)—XI

  


SHORT ANSWER QUESTIONS—Each Question Carries 3 Marks

  

 
GRAVITATION 8.49

D D

  
(1) (2)

D
D

(3) (4)

Fig. 8.42






8.50 PHYSICS (Vol. II)—XI

–2
g = 9.81 m s
due to gravity (g)
Acceleration

Radius of Earth

Fig. 8.43

m1

m m O m m X

Fig. 8.45

Ep
O
e=0 O
0.5 r
0.8
0.9
M

Fig. 8.46

Fig. 8.44
GRAVITATION 8.51

 


1 6 9
2M 4M
M

7M 2 10 5M
7
3M 3
M
5M 4 11 7M

M
4M 2M
5 8 12

Fig. 8.47

 
  

 
   m 3m

Fig. 8.48
 

 
   
 

   

LONG ANSWER QUESTIONS


8.52 PHYSICS (Vol. II)—XI

 
 
 
GRAVITATION 8.53

LM  
OP
N Q

500 kg 300 kg

100 kg 500 kg

Fig. 8.49
8.54 PHYSICS (Vol. II)—XI

 
  

 
  

    
 
 
GRAVITATION 8.55

0.06 m

6.40 kg Q 6.40 kg

Fig. 8.50

   
   
   
8.56 PHYSICS (Vol. II)—XI

 
 
 

 

 
 
 

 
 
 

 
 
  
GRAVITATION 8.57

 
   

 
  
 
8.58 PHYSICS (Vol. II)—XI

      
       
 

   
  
  
 

 
 

cm /2
   
10

6 cm
 
    
  
 
8 cm 8 cm 
     
  
     
   

 
   


   
   
    
       
 
  
   
   
 


    
   

 
      

  


    
  
   



 

  


     

GRAVITATION 8.59


  


 
  

  
 

 
 


  
 

  


 
    
  


    
   

     

     



 

 
 

  

 

  
     
 
   
    
    
 

   
8.60 PHYSICS (Vol. II)—XI

 
     
      
   
   


 
     
  
   
 

P. 1. Two bodies of masses m1 and m2 are placed at sphere. The sphere (with cavity) exerts a gravitational
a distance r apart. Show that at the position where the
gravitational field due to them is zero, the potential is force F2 on the same particle at P. Calculate the ratio

given by
Solution.  

Solution. =

  
 
or = or =
 
    
 
 


   
 
  
     
     

   
     
P. 2. A solid sphere of uniform density and radius R     
exerts a gravitational force of attraction F1 on a particle 
placed at P. The distance of P from the centre of the sphere 

is 2R. A spherical cavity of radius is now made in the



GRAVITATION 8.61
P. 3. A body is projected vertically upwards from
the surface of the Earth so as to reach a height equal to the
radius of the Earth. Neglecting resistance due to air,
calculate the initial speed which should be imparted to
the body.

Solution. Work done in moving a body from the surface


of the Earth (x = R) to a height h = R(x = 2R) is given by  

  
     

 
     
  
  
 


=  s = 5060 s.
  
P. 5. A planet of mass M moves around the Sun along
 an ellipse so that its minimum distance from the Sun is
equal to r and the maximum distance equal to R. Making
use of Kepler’s laws, find its period of revolution around
the Sun. Take mass of Sun as Ms .
 


Solution. Semi-major axis = = x (say)

P. 4. Imagine a tunnel dug along a diameter of the


Earth. Show that a particle dropped from one end of the
  
tunnel executes simple harmonic motion. What is the time   
period of this motion? Assume the Earth to be a sphere of
uniform mass density (equal to its known average density) 

= 5520 kg m–3, G = 6.67 × 10–11 N m2 kg–2. Neglect all dam-
ping forces.
Solution. The acceleration due to gravity at a depth d
below the free surface of Earth is given by

  
  
    
   
  
R
P. 6. A body is projected vertically upwards from

the bottom of a crater of Moon of depth , where R is the

radius of Moon, with a velocity equal to the escape velocity


Fig. 8.51
on the surface of Moon. Calculate maximum height
attained by the body from the surface of the Moon.
Solution. Escape velocity at the surface of Moon,


ve =

8.62 PHYSICS (Vol. II)—XI
Find the semi-major axis, eccentricity and period of the
orbit. Given : mass of Sun = 1.99 × 1030 kg and 3.156 × 107 s
= 1 year. [Ans. 2.67 × 109 km, 0.967, 75.4 years]

Hint. The length of the major axis equals the sum of the
comet-Sun distance at perihelion plus the comet-Sun distance

at aphelion. The length of the major axis is 2a.





P. 7. Three particles, each of mass m, are situated at
the vertices of an equilateral triangle of side a. The only
forces acting on the particles are their mutual P. 10. To what latitude does the SYNCOMS
gravitational forces. It is desired that each particle moves (Communication satellite) coverage extend ? What is the
in a circle while maintaining the original mutual orbital speed of a SYNCOMS ? Given : RE = 6.37 × 103 km
separation a. Calculate the initial velocity that should be and distance of SYNCOMS from the centre of Earth = 4.22
given to each particle and also the time period of the × 104 km. [Ans. 81.3°, 3.07 × 102 m s–1]
circular motion. Hint.

   P
 
 

Hint. F2 =    O  S
Equator
C

Earth

FAC
Fig. 8.53
o
60
A B
FAB

Fig. 8.52

 
  



P. 8. If a satellite is revolving around a planet of
mass M in an elliptical orbit of semi-major axis a, show
that the orbital speed of the satellite when it is at a distance
r from the focus will be given by
   
  
 
  P. 11. Sun and the Earth revolve around a common
centre of mass with common period of revolution T. If M1
Hint. mv2 – =– . and M2 are the masses of the Sun and Earth respectively
and a is the separation between them, then prove that
P. 9. Comet Halley moves in an elongated elliptical 
orbit around the Sun. At perihelion, the comet is 8.75 × 107 km  
from the Sun ; at aphelion, it is 5.26 × 109 km from the Sun.
GRAVITATION 8.63
Solution. It has been found that both the Sun (mass
M1) and the Earth (mass M2) revolve around a common centre of
mass O and thus, form a binary system. Let a1 and a2 be the radii
of revolution. Then by definition of centre of mass,
 
 

 

 

 
 
  



  

M 1 a1 a2 M2 
O 
 
a 


Fig. 8.54

NCERT EXERCISES (With Solutions)

 
  
 
8.64 PHYSICS (Vol. II)—XI

 
 
 

 
 
 


 
  

 
  

   
 
 

   
     
   
      
 
 


NCERT EXERCISES (UNIT VI) 8.65

g b  
P
f C a
d
e c

 
   
 

Fig. 8.55    

  

  

   
   
   

    

S


r P
  x
  
  
E
     
Fig. 8.56       
      
   
 

 

  

 

 
8.66 PHYSICS (Vol. II)—XI



   
         


  

  
 

  

  
   



  

  

 


  
 


    
   
        


  
       
     


NCERT EXERCISES (UNIT VI) 8.67

 
   
   


   

    


 


     
  


  


 




    



  

  
 
  
8.68 PHYSICS (Vol. II)—XI

 
  
  

    
    
   

  
    
 


    

     

 

  

  

 
 


  
  


 
   

 
 
   
    

       
NCERT EXEMPLAR PROBLEMS (UNIT VI) 8.69

NCERT EXEMPLAR PROBLEMS (With Answers)


[Based on Higher Order Thinking Skills].
® ®
MCQ I (Only one option is correct.)
 


8.70 PHYSICS (Vol. II)—XI

A B C
2M m M
Fig. 8.57

´
-

MCQ II (Any number of options may be


correct.)

® ®

® ®

®
® ® æ ö
=- =- ç ÷
+ è ø


    ® ® ®

= -
NCERT EXEMPLAR PROBLEMS (UNIT VI) 8.71

VSA

Areal
Velocity
O R r

Fig. 8.59
t

Fig. 8.58

® ® ® ® ® ® ® ®
´ ´ ´
= = = =

® ®
´
® ®
8.72 PHYSICS (Vol. II)—XI

SA

(a) (b)

(c) (d)

- +

10 R

M M
m R

5R 5R
(f)
Fig. 8.60

(e)

Fig. 8.62

=- =-

KE æ ö æ ö
çè - ÷ø - çè - ÷ø =

TE R =
PE

Fig. 8.61
NCERT EXEMPLAR PROBLEMS (UNIT VI) 8.73

LA

P 
o h m

M
p p
 = 
Fig. 8.63

´
¢=
+ +

= = -
+

´ =
¢ =
+

¢
= ¢=

8.74
GRAVITATION (UNIT VI) 8.75

 
 

 
 


 

 

 

 
8.76 PHYSICS (Vol. II)—XI

 
  
 

 
  
 

 
 
 

 

 

   
    
   

  
  
 

GRAVITATION (UNIT VI) 8.77

 
 
 

 
 
 
8.78 PHYSICS (Vol. II)—XI

   
 
   
   

   
     
   
GRAVITATION (UNIT VI) 8.79

 


 

    
  


   

    
   
    
8.80 PHYSICS (Vol. II)—XI


  

   
     
   

   
     
   
GRAVITATION (UNIT VI) 8.81

   
   
     
8.82 PHYSICS (Vol. II)—XI

 


 

 


GRAVITATION (UNIT VI) 8.83

Earth

   

   
8.84 PHYSICS (Vol. II)—XI

 
 
 
GRAVITATION (UNIT VI) 8.85

   
   
   

   
   
   
8.86 PHYSICS (Vol. II)—XI

E E
R O
O R
r r

Energy
P
r
R
S

E E
R O
O
r R r
GRAVITATION (UNIT VI) 8.87

     
 
 

  
 

 



8.88 PHYSICS (Vol. II)—XI

   


  

 

  

 
 


  


    
 
  

 
   
  
  
 
 
 

       
     
    
  


GRAVITATION (UNIT VI) 8.89

  
  
 

   
     
   

 
 
 
 
   
   
 

 
    


 
 
 
 
   
   
 



 
   
 
  
 


8.90 PHYSICS (Vol. II)—XI

 
    
 
  
  
   



 
  
  

 
 
  

   
 
 

  

 

 
 
  


 
   
    
 

 



GRAVITATION (UNIT VI) 8.91


 
   
     
 

 

 
   

  
        
          

 


   

   

   
   

 
      

 
  
 
 
 
     

 
    
  
 
 
   
   

 
      
   

 
  
  

 
    
 



8.92 PHYSICS (Vol. II)—XI

+ + ¢= æ ö

çè ÷ø

 + ¢=

= =
 + = æ ö
çè + ÷ø
 +

 æ ö æ ö
çè + ÷ø = + çè - ÷ø =
p p
=
 + 
= = = =
+
  


 = - =-
æ ö
-  = çè ÷ø =-

p
GRAVITATION (UNIT VI) 8.93

4.
UNIT—VII
PROPERTIES OF BULK MATTER
24 Periods

Chapter–9: Mechanical Properties of Solids

Chapter–10: Mechanical Properties of Fluids

Chapter–11: Thermal Properties of Matter

SOLVED NUMERICALS 110 PLUS

UNSOLVED NUMERICALS 220 PLUS

VERY SHORT, SHORT & LONG ANSWER QUESTIONS 350 PLUS

QUESTIONS & PROBLEMS BASED ON HOTS 165 PLUS

NCERT EXERCISES, EXAMPLES & EXEMPLAR PROBLEMS 185 PLUS

MULTIPLE CHOICE QUESTIONS 260 PLUS


9.3
9.4 PHYSICS (Vol. II)—XI

Fig. 9.1. Spring-ball model for the


illustration of elastic behaviour of solids
MECHANICAL PROPERTIES OF SOLIDS 9.5

F F

Fig. 9.2. Tensile stress

F F

l

Fig. 9.3. Compressive stress


9.6 PHYSICS (Vol. II)—XI
Top face

Fixed face
Fig. 9.4

V

Fig. 9.5

  
 

 


  
  

  
      
 
MECHANICAL PROPERTIES OF SOLIDS 9.7


l
E E F F

l 
F
D C C
D

l 

G H

A B
    Fig. 9.6. Shear strain


9.8 PHYSICS (Vol. II)—XI

SUPPORT

SPRING
POINTER
0

SCALE
10 PAN
20
30 WEIGHT

TABLE

Fig. 9.7. Experimental verification


  l
A

l

Fig. 9.8. Young’s modulus of


elasticity
MECHANICAL PROPERTIES OF SOLIDS 9.9



  

 L
  2
 G
L

Mg
 
  
Fig. 9.9

 

  

 

    
 


9.10 PHYSICS (Vol. II)—XI

 
 


  
   

 
  
  

 






    
   
   

    
   
    

    
   
    
 
 

   
 

  
 

  
  
 
  


MECHANICAL PROPERTIES OF SOLIDS 9.11



 

 
  
 

 
 Fig. 9.10. Bulk modulus of elasticity


  

 
 
 




9.12 PHYSICS (Vol. II)—XI

    
 
 

 


 

   

  
 

    

   

 
 



  


   

l F
D D C C

 l  
F
A B
Fig. 9.11. Modulus of rigidity
MECHANICAL PROPERTIES OF SOLIDS 9.13


 

 


 
 
 
 
 

 
    
   

   
9.14 PHYSICS (Vol. II)—XI

Reference wire

Experimental wire
A B

0
5
10 V
M
15
20

Fig. 9.12. Experimental


arrangement
Elastic limit
or yield point D
Su
E
Fracture point
Sy
A B C Plastic behaviour
Proportional limit

Stress
Hooke’s Law region

Permanent set
O O 30%
<1% Strain
Fig. 9.13. Stress-strain curve


MECHANICAL PROPERTIES OF SOLIDS 9.15

1.0
Stress (10 N m )
–2
6

0.5

0
0 0.5 1.0
Strain
Fig. 9.14. Stress-strain curve for the
elastic tissue of Aorta, the large tube
(vessel) carrying blood from the
heart.
9.16 PHYSICS (Vol. II)—XI

 
   
 

L
Fig. 9.15. A test specimen, used to determine
a stress-strain curve.
The change L, that occurs in a certain length L, is
measured in a tensile stress-strain test.




MECHANICAL PROPERTIES OF SOLIDS 9.17

b
(a) (b) (c) (a) (b)

Fig. 9.17. (a) A pillar with rounded ends


Fig. 9.16. (a) Rectangular section of a bar
(b) A thin bar and how it can buckle (b) Pillar with distributed ends
(c) Commonly used section for a load bearing bar


  


 

  

   +


p p   
p

 +
>  > 
 

 p
9.18 PHYSICS (Vol. II)—XI

ing
as
re
nc
a di g
Lo s in

Load
a
e c re
a dd
Lo
Extension
Fig. 9.18. Elastic hysteresis

L
L + L

D – D
Fig. 9.19




 






  
    
  

MECHANICAL PROPERTIES OF SOLIDS 9.19

 
    

p p + p

p p
 +  +
p p
   
     


    


  + 
  


 
   

    

 



 
 
 





9.20 PHYSICS (Vol. II)—XI


  
       



 

 
          

 
   

 

 

  
 

   
 

  


MECHANICAL PROPERTIES OF SOLIDS 9.21



VERY SHORT ANSWER QUESTIONS—Each Question Carries 1 Mark Only

A n s.
9.22 PHYSICS (Vol. II)—XI

SHORT ANSWER QUESTIONS—Each Question Carries 2 Marks


 p



p p
Stress

Stress

A B
C
E A
P B
Strain Strain
Stress

Fig. 9.21

O O Strain

Fig. 9.20
MECHANICAL PROPERTIES OF SOLIDS 9.23

SHORT ANSWER QUESTIONS—Each Question Carries 3 Marks

8
3 × 10

8 Steel
2 × 10
Stress (Pa)

8 Aluminium
1 × 10

0
0 0.0002 0.0004 0.0006 0.0008 0.0010 0.0012 0.0014
Strain
Fig. 9.22
+
 

F I
GH JK 

F IF I
  GH JK GH JK 
9.24 PHYSICS (Vol. II)—XI

RUBBER (A) RUBBER (B)

STRESS

STRESS

STRAIN STRAIN

Fig. 9.23

z z  
z 
LM
MN
OP
PQ 


  
  

   

P
P P P

l
Q l l l
0 0 0
O0 0
F 0 F 0 F 0 F

Fig. 9.24
P
P

l
l
0
0 0
0 F
F

Fig. 9.25
MECHANICAL PROPERTIES OF SOLIDS 9.25



   


  

 


1 kg
 2 kg

Fig. 9.26

 

  
 

  

   

F/N

40 P

30
20
10
O
0 1 2 3 4 5 /m
Fig. 9.27
9.26 PHYSICS (Vol. II)—XI

12

)
–2
Stress (10 N m
10
A
8

7
6
 
      4
2
O 2 4 6 8 10 12
–4
Strain (10 )

Fig. 9.29



   


   

 
   


   




    
  

Fig. 9.28
  

     

  
   
 


MECHANICAL PROPERTIES OF SOLIDS 9.27

LONG ANSWER QUESTIONS

 

9.28 PHYSICS (Vol. II)—XI

 




MECHANICAL PROPERTIES OF SOLIDS 9.29




 

     
   
    


 
 

    
 

  
     
  
   

 

 
 
  

 
  
  
9.30 PHYSICS (Vol. II)—XI


  
 

  

 
 
 

     

  

   
  




 

 
    



P. 1. A light rod of length 2 m is suspended from the


 
ceiling horizontally by means of two vertical wires of equal
length tied to its ends. One of the wires is made of steel and
is of cross-sectional area 0.1 sq. cm and the other is of  
brass of cross-sectional area 0.2 sq. cm. Find out the posi-
tion along the rod at which a weight may be hung to pro- 
 
duce (i) equal stresses in both wires (ii) equal strains in
both wires. Given : Y for steel = 20 × 1011 dyne cm–2 and Y

for brass = 10 × 1011 dyne cm–2.

 
Solution.   or 

 
  
Steel Brass  
T1 T2

 

x 2–x
P. 2. A cube of aluminium of each side 4 cm is
W
subjected to a tangential (shearing) force. The top face of
Fig. 9.30 the cube is sheared through 0.012 cm with respect to the
MECHANICAL PROPERTIES OF SOLIDS 9.31
bottom face. Find (i) shearing strain (ii) shearing stress
and the shearing force. Given : G = 2.08 × 1011 dyne cm–2. Solution.   

Solution. l = 4 cm, l = 0.012 cm,

 
   


 
 

   
     
  



    
 
 
 

P. 5. A steel wire has length 2 m, radius 1 mm and


Y = 2 × 1011 N m–2. A 1 kg sphere is attached to one end of the
wire and whirled in a vertical circle with an angular
velocity of 2 revolutions per second. What is the elongation
of the wire when the sphere is at the lowest point of the
vertical circle ?

P. 3. Find the depth of a lake for which the density    


 
of water will be 10% higher than the density at the surface.   
Compressibility of water is 0.00005 per atmosphere.
Given : average density of sea-water = 1100 kg m–3.    
  
   
Solution.     
  
 
  



   
     P. 6. A uniform cylindrical wire is subjected to a
 
longitudinal tensile stress of 5 × 107 N m–2. The Young’s
modulus of the material of the wire is 2 × 1011 N m–2. The

volume change in the wire is 0.02%. Calculate the
 fractional change in the radius.



 
 

     
    
 



       
 

P. 4. If the normal density of sea water is 1.00 g cm , –3 
what will be its density at a depth of 3 km? Given :      
compressibility of water = 0.000048 per atmosphere,    
1 atmospheric pressure = 1.01 × 106 dyne cm–2.
9.32 PHYSICS (Vol. II)—XI

 
  

  
    

P. 7. A material has normal density  and bulk


modulus K. Find the increase in the density of the material
when it is subjected to an external pressure P from all
sides.

  
 

 


 



      P. 9. A wire that obeys Hooke’s law is of length l1
 when it is in equilibrium under a tension F1. Its length
 
becomes l2 when the tension is increased to F2. Calculate
 the energy stored in the wire during this process.
    
   
   

     
   

P. 8. In a human   
pyramid in a circus, the entire
weight of the balanced group 
   
is supported by the legs of a 
performer who is lying on his
back (as shown in Fig. 9.31). 
   
The combined mass of all the 
persons performing the act,
and the tables, plaques, etc.
involved is 280 kg. The mass
  
of the performer lying on his        
back at the bottom of the pyra-   
mid is 60 kg. Each thigh-bone
(femur) of this performer has       
a length of 50 cm and an effec-
tive radius of 2.0 cm. Deter-
mine the amount by which Fig. 9.31. Human
each thighbone gets com- pyramid in a circus F2
pressed under the extra load.
Given : Young’s modulus for bone = 9.4 × 109 N m–2. F1
Force

l1 – l l2 – l
Extension
Fig. 9.32


NCERT EXERCISES (UNIT VII) 9.33
P. 11. A wire of radius r stretched without tension
along a straight line is lightly fixed at A and B (Fig. 9.33).
  What is the tension in the wire when it is pulled into the
shape ACB ? Assume Young’s modulus of the material of
the wire to be Y.
  
         
   

A l l
B
    

   C

Fig. 9.33
P. 10. A copper wire 2 m long is stretched by 1 mm. If
the energy stored in the stretched wire is converted to heat, Hint. Extension = (AC + CB) – AB
calculate the rise in temperature of the wire. (Given:
Y = 12 × 1011 dyne cm–2, density of copper = 9 g cm–3 and
 
specific heat of copper = 0.1 cal g–1°C–1) [Ans. 0.004°C]     
 

Hint.
 

 A l l
B



C

Fig. 9.34



 
 

NCERT EXERCISES (With Solutions)



 


 
 
 
 


  
  
9.34 PHYSICS (Vol. II)—XI

A
300

Stress (10 Nm )
–2 250
200
6

150
100
50
0
0 0.001 0.002 0.003 0.004
Strain

Fig. 9.35
1.5 m
Steel
4.0 kg
1.0 m
 Brass
6.0 kg
 
  Fig. 9.37

  
Fig. 9.36 

 
  
   

  
    
 

  
   
   
NCERT EXERCISES (UNIT VII) 9.35

  
L

10 cm
 Fig. 9.39


   
 
  
100 kg

100 × 9.8 N

Fig. 9.38

 

   


   



  
  
 

 
 
 


 
9.36 PHYSICS (Vol. II)—XI

   
   
    

  
  

 


  

  
 


 

 

   

  
 


 




 

 
     
   

 
      



NCERT EXERCISES (UNIT VII) 9.37

Diamond Metal
cones gasket
 


 
Fig. 9.40

 
  
 
 


A B
T T
 
D

mg
 
    Fig. 9.42
 
A B

1.05 m
T cos 
T cos 
 
    T sin  T sin 
 
Fig. 9.41 
mg
Fig. 9.43



    
     
   

 

 
 
 
9.38 PHYSICS (Vol. II)—XI

 
 
 

  
  
    

 

 

NCERT EXEMPLAR PROBLEMS (With Answers)


[Based on Higher Order Thinking Skills].

MCQ I (Only one option is correct.)

 
 
 
  
  

a 
NCERT EXEMPLAR PROBLEMS (UNIT VII) 9.39

2L
D
B C
O

Fig. 9.44

    MCQ II (Any number of options may be


        
correct.)
   


Ultimate Tension
Strength Fracture Point

Stress Linear
Limit

Strain E
Material (i )

Fig. 9.45
Ultimate Tension
Strength
Linear Fracture Point
Limit

Stress

T sin  T sin 
T T
Strain E
  Material (ii )

T cos  T cos 
m Fig. 9.47

Fig. 9.46
 
9.40 PHYSICS (Vol. II)—XI

A L L B

F1 F2

x l–x
m

Fig. 9.49


 

 


 
 


 

A B

steel Al

m 


Fig. 9.48


 
  

 

NCERT EXEMPLAR PROBLEMS (UNIT VII) 9.41

VSA   
      




>  


 
   
 
 


  

 
   


   
  
    
SA




 LA


 
 
a

 Fig. 9.50
a
9.42 PHYSICS (Vol. II)—XI
 
  

   
®  

 

N  
a FN   

/2 – 
O F
 

 FP
a
 
Fig. 9.51  


 
 
    
 
10.1
10.2 PHYSICS (Vol. II)—XI

 



 

 
 
 

 

 


 




 

 
 

  




 

FLUIDS AT REST [MECHANICAL PROPERTIES OF FLUIDS (I)] 10.3

Y P

A X

Fig. 10(a).1. Fluid in equilibrium


h

Fig. 10.(a).2. Pressure due


 to liquid column
10.4 PHYSICS (Vol. II)—XI


  




–3 2
area = 10 m

h = 0.4 m

 –3 2
area = 2  10 m

Fig. 10(a).4

 
 
Fig. 10(a).3
 
FLUIDS AT REST [MECHANICAL PROPERTIES OF FLUIDS (I)] 10.5

O1
O2

Fig. 10(a).5. Pascal’s law

P 1A
1

P 2A

Fig. 10(a).6. Effect of


gravity on fluid pressure
 




  
10.6 PHYSICS (Vol. II)—XI

D B

  A
Fig. 10(a).7. Experimental proof of
Pascal’s law

F1

A2
A1

F2

Fig. 10(a).8. Schematic diagram illustrating


the principle behind the hydraulic lift, a
device used to lift heavy loads
FLUIDS AT REST [MECHANICAL PROPERTIES OF FLUIDS (I)] 10.7

5 kgf LOAD

Fig. 10(a).9. Hydraulic lift

E
AK L
BR D A
MASTER PE
CYLINDER
PIPE LINE TUBE T

P
TO OTHER LEVER
WHEELS SYSTEM
BRAKE OIL

P2 P1
INNER RIM OF
THE WHEEL
WHEEL
S2 CYLINDER S1

Fig. 10(a).10. Hydraulic brake


10.8 PHYSICS (Vol. II)—XI
RIGID CEILING

 F H

P F
C
P1
P2

F2 C2 C1
RV
V1
V2
T1

T2
WATER
RESERVOIR

Fig. 10(a).11. Hydraulic press






 

 
 
   


 
 

A B C

Fig. 10(a).12. Pascal’s vases


FLUIDS AT REST [MECHANICAL PROPERTIES OF FLUIDS (I)] 10.9

V
R
P
R H
H
R
V
(a) (b) (c)

Fig. 10(a).13

Torricellian
vacuum

76 cm 76 cm

Fig. 10(a).14. Torricelli’s experiment


10.10 PHYSICS (Vol. II)—XI

P0

P
A B


Fig. 10(a).15. Open-tube manometer



FLUIDS AT REST [MECHANICAL PROPERTIES OF FLUIDS (I)] 10.11

Atmospheric pressure
1.0 C

 
0.5
A
  
  B
 0 8 16 24
Height
Fig. 10(a).16. Variation of atmospheric
pressure with height

 
 




 
10.12 PHYSICS (Vol. II)—XI

LOAD

1.5 m
OIL

 
 
 
Fig. 10(a).17


 
 



 


 
 


 
 
   
FLUIDS AT REST [MECHANICAL PROPERTIES OF FLUIDS (I)] 10.13

Fig. 10(a).18. Proof of Archimedes’


 principle

  

  

 

 
æ ö æ ö
 ç  ÷ çè   ÷ø
è ø



10.14 PHYSICS (Vol. II)—XI

 

 

 

B B
G G

(a) (b) (c)

Fig. 10(a).19. Stability of floating body


FLUIDS AT REST [MECHANICAL PROPERTIES OF FLUIDS (I)] 10.15

  

 æ ö
çè  ÷ø  

Fig. 10(a).20

 
   
 
   
      
   

  


10.16 PHYSICS (Vol. II)—XI

 

   

 


          

  
  

  
   
  
 


   

  æ ö 
      ç ÷ø  
  è
 

 

  
   
  

 
   
 

SUMMARISING
FLUIDS AT REST [MECHANICAL PROPERTIES OF FLUIDS (I)] 10.17

VERY SHORT ANSWER QUESTIONS—Each Question Carries 1 Mark Only

SHORT ANSWER QUESTIONS—Each Question Carries 2 Marks



10.18 PHYSICS (Vol. II)—XI



 

A B C

Fig. 10(a).21
FLUIDS AT REST [MECHANICAL PROPERTIES OF FLUIDS (I)] 10.19


SHORT ANSWER QUESTIONS—Each Question Carries 3 Marks


 

F1
F2

oil

Fig. 10(a).22

C A B

 

   Fig. 10(a).23
10.20 PHYSICS (Vol. II)—XI

æ  ö
ç  ÷
 è ø
æ  ö
ç  ÷
è ø

æ  ö
ç  ÷
è ø
æ  ö
ç  ÷
è ø
Q. 52. A cube of wood supporting a 200 g mass just
floats in water. When the mass is removed, the cube rises
by 2 cm. What is the side of the cube ?
Answer. When the mass is removed, the volume of the
cube that has come out of water is 2 l2 cm3, where l cm is the
length of each side of the cube. Decrease in upthrust = 2l2 g dyne.
Loss in weight = 200 g dyne. Now, 2l2 = 200 or l = 10 cm.
Q. 53. A penguin floats first in a fluid of density
0 , then in a fluid of density 0.950 , and then in a fluid of
density 1.1 0 . (a) Rank the densities according to the
buoyant force on the penguin, greatest first. (b) Rank the
densities according to the amount of fluid displaced by
the penguin, greatest first.
Answer. (a) All tie (b) 0.95 0, 0,1.10.

 

FLUIDS AT REST [MECHANICAL PROPERTIES OF FLUIDS (I)] 10.21
Q. 54. To what height should a cylindrical vessel be 

filled with a homogeneous liquid to make the force with
  
which the liquid presses on the side of the vessel equal to 

the force exerted by the liquid on the bottom of the vessel ?
Answer. Thrust on the bottom of the vessel  

     
  
  

  
 
  

Q. 55. An office window has dimensions 3.4 m by

2.1 m. As a result of the passage of a storm, the outside air 
 
pressure drops to 0.96 atm, but inside the pressure is held   

at 1.0 atm. What net force pushes out on the window ?
Q. 58. A tank containing water is placed on a spring
scale which registers a total weight W. A stone of weight w
Answer. P = or F=P×A is hung from a string and is lowered into water without
touching the sides or bottom of the tank. What will be the
reading on the spring scale ?
Answer. First, for the stone alone, the following forces
act.

Q. 56. A string is holding


a solid block below the surface
of a liquid. The density of liq-
uid is greater than that of
solid. The tension in the string
is T0 , when the system is at rest.
What will be the tension in the
string if the system has upward
acceleration a ? Fig. 10(a).24

Answer. Let V be the volume of block. Let  and  be the


densities of the block and liquid respectively.
 
 

 
  
  Q. 60. A piece of brass (Cu and Zn) weighs 12.9 g in
Q. 57. An object of density 12 g cm–3
is weighed with air. When completely immersed in water, it weighs 11.3 g.
brass weights of density of 8 g cm–3 by a physical balance. The relative densities of Cu and Zn are 8.9 and 7.1 respec-
If the density of air is 1.2 × 10 –3 g cm–3, then calculate the tively. Calculate the mass of copper in the alloy.
percentage error in weighing. Answer. Loss of weight = (12.9 – 11.3) gf = 1.6 gf
Answer. When the beam is horizontal,

       
   
 





10.22 PHYSICS (Vol. II)—XI

Q. 61. A jeweller claims that he sells ornaments made


of pure gold that has the relative density of 19.3. He sells a
bangle weighing 25.250 gf to a person. The clever customer
weighs the bangle when immersed in pure water and finds
that it weighs 23.075 gf in water. Is the ornament made of Q. 64. The manual of a car instructs the owner to
pure gold ? inflate the tyres to a pressure of 200 kPa. (a) What is the
recommended gauge pressure ? (b) What is the recom-
Answer. Weight of ornament in air = 25.250 gf mended absolute pressure ? (c) If, after the required infla-
tion of the tyres, the car is driven to a mountain peak where
the atmospheric pressure is 10% below that at sea level,
what will the tyre gauge read ?
Answer. (a) When the manual says pressure, it means
gauge pressure Pg. Thus

 

Q. 62. Fig. 10(a).25 shows four containers of olive 


oil. Rank them according to the pressure at depth h, great-
est first.

Q. 65. A spring balance reads 10 kg when a bucket of


water is suspended from it. What is the reading on the
spring balance when
Fig. 10(a).25
Answer. All tie.
 

Q. 63. A tube 1 cm2 in cross-section is attached to the


Answer. (i) When the ice cube is put in the bucket, then
top of a vessel 1 cm high and of cross-section 100 cm2. Water
the total mass of the system suspended from the spring balance
is poured into the system filling it to a depth of 100 cm
will be 10 kg + 1.5 kg, i.e., 11.5 kg. So, the balance will read
above the bottom of the vessel as shown in Fig 10(a).26.
11.5 kg.
2
1 cm

99 cm





1 cm

Fig. 10(a).26

Answer. (a) P = 100 cm × 1 g cm–3 × 1000 cm s–2 


FLUIDS AT REST [MECHANICAL PROPERTIES OF FLUIDS (I)] 10.23
Answer. Weight must be equal to the upthrust.

      

Q. 66. In order to determine the density of a small 


marine animal, benzene and chloroform were mixed in a
proportion of 78% and 22% by volume respectively. The
animal was observed to remain suspended in the resulting
mixture. Calculate the average density of the animal. Given :
density of benzene = 879 kg m–3; density of chloroform =
1527 kg m–3.

LONG ANSWER QUESTIONS



Water
main

100 m

Fig. 10(a).27
 
 
 
10.24 PHYSICS (Vol. II)—XI

 
 
  

  
 
 

Fig. 10(a).28
FLUIDS AT REST [MECHANICAL PROPERTIES OF FLUIDS (I)] 10.25





 
  





 


   



   

 
   

   
     
   

  
 
10.26 PHYSICS (Vol. II)—XI

   


  

V = V
   

 
       
 

 
 

 

 
  


P

    40 cm
60°

 
O

 
 


 

P. 1. A block of cedar wood has a mass of 200 kg and Solution. The wood will float when it displaces a weight
a density of 570 kg m–3. What fraction of the wood will be of water equal to its own weight, which is equivalent to saying
under the surface when the block is floating in water that the wood and the displaced water have the same mass.
(density = 1000 kg m–3) ?
FLUIDS AT REST [MECHANICAL PROPERTIES OF FLUIDS (I)] 10.27

P. 2. A car has a total mass of 1800 kg. The volume


of air space in the passenger compartment is 5.00 m3. The
volume of the motor and front wheels is 0.750 m3, and the
volume of the rear wheels, gas tank and trunk is 0.800 m3.

Water cannot enter these areas. The car is parked on a hill ; 
the hand brake cable snaps and the car rolls down the hill
into a lake (Fig. 10(a).29). (a) At first, no water enters the
passenger compartment. How much of the car, in cubic
metre, is below the water surface with the car floating as
shown ? (b) As water slowly enters, the car sinks. How

many cubic metre of water are in the car as it disappears
below the water surface ? (The car remains horizontal,
owing to a heavy load in the trunk.)

 
5.00 m
3   
0.800 m
3  
3
0.750 m
P. 4. What is the pressure at the centre of the Earth
and at the centre of the Sun ? Assume they are spheres of
constant density where Re = 6.36 × 106 m, Rs = 6.95 × 108 m,
Fig. 10(a).29 e = 5.52 × 103 kg m–3 and s = 1.42 × 103 kg m–3. The
acceleration due to gravity at the surface of the Sun is
274 m s–2.

Solution. The pressure at a depth h is P =  h where


is the average acceleration due to gravity. For a depth h = R,
P= R.

P. 3. An ice cube, of mass ‘ m ’ gram, containing a


piece of iron, of mass ‘ M ’ gram and density ‘D’ g cm–3,
floats in water in a beaker. What will be the effect on water
level when the ice melts?
Solution. When the ice cube floats,


10.28 PHYSICS (Vol. II)—XI
P. 5. Calculate the absolute and gauge pressure O
of gas above the liquid surface in the tank shown in 
Fig. 10(a).30. Density of oil = 820 kg m–3, density of mercury
= 13.6 × 103 kg m–3 ; 1 atmospheric pressure = 1.01 × 105 Pa. 
 h1
H
h2

GAS
 A d B

1.50 m
OIL L
 O
 Fig. 10(a).32
MERCURY

Fig. 10(a).30

Solution. The points A and B of the mercury column in
the U-tube are at the same level. So, they are at the same pres-
sure.

 


 

P. 7. A uniform solid cylinder of density 0.8 g cm–3
floats in equilibrium in a combination of two non-mixing
liquids A and B with its axis vertical. The densities of the
 liquids A and B are 0.7 g cm–3 and 1.2 g cm–3 respectively.
The height of liquid A is hA = 1.2 cm. The length of the part

of the cylinder immersed in liquid B is hB = 0.8 cm.

Air h
 
A hA
P. 6. A U-tube of base length L contains a liquid of
density  in it. The tube is rotated about one of its vertical B hB
limbs with angular velocity  as shown in Fig 10(a).31.
Find the difference in height H of liquid in the two limbs.
The diameter of tube is negligibly small as compared to Fig. 10(a).33
length of the tube.

L
Solution. (a) As the pressure at any level in the liquid,
Fig. 10(a).31 acts equally in all possible directions, therefore force exerted by
liquid A on the cylinder will be radially symmetric and hence its
Solution. Due to rotation about the axis OO, a force is
value will be zero.
acting on the liquid column AB outwards, pushing up the liquid
column on the right. This centrifugal force acting on the small
element dx of area of cross-section dx, distant x from the axis is
   
FLUIDS AT REST [MECHANICAL PROPERTIES OF FLUIDS (I)] 10.29

         
          
        

  
    
   
   
  
P. 9. Two identical cylindrical vessels with their
bases at the same level, each contains a liquid of density
 . The height of the liquid in one vessel is h1 and that in
the other vessel is h2 . The area of either base is A. What is
        the work done by gravity in equalising the level when the
   two vessels are connected ?

P. 8. A container of a large uniform cross-sectional


area A resting on a horizontal surface holds two immiscible,
non-viscous and incompressible liquids of densities d and h2
h1 + h2
2d, each of height as shown in Fig. 10(a).34. The lower 2
h1
density liquid is open to atmosphere having pressure P0 .
 
A homogeneous solid cylinder of length L   , cross- Fig. 10(a).35
 
    
sectional area is immersed such that it floats with its     
   

axis vertical at the liquid-liquid interface with length Hint. Initial potential energy of the system,

in denser liquid. Determine   


Ui = Ag  
 
 

  
  
 
d
H/2 P. 10. Assume the density of brass weights to be 8 g
3L/4 cm–3 and that of air to be 0.0012 g cm–3. What fractional
error arises from neglecting the buoyancy of air in weighing
an object of density 3.4 g cm–3 on a beam balance ?
L/4
2d
H/2


Fig. 10(a).34
Solution. (i) According to principle of floatation,

 
  
 

  
  
  
10.30 PHYSICS (Vol. II)—XI

VALUE BASED QUESTION


Fig. 10(b).1

10.31
10.32 PHYSICS (Vol. II)—XI

 = t F
B E C F


A D


(a)

(b)

 Fig. 10(b).2. (a) A layer of liquid sandwiched between


two parallel glass plates in which the lower plate is

fixed and the upper one is moving to the right with

velocity (b) velocity distribution for viscous flow in
a pipe

SURFACE LAYER

FIXED
LAYER
Fig. 10(b).3. Steady flow of liquid
FLUIDS IN MOTION [MECHANICAL PROPERTIES OF FLUIDS (II)] 10.33

Q d
F
dx
P

x
  
FIXED LAYER

Fig. 10(b).4. Newton’s law of



viscous flow

 

 


 
  

10.34 PHYSICS (Vol. II)—XI

film 

 

0.01 kg 

Fig. 10(b).5



 
  
 
 

 
 
  

FLUIDS IN MOTION [MECHANICAL PROPERTIES OF FLUIDS (II)] 10.35

  

  


 



10.36 PHYSICS (Vol. II)—XI

F
U

W
Fig. 10(b).6. Sphere falling in a fluid



  

 

0
       VELOCITY


    

TIME
Fig. 10(b).7. Variation of with t


FLUIDS IN MOTION [MECHANICAL PROPERTIES OF FLUIDS (II)] 10.37

   

 


  


   B


     C

   Fig. 10(b).8. Experimental
determination

 


   


10.38 PHYSICS (Vol. II)—XI




 
 
 

    

 
 

  

  
  


 
  

 
  

     


     
     

 
     
  
    
  

Fig. 10(b).9. Flow of liquid through a pipe


FLUIDS IN MOTION [MECHANICAL PROPERTIES OF FLUIDS (II)] 10.39


 



 
   
 

 
  
 

    
 
      

 
    





10.40 PHYSICS (Vol. II)—XI


h
 l

 OUTFLOW
 TUBE

Fig. 10(b).10. Poiseuille’s method for


determination of 

B
A
V

O p

Fig. 10(b).11. Variation of V with p


 

  

 

FLUIDS IN MOTION [MECHANICAL PROPERTIES OF FLUIDS (II)] 10.41

 
  

   
  

 
 

  

1 2
3
 b c
a
Fig. 10(b).12. Streamline flow

 
10.42 PHYSICS (Vol. II)—XI

Fig. 10(b).13. Flow of non-viscous


liquid

Fig. 10(b).14. Flow of viscous liquid

Fig. 10(b).15. Fluid flow changing from


laminar to turbulent while flowing
past a cylindrical obstacle
FLUIDS IN MOTION [MECHANICAL PROPERTIES OF FLUIDS (II)] 10.43

Fig. 10(b).16. Rising smoke becomes


turbulent after some distance








  
10.44 PHYSICS (Vol. II)—XI

 

 

   

     


   




 


   
  

FLUIDS IN MOTION [MECHANICAL PROPERTIES OF FLUIDS (II)] 10.45

 


 

 
 
 

 

   


10.46 PHYSICS (Vol. II)—XI

 

     
 
   
 




      
    
  
  
   


  
  

  


 
 
 

 
FLUIDS IN MOTION [MECHANICAL PROPERTIES OF FLUIDS (II)] 10.47

B
2
A

Fig. 10(b).17. Equation of continuity




  
  

 


10.48 PHYSICS (Vol. II)—XI

 
 

 

 

 
  
 

 
  
 

 
 

Fig. 10(b).18
Fig. 10(b).20

2 –1 2 –1
0.12 m 3ms 0.12 m 1.5 m s

0.18
m2


 
 
Fig. 10(b).19
FLUIDS IN MOTION [MECHANICAL PROPERTIES OF FLUIDS (II)] 10.49


      

 
    

  
    


 
 

T

h

x
 P

Fig. 10(b).21. Pressure energy

 
 



10.50 PHYSICS (Vol. II)—XI

 

  

    

 



  

P2a2
B
2

A
P1a1 h2
1

 h1

 DATUM LEVEL
Fig. 10(b).22. Bernoulli’s theorem
FLUIDS IN MOTION [MECHANICAL PROPERTIES OF FLUIDS (II)] 10.51

  

   

   

  

 

     
 

         
   

  

  


10.52 PHYSICS (Vol. II)—XI

 

   


    
     

   
  
   
 


FLUIDS IN MOTION [MECHANICAL PROPERTIES OF FLUIDS (II)] 10.53

p1
p2
A 1 B 2

a1 a2
Fig. 10(b).23. Venturimeter

     

    

    




 

    
      
   

 



10.54 PHYSICS (Vol. II)—XI

 

h MANOMETER
TUBE

Fig. 10(b).24. Venturi tube

h
T1 T2

  
A B
H

Fig. 10(b).25. Pitot tube

  

     = -

 

  

L R

Angle of 1 1
attack A

2
2 B

Fig. 10(b).26. Streamlines around an air foil or airplane wing


FLUIDS IN MOTION [MECHANICAL PROPERTIES OF FLUIDS (II)] 10.55

Air

T
 
d d
d d
 
Fig. 10(b).27. Attracted disc paradox
 
B

      h

Fig. 10(b).28. Pressure due to


liquid column


p1

WIND p2

   

Fig. 10(b).29. Blowing of roofs


10.56 PHYSICS (Vol. II)—XI

AIR AIR

GAS

Fig. 10(b).30. Bunsen Burner

Air
Spray

Rubber
Balloon

Fig. 10(b).31. Sprayer

Fig. 10(b).32. Ball in liquid stream

Fig. 10(b).33. Purely spin motion of a ball

Fig. 10(b).34. Linear motion of ball

Fig. 10(b).35. Combination of


linear and spin motions
FLUIDS IN MOTION [MECHANICAL PROPERTIES OF FLUIDS (II)] 10.57

B WATER

P
WATER JET

AIR

VESSEL

Fig. 10(b).36. Filter pump

h
H

O
h

R

Fig. 10(b).37. Torricelli’s theorem

     

 = -   -  =

 = =
10.58 PHYSICS (Vol. II)—XI


   


 = 

= 

  =  

    =    


=

=
FLUIDS IN MOTION [MECHANICAL PROPERTIES OF FLUIDS (II)] 10.59




   =   
- = =

 -  -   -

  
    - 
  
 

- 
=
  

   - 
  
 

=

 =  

  

   

  - -  -

- -
=   =


10.60 PHYSICS (Vol. II)—XI


l
h


h'

 
 
 
R  
Fig. 10(b).38

= 

VERY SHORT ANSWER QUESTIONS—Each Question Carries 1 Mark Only

Fig. 10(b).39

Fig. 10(b).40
FLUIDS IN MOTION [MECHANICAL PROPERTIES OF FLUIDS (II)] 10.61

[... 

SHORT ANSWER QUESTIONS—Each Question Carries 2 Marks


10.62 PHYSICS (Vol. II)—XI

SHORT ANSWER QUESTIONS—Each Question Carries 3 Marks


- -
  

2 5
6

4
4 8

Fig. 10(b).41

1 2
Flow 3

  Fig. 10(b).42

-
 
-
 
FLUIDS IN MOTION [MECHANICAL PROPERTIES OF FLUIDS (II)] 10.63

 
 - 
 
 
 - 
 

   
=  
   
 
=   =

 

  
 
  

 =  = 
=
p1 
-
Wind p2   
   
  

Fig. 10(b).43

 - 

 

-  -    
  
  
 
  -  
 
10.64 PHYSICS (Vol. II)—XI

 - 

 



  = =

LONG ANSWER QUESTIONS

 

FLUIDS IN MOTION [MECHANICAL PROPERTIES OF FLUIDS (II)] 10.65

 

- 






10.66 PHYSICS (Vol. II)—XI


 

2
A 2 = 0.250 cm
2 =?

1.50 m
2
A 1 = 3.60 cm
–1
1 = 50.0 cm s


 Fig. 10(b).44
FLUIDS IN MOTION [MECHANICAL PROPERTIES OF FLUIDS (II)] 10.67


     
 
 
  
 
  



  = 
   

 

  
  


  
 
 
  
 

=
  
=
 
 
=
 
= 
 
 


10.68 PHYSICS (Vol. II)—XI

 

 
  =

=


 =   




=


= =
 
 

  

   =

  

  
 

   = 
  

     


  
=

 
   

 = 
 



  
FLUIDS IN MOTION [MECHANICAL PROPERTIES OF FLUIDS (II)] 10.69

 
 
= 


  
=


A B
C

Fig. 10(b).46

 
    



 

 
D = 8 mm d = 2 mm
  
    
= 
  

1.25 m

Ground

Fig. 10(b).45


 
10.70 PHYSICS (Vol. II)—XI

 


 


 




a b

h B
A

100 cm

Fig. 10(b).48
30 cm

10 cm B
A
   
Fig. 10(b).47

   




 
    =  = =
 


 


   
 
ATTRACTION REPULSION
DISTANCE

50 nm 100 nm

35 nm

Fig. 10(c).1. Variation of intermolecular


force with distance

10.71
10.72 PHYSICS (Vol. II)—XI

B
A

Fig. 10(c).2. Force of surface tension


SURFACE TENSION [MECHANICAL PROPERTIES OF FLUIDS (III)] 10.73


 
 
 

 
 
 
  

Fig. 10(c).3. Contraction


due to surface tension
10.74 PHYSICS (Vol. II)—XI

S S

Fig. 10(c).4. Behaviour of free


surface as stretched membrane

Fig. 10(c).5. Minimum surface area


due to surface tension

DROP

MIXTURE

Fig. 10(c).6. Shape of liquid drop in


the absence of gravity

A S
B
C D
Q

Fig. 10(c).7. Molecular theory


of surface tension
SURFACE TENSION [MECHANICAL PROPERTIES OF FLUIDS (III)] 10.75


l
C D

A B
x
A B

F
Fig. 10(c).8. Surface energy
10.76 PHYSICS (Vol. II)—XI

 

 
 

 




 

 



 

  


    

 
     
 
 
   



SURFACE TENSION [MECHANICAL PROPERTIES OF FLUIDS (III)] 10.77

Fig. 10(c).9. Measuring surface tension

 Q

P 
R
P R

Fig. 10(c).10. Angle of contact

R
Q  Q

P P

Fig. 10(c).11. Angle of contact

Sla


 Ssa Ssl
10.78 PHYSICS (Vol. II)—XI

S la


S sa S sl

Fig. 10(c).12. Different shapes of water


 drops with interfacial tensions (a) on a
lotus leaf (b) on a clean plastic plate.



  


  
P C

   
   Fig. 10(c).13. Experimental
determination of angle of contact

 
SURFACE TENSION [MECHANICAL PROPERTIES OF FLUIDS (III)] 10.79

A F1 O

 F2
45°

R
C

A F1 O

F2
45°
C
R


A F1 O

 
  F2
45°
 
R C

Fig. 10(c).14. Shape of a liquid


meniscus

  
 

   
10.80 PHYSICS (Vol. II)—XI

  

    

Fig. 10(c).15. Capillarity

(Balance of Forces Method) [Application of


surface tension idea]
SURFACE TENSION [MECHANICAL PROPERTIES OF FLUIDS (III)] 10.81

R cos 

R cos 
 R   R
A B
R sin  R sin 
  C
S S
h G

L W L


Fig. 10(c).16. Ascent formula



   

 r

Fig. 10(c).17. Liquid meniscus

   
        
   

    
10.82 PHYSICS (Vol. II)—XI

 
      
 

 
    

 
   
 

 
        
 
   
  
  


 


 


 

   

r

Fig. 10(c).18

 
SURFACE TENSION [MECHANICAL PROPERTIES OF FLUIDS (III)] 10.83


 S S
h
S
R sin    R sin 

R cos 

R cos 
  R R

   


 Fig. 10(c).19. Descent formula

 
  



C C’
R

A r O R’

    h
S
h’


 
 

 

Fig. 10(c).20. Rise of liquid in a
capillary tube of insufficient length

    
10.84 PHYSICS (Vol. II)—XI

 
 
 

  
 
   

    

  
    

 


   
  
   


  

 
 

  
  
  

  
    
    
   
 
 


 
      

  
  
      
     
  
   



 
SURFACE TENSION [MECHANICAL PROPERTIES OF FLUIDS (III)] 10.85

 
   
 
   

 
      
     

 
     
  
   

S S

Fig. 10(c).21

S S
R

EXCESS PRESSURE
Fig. 10(c).22. Excess of
pressure on concave side

EXCESS PRESSURE

R
S S

Fig. 10(c).23. Excess of


pressure on concave side
10.86 PHYSICS (Vol. II)—XI

p
dR

 Fig. 10(c).24. Excess pressure


inside a liquid drop

 
   

 


 

2 R S

 p R
2

Fig. 10(c).25. Upper hemisphere


  

Fig. 10(c).26. Air bubble in a liquid


SURFACE TENSION [MECHANICAL PROPERTIES OF FLUIDS (III)] 10.87

p
 dR

 
  Fig. 10(c).27. Soap bubble

  

B A C

Y X

Fig. 10(c).28. Experimental


demonstration

D ensity,


10.88 PHYSICS (Vol. II)—XI

 


    
 

   
    
   

 



    

  
  
 

      
      
      
SURFACE TENSION [MECHANICAL PROPERTIES OF FLUIDS (III)] 10.89



PA

B
P–2S
R
h

P PA P

P B P


P–2S
R

Fig. 10(c).29. Rise of liquid

r

R

 

  Fig. 10(c).30
   
 

PA P
P
  P
P B
  PA
 
P + 2S
R
C B

P+ 2S P+ 2S
R R

(a) (b)
Fig. 10(c).31. Fall of liquid
   
10.90 PHYSICS (Vol. II)—XI


 –



   
      –  R
    r




Fig. 10(c).32

S 

SURFACE TENSION [MECHANICAL PROPERTIES OF FLUIDS (III)] 10.91

Soap
molecules

Water

Soap molecules with head attracted to water Platter with particles of greasy dirt Water is added ; dirt is not dislodged
(a) (b) (c)

Detergent is added. The ‘inert’ waxy ends of Inert ends surround dirt and the platter dirt Dirt is held suspended, surrounded by soap
its molecules are attracted to boundary where can now be dislodged say by moving water molecules
water meets dirt (e) (f)
(d)

Fig. 10(c).33. Pictorial representation of detergent action

VERY SHORT ANSWER QUESTIONS—Each Question Carries 1 Mark Only


10.92 PHYSICS (Vol. II)—XI

 

 
   
 
SURFACE TENSION [MECHANICAL PROPERTIES OF FLUIDS (III)] 10.93

SHORT ANSWER QUESTIONS—Each Question Carries 2 Marks

 
 Fig. 10(c).34

S2 S3
S1

A B

Fig. 10(c).35
10.94 PHYSICS (Vol. II)—XI

SHORT ANSWER QUESTIONS—Each Question Carries 3 Marks

 

  
   
  


    
 


  
  
  


  



t

  Fig. 10(c).36
SURFACE TENSION [MECHANICAL PROPERTIES OF FLUIDS (III)] 10.95

Fig. 10(c).37


     
        
     

 

 
 
  

 
10.96 PHYSICS (Vol. II)—XI



  
SURFACE TENSION [MECHANICAL PROPERTIES OF FLUIDS (III)] 10.97

 

  
      

   

LONG ANSWER QUESTIONS



 
 
  
10.98 PHYSICS (Vol. II)—XI
SURFACE TENSION [MECHANICAL PROPERTIES OF FLUIDS (III)] 10.99



10.100 PHYSICS (Vol. II)—XI


SURFACE TENSION [MECHANICAL PROPERTIES OF FLUIDS (III)] 10.101

 
     
  
  
 



 
 
     
   
   
10.102 PHYSICS (Vol. II)—XI

  
   

   

     
 

 
   
 
  

   
     
   




 
 
    
 

 

    
      


   
       
  

     
      
   
 

  

       

F I
 
     
GG
  



JJ

H K
 
 
    
 

    
   
 
 
   

   
SURFACE TENSION [MECHANICAL PROPERTIES OF FLUIDS (III)] 10.103

  
  
 
  

  

    
      
= 4.898 × 10  

  
 


 
   
 
 
     
  
   

   
       
    
    
   
   

    
  

 
  = 7.2 × 10
 
  
   
 '
 
  
        
   
 
 
     

 

  
  

F I  
 GH JK
  
'  

P. 1. 125 water droplets, each of radius r, coalesce to   


form a single drop. The energy released raises the   
temperature of the drop. If S represents surface tension, 
represents density, S represents specific heat and J 
represents mechanical equivalent of heat, then calculate
the rise in temperature of the drop.      

    
  
   
10.104 PHYSICS (Vol. II)—XI

P. 2. A drop of liquid of density  is floating half-


immersed in a liquid of density d. If S is the surface tension,
then what is the diameter of the drop of the liquid ?  
   
 
Solution.       
  
   
    
   
 

 
  

   
   
  

P. 3. Two spherical soap bubbles coalesce. If V is the
change in volume of the enclosed air and A is net change   
in surface area, then prove that 3PV + 4AS = 0.        
 

 
   
Solution. Using Boyle’s law,  
P. 5. A glass U-tube is such
   
that the diameter of one limb is
3.0 mm and that of the other is A
     
         6.0 mm. The tube is inverted
      B h C
vertically with the open ends
below the surface of water in a D
      beaker. What is the difference
    

         between the heights to which
 
water rises in the two limbs ?
Surface tension of water is
0.07 N m–1. Assume that the angle Fig. 10(c).38
of contact between water and
P. 4. If a number of little droplets of water, each of glass is 0°.
radius r, coalesce to form a single drop of radius R, show
that the rise in temperature will be given by [Ans. 4.76 mm]
Hint. Let PA, PB, PC and PD be the pressures at the points
 , A, B, C and D respectively.
 
 

Solution. Let n be the number of little droplets. 


    
    

      

   
            
   

   
         
   
SURFACE TENSION [MECHANICAL PROPERTIES OF FLUIDS (III)] 10.105

 
    
 

 
   
   
 
P. 6. A vessel filled with air under pressure p0 con-  
tains a soap bubble of diameter d. The air pressure having
been reduced isothermally n fold, the bubble diameter in- P. 8. A bubble having
creased  fold. Calculate the surface tension of soap wa- surface tension T and
ter solution. radius R is formed on a ring R
b
of radius b(b < < R). Air is
        blown inside the tube with
Solution.      =    
         velocity v as shown. The air
molecule collides perpendi-
   cularly with the wall of the
    
   bubble and stops. Calculate
the radius at which the Fig. 10(c).39
  bubble separates from the
    ring.
 
 
   Solution. Excess pressure of bubble =
  
  
 
P. 7. Water rises to a height of 10 cm in a capillary    
= 
tube dipped in water. When the same tube is dipped in
mercury, it is depressed by 2.60 mm. Compare the surface
tensions of water and mercury. Density of water = 1000 kg m–3, 
that of mercury = 13600 kg m–3. Angle of contact for water
is 5° and that for mercury is 135°.
Solution. For water, h1 = 10 cm = 0.1 m, 
  P. 9. A container of width
2a is filled with a liquid. A thin y
wire of weight per unit length  is
gently placed over the liquid

surface in the middle of the
 
surface as shown in the figure. As
a result, the liquid surface is
   depressed by a distance y (y < < a). 2a
  Determine the surface tension of Fig. 10(c).40
the liquid.

Solution. The forces acting on the part of the wire of


length l are shown in figure.
 

 S  S


     

 

  lg
  
   
Fig. 10(c).41
10.106 PHYSICS (Vol. II)—XI
R. If S is the surface tension, calculate the release of energy.
If all the energy released is converted into kinetic energy,
   find the velocity acquired by the drop.
   
    
    
  
 Hint. Since the volume remains constant,

        .

P. 10. Several spherical drops of a liquid of radius


r each coalesce to form a single spherical drop of radius

NCERT EXERCISES (With Solutions)





  

  
 

 
 
NCERT EXERCISES (UNIT VII) 10.107


  



 



 



10 cm 12.5 cm

Fig. 10(c).42
10.108 PHYSICS (Vol. II)—XI

  
 



   



SPIRIT 

WATER 
27.5 cm
25 cm


h        
A B
MERCURY
 
 
Fig. 10(c).43

  
    

  

 

 

   

(a) (b)

Fig. 10(c).44
NCERT EXERCISES (UNIT VII) 10.109


 
 


 
      

 
 


  

  


 

 
 
 
 

 

Fig. 10(c).45
10.110 PHYSICS (Vol. II)—XI

TO PUMP


  

Fig. 10(c).46

 



 


  
 
  
NCERT EXERCISES (UNIT VII) 10.111


     

   
 
   
 
  

   





    

   
  




 
         
    
      



  
       

10.112 PHYSICS (Vol. II)—XI
 

 

  


 
 
p 2 = p – dp
dy
p1 = p 

y

sea level

Fig. 10(c).47




  


 



NCERT EXEMPLAR PROBLEMS (UNIT VII) 10.113

NCERT EXEMPLAR PROBLEMS (with Answers)

[Based on Higher Order Thinking SKills].

MCQ I (Only one option is correct.)

    
      
 

Fig. 10(c).48

MCQ II (Any number of options may be


correct.)

Fig. 10(c).49
10.114 PHYSICS (Vol. II)—XI

 

Coin

Fig. 10(c).50

Fig. 10(c).51


  



  

SA

VSA


  

   




  


NCERT EXEMPLAR PROBLEMS (UNIT VII) 10.115

  


  

    
   

  
 
 

 
  
  

Fig. 10(c).52  
  
  

 
  
  

   



 
    


11.1
11.2 PHYSICS (Vol. II)—XI

tF

212°

Temperature (°F)
tF = 180°

32° tC = 100°


tC
0 Temperature (°C) 100

Fig. 11(a).1. A plot of Fahrenheit temperature (tF)


versus Celsius temperature (tC)


THERMAL PROPERTIES OF MATTER 11.3

°F

 212°


180° F

32°
    100°C
   
  
0
100 °C
Fig. 11(a).2

CHECKPOINT

O O O
70 X 120 W 90 Y BOILING POINT

O O O
–20 X 30 W 0 Y FREEZING POINT

Fig. 11(a).3
   
   



11.4 PHYSICS (Vol. II)—XI

Pressure

–273.15 °C

–200°C –100°C 0°C 100°C Temperature

 Fig. 11(a).4. Pressure versus temperature of


a low density gas kept at constant volume

Pressure
 Gas A

Gas B

Gas C


–273.15°C 0°C Temperature
(0 K)
 Fig. 11(a).5. A plot of pressure versus
temperature and extrapolation of lines for low
density gases indicates the same absolute
zero temperature

Steam
point 373.15K 100.00°C 212.00°F

Ice
point 273.15K 0.00°C 32.00°F

Absolute
Zero 0.00K –273.15°C –459.69°F

Fig. 11(a).6. Comparision of the Kelvin,


Celsius and Fahrenheit temperature scales

T =
THERMAL PROPERTIES OF MATTER 11.5






 
Fig. 11(a).7. Linear expansion

  



 


 




11.6 PHYSICS (Vol. II)—XI

 



 


  


 A
A


Fig. 11(a).8. Superficial expansion



THERMAL PROPERTIES OF MATTER 11.7


 

 v

v

 Fig. 11(a).9. Cubical expansion


 


  



6
K )
–1


 
–5
 (10

 3


250 500
T (K)

Fig. 11(a).10. Coefficient of


volume expansion of copper
as a function of temperature
11.8 PHYSICS (Vol. II)—XI







  
 
 


CHECKPOINT

(1) (2) (3) (4)


  Fig. 11(a).11
 
THERMAL PROPERTIES OF MATTER 11.9

 


  
 
 


 

  
  
    
  


  

 


  
      
  
 


 

  
  
  

  

  
  
11.10 PHYSICS (Vol. II)—XI

Fig. 11(a).12

Iron Iron
Bimettalic strip
Brass Brass

Current

(i) (ii)

(iii)
Fig. 11(a).13. (i ) When cool, contact closed (ii ) When
hot, contact open, (iii ) Thermoswitch in electric iron

+

Iron

Brass

Fig. 11(a).14
THERMAL PROPERTIES OF MATTER 11.11

Fish plate Gap Gap


Expands towards
the left

Rail
Expands towards
(i) (ii) the right

Fig. 11(a).15

Pipelines

Fig. 11(a).16
11.12 PHYSICS (Vol. II)—XI

 

Fig. 11(a).17

Fig. 11(a).18. Thermal expansion in


terms of vibrational motion of atoms
THERMAL PROPERTIES OF MATTER 11.13



 
  

 
 

  
 

  
 

   
  

     

    
     
 A 3 = ( ) (  )        
A 1 = ( )    

  
   
  

A2 = (  )

Fig. 11(a).19
11.14 PHYSICS (Vol. II)—XI


 
 
   



  
 
 



 

 
 


   

 l   q
 
  

     
  


  



 
THERMAL PROPERTIES OF MATTER 11.15

  

Volume of one kg of water (x10 m )


3
V

3
1.04343

1.00013
1.00000
T
0 5 10 100
Temperature (°C)


1.00000

Density (kg/m x10 )


3
0.99995

3
0.99990
0.99985
0.99980
0.99975
0.99970 T
0 5 10
Temperature (°C)

Fig. 11(a).20. Thermal expansion of water

  
 



11.16 PHYSICS (Vol. II)—XI






 




THERMAL PROPERTIES OF MATTER 11.17

 


  
11.18 PHYSICS (Vol. II)—XI

3R

0K T

Fig. 11(a).21. Variation of C with T

–1
Sp. HEAT IN°C cal g °C
1.008
–1

1.004

1.000

15 0C
0.996
20 40 60 80 100
O
TEMPERATURE IN C

Fig. 11(a).22. Variation of specific heat


of water with temperature
THERMAL PROPERTIES OF MATTER 11.19



   


  


    

FG  IJ
 
H  K

FG  IJ
 
H  K
11.20 PHYSICS (Vol. II)—XI

 
 

 
 

THERMAL PROPERTIES OF MATTER 11.21

  





P + P V P V P V + V
Q1 Q2 = Q1 + dW
T + T T T + T

   
Fig. 11(a).23. Relation between Cp and C

 

   
 


11.22 PHYSICS (Vol. II)—XI

  

  

 
 
 
   

 

 

 





THERMAL PROPERTIES OF MATTER 11.23

 

    


  
 

     
  
 

   
  


 
 

11.24 PHYSICS (Vol. II)—XI

  

 


THERMAL PROPERTIES OF MATTER 11.25

100

Temperature (°C)
0 tm Time (min)

Fig. 11(a).24. A plot of temperature


versus time showing the changes in
the state of ice on heating (not to scale)

Fig. 11(a).25
11.26 PHYSICS (Vol. II)—XI

5
22.6 × 10 J/kg
(540 kcal/kg)
Boiling Phase
Gas phase

Temperature (°C)
Point Change
100 5 (steam)
3.33 × 10 J/kg
(80 kcal/kg)
Melting Phase
Point Change Liquid phase
0 (water)
Solid phase
(ice)

Heat (energy)
Fig. 11(a).26. Temperature versus heat for
water (Not to scale)

 



THERMAL PROPERTIES OF MATTER 11.27

P
(atm)
B

218 A
Ice

Water
P Steam
0.006
C
–220 0.01 374 T(°C)

Fig. 11(a).27
11.28 PHYSICS (Vol. II)—XI

P
(atm)
B
73.0 A
56.0
u id
Solid L iq
P
5.11
1.0 Vapour
C
–78.5 –56.6 20 31.1 T(°C)

Fig. 11(a).28

Fu
sio Liquid
nL
ine
Solid on Line
e risati
tio n Lin Vapo
lima
P Sub

Vapour

T
Fig. 11(a).29
THERMAL PROPERTIES OF MATTER 11.29

VERY SHORT ANSWER QUESTIONS—Each Question Carries 1 Mark Only


   

SHORT ANSWER QUESTIONS—Each Question Carries 2 Marks


11.30 PHYSICS (Vol. II)—XI

SHORT ANSWER QUESTIONS—Each Question Carries 3 Marks

C D
TEMPERATURE

A B

O HEAT INPUT

Fig. 11(a).30
THERMAL PROPERTIES OF MATTER 11.31

 
 

Q. 46.


Answer. The increase in volume of a body depends upon
its original volume, rise in temperature and the value of
 
coefficient of cubical expansion. Therefore, increase in volume in Answer.

these two cases will be same.
  

Q. 47.
11.32 PHYSICS (Vol. II)—XI


THERMAL PROPERTIES OF MATTER 11.33



 

 
 
  

Q. 63.

 
 
 


 
Answer.


     

LONG ANSWER QUESTIONS

   
 
11.34 PHYSICS (Vol. II)—XI

 

 

 
    
  

 

 

 


THERMAL PROPERTIES OF MATTER 11.35

  
 
     
 
  

       


    

 
   
 


   

  

  
  

11.36 PHYSICS (Vol. II)—XI

 

 
  

 

  
    Fig. 11(a).31
  



   

 
     

   

 
 

 


THERMAL PROPERTIES OF MATTER 11.37

  
    
 

 

  
 

 
    
 


 
   
11.38 PHYSICS (Vol. II)—XI


  

 


  

 
  

 

 

   

F I LMF F  I  I OP 
 GH   JK MGH  GH JK JK P
N Q 

  
      
   
   

  

 
   
  

Conduction

Convection

Radiation
Fig. 11(b).1. Heating by conduction,
convection and radiation

T1 L T2

Fig. 11(b).2. Steady state heat flow by


 conduction in a bar with its two ends
maintained at temperatures T 1 and T2

11.39
11.40 PHYSICS (Vol. II)—XI



HEAT TRANSFER 11.41


 
11.42 PHYSICS (Vol. II)—XI

 
 
HEAT TRANSFER 11.43
11.44 PHYSICS (Vol. II)—XI

 

 
  
  


 
 
 

A B
Fig. 11(b).3

  

 





   

  

 
 
 
 

  

 

 

T0
T1 = 373 K T2 = 273 K

Iron Brass

Fig. 11(b).4
HEAT TRANSFER 11.45

Furnace Ice box


o Steel o
300 C 0 C

   Insulating Copper

material
Fig. 11(b).5

 

    
 

 

11.46 PHYSICS (Vol. II)—XI


Day

Air current
(sea breeze)

Land warmer than water

Night

Air current
(land breeze)

Water warmer than land

Fig. 11(b).6. Convection cycles


HEAT TRANSFER 11.47


11.48 PHYSICS (Vol. II)—XI

P
O

Fig. 11(b).7. Black body


HEAT TRANSFER 11.49

 


 

  
 


 

  


11.50 PHYSICS (Vol. II)—XI

 

G
Bolometer

Prism
C1 C2

Fig. 11(b).8. Experimental arrangement for studying the


distribution of energy in the spectrum of black body

1650K

1450K
1260K
  E 1000K

O

violet red
Visible Region

Fig. 11(b).9. Variation of E with


 at different temperatures

HEAT TRANSFER 11.51

 

 


  




 
  


 

 


11.52 PHYSICS (Vol. II)—XI

 
   

  
   
  

 
 

 

  

 
  
 



HEAT TRANSFER 11.53

  

 

Y
loge (T – T0 )

O X
  t
Fig. 11(b).10
 
   
 


T

  
 t

Fig. 11(b).11. Cooling curve


11.54 PHYSICS (Vol. II)—XI


Rate of fall of
temperature
as

Excess temperature

Fig. 11(b).12

 

T2 T1


 C
V


Fig. 11(b).13. Experimental verification
 of Newton’s law of cooling


loge(T2 – T1)

t
Fig. 11(b).14
HEAT TRANSFER 11.55


 


 




  
 

 
 

  
 


   

  
     

  
 
   
 


     



   
 

  


11.56 PHYSICS (Vol. II)—XI

   
 

  
 


  










   
   
 

   
   
      

   

   
  
 

  



 



HEAT TRANSFER 11.57

tion
r radia
a
Sol
CO2 and other
gases in lower
atmosphere

ud Infrared
Clo
rays are
reflected

Earth surface Infrared rays


are radiated
by Earth
Fig. 11(b).15. Greenhouse effect

VERY SHORT ANSWER QUESTIONS—Each Question Carries 1 Mark Only


11.58 PHYSICS (Vol. II)—XI

SHORT ANSWER QUESTIONS—Each Question Carries 2 Marks


HEAT TRANSFER 11.59

SHORT ANSWER QUESTIONS—Each Question Carries 3 Marks


    



      
      
      

  
11.60 PHYSICS (Vol. II)—XI

 

   
  
 
   
  
  

   
   
  
 

Radiation of
1400 W power
Sun

1 m 2 near
the Earth

Fig. 11(b).16
HEAT TRANSFER 11.61

LONG ANSWER QUESTIONS


11.62 PHYSICS (Vol. II)—XI

 
 


50°C 60°C

20 m
T

Fig. 11(b).17


HEAT TRANSFER 11.63

 


 




 
 

 

 

 



     
   
  
  
  
  

 
  
   
11.64 PHYSICS (Vol. II)—XI

P. 1. The opposite faces of a cubical block of iron of P. 3. A rectangular slab is made of two parallel-
2
cross-section 4 cm are kept in contact with steam and sided plates of copper and brass. The copper plate is 4 cm
thick and brass plate is 3 cm thick. Coefficients of thermal
melting ice. Calculate the quantity of ice melted at the end
conductivity of copper and brass are 0.92 cgs units and
of 10 minute. [K for iron = 0.2 cgs units, Latent heat of 0.62 cgs units respectively. The outer surface of copper
fusion of ice = 80 cal g–1] plate is at 200°C and that of brass plate is 20°C. Find the
temperature of the interface.
Solution. If l be the edge of the cubical block, then
Solution. Let the area of compound plate be A cm 2.

   

T1 = 200°C

x1 = 4 cm Copper K1 = 0.92
T

x2 = 3 cm Brass K2 = 0.62

   T2 = 20°C

Fig. 11(b).18

   
P. 2. Water is boiled in a rectangular steel tank
of thickness 2 cm by a constant temperature furnace. Due
to vaporisation, water level falls at a steady rate of 1 cm       
in 9 minute. Calculate the temperature of furnace. Given : 
K for steel = 0.2 cgs units.
Solution. Let A sq. cm be the surface area of the bottom
P. 4. Steam at 100°C is passed through a metal tube
of the tank.
of 0.5 m diameter. The tube is covered with material of
uniform thickness 0.25 cm, and thermal conductivity
0.5 × 10–3 cgs units. Find how much steam is condensed per
metre length in 5 minute ? Latent heat of steam is 537
calorie per gram and atmospheric temperature is 20°C.
Solution. T1 – T2 = (100 – 20)°C = 80°C ;



   
HEAT TRANSFER 11.65


       


  

P. 5. A 10 cm layer of ice has been formed over a


pond of water. The temperature of air above is – 5°C. How
long will it take the layer to become 10.1 cm thick ? K for
ice = 0.008 cgs units, denstiy of ice = 1 g cm –3 and latent
heat of ice = 79.8 cal g–1.
Solution. Let A cm2 be the area of layer of ice.

  

P. 7. A room is maintained at 20°C by a heater of


resistance 20  connected to 200 V mains. The temperature
is uniform throughout the room and the heat is transmitted
through a glass window of area 1 m2 and thickness 0.2 cm.
Calculate the temperature outside. Given that thermal
conductivity of glass, K = 0.2 cal m–1 s–1 °C –1 and mechanical
equivalent of heat, J = 4.2 J cal–1.
Solution. Heat generated by heater per second,





 
  
P. 6. A layer of ice is present on a pond. It will take 
3 hour and 20 minute for the thickness of ice to increase by 
2 mm. Find the thickness of ice present in the beginning
from the following data :

  

Solution. t = 3 hour 20 minute = 12000 s


11.66 PHYSICS (Vol. II)—XI

NCERT EXERCISES (With Solutions)






 
   
  


 


 

   
  
 

  
 
 


NCERT EXERCISES (UNIT VII) 11.67



   


 
 
  



 
   
11.68 PHYSICS (Vol. II)—XI





    

 
  
    
 
   



 
       

  


    
 




 

NCERT EXERCISES (UNIT VII) 11.69

  




11.70 PHYSICS (Vol. II)—XI

 

    

 

 

 
  

 

 
NCERT EXERCISES (UNIT VII) 11.71

  

 
   

   


NCERT EXEMPLAR PROBLEMS (With Answers)


[Based on Higher Order Thinking Skills].
MCQ I (Only one option is correct.)

 

 



180
Temperature (°A)


 tA = 150°



tB = 100°

 0
 Temperature (°B) 100

Fig. 11(b).19
11.72 PHYSICS (Vol. II)—XI


   MCQ II (Any number of options may be


correct.)


 
 



   
   

 

   
NCERT EXEMPLAR PROBLEMS (UNIT VII) 11.73

Temperature (°C) E

 
100  
C D  

A B
O tm time (min)
 

Fig. 11(b).20
 

q q

VSA
SA


 

 

 

 
11.74 PHYSICS (Vol. II)—XI


 

  

 
  
LA

 

q

P
(atm)  q
218  q
Liquid
1 q  
0.006
Solid
Gas
374   
0 0.01 T(K) 
 
 

Fig. 11(b).21  



q  

 q
 q
q  

  
NCERT EXEMPLAR PROBLEMS (UNIT VII) 11.75

     æ  ö æ ö
   
 çè ÷ø  çè ÷ø   
   

  

 

  q  q


 


q q
q
q q qq

 q q q q
L q q
q

q
1/2 (L +  L)
é æq  q öù
Fig. 11(b).22 ê   çè ÷ø ú
ë û
 

 

11.76
PROPERTIES OF BULK MATTER (UNIT VII) 11.77

 

 

 

2M 

 
11.78 PHYSICS (Vol. II)—XI




 
PROPERTIES OF BULK MATTER (UNIT VII) 11.79

 
 

= =

æ ö
çè ÷ø

æ ö æ ö
çè ÷ø çè ÷ø

æ ö æ ö
çè ÷ø çè ÷ø

æ ö
çè ÷ø
11.80 PHYSICS (Vol. II)—XI

Strain
P
Q

Stress

  
   



 
   
   
  

  
 

PROPERTIES OF BULK MATTER (UNIT VII) 11.81


  

 F Copper Copper F
2L L
 









  
= ´ =
 

æ  ö
ç ÷
è  ø =
 
  
  = 

 
     =

 
 


=

´ 
æ ö æ ö æ ö  ´
ç ÷ ç ÷ ç ÷
è ø è ø è ø
´ =

 ´ ´ ´
´ ´ ´


=
11.82 PHYSICS (Vol. II)—XI
PROPERTIES OF BULK MATTER (UNIT VII) 11.83


  

 
      
  
 
 

 

  
     
     
11.84 PHYSICS (Vol. II)—XI

  

æ ö æ ö
æ  ö  ç  ÷ ç  ÷
ç ÷ è ø è ø
è  ø 

 æ  ö
ç ÷ 
 è  ø

æ ö
ç ÷
è ø


 

 

 

 

 
 

 
 
PROPERTIES OF BULK MATTER (UNIT VII) 11.85
11.86 PHYSICS (Vol. II)—XI

´  ´  

 

  

   

   

   


 æ ö
ç ÷ 
è ø

æ ö æ ö
ç ÷ ç  ÷
è ø è ø

 

 




  

 



   
 
 
  
 
PROPERTIES OF BULK MATTER (UNIT VII) 11.87

     
  
      
 
  
 
  
æ  ö
 ç ÷ 
è  ø

æ ö
 ç ÷   
è ø

 
 

 
 

 ´   ´ 
  





=

 
 

 

æ ö
  çè  ÷ø





  

æ ö
 ç  ÷
è ø

æ ö
ç  ÷
è ø
11.88 PHYSICS (Vol. II)—XI

 

 

æ  ö
 ç ÷
è ø
PROPERTIES OF BULK MATTER (UNIT VII) 11.89

 

 
   


  

  

  
11.90 PHYSICS (Vol. II)—XI

 
 



 

 

æ ö æ ö
ç  ÷ ç  ÷
è ø è ø


 
 
PROPERTIES OF BULK MATTER (UNIT VII) 11.91

  
 = 
 




  
11.92 PHYSICS (Vol. II)—XI

 

æ ö
ç  ÷
è  ø

æ ö æ ö
ç  ÷ ç  ÷
è  ø è  ø
PROPERTIES OF BULK MATTER (UNIT VII) 11.93







 

 





=

æ  ö æ  ö
ç ÷ ç ÷
è ø è ø
æ  ö æ  ö
 = 
èç ø÷ èç ø÷
11.94 PHYSICS (Vol. II)—XI

 
  
   

  
 
 

 
æ ö æ ö ´
ç ÷ ç ÷ æ ö 
è ø è ø  ç  ÷
è ø

 æ ö
 ç  ÷
 è ø
  

´ ´ 


   ´

 
   




  


  

  
  

  
  
 
=
 æ ö 
 ç ÷    
 è ø 
 
 
æ ö 
ç ÷
è ø 
PROPERTIES OF BULK MATTER (UNIT VII) 11.95

æ ö
=ç ÷ æ ö
è ø =ç ÷
è ø
æ ö æ ö 
=ç ÷ ç ÷ =
´ 
´ è ø è ø =

=
´
  
= = =

   
 
´ ´ 

´
  



 
 

  
 æ ö  = =
´  
 èç ÷ø 

 =   =


= 
  = 
 

æ ö ´ 
=ç ÷
è ø 

 

( ) 
11.96 PHYSICS (Vol. II)—XI

 

 
 
PROPERTIES OF BULK MATTER (UNIT VII) 11.97

 

 

 
  
11.98 PHYSICS (Vol. II)—XI

 

 
 

 
 

 
 

 


PROPERTIES OF BULK MATTER (UNIT VII) 11.99

æ ö
çè  ÷ø

æ ö
çè  ÷ø

æ ö
çè ÷ø

 

Valve

  

= 
11.100 PHYSICS (Vol. II)—XI

æ ö
   ç ÷  
è  ø


  

 

  

  



 

 


 

 ´
´ ´

´  = 
æ ö
 ç  ÷
è ø
=

´  ´ 
PROPERTIES OF BULK MATTER (UNIT VII) 11.101

æ ö æ ö
 =ç  ÷ø  = çè  ÷ø
è
´   ´ 

= ´ 
æ ö
 =ç  ÷ø =
è  =


 

   
     
11.102 PHYSICS (Vol. II)—XI




PROPERTIES OF BULK MATTER (UNIT VII) 11.103

Temp.

Temp.




Temp.

Temp.
11.104 PHYSICS (Vol. II)—XI
  
  


 

 

 

  


   

     


PROPERTIES OF BULK MATTER (UNIT VII) 11.105



´ ´ ´ 


  
 

 
 
11.106 PHYSICS (Vol. II)—XI

  


 

  Temp.

 

´ ´



     ´
´  ´   ´ ´
   

æ ö
 ç ÷
è ø 

 

 


 

  

  
PROPERTIES OF BULK MATTER (UNIT VII) 11.107

 

 
11.108 PHYSICS (Vol. II)—XI

 
 
 
PROPERTIES OF BULK MATTER (UNIT VII) 11.109

  
   
 

   

 

11.110 PHYSICS (Vol. II)—XI

 
 

 
 

   

   

 


PROPERTIES OF BULK MATTER (UNIT VII) 11.111

 
 

 
 

   
   
     
 
    
   
     
11.112 PHYSICS (Vol. II)—XI

   

     
  

     
 

 

 
 



 
PROPERTIES OF BULK MATTER (UNIT VII) 11.113

   
   
   

 
 
 



 


11.114 PHYSICS (Vol. II)—XI

 
 
    


    
   
     

   
   
   

 
   
 

Configuration II
Configuration I
2K
K 2K K
PROPERTIES OF BULK MATTER (UNIT VII) 11.115


T T

0

O t O t
T T

0 0

O t O t

 
11.116 PHYSICS (Vol. II)—XI

      
  
 

  



  
 

 


   

 


  

   
  

 
      
    



    

    
   
  

 
  
  

 
 

  
  

 


   
  
  
PROPERTIES OF BULK MATTER (UNIT VII) 11.117



cylinder = 
   
    





 
 


 =

   
  
   


 
   
  =

 =  

 
  
  
  

 =

= = =

 = = 
   
=   


=


  

= = =
  = 

   
    
    
 = = 
 
  =
= = 
11.118 PHYSICS (Vol. II)—XI

 
= = =

=
 = 

 =
=
 =

=

PROPERTIES OF BULK MATTER (UNIT VII) 11.119
UNIT—VIII
THERMODYNAMICS 12 Periods

Chapter–12: Thermodynamics

This chapter has been divided into two chapters.

Chapter–12(a): First Law of Thermodynamics


Thermal equilibrium and definition of temperature (zeroth law of thermodynamics), heat,
work, and internal energy. First law of thermodynamics, isothermal and adiabatic processes.

Chapter–12(b): Second Law of Thermodynamics


Second law of thermodynamics: reversible and irreversible processes. Heat engine and
refrigerator.

SOLVED NUMERICALS 18 PLUS

UNSOLVED NUMERICALS 52 PLUS

VERY SHORT, SHORT & LONG ANSWER QUESTIONS 78 PLUS

QUESTIONS & PROBLEMS BASED ON HOTS 38 PLUS

NCERT EXERCISES, EXAMPLES & EXEMPLAR PROBLEMS 30 PLUS

MULTIPLE CHOICE QUESTIONS 110 PLUS


Pioneers of Thermodynamics
In previous chapter, we have studied thermal properties of matter. In
chapters 12(a) and 12(b), we shall study laws that govern thermal energy. We
shall study the processes where work is converted into heat and vice versa. In
winter, when we rub our palms together, we feel warmer. This is because work
done in rubbing produces ‘heat’. Conversely, in a steam engine, the ‘heat’ of the
steam is used to do useful work in moving the pistons, which in turn rotate the
wheels of the train.
In Physics, we need to define the notions of heat, temperature, work, etc.
carefully. Historically, it took a long time to arrive at the proper concept of Temperature is one of the most
‘heat’. Before the modern picture, heat was regarded as a fine invisible fluid important concepts of thermodynamics.
filling in the pores of a substance. This fluid was called “caloric”. In time, the
picture of heat as a fluid was discarded in favour of the modern concept of heat
as a form of energy. An important experiment in this connection was due to
Benjamin Thomson (also known as Count Rumford) in 1798. He observed that
boring of a brass cannon generated a lot of heat, indeed enough to boil water.
More significantly, the amount of heat produced depended on the work done. The
experiment demonstrated conversion of energy from one form to another-
from work to heat.
Thermodynamics is the branch of Physics that deals with the
concepts of heat and temperature and the inter-conversion of heat and
other forms of energy. In this branch of Physics, we deal with the processes
involving heat, work and internal energy.This branch of Physics can successfully
explain the bulk properties of matter in terms of molecules. The word
‘thermodynamics’ is a combination of two words (i) thermal (heat) (ii) dynamics
(motion). So, thermodynamics is basically a study of the relationship between
heat and motion.
Thermodynamics is a macroscopic science. It deals with bulk systems
and does not go into the molecular constitution of matter. In-fact, its concepts
and laws were formulated in the nineteenth century before the molecular picture
of matter was firmly established.
Thermodynamic description of a gas avoids the molecular description
altogether. Instead, the state of a gas in thermodynamics is specified by
macroscopic variables such as pressure, volume, temperature, mass and
composition that are felt by our sense perceptions and are measurable.*

12.3
12.4 PHYSICS (Vol. II)—XI
The distinction between mechanics and thermodynamics must be
clearly understood. In mechanics, our interest is in the motion of particles or
bodies under the action of forces and torques. Thermodynamics is not concerned
with the motion of the system as a whole. It is concerned with the internal
macroscopic state of the body. When a bullet is fired from a gun, what changes is
the mechanical state of the bullet (its kinetic energy, in particular), not its
temperature. When the bullet pierces a wood and stops, the kinetic energy of the
bullet gets converted into heat, changing the temperature of the bullet and the
surrounding layers of wood. Temperature is related to the energy of the internal
(disordered) motion of the bullet, not to the motion of the bullet as a whole.

A system in a state of
thermodynamic equilibrium
possesses the following
(i) Thermodynamical System. It is an assembly of an extremely
equilibria simultaneously.
large number of particles (atoms or molecules) so that the assembly has a certain
(i) Thermal equilibrium
value of pressure, volume and temperature. The thermodynamic system may
(ii) Mechanical equilibrium
exist in the form of a solid, a liquid or a gas or a combination of two or more of
(iii) Chemical equilibrium
these. The only condition is that the number of particles should be very large.
(ii) Surroundings. Everything outside the system which has a direct
effect on the system is called its surroundings.Consider a gas enclosed in cylinder
fitted with a frictionless piston. The gas forms the ‘thermodynamical system’.
The atmospheric air surrounding the cylinder forms the ‘surroundings’. The
movable piston and a flame also form the surroundings because they have a
direct effect on the gas.

(i) Introduction. In mechanics, equilibrium means that the net


external force and torque on a system are zero. However, in thermodynamics,
the term ‘equilibrium’ appears in a different context.
(ii) Thermodynamic Equilibrium. A system is in a state of
thermodynamic equilibrium if the macroscopic variables (pressure, volume,
temperature, mass and composition) that characterise the system do not change
with time. (a)

Consider a gas inside a closed rigid container, completely insulated from


its surroundings. Since the pressure, volume, temperature, mass and composition
of the gas do not change with time therefore the gas is said to be in a state of
thermodynamic equilibrium.
Whether or not a system is in a state of thermodynamic equilibrium
depends on the surroundings and the nature of the wall that separates the system
from the surroundings. Consider two gases A and B occupying two different
containers. Let the pressure and volume of the gases be (P A, VA) and (PB, VB)
(b)
respectively. Suppose first that the two systems are put in proximity but are
Fig. 12(a).1. (a) Systems A and B (two
separated by an adiabatic wall—an insulating wall that does not allow flow of
gases) separated by an adiabatic wall–
energy (heat) from one to another. The systems are insulated from the rest of the
an insulating wall that does not allow
surroundings also by similar adiabatic walls. The situation is shown schematically
flow of heat. (b) The same systems A
in Fig. 12(a).1(a). In this case, it is observed that any possible pair of values (PA,
and B separated by a diathermic wall–a
VA) will be in equilibrium with any possible pair of values (PB, VB). conducting wall that allows heat to flow
from one to another. In this case, thermal
(iii) Thermal Equilibrium. Now suppose that the adiabatic wall is equilibrium is attained in due course
replaced by a diathermic wall—a conducting wall that allows energy flow
FIRST LAW OF THERMODYNAMICS 12.5
(heat) from one to another. It is then found that the macroscopic variables of the
systems A and B change spontaneously until both the systems attain equilibrium
states. After that there is no change in their states. The situation is shown in
Fig. 12(a).1(b). The pressure and volume variables of the two gases change to
(PB, VB) and (PA, VA) such that the new states of A and B are in equilibrium
with each other. There is no more energy flow from one to another. We then say
that the system A is in thermal equilibrium with the system B.
Two systems are said to be in thermal equilibrium with each
other if they are at the same temperature.

Imagine two systems A and B, separated by an adiabatic wall, while each


is in contact with a third system C, via a conducting wall [Fig. 12(a).2(a)]. The
states of the systems (i.e., their macroscopic variables) will change until both A
and B come to thermal equilibrium with C. After this is achieved, suppose that
the adiabatic wall between A and B is replaced by a conducting wall and C is
insulated from A and B by an adiabatic wall [Fig. 12(a).2(b)]. It is observed that
the states of A and B change no further i.e., they are found to be in thermal
(a)
equilibrium with each other. This observation forms the basis of the Zeroth
Law of Thermodynamics, which is stated as under : Two systems in thermal
equilibrium with a third system separately are in thermal equilibrium
with each other.
R.H. Fowler formulated this law in 1931 long after the first and second
Laws of thermodynamics were stated and so numbered.
The Zeroth Law clearly suggests that when two systems A and B are in
thermal equilibrium, there must be a physical quantity that has the same value
for both. This thermodynamic variable whose value is equal for two systems in
thermal equilibrium is called temperature (T). Thus, if A and B are separately in (b)
equilibrium with C, TA = TC and TB = TC. This implies that TA = TB i.e., the Fig. 12(a).2. (a) Systems A and B are
systems A and B are also in thermal equilibrium. separated by adiabatic wall, while each
is in contact with a third system C via a
On the basis of the above discussion, the Zeroth Law of thermodynamics
conducting wall. (b) The adiabatic wall
may be stated as follows : between A and B is replaced by a
conducting wall while C is insulated
from A and B by an adiabatic wall

On the basis of discussion of thermal equilibrium and Zeroth law of


thermodynamics, the temperature can be defined in any of the following ways.
Temperature is that thermodynamic variable which determines whether
the two systems in contact will be in thermal equilibrium or not.
Temperature of a body is that parameter which determines the degree of
hotness or coldness of a body.
Temperature of a body is that parameter which determines the direction
of flow of heat when the body is brought in contact with another body.
Temperature of a system is that physical quantity, equality of which is
the only condition for the thermal equilibrium of two systems or bodies in contact.
12.6 PHYSICS (Vol. II)—XI

(i) Heat. Heat and work are the terms used to describe energy in the
Heat is a form of energy in transit
process of transfer.
from hot to cold.
Heat is the energy that flows by conduction, convection or
radiation from one body to another

P, V

A V

Fig. 12(a).3

In 1798, Count
Rumford observed that the amount of heat produced is proportional to the amount
of mechanical work done. Later, Dr. James Prescott Joule of Manchester
established a definite relation between the work done and the heat produced. It
was shown that when a certain amount of mechanical work is done, an equivalent
amount of heat is always produced.
If W is the amount of mechanical work done and Q is the quantity of heat
produced, then
WQ or W = JQ
where J is a constant of proportionality.
It is known as Joule’s constant or Joule’s mechanical equivalent of heat.
If Q = 1, then J = W.
FIRST LAW OF THERMODYNAMICS 12.7
Joule’s mechanical equivalent of heat may be defined as the
amount of work required to be done to produce a unit quantity of heat.
If Q is measured in calorie and W in erg, then
J = 4.186 × 107 erg cal–1
So, 4.186 × 107 erg of work is required to produce 1 calorie of heat.
If Q is measured in calorie and W in joule, then
J = 4.186 J cal–1 [... 1 J = 107 erg]
In SI, both Q and W are measured in joule. In this case, Q = W.
So, work and heat are two equivalent forms of energy. However, the
following distinction between work and heat must be clearly noted.
Work is the transfer of mechanical energy irrespective of temperature
difference whereas heat is the transfer of thermal energy because of temperature
difference only.
(iv) Internal Energy of a System. The internal energy of a system
is the sum of kinetic and potential energies of the molecules of the system. This
energy is possessed by a system due to its molecular motion and molecular
configuration. The energy due to molecular motion is called internal kinetic  The internal energy of an ideal
energy (Ek). The motion includes translational, rotational and vibrational motion gas consists only of internal
of the molecules. The energy due to molecular configuration is called internal kinetic energy—which depends
potential energy (E p). The internal energy is denoted by U. only on temperature of the gas.
 The internal energy of a real gas
U = Ek + E p is sum of internal kinetic energy
In thermodynamics, the kinetic energy of the system, as a whole, is not and internal potential energy of
relevant. Thus, the internal energy is the sum of molecular kinetic and potential the molecules of the gas. It
energies in the frame of reference relative to which the centre of mass of the depends on both temperature
and volume of gas.
system is at rest.
(v) Internal Energy of an Ideal Gas. Kinetic theory of gases gives
the model of an ideal gas. In the case of an ideal gas, the intermolecular force is
zero. As a result of this, the molecular potential energy of the gas molecules is
zero. So, the internal energy of the ideal gas is wholly kinetic in nature. According
to kinetic interpretation of temperature, the kinetic energy of gas molecules
depends only upon temperature. So, the internal energy of an ideal gas depends
only upon the temperature of the gas. Mathematically, the internal energy U of
an ideal gas is a function of the absolute temperature T of the gas.
(a)
U = f (T)
(vi) Internal Energy of a Real Gas. The molecules of a real gas exert
forces on one another. Due to intermolecular forces, the molecules of a real gas
possess potential energy. The intermolecular forces depend upon intermolecular
distance which further depends upon the volume of the gas. So, the potential
energy of the gas molecules depends upon the volume of the gas.
Due to molecular motion, the molecules of a real gas possess kinetic energy. (b)
According to kinetic interpretation of temperature, the kinetic energy of gas Fig. 12(a).4. (a) Internal energy U of a
molecules depends upon temperature. gas is the sum of the kinetic and
The internal energy of a real gas is the sum of potential energy and kinetic potential energies of its molecules
energy of gas molecules. Potential energy depends upon volume. Kinetic energy when the box is at rest. Kinetic energy
due to various types of motion
depends upon temperature. So, the internal energy of a real gas depends upon (translational, rotational, vibrational) is
both volume and temperature. Mathematically, the internal energy U of a real to be included in U. (b) If the same box
gas is a function of volume V and temperature T of gas. is moving as a whole with some velocity,
U = f (V, T) the kinetic energy of the box is not to be
included in U.
12.8 PHYSICS (Vol. II)—XI

(vii) Internal Energy is Thermodynamic State Variable of the


System (gas). We have invoked the molecular picture to understand the meaning
of internal energy. But as far as thermodynamics is concerned, U is simply a
macroscopic variable of the system. The important thing about internal energy
is that it depends only on the state of the system, not on how that state was
achieved. Internal energy U of a system is an example of a thermodynamic ‘state
In a gas, the motion of molecules is
variable’. Its value depends only on the given state of the system, not on the
not only translational (i.e., motion
‘path’ taken to arrive at that state. Thus, the internal energy of a given mass of from one point to another in the
gas depends on its state described by specific values of pressure, volume and volume of the container); it also
includes rotational and vibrational
temperature. It does not depend on how this state of the gas came about. Pressure,
motion of the molecules.
volume, temperature, and internal energy are thermodynamic state variables of
the system (gas).
(viii) Heat and Work can Change Internal Energy. Consider the (b)
system to be a gas contained in a cylinder with a movable piston as shown in
Fig. 12(a).5. There are two ways of changing the state of the gas (and hence its
internal energy). One way is to put the cylinder in contact with a body at a
higher temperature than that of the gas. The temperature difference will cause
a flow of energy (heat) from the hotter body to the gas. This increases the internal
energy of the gas. The other way is to push the piston down i.e., to do work on
the system. This again results in increasing the internal energy of the gas. Of
course, both these things could happen in the reverse direction. With surroundings
at a lower temperature, heat would flow from the gas to the surroundings.
Likewise, the gas could push the piston up and do work on the surroundings. In
short, heat and work are two different modes of altering the state of a
thermodynamic system and changing its internal energy.
(a)
(ix) Heat and Work are Different from Internal Energy. The notions
of heat and work should be carefully distinguished from the notion of internal
energy. The state of a thermodynamic system is characterised by its internal Fig. 12(a).5. Heat and work are two
energy and not by heat and work. In other words, while internal energy is a state distinct modes of energy transfer to a
variable, heat and work are not state variables. A statement like ‘a gas in a system that results in change in its
internal energy. (a) Heat is energy
given state has certain amount of internal energy’is a perfectly meaningful
transfer due to temperature difference
statement. Similar statements for heat and work are meaningless. Heat and between the system and the
work are modes of energy transfer to a system which results in change in its surroundings. (b) Work is energy
transfer brought about by means (e.g.,
internal energy.
moving the piston by raising or lowering
some weight connected to it) that do not
involve such a temperature difference

The internal energy U of a system can change through two modes of


energy transfer : heat and work. We will now establish a quantitative relationship
of internal energy with heat and work.
Let,
Q = Heat supplied to the system by the surroundings
W = Work done by the system on the surroundings
U = Change in internal energy of the system
FIRST LAW OF THERMODYNAMICS 12.9
The general principle of conservation of energy then implies that
Q = U + W ...(1)
i.e., the energy (Q) supplied to the system goes in partly to increase the
internal energy of the system (U) and the rest in work on the environment
(W). Equation (1) is the mathematical statement of the First Law of
Thermodynamics. It is simply the general law of conservation of energy applied
to any system in which the energy transfer from or to the surroundings is taken
into account.
The first law of thermodynamics can be stated as follows :
If some quantity of heat is supplied to a system capable of doing
external work, then the quantity of heat absorbed by the system is equal
to the sum of the increase in the internal energy of the system and the
external work done by the system.
The equation (1) applies to a process in which the initial and final states of
the system differ by a finite amount. However, if the end states differ by an
infinitesimally small amount, then the equation (1) takes the following form :

dQ = dU + dW

This equation is known as the differential form of the first law of


thermodynamics.
Discussion. (i) The first law of thermodynamics is essentially a
restatement of the law of conservation of energy, i.e., energy can neither
be created nor be destroyed but may be converted from one form to another.
(ii) In applying the first law of thermodynamics, all the three quantities,
i.e., dQ, dU and dW must be expressed in the same units, i.e., either in units of
work or in units of heat.
(iii) This law is applicable to every process in nature.
(iv) This law is applicable to all the three phases of matter i.e., solid,
liquid and gas.
(v) U may be any type of internal energy-translational kinetic energy,
rotational kinetic energy, vibrational kinetic energy. It is a characteristic of the
state of a system. Since U is a state variable therefore it depends only on the
initial and final states. U does not depend on the path taken by the system to go
from one state to another.
(vi) dW may be any type of work done by the system. It depends upon the
manner in which the system has been changed from one state to another. So, it
is not a characteristic of the state of the system. The same is true for dQ.
(vii) If the system is a gas in a cylinder with a movable piston, the gas in
moving the piston does work. Since force is pressure times area, and area times
displacement is volume, work done by the system against a constant pressure P
is
W = P V
where V is the change in volume of the gas. Thus, for this case, Eq. (1) gives
Q = U + P V
(viii) Let us put equation (1) in the alternative form
Q – W = U
Q and W will, in general, depend on the path taken to go from initial to
final states. But the combination Q – W is path-independent.
12.10 PHYSICS (Vol. II)—XI
(ix) If the system is taken through a process in whichU = 0 (for example,
isothermal expansion of an ideal gas), then
Q = W
In this case, the heat supplied to the system is used up entirely by the
system in doing work on the environment.
(x) *The first law of thermodynamics introduces the concept of internal
energy.
(xi) The system may go from an initial state to the final state in a number
of ways. For example, to change the state of a gas from (P 1, V1) to (P2, V2), we
can first change the volume of the gas from V1 to V2, keeping its pressure constant
i.e., we can first go the state (P1, V2) and then change the pressure of the gas
from P1 to P2, keeping volume constant, to take the gas to (P2, V2). Alternatively,
we can first keep the volume constant and then keep the pressure constant.
An Important Note. Both dQ and dW are path functions. dQ cannot be
written as (Q2 – Q1) and dW cannot be written as (W2 – W1). For the same
reason, ‘the initial heat Q1’, ‘the final heat Q2’, ‘initial work W1’, ‘final work W2’
are meaningless terms.
Significance. The first law of thermodynamics tells us that it is impossible
to get work from any machine without giving it an equivalent amount of energy.
The internal energy Eint of a system
Sign Convention. (i) When heat is supplied to a system, dQ is taken as tends to increase if energy is added
positive. When heat goes out of the system, dQ is taken as negative. via heat Q and tends to decrease if
(ii) When the gas expands, dW is taken as positive. This is because the energy is lost via work W done by the
force exerted by the system and the displacement are in the same direction. system.
When the gas is compressed, dW is taken as negative. This is because the force
exerted by the system and the displacement are in opposite directions.
(iii) dU is positive for increase in internal energy and negative for decrease
in internal energy.

(i) Thermodynamic state variables are those macroscopic


variables whose specific values completely describe the equilibrium (a)
state of a thermodynamic system. Thermodynamical state variables are
also known as thermodynamic co-ordinates or thermodynamic parameters. When
these variables change with time, we say that a thermodynamic process is taking
place.
An equilibrium state of a gas is completely specified by the values of
pressure, volume, temperature, and mass (and composition if there is a mixture
of gases). A thermodynamic system is not always in equilibrium. For example, a
(b)
gas allowed to expand freely against vacuum is not in an equilibrium state
Fig. 12(a).6. (a) The partition in the box
[Fig. 12(a).6(a)]. During the rapid expansion, pressure of the gas may not be
is suddenly removed leading to free
uniform throughout. Similarly, a mixture of gases undergoing an explosive expansion of the gas. (b) A mixture of
chemical reaction (e.g., a mixture of petrol vapour and air when ignited by a gases undergoing an explosive
spark) is not in an equilibrium state; again its temperature and pressure are not chemical reaction. In both situations, the
uniform [Fig. 12(a).6(b)]. Eventually, the gas attains a uniform temperature and gas is not in equilibrium and cannot be
pressure and comes to thermal and mechanical equilibruim with its surroundings. described by state variables.

*Zeroth law of thermodynamics introduces thermodynamic quantity called temperature. The second law of thermodynamics
brings out the concept of entropy.
FIRST LAW OF THERMODYNAMICS 12.11
In short, thermodynamic state variables describe equilibrium states of
systems.
(ii) Equation of state. The various state variables are not necessarily
independent. The connection between the state variables is called the equation of
state. For example, for an ideal gas, the equation of state is the ideal gas relation
PV = RT.
For a fixed amount of the gas i.e., given , there are thus, only two
independent variables, say P and V or T and V. Real gases may have more
complicated equations of state.
(iii) Types of thermodynamic variables. The thermodynamic state
variables are of two types: extensive and intensive. Extensive variables indicate
the ‘size’ of the system. Intensive variables such as pressure and temperature do
not. To decide which variable is extensive and which intensive, think of a relevant
system in equilibrium, and imagine that it is divided into two equal parts. The
variables that remain unchanged for each part are intensive. The variables whose
values get halved in each part are extensive.
Extensive variables are those variables which depend on the size
or mass of the system. Examples : volume V, internal energy U, total mass
M, heat capacity, entropy and enthalpy.
Intensive variables are those variables which are independent
of the size of the system. Examples : Temperature T, pressure P, density 
and specific heat capacity.
It is a good practice to check the consistency of thermodynamic equations
using this classification of variables. For example, in the equation
Q = U + P V
quantities on both sides are extensive. (The product of an intensive variable like
P and an extensive quantity V is extensive.)
As emphasised earlier, Q is not a state variable. However, Q is clearly
proportional to the total mass of system and hence is extensive.

12(a).09.1. QUASI-STATIC PROCESS


(i) Non-equilibrium states. Consider a gas in thermal and mechanical
equilibrium with its surroundings. The pressure of the gas in that case equals
the external pressure and its temperature is the same as that of its surroundings.
Suppose that the external pressure is suddenly reduced (say by lifting the weight
on the movable piston in the container). The piston will accelerate outward.
During the process, the gas passes through states that are not equilibrium states.
The non-equilibrium states do not have well-defined pressure and tempearture.
In the same way, if a finite temperature difference exists between the gas and its
surroundings, there will be a rapid exchange of heat during which the gas will
pass through non-equilibrium states. In due course, the gas will settle to an
equilibrium state with well-defined temperature and pressure equal to those of
the surroundings. The free expansion of a gas in vacuum and a mixture of gases
undergoing an explosive chemical reaction are also examples where the system
goes through non-equilibrium states.
12.12 PHYSICS (Vol. II)—XI

(ii) Quasi-static process. Non-equilibrium states of a system are difficult


to deal with. It is, therefore, convenient to imagine an idealised process in which
at every stage the system is in an equilibrium state. Such a process is, in principle,
infinitely slow-hence the name quasi-static (meaning nearly static).Quasi-static
process is that infinitely slow process in which the system changes
its variables (P, T, V) so slowly that it remains in thermal and
mechanical equilibrium with its surroundings throughout. In a quasi-
static process, at every stage, the difference in the pressure of the system and the
external pressure is infinitesimally small. The same is true of the temperature
difference between the system and its surroundings. To take a gas from the state
(P, T) to another state (P, T) via a quasi-static process, we change
the external pressure by a very small amount, allow the system
to equalise its pressure with that of the surroundings and continue
the process infinitely slowly until the system achieves the
pressure P. Similarly, to change the temperature, we introduce
P + P
an infinitesimal temperature difference between the system and (P, T)
the surrounding reservoirs and by choosing reservoirs of
progressively different temperatures T to T, the system achieves
the temperature T.
A quasi-static process is obviously a hypothetical T + T
construct. In practice, processes that are sufficiently slow and
do not involve accelerated motion of the piston, large temperature Fig. 12(a).7. In a quasi-static process, the
gradient, etc. are reasonable approximation to an ideal quasi- temperature of the surrounding reservoir and the
static process. We shall from now on deal with quasi-static external pressure differ only infinitesimally from the
processes only, except when stated otherwise. temperature and pressure of the system.

12(a).09.2. ISOTHERMAL PROCESS


Isothermal process is a thermodynamic process in which the pressure Essential conditions for an
and volume of system change but temperature remains constant.In an isothermal isothermal process to take place
process, the exchange of heat between the system and the surroundings is allowed. (i) The container should be
perfectly conducting to allow
In other words, an isothermal process is carried out by either supplying heat to
free exchange of heat between
the substance or by extracting heat from it. the system and surroundings.
A process has to be extremely slow to be isothermal. (ii) The process should be very slow
Examples. (1) The expansion of a gas in a metallic cylinder placed in a so that there is sufficient time
large reservoir of fixed temperature is an example of an isothermal process. for the exchange of heat.
(Heat transferred from the reservoir to the system does not materially affect the
temperature of the reservoir, because of its very large heat capacity.)
(2) The temperature of a substance remains constant during melting. So,
the melting process is an isothermal process.
(3) When a substance boils, its temperature remains constant. So, boiling
is an isothermal process.
(4) Consider an ideal gas enclosed in a conducting cylinder fitted with a
conducting piston. Let the gas be allowed to expand very very slowly. This shall
cause a very slow cooling of the gas. But heat will be conducted into the cylinder
from the surroundings. Thus, the temperature of the gas remains constant.
If the gas is compressed very very slowly, heat will be produced. But this
heat will be conducted to the surroundings. So, the temperature of the gas shall
remain constant.
FIRST LAW OF THERMODYNAMICS 12.13
Equation of isothermal process
We know that PV = RT [Perfect gas equation]
In an isothermal process, T is constant. T3 > T2 > T1

 PV = constant [... R is constant.]


So, the product of the pressure and volume of a given mass of a perfect gas T3
remains constant in an isothermal process. In other words, Boyle’s law is obeyed P T2
in an isothermal process. T1

A graph between pressure and volume of a given mass of a gas at constant


temperature is known as isotherm or isothermal of the gas. In Fig. 12(a).8, V
three isotherms for a given gas at three different temperatures T1, T2 and T3 are Fig. 12(a).8. Isotherms of a gas.
shown.
12(a).09.3. ADIABATIC PROCESS
Adiabatic process is that thermodynamic process in which pressure, Essential conditions for an
volume and temperature of the system change but there is no exchange of heat adiabatic process to take place
between the system and the surroundings. (i) The container should be
A process has to be sudden and quick to be adiabatic. perfectly insulated so that
In an adiabatic process, the system is insulated from the surroundings. there is no exchange of heat
between the system and
Heat absorbed or released is zero.
surroundings.
Examples. (1) Consider a gas enclosed in a thermally insulated cylinder (ii) The process should be sudden
fitted with a non-conducting piston. If the gas is compressed suddenly by moving so that heat does not get time
the piston downwards, some heat is produced. This heat cannot escape the cylinder. to be exchanged with the
Consequently, there will be an increase in the temperature of the gas. surroundings.
(2) If a gas is suddenly expanded by moving the piston outwards, there
will be a decrease in the temperature of the gas.
(3) Bursting of a cycle tube.
(4) Propagation of sound waves in a gas.
(5) Expansion of hot gases in internal combustion engine.
(6) Expansion of steam in the cylinder of a steam engine.
Adiabatic relation between pressure P and volume V
PV = K ...(1)
where  is the ratio of specific heats (ordinary or molar) at constant pressure and Isothermal

at constant volume.
Adiabatic

Adiabatic

Cp
= P
Cv
If an ideal gas undergoes a change in its state adiabatically from (P1, V1) Isothermal
to (P2, V2), then
V
P1V1 = P2V2
Fig. 12(a).9 shows the P-V curves of an ideal gas for two adiabatic processes Fig. 12(a).9
connecting two isotherms.

Adiabatic relation between volume and temperature

For a perfect gas, PV = RT or

 
From equation (1),  
12.14 PHYSICS (Vol. II)—XI


or  

 T V– 1 = constant

If V1, T1 be the initial and V2, T2 be the final volumes and temperatures
respectively of the gas for an adiabatic change, then

T1V1– 1 = T2V2– 1

Adiabatic relation between pressure and temperature

For a perfect gas, PV = RT or

 
  
From equation (1),    or 


or 
 

TP1 – = constant

If P1, T1 be the original and P2, T2 the final pressures and temperatures
respectively of gas for an adiabatic change, then
T1P11 – = T2P21 –

Initial pressure, P1 = 76 cm of mercury column


–2
or P 1 = 76 × 13.6 × 981 dyne cm
Final pressure, P2 = ?
Example 1. A certain gas at atmospheric pressure is
compressed adiabatically so that its volume becomes half We know that P2V2 = P1V1 

of its original volume. Calculate the resulting pressure in



dyne cm–2. Given :  = 1.4.    
or      
Solution. Initial Volume, V 1 = V (say)    

or P2 = P1 (2)1.4
Final Volume, 
= 76 × 13.6 × 981 × (2) 1.4 dyne cm–2
= 2.7 × 106 dyne cm–2

12(a).09.4. ISOCHORIC PROCESS


It is also known as isovolumic process. Isochoric process is a
thermodynamic process that takes place at constant volume of the system.
The pressure-volume plot for an isochoric process is a vertical straight P
line parallel to pressure axis as shown in Fig. 12(a).10.
Examples. (1) Consider a gas enclosed in a cylinder having rigid walls and
a fixed piston. When heat is added to the gas, there would be no change in the
volume of the gas. V

(2) When a substance melts, the change in volume is negligibly small. So, Fig. 12(a).10
this may be regarded as a nearly isochoric process.
FIRST LAW OF THERMODYNAMICS 12.15

12(a).09.5. ISOBARIC PROCESS


Isobaric process is a thermodynamic process that takes place at constant
pressure.
P
The pressure–volume plot for an isobaric process is a horizontal straight
line parallel to volume axis as shown in Fig. 12(a).11.
Example. Heating of water at atmospheric pressure.

12(a).09.6. CYCLIC AND NON-CYCLIC PROCESSES V


Cyclic process. Cyclic process is that thermodynamic process in which
Fig. 12(a).11
the system returns to its initial stage after undergoing a series of changes.
Non-cyclic process. Non-cyclic process is that process in which the system
does not return to its initial stage.

The following sign conventions are adopted in the study of thermodynamical


process.
(i) Heat gained by a system is taken as positive while that lost by a
system is taken as negative.
(ii) The work done by a system is taken as positive while that done on
the system is taken as negative.
(iii) Increase in the internal energy of a system is taken as positive.
Decrease in the internal energy of system is taken as negative.

(i) Isothermal Process


According to the first law of thermodynamics,
dQ = dU + dW ...(1)
The internal energy of an ideal gas depends only on temperature. In an
isothermal process, temperature remains constant.
 Change in internal energy, dU = 0
From equation (1), dQ = dW
When an ideal gas expands isothermally, it does mechanical work dW
Type of Feature
and absorbs an equivalent amount of heat dQ from the surroundings. Similarly,
processes
when an ideal gas is compressed isothermally by doing a mechanical work dW
on it, it rejects an equivalent quantity of heat dQ to the surroundings. Isothermal Temperature
(ii) Adiabatic Process constant
In an adiabatic process, the system is insulated from the surroundings. Isobaric Pressure
So, heat absorbed or released by the system is zero. constant
 dQ = 0 Isochoric Volume constant
It follows from first law of thermodynamics (dQ = dU + dW) that
Adiabatic No heat flow
dU + dW = 0 between the
system and the
or dU = – dW
surroundings
When a gas expands adiabatically, dW is positive. So, dU must be negative (dQ = 0)
i.e., the internal energy of the system will decrease. This will cause cooling of the
gas. In the case of adiabatic compression, the gas will be heated up.
12.16 PHYSICS (Vol. II)—XI
(iii) Isochoric Process
It follows from first law of thermodynamics that
dQ = dU + PdV
In an isochoric process, V is constant.  dV = 0
In this process, no work is done on or by the gas.

Now, dQ = dU

The heat absorbed by the gas goes entirely to increase the internal energy
of the gas and its temperature. The change in temperature for a given amount of Free expansion
heat is determined by the specific heat of the gas at constant volume. In free expansion, there is no
(iv) Isobaric Process transfer of heat between the system
and surroundings and no work is
In an isobaric process, P is fixed. The work done by the gas is done on or by the system.
W = P(V2 – V1) = R(T2 – T1) dQ = 0 and dW = 0
The heat absorbed goes partly to increase internal energy and partly to do  dU = dQ – dW = 0
work. This is in accordance with first law of thermodynamics.
Due to change in internal energy, the temperature changes. The change
in temperature for a given amount of heat is determined by the specific heat of
the gas at constant pressure.
(v) Cyclic Process
In a cyclic process, the system returns to its initial state. Since internal
energy is a state variable, dU = 0 for a cyclic process.
It follows from first law of thermodynamics,

dQ = dW

The total heat absorbed equals the work done by the system.
(vi) Relation between two Molar Specific Heats of a Gas (Mayer’s
Formula)
Let one mole of an ideal gas be heated at constant volume so that its
temperature increases by dT. Since the gas is heated at constant volume therefore
no work will be done by the gas. The whole of the heat supplied will increase only
the internal energy. Let dU be the increase in internal energy. If Cv is the molar
specific heat at constant volume, then
dU = Cv dT ...(1)
Let one mole of an ideal gas be heated at constant pressure so that the
temperature increases by dT. Let dQ be the heat supplied.
Then, dQ = 1 × Cp × dT = Cp dT ...(2)
where Cp is the molar specific heat at constant pressure.
If dW is the work done by gas, then
dW = PdV ...(3)
where P is the constant pressure at which the gas is heated anddV is the increase
in the volume of the gas.
But dQ = dU + dW [First law of thermodynamics]
 CpdT = Cv dT + PdV ...(4)
[From equations (1), (2) and (3)]
Also, PV = RT [Ideal gas equation]
Differentiating, PdV = R dT [Here, P is constant]
From equation (4), CpdT = Cv dT + R dT

or Cp = C v + R
which is the required relation.
FIRST LAW OF THERMODYNAMICS 12.17
(vii) Equation of Adiabatic Process (Adiabatic Relation between
P and V for Ideal Gas)
Consider one mole of a gas contained in a perfectly non-conducting cylinder
fitted with a non-conducting piston. Let P, V and T be the pressure, volume and
temperature respectively of the gas. Let the gas be compressed adiabatically so dx

that the piston moves inwards through a distancedx. Let A be the cross-sectional
area of the piston.
Force acting on the piston = P × A
Work done, dW = Force × Distance
= PAdx = PdV ...(1)
Fig. 12(a).12. Work done in
where dV is the decrease in the volume of the gas. adiabatic process
The heat generated due to compression causes a rise of temperature dT.
This heat energy is equal to C vdT, where Cv is gram molecular specific heat at
constant volume. This is equal to change in the internal energy of the gas.
 dU = Cv dT ...(2)
According to the first law of thermodynamics,
dQ = dU + dW ...(3)
In an adiabatic process, no exchange of heat between the system and the
surroundings is allowed.
 dQ = 0 ...(4)
 Cv dT + P dV = 0 ...(5) [From (1), (2), (3) and (4)]
For an ideal gas, PV = RT

Differentiating, PdV + VdP = RdT or

From equation (5),  


 
 
or CvPdV + CvVdP + RPdV = 0
or (Cv + R) PdV + CvVdP = 0
But Cp – C v = R or Cp = C v + R
 CpPdV + CvVdP = 0
Dividing both sides by CvPV, we get

 

 
or      
 

Integrating,
  

or  
 
or     
 
or log V + log P = constant or log PV = constant
or PV  = antilog (constant) = another constant K

 PV  = K
12.18 PHYSICS (Vol. II)—XI
(viii) Melting Process
When a substance melts, the change in volume (dV) is very small and
can, therefore, be neglected. The temperature of a substance remains unchanged
during melting process.
Consider the melting of a mass m of the solid. Let L be the *latent heat of
fusion i.e., the heat required to change a unit mass of a solid to liquid phase at
constant temperature.
Heat absorbed during melting process, dQ = mL
According to the first law of thermodynamics,
dQ = dU + dW
or mL = dU [... dW = PdV = P × 0 = 0]
So, the internal energy increases by mL during the melting process.
Note. The latent heat of a substance is related to the increase in the
internal energy per unit mass.
(ix) Boiling Process
When a liquid is heated, it changes into vapour at constant temperature
(called boiling point) and pressure. When **water is heated at normal atmospheric
pressure, it boils at 100°C. The temperature remains unchanged during the
boiling process.
Consider the vaporisation of liquid of massm. Let Vl and Vv be the volumes
of the liquid and vapours respectively.
Then, the work done in expanding at constant temperature and
pressure P,
dW = PdV = P (Vv – Vl)
Let L be the latent heat of vaporisation, i.e.,the heat required to change a
unit mass of a liquid to vapour phase at constant temperature and pressure.
 Heat absorbed during boiling process, dQ = mL
Let Ul and Uv be the internal energies of the liquid and vapours respectively.
Change in internal energy, dU = Uv – Ul
According to the first law of thermodynamics,
dQ = dU + dW
 mL = (Uv – Ul) + P(Vv – Vl)
or Uv – Ul = mL – P(Vv – Vl)
Knowing all the quantities on right hand side, we can calculate the gain
in the internal energy.

Pressure, P = 1 atmosphere = 0.76 m of mercury


= 0.76 × 13600 × 9.8 N m–2
[... density of mercury = 13600 kg m–3]
Example 2. 1 kg of water at 373 K is converted into Small work done,
steam at same temperature. Volume of 1 cm3 of water
dW = PdV = 0.76 × 13600 × 9.8 × 1.67 J
becomes 1671 cm3 on boiling. What is the change in the
internal energy of the system if the latent heat of 
or dW =  cal
vaporisation of water is 5.4 × 105 cal kg–1 ?
4
Solution. Volume of 1 kg of water = 4.026 × 10 cal
= 1000 cm3 = 103 cm3 = 10–3 m3 But dU = dQ – dW
Volume of 1 kg of steam = 10 –3 × 1671 m3 (First law of thermodynamics)
= 1.671 m 3 or dU = (5.4 × 105 – 0.4026 × 105) cal
Change in volume, dV = (1.671 – 10 –3) m3 = 1.670 m 3 = 4.9974 × 105 cal

*The latent heat of fusion of ice is 80 cal g –1 or k cal kg –1 at normal atmospheric pressure.
**The latent heat of vaporisation of water is 540 k cal kg –1 or 540 cal g –1 under normal atmospheric pressure. It varies with
pressure.
FIRST LAW OF THERMODYNAMICS 12.19

Cv for a mixture of gases


Let n1 moles of a gas A be mixed with n2 moles of gas B. Let Cv and Cv
1 2
be the specific heats of A and B respectively. Let the temperature be raised
through dT. Change in internal energy of A, dU1 = n1 Cv dT; change in internal
1
energy of B, dU2 = n2Cv dT; change in internal energy of mixture, dU = dU1 + dU2.
2
Let Cv be the specific heat of the mixture.
Then (n1 + n2) Cv dT = n1Cv dT + n2Cv dT
1 2

or Cv =

Cp for a mixture of gases

Cp = C v + R = +R

 
=


=

The equation of state of a gas is PV = RT. Out of the three variables P, V A(P 1, V 1)
and T, if any two are known, the third can be calculated. So, two thermodynamic
variables are sufficient to describe the behaviour of a thermodynamic system.
P-V diagram is a graph between the volume V of a system and the pressure P B(P2, V2)

P of the system. The volume is plotted against X-axis while the pressure is
plotted against Y-axis. O V X

Fig. 12(a).13 shows an indicator or P-V diagram. The point A represents Fig. 12(a).13. Indicator diagram
the initial stage of the system. P1 and V1 are the initial pressure and initial
volume respectively of the system. The point B represents the final state of the For one complete cycle as shown
system. P2 and V2 are the final pressure and final volume respectively of the in the P-V diagram, are (a) Eint.
for the gas and (b) the net heat
system. The points between A and B represent the intermediate states of the
transfer Q positive, negative, or
system.
zero ?
The indicator diagram helps us to calculate the amount of work done by
the gas or on the gas during expansion or compression.

Let us calculate the work done in the case of (a) a non-cyclic process V
(b) cyclic process. Fig. 12(a).14
(a) Work done in non-cyclic process. Consider the expansion of an Answer. (a) Zero (b) Negative.
ideal gas enclosed in a perfectly insulated cylinder fitted with a non-conducting Explanation. For the case of
and frictionless piston. Let P be the pressure of the gas at any time. Let A be the closed cycle, E int. = 0. W net < 0,
cross-sectional area of the piston. Q = W.
12.20 PHYSICS (Vol. II)—XI

Force exerted by the gas on the piston = P × A


Let the gas push the piston up through an infinitesimally small distance
dx. Let dW be the corresponding work done by the gas.
Then, dW = P × A × dx
or dW = PdV
where dV is the infinitesimally small increase in the volume of the gas.
Here, it is assumed that the pressure of the gas remains constant during
the infinitesimally small expansion of the gas. (FINAL
The total work W done by the gas in expanding from initial state i (Pi, Vi) STATE)

to final state f (Pf , Vf) is given by

 (INITIAL
STATE)
The whole operation is represented in the indicator diagram. At any point
‘a’, let P and V be the pressure and volume respectively of the gas. The Fig. 12(a).15. Expansion of a gas
infinitesimally small increase dV in volume is represented by cd. The points a
and b on the curve are so close to each other that the shaded strip may be regarded
as a rectangle of area PdV. So, the area of the strip gives the work dW done by
the gas in expanding from volume Oc to volume Od [Fig. 12(a).16].
i(P ,V )
The whole of the area between the P-V diagram and the volume-axis can POSITIVE
P WORK
be divided into a large number of small strips. The sum of the areas of these
strips gives the work W done by the gas during expansion from volume V i to a b f(P , V )
volume Vf . P

Discussion. (i) The work done by an expanding gas is equal to the area P
enclosed between the P-V diagram and the volume-axis. c d
(ii) The two points i and f can be joined by a number of different curves. O V
V
V
The areas enclosed by these curves will be different. But the ‘area’ determines Fig. 12(a).16. PV diagram of
the work done by the gas. Thus, the work done by a gas in a non-cyclic process is expansion of a gas
a function of the path chosen.
(iii) If the gas is taken from the state f (Pf , Vf) to the state i (Pi , Vi), along
the path f to i by compressing the gas, then the work done will be given by

   

= – (area between the PV diagram and the volume-axis)


So, the work done on the gas is negative.
(iv) If the gas is taken first along the path i to f and then back from f to
i along the same path, then the net work done on the system is zero.
(b) Work done in cyclic process. Consider an ideal gas enclosed in a
cylinder fitted with a piston. Let the gas undergo a succession of changes in Fig. 12(a).17. PV diagram of
pressure and volume such that it goes from i (Pi , Vi) to f (Pf , Vf) along the path compression of a gas
i j f and back to the initial state i under different conditions represented by the
Y
path f k i. Let W1 be the work done by the gas during expansion from volume V i i(P ' V )
to volume Vf . P
j
 W1 = area i j f m l i
Let W2 be the work done on the gas during compression from volume Vf to P
k
f(P ' V )
volume Vi.
P
 W2 = area f k i l m f
V V
If W be the total work done during the cyclic process, then O l m X
W = area i j f m l i– area f k i l m f = area i j f k i V
= area of the loop Fig. 12(a).18. PV diagram of a cyclic process
FIRST LAW OF THERMODYNAMICS 12.21

Discussion. (i) The net work done during the cyclic process is not zero.
This is because the gas receives heat during compression which is converted into
work.
(ii) The work done per cycle is numerically equal to the area of the loop
representing the cycle.
(iii) If the loop is traced in clockwise direction, its area represents the
work done by the system. So, the work done is positive.
(iv) If the loop is traced anticlockwise, its area represents the work done
on the system by some external agency. So work done is taken as negative.

Then, WAB = area of trapezium ABEDA

= (DA + EB) CB
Example 3. One mole of an ideal gas undergoes a
cyclic change ABCD. Calculate the net work done in the = (4 + 2) N m–2 × *3 × 10–3 m3
process. Given : 1 atmosphere = 106 dyne cm–2.
= 9 × 10–3 J
Solution. Since the loop ABCD is traced in the clockwise
direction therefore the work done is positive. It is a case of an expanding gas. So, work done is positive.
Work done, W = + area ABCD (ii) Let WBC be the work done during the process from
B to C.
= DC × DA
= (4 – 1) litre × (5 – 2) atm 5
= 3 litre × 3 atm E B (2, 4)
VOLUME V (IN LITRE)
4
A B
5 3

4 2
(IN ATMOSPHERE)

D
PRESSURE P

3 1 A (4, 1)
C (2, 1)
2 D C
0 1 2 3 4 5
–2
1 PRESSURE P (IN N m )

Fig. 12(a).20
0 1 2 3 4 5 Then, WBC = area of rectangle BCDEB
VOLUME V
(IN LITRE) = DC × DE
= 2 N m–2 × 3 × 10–3 m3
Fig. 12(a).19
= 6 × 10 –3 J
= 3 × 103 cm3 × 3 × 106 dyne cm–2
In this case, the gas is being compressed. So, the work
= 9 × 109 erg
done is negative.
Example 4. The P–V diagram for a cyclic process is
a triangle ABC drawn in order. The co-ordinates of A, B (iii) Let WCA be the work done during the process from
and C are (4, 1), (2, 4) and (2, 1) respectively. The C to A. Corresponding area is zero.
co-ordinates are in order P, V. Pressure is in N m –2 and Then, WCA = zero
volume is in litre. Calculate the work done during the
(iv) Work done during the cycle ABCA
process from A to B, B to C and C to A. Also calculate the
work done in complete cycle. = WAB + WBC + WCA
Solution. (i) Let WAB be the work done during the = 9 × 10 –3 J + (–6 × 10–3 J) + 0 = 3 × 10–3 J
process from A to B.

*1 litre = 10–3 m3
12.22 PHYSICS (Vol. II)—XI

Fig. 12(a).21. Work done in


isothermal process


RT

V


   

     

  

   

 

 
FIRST LAW OF THERMODYNAMICS 12.23

 
  

 
   

   

    





 

If W


K
V
12.24 PHYSICS (Vol. II)—XI

V2 1
V dV
1 V
V2
V V  dV
1

V2
V   1
  1 V1

V2
K 1
1  V  1 V1

K  1 1 
  1   1 
1    V2 V1 

1  K K 
  1   1 
1    V2 V1 
 

1  P2 V2  P1V1 
  1   1 
1   V2 V1 

1
1 
 
B ut P   

R
1 

R
 1

R
 1

 



FIRST LAW OF THERMODYNAMICS 12.25


   

 

  


 

12.26 PHYSICS (Vol. II)—XI

   

 
      
     


   
      
    

   
   
   

VERY SHORT ANSWER QUESTIONS—Each Question Carries 1 Mark Only


FIRST LAW OF THERMODYNAMICS 12.27

SHORT ANSWER QUESTIONS—Each Question Carries 2 Marks


12.28 PHYSICS (Vol. II)—XI

SHORT ANSWER QUESTIONS—Each Question Carries 3 Marks

dP P

dV V

dP
dV

 
   
A Isotherm  
B

C
Adiabatic
 

 



 
 

Fig. 12(a).22. Works done in isothermal and
adiabatic expansions.
  
  
 

     
       
     


C Adiabatic
B A

Isotherm

Fig. 12(a).23. Works done in isothermal and


adiabatic compressions.
FIRST LAW OF THERMODYNAMICS 12.29

V
2


P

V   

Fig. 12(a).24

  
 


  
 
 
 


   

   
 

   
12.30 PHYSICS (Vol. II)—XI

   

C D
Pressure/Pa

200000
C B B A
100000 A
B
V P
0 0.0001 0.0003 A C
3
Volume/m

Fig. 12(a).25 T V

Fig. 12(a).27 Fig. 12(a).28




   

300
V (in cc)

100

0 100 300 A B
P (in kPa)
Pressure (P)

Fig. 12(a).26
C
D

 P   V 
    
 2   2  V1 V2
Volume (V)
 300  100   300  100 
  Fig. 12(a).29
   2 
 2    
FIRST LAW OF THERMODYNAMICS 12.31

 


P     
   
   
1
2

3 T3   
 
T2  
4 5
T1

Fig. 12(a).30

 

a b T1

d c T2

Va Vd Vb Vc
V

Fig. 12(a).31
12.32 PHYSICS (Vol. II)—XI

LONG ANSWER QUESTIONS

A
40

30 B
Pressure (Pa)

C
20

10

0 1.0 2.0 3.0 4.0


3
Volume (m )

Fig. 12(a).32
FIRST LAW OF THERMODYNAMICS 12.33

A B
5
50

Pressure (N/m )
4

2
C
(IN ATMOSPHERE)

40
PRESSURE P

3
30
2 D C
20
1
10
A B
0 1 2 3 4 5
VOLUME V (IN LITRE) 0 1 2 3 4 5 6
3
Volume (m )
Fig. 12(a).33
Fig. 12(a).34

A B
15
P (N/m )
2

12

9
6
D C
3
 0 1 2 3 4 5 6 7 8
V (litre)

Fig. 12(a).35



12.34 PHYSICS (Vol. II)—XI

R
20

Pressure (N/m )
15

2
 10

5
Q P
N M
0 5 10 15 20 25 30
3
Volume (m )

50 Fig. 12(a).37
B C
40

30
P (N/m )
2

20

10 D
A

0 1 2 3 4 5 6
3
Volume (m )
Fig. 12(a).36


 

 


   

    
 
 
    
 

  

 

FIRST LAW OF THERMODYNAMICS 12.35

  
 
 
 

   



 

   
 
  
  
     

  
    
    
    
   
  
    
    
 
   
 

 
 
    

      
   
     
    
 

 
    
     
  
     
     
    
    

 
  
W = 1.01 × 10   
 
  
  
  


  
  
   


 
12.36 PHYSICS (Vol. II)—XI

  –1
 V1 
 
 V2 

5
–1
 V 3
 
2 2 V

5

3

1 300
3/2 2/3 2
(2 )

 

3
2

  7
 
   = 5 
 
 

   
    
    


 
 
 

 
 
 

  
  

 

1m
  

Fig. 12(a).38

  4
TV 
  3

FIRST LAW OF THERMODYNAMICS 12.37


   
  
 

 
 
 

  


 
   
 

  

   

C B 

P
A D

V

Fig. 12(a).39 
12.38 PHYSICS (Vol. II)—XI

 
  


    
  
   




  

 

TB  
 
5  
p/10 Pa Isothermal

A B Total work done


2.0
by gas

VA VB VC
0 –2 3
0 0.5 1 1.5 V/10 m

Fig. 12(a).40


 


   
   
   


  

FIRST LAW OF THERMODYNAMICS 12.39
 


 
 
 

   
 
 

 
 
 

   

 


P
Open atmosphere B C
2P1

Heater
P1 A
Rigid support

Fig. 12(a).41 0 T
T1 2T1

Fig. 12(a).42


   
   
 



 
 

 
  
 
 

    
 
   
12.40 PHYSICS (Vol. II)—XI

VALUE BASED QUESTION


(i) Introduction. It is very easy to convert work into heat. As an
example, when two bodies are rubbed, heat is produced. However, for the conversion
of heat into work, we require a device. This device is called heat engine.
Heat engine is a device by which a system is made to undergo a
cyclic process that results in conversion of heat to work.
(ii) Principle. A system whose different parts are at different
temperatures tends to change towards the state of thermal equilibrium.
(iii) Construction. A heat engine consists essentially of the following
parts :
1. Hot reservoir or source. It is the supplier of heat energy. It is
maintained at very high temperature (say, T 1).
2. Cold reservoir or sink. That heat which has not been converted into
work is rejected to the cold reservoir. It is maintained at low temperature (say,
T2).
3. Working substance—the system. For example, a mixture of fuel
vapour and air in a gasoline or diesel engine or steam in a steam engine are the
working substances.
(iv) Working. The working substance goes through a cycle consisting of
several processes. In some of these processes, it absorbs a total amount of heat Q1
from an external reservoir at some high temperature T 1.
In some other processes of the cycle, the working substance W
releases a total amount of heat Q2 to an external reservoir at some
lower temperature T2. Hot Q1 Q2 Cold
Reservoir Reservoir
The work done (W) by the system in a cycle is transferred to T1 T2
the environment via some arrangement (e.g., the working substance
may be in a cylinder with a moving piston that transfers mechanical
Fig. 12(b).1. Schematic representation of a heat
energy to the wheels of a vehicle via a shaft). engine. The engine takes heat Q 1 from a hot
The cycle is repeated again and again to get useful work for reservoir at temperature T 1, releases heat Q 2 to a
some purpose. cold reservoir at temperature T 2 and delivers work
The basic features of a heat engine are schematically represented W to the surroundings.
in Fig. 12(b).1.
(v) Thermal Efficiency of Heat Engine. It is defined as the ratio of the
work done, by the system on the environment in one complete cycle to the heat
absorbed by the system in one cycle. It is denoted by .
The work W obtained in each cycle is called the output while the heat Q 1
absorbed by the system in one cycle is the heat input.

12.41
12.42 PHYSICS (Vol. II)—XI

  

Since the working substance returns to its initial state after completing
one cycle therefore there is no change in internal energy. So, applying first law of
thermodynamics, we get
Q1 – Q 2 = W
Here, Q1, Q2 and W are all measured in the same units.


  

Discussion. (i) For a given value of Q 1, the smaller the value of Q2, the
higher is the efficiency of the heat engine.
(ii) For Q2 = 0,  = 1, i.e., the engine will have 100% efficiency in
converting heat into work. In such a case, the whole of the heat extracted from
the source will be converted into mechanical work. No heat will be rejected to the
cold reservoir. Note that the First Law of Thermodynamics i.e., the energy
conservation law does not rule out a 100% efficient heat engine. But experience
shows that such an ideal engine with  = 1 is never possible, even if we can
eliminate various kinds of losses associated with actual heat engines. It turns
out that there is a fundamental limit on the efficiency of a heat engine set by an
independent principle of nature, called the Second Law of Thermodynamics.
(iii) For the sake of discussion, let us discuss the purely hypothetical
case of an engine whose efficiency is greater than 1. Such a *heat engine will
extract heat both from the source and the sink.
(iv) In actual practice, the efficiency of heat engine does not exceed 55%,
i.e., at the most only 55% of the total heat energy extracted from the source is
converted into mechanical work. Total efficiency of a steam engine varies from
12% to 17%. The maximum efficiency of a diesel engine is 40% while that of a
petrol engine is only up to 26%.
(v) The low efficiency of a heat engine is not due to the bad designing of
a heat engine but it is consequence of a certain law of nature. It was shown by a
French engineer Sadi Carnot that there is something in nature which forbids
the complete conversion of heat into work.
Types of Heat Engines
The mechanism of conversion of heat into work varies for different heat
engines. Basically, there are two ways : the system (say a gas or a mixture of
gases) is heated by an external furnace, as in a steam engine; or it is heated
internally by an exothermic chemical reaction as in an internal combustion
engine. The various steps involved in a cycle also differ from one engine to another.
There are basically two types of heat engines—External combustion engine
and Internal combustion engine.
Steam engine is an example of external combustion engine. In such an
engine, the fuel is burnt outside the working portion of the engine.
Internal combustion engines are used in motor cars and aeroplanes. Petrol
engine and Diesel engine are examples of internal combustion engines. In such
engines, the fuel is burnt within the working portion of the engine.
SECOND LAW OF THERMODYNAMICS 12.43

The first law of thermodynamics gives us an equivalence of the work done


and the heat produced. It merely tells us that heat and work are interconvertible
and the rate of exchange is fixed, i.e., 4.2 joule is required to produce 1 calorie of
heat. This law suffers from the following limitations :
1. It does not indicate the direction in which the change can
proceed.
Illustrations. (i) When a hot body is brought in thermal contact with a
cold body, heat always flows from the hot body to the cold body. Why heat does
not flow from the cold body to the hot body ? The first law of thermodynamics is
silent about it. So, this law does not indicate the direction of heat transfer.
(ii) When brakes are applied to a moving car, the wheels stop rolling.
The work done against friction is converted into heat. Why this heat is not
reconverted into work so that the wheels may start rolling again? The first law
of thermodynamics has no answer to this question. So, the first law of
thermodynamics does not tell us anything about the direction in which certain
process can take place.
(iii) It is not possible for a ship to use the huge amounts of heat of the sea
waters to operate its engine. What prevents the conversion of heat into work ?
The first law is again silent about this. So, the first law of thermodynamics does
not specify the conditions under which heat is converted into work.
2. The first law of thermodynamics gives no idea about the extent
to which the change takes place.
It has been observed that no heat engine can convert all the heat extracted
from the source into mechanical energy. Why the whole of the heat cannot be
converted into mechanical energy ? The first law is silent about this question.
These limitations led to the formulation of another law called the“second
law of thermodynamics”.

Entropy is that physical quantity


which remains constant when the
This law specifies the conditions for the conversion of heat into work. substance undergoes a reversible
There are several statements of this law but the following two are the most adiabatic change.
significant : Entropy, S = k log W
(i) Kelvin-Planck Statement where W is the thermodynamic
probability.
No process is possible whose sole result is the absorption of heat

from a reservoir and the complete conversion of the heat into work. dS =
“It is impossible to construct an engine, operating in a cycle, which will According to second law of
produce no effect other than extracting heat from a reservoir and performing thermodynamics, if a process
an equivalent amount of work.” In simple words, it is not possible to get a occurs in a closed system, the
continuous supply of work from a body by cooling it to temperature lower than entropy increases for irreversible
process, remains constant for
that of the surroundings.
reversible process.
This form of the law is applicable to heat engines. The working substance So, dS  0.
of a heat engine, operating in a cycle, cannot convert all the extracted heat into This may be regarded as another
work. It must reject a part of the heat to the sink at a lower temperature. So, in form of second law of thermo-
order to convert heat into work, it is necessary to have both source and sink. dynamics.
12.44 PHYSICS (Vol. II)—XI
Since all the heat extracted from the source can never be converted into work
therefore the efficiency of the engine is never* one.
(ii) Clausius Statement
No process is possible whose sole result is the transfer of heat from
a colder object to a hotter object.
“It is impossible to make heat flow from a body at a lower temperature to
a body at a higher temperature without doing external work on the working
substance”. In simple words, heat cannot, by itself, flow from a body at a lower
temperature to a body at a higher temperature.
This form of the law is applicable to ice plants and refrigerators. The
refrigerant absorbs heat from inside the refrigerator and rejects a greater quantity
of heat to the surroundings (at higher temperature) with the help of an external
agency, say an electric motor. In ammonia ice plant, heat is absorbed from the
brine solution at a lower temperature and rejected into water at a higher
temperature. This is achieved with the help of an external agency like a pump.

The Second Law of Thermodynamics gives a fundamental limitation


to the efficiency of a heat engine and the coefficient of performance of a
refrigerator. In simple terms, it says that efficiency of a heat engine can never be
unity. This implies that heat released to the cold reservoir can never be made
zero. For a refrigerator, the Second Law says that the coefficient of performance
can never be infinite. This implies that external work (W) can never be zero. The
two statements, one due to Kelvin and Planck denying the possibility of a perfect
heat engine and another due to Clausius denying the possibility of a perfect
refrigerator or heat pump, are a concise summary of these observations.

It appears as if the two statements of second law of thermodynamics are


unrelated to each other. But the two statements are equivalent. Any violation of
one statement will violate the other statement.
Suppose we can get a continuous supply of work by cooling a body to a
temperature lower than that of the coldest surroundings. This violates Kelvin’s
statement. Suppose the continuous supply of work is used to produce electric
current with the help of a generator. When the electric current is passed through
a conducting wire, heat will be produced. So, it will be possible for us to transfer
heat from a lower temperature to a higher temperature without doing external
work. This violates Clausius statement. Similarly, if Clausius statement is
violated, the Kelvin statement will also be violated. Thus, we conclude that the
two statements of second law of thermodynamics are equivalent.
Verification. Both the statements of second law of thermodynamics are
negative statements. So, they are incapable of direct experimental proof. The
greatest proof of the second law of thermodynamics is that no violations have
been so far noted.
SECOND LAW OF THERMODYNAMICS 12.45

(i) The law cannot be proved directly. But its validity has not been
contradicted by any machine designed so far.
(ii) It is applicable only to a cyclic process in which the system returns to
its original state after a complete cycle of changes.
(iii) It makes no prediction as to what will happen under certain conditions.
It simply states what will happen under a given set of conditions.

A thermodynamic process (state i  state f) is reversible if the


process can be turned back such that both the system and the
surroundings return to their original states, with no other change
anywhere else in the universe.
It is that process which can be retraced in the opposite direction so that
the system and the surroundings pass through exactly the same state at each
stage as in the direct process. If some heat is absorbed from the surroundings in
the direct process, the same amount of heat is given out in the reverse process.
Similarly, if some work is done by the system in the direct process, the same
amount of work is done on the system in the reverse process. At the end of a
reversible process, the system and the surroundings should be restored to their
initial stage.
A reversible process is an idealised notion. A process is reversible only if it
is quasi-static (system in equilibrium with the surroundings at every stage) and
there are no dissipative effects. For example, a quasi-static isothermal expansion
of an ideal gas in a cylinder fitted with a frictionless movable piston is a reversible
process.
In order that a process may be reversible, it should satisfy the following
conditions.
(i) The process should proceed at an extremely slow rate so that
the following requirements are met.
(a) The system should remain in mechanical equilibrium, i.e., there i
should be no unbalanced force either within the system or between the system
and the surroundings. The pressure of the working substance must not differ
appreciably from that of the surroundings at any stage of the cycle of operations.
P
(b) The system should remain in thermal equilibrium, i.e., all parts of
the system and the surroundings should remain at the same temperature. The
f
temperature of the working substance must not differ appreciably from that of
the surroundings at any stage of the cycle of operations. V
(c) The system should remain in chemical equilibrium i.e., the internal Fig. 12(b).2. PV diagram of a reversible
structure should not change due to chemical reaction, diffusion etc. In other process
words, no new products should be formed in the process.
(ii) No dissipative forces (friction, inelasticity, viscosity,
electrical resistance etc.) should be present.
A completely reversible process is only an idealised concept meant only for
theoretical considerations. In actual practice, there is always some loss of heat
due to friction, conduction, radiation etc. In the following examples, the conditions
for a reversible process are only approximately satisfied.
(1) When current is passed through an electrolyte, the ions are formed.
The positive ions move towards the cathode while the negative ions move towards
12.46 PHYSICS (Vol. II)—XI
anode. If the direction of electric current is reversed, the direction of motion of
ions will also be reversed. So, electrolysis is a reversible process. Here it is
assumed that the electrical resistance offered by an electrolyte is zero.
(2) When an elastic spring is compressed gradually, some work is done on
the spring. When the applied force is reduced, the spring expands and does nearly
the same amount of work as was initially done upon it. So, slow compression
or extension of a spring is a reversible process.
Why is reversibility such a basic concept in thermodynamics ?
As we have seen, one of the concerns of thermodynamics is the efficiency with
which heat can be converted into work. The Second Law of Thermodynamics
rules out the possibility of a perfect heat engine with 100% efficiency. But what
is the highest efficiency possible for a heat engine working between two reservoirs
at temperatures T 1 and T2 ? It turns out that a heat engine based on idealised
reversible processes achieves the highest efficiency possible. All other engines
involving irreversibility in any way (as would be the case for practical engines)
have lower than this limiting efficiency.

It is a process which is not exactly reversed, i.e., the system does not
pass through the same intermediate state as in the direct process.
Imagine some process in which a thermodynamic system goes from an
initial state i to a final state f. During the process the system absorbs heat Q
from the surroundings and performs work W on it. Can we reverse this process
and bring both the system and surroundings to their initial states with no other i
effect anywhere? Experience suggests that for most processes in nature this is
not possible. The spontaneous processes of nature are irreversible.
Irreversibility is a rule rather an exception in nature. Irreversibility arises P
mainly from two causes: one, many processes (like a free expansion, or an
explosive chemical reaction) take the system to non-equilibrium states; two, most f

processes involve friction, viscosity and other dissipative effects (e.g., a moving
V
body coming to a stop and losing its mechanical energy as heat to the floor and
Fig. 12(b).3. PV diagram of irreversible
the body; a rotating blade in a liquid coming to a stop due to viscosity and losing
process
its mechanical energy with corresponding gain in the internal energy of the
liquid). Since dissipative effects are present everywhere and can be minimised
but not fully eliminated, most processes that we deal with are irreversible.
Some examples are as follows :
(1) All chemical reactions are irreversible because they involve
changes in the internal structure of the constituents. The constituents can be
restored only when energy is supplied from outside. Rusting of iron is one example
of a chemical reaction. When iron rusts, iron is converted to iron oxide. Iron and
oxygen can be separated only by supplying energy from outside. So, rusting of
iron is an irreversible process.
(2) The decay of organic matter is an irreversible process.
(3) When an electric current flows through a conductor, some heat is
produced. If the electric current is made to flow in the reverse direction, some
heat will be again produced. So, flow of current through a conductor is an
irreversible process.
(4) When a body is slided along a rough surface, work is done against the
force of friction. When the body is slided in the reverse direction, some work is
again done against the frictional force. So, it is an irreversible process.
SECOND LAW OF THERMODYNAMICS 12.47
(5) The base of a vessel on an oven is hotter than its other parts. When the
vessel is removed, heat is transferred from the base to the other parts, bringing
the vessel to a uniform temperature (which in due course cools to the temperature
of the surroundings). The process cannot be reversed; a part of the vessel will not
get cooler spontaneously and warm up the base. It will violate the Second Law of
Thermodynamics, if it did.
(6) The free expansion of a gas is irreversible.
(7) The combustion reaction of a mixture of petrol and air ignited by a
spark cannot be reversed.
(8) Cooking gas leaking from a gas cylinder in the kitchen diffuses to the
entire room. The diffusion process will not spontaneously reverse and bring the
gas back to the cylinder.
(9) The stirring of a liquid in thermal contact with a reservoir will convert
the work done into heat, increasing the internal energy of the reservoir. The
process cannot be reversed exactly; otherwise it would amount to conversion of
heat entirely into work, violating the Second Law of Thermodynamics.

A reversible heat engine operating between two temperatures is


called a Carnot engine.
It is an ideal heat engine which is free from all the imperfections
of an actual engine. So, it cannot be realised in actual practice. It was
conceived by *Niolas Le’onard Sadi Carnot, a French Engineer. This
engine serves us a standard by which the performance of actual engines
can be judged. lt consists essentially of the following parts.
(i) Source. It serves as source of heat. It is maintained at a
constant high temperature T1 K. It has infinite thermal capacity, i.e.,
SOURCE SINK
any amount of heat may be extracted from it at a constant temperature (T1) (T2)
STAND
T1 .
(ii) Sink. It is a cold body maintained at constant low
temperature T2 K. It also has infinite thermal capacity, i.e., any amount Fig. 12(b).4. Carnot’s Ideal Heat Engine
of heat rejected to it will not affect its temperature.
(iii) Insulating stand. It is a perfectly non-conducting pad.
(iv) A cylinder with perfectly non-conducting walls but with a perfectly
conducting bottom. It is fitted with a perfectly non-conducting and frictionless
piston over which some weights are placed. One mole of an ideal gas is enclosed
in the cylinder. The ideal gas acts as the working substance.
The working substance is subjected to the following four successive
reversible operations so as to complete a reversible cycle. This cycle is called
Carnot’s cycle. The reversibility of operations is a very important assumption
because our aim is to find out the maximum efficiency attainable by engine
where all sources of irreversibilities are absent.
To begin with, let the pressure, volume and temperature be P 1, V1 and T1
respectively. The state of the working substance is represented by the point A in
the P-V diagram. The following sequence of steps constitute one cycle called the
Carnot cycle.
12.48 PHYSICS (Vol. II)—XI
(a) Step 1  2: Isothermal expansion of the gas taking its state
from (P1, V1, T1) to (P2, V2, T1)—OPERATION I.
The cylinder is placed on the source. The piston is allowed to move out
A (P1, V1, T1)
infinitely slowly by reducing very gradually the weights on the piston. The gas
B (P2, V2, T1)
expands extremely slowly. As the gas expands, its temperature tends to fall. But
since it is in thermal contact with the heat source therefore it will extract a Q1
P
certain amount of heat Q 1 from the source. In this way, the temperature of the
gas will remain T1 throughout the process of expansion. In other words, the gas C (P3, V3, T2)
expands isothermally at temperature T 1 K. This isothermal expansion is Q2
represented by the curve AB on the indicator diagram. Let W 1 be the work done
by the gas in expanding from volume V 1 to volume V 2. The pressure decreases
O a d b c
from P1 to P2.
V
Applying first law of thermodynamics, Fig. 12(b).5. Carnot cycle for a heat
engine with an ideal gas as the working
      substance.

The cylinder is placed on the insulated stand and the piston is allowed to
move out. The gas expands adiabatically from volume V 2 to volume V 3 till its
temperature falls to T2 K.
The pressure falls from P2 to P3.
The adiabatic expansion is represented by the curve BC in the indicator
diagram.
The work done by the gas is given by

      


The cylinder is placed on the sink and the gas is isothermally compressed
until the pressure and volume become P4 and V4 respectively. The operation is
represented by the isothermal curve CD. The heat Q2 developed in compression
is absorbed by the sink. Let W3 be the work done on the gas.
Applying first law of thermodynamics,

      

The cylinder is placed on the insulating stand and the gas is compressed
adiabatically till it attains its initial pressure P1 , volume V1 and temperature
T1. The adiabatic compression is represented by the curve DA in the indicator
diagram. Let W4 be the work done on the gas.

Then      

Let W be the net external work done by the working substance during one
cycle.
Then, W = Work done by the gas – Work done on the gas
= W1  2 + W2  3 – W3  4 – W4  1 = W1  2 – W3  4
[... W2  3 = W4  1]
= area AB ba A – area C cd DC = area ABCDA
SECOND LAW OF THERMODYNAMICS 12.49
The working substance can be taken through the cycle again and again.
In this way, more and more work can be done by the engine.
Thermal Efficiency of a Carnot engine is defined as the ratio of the
external work done in one cycle to corresponding amount of heat extracted
from the source.
Since the working substance is restored to its initial state therefore there
is no change in its internal energy.
Applying first law of thermodynamics,
W = Q1 – Q2
W, Q1 and Q2 are all measured in the same units, i.e., either in units of
heat or in units of work.

Thermal efficiency,  


or   = 

Here,    and   

F I
GH JK 





Now, 
F I or 
 
...(1)

GH JK 


The points B and C lie on the same adiabatic.


 
  ...(2)
The points A and D lie on the same adiabatic.
 
  ...(3)
Dividing (2) by (3), we get

F I 
F I 

GH JK G J
H K
or 

 From equation (1),     

Discussion. (i) The efficiency of the Carnot’s ideal engine is independent of


the nature of the working substance. It depends only upon the temperatures of the
source and sink. The greater the difference between the two temperatures, higher
is the efficiency of the Carnot engine.
(ii) Efficiency is the same for all reversible engines working between
temperatures T1 and T2.
(iii)  is always less than one. The value of  can be one only if T2 = 0 i.e.,
if the sink is at absolute zero of temperature. Since the absolute zero of
temperature cannot be attained therefore  cannot be equal to one.
(iv) When T2 = T1, then  = 0.
12.50 PHYSICS (Vol. II)—XI

So, heat cannot be converted into work without a temperature difference. In


other words, heat can be converted into work only if a sink at a lower temperature
is available. This explains as to why the large amount of heat energy of sea-water
cannot be used for deriving mechanical work.

W
A refrigerator is the reverse of a heat engine. Here the working
substance extracts heat Q2 from the cold reservoir at temperature T 2, Hot Cold
Q1 Q2
some external work W is done on it and heat Q 1 is released to the hot Reservoir Reservoir
T1 T2
reservoir at temperature T 1 (Fig. 12(b).6).
A heat pump is the same as a refrigerator. What term we use Fig. 12(b).6. Schematic representation of a
refrigerator or a heat pump, the reverse of a heat
depends on the purpose of the device. If the purpose is to cool a portion
engine.
of space, like the inside of a chamber and higher temperature reservoir
is surrounding, we call the device a refrigerator; if the idea is to pump heat into
a portion of space (the room in a building when the outside environment is cold),
the device is called a heat pump.
In a refrigerator, the working substance (usually, in gaseous form) goes
through the following steps :
(a) Sudden expansion of the gas from high to low pressure which cools it
and converts it into a vapour-liquid mixture.
(b) Absorption by the cold fluid of heat from the region to be cooled converting
it into vapour.
(c) Heating up of the vapour due to external work done on the system.
(d) Release of heat by the vapour to the surroundings, bringing it to the
initial state and completing the cycle.
In an actual refrigerator, the vapours of some low boiling point liquid
(ammonia or *freon—l2) act as the working substance. The working substance
**absorbs a certain quantity of heat Q 2 from the cold body or sink at lower
temperature T2. In a household refrigerator, the ice cubes in the freezer
compartment and food constitute the cold body. A certain amount of work W is
performed by the compressor of the refrigerator on the working substance. The
compressor is operated by an electric motor. The quantity of heat Q 1 is rejected
to the hot body (atmosphere) at temperature T 1 K by the radiator (fixed at the
back of the refrigerator).
Coefficient of performance. It measures the efficiency of a refrigerator.
It is defined as the ratio of the quantity of heat extracted per cycle from
the contents of the refrigerator to the mechanical work W done by the external
agency to do so.
It is denoted by  or K or .

 
SECOND LAW OF THERMODYNAMICS 12.51

Smaller the amount of mechanical work done in removing heat Q2, greater
will be the coefficient of performance.
Applying first law of thermodynamics,

W = Q1 – Q2   ...(1)

This expression may be put in another form also.



[Dividing the numerator and denominator of equation (1) by Q 2.]

But    

or  ...(2)

Discussion. (i) In actual practice,  varies from 2 to 6. For an actual 



refrigerator, the value of  is less than that calculated from equations (1) or (2).
(ii) Lesser the difference in the temperatures of the cooling chamber and 
the atmosphere, higher is the coefficient of performance of the refrigerator. 
(iii) In a heat engine, the efficiency can never exceed 100%. But in the
case of a refrigerator, the coefficient of performance may be much higher than  
100%.

(iv) As the refrigerator works, T2 goes on decreasing due to formation of  

too much ice. There is practically no change in T 1. This decreases the value of .
However, if the refrigerator is defrosted, T2 shall increase and consequently the
value of . So, it is necessary to defrost the refrigerator.

Carnot showed that no engine can be more efficient than the perfectly
reversible engine. This is known as Carnot’s theorem and is stated as follows :
No heat engine working between given temperatures can
have efficiency greater than that of a reversible engine working
W
between the same temperatures.
Imagine a reversible (Carnot) engine R and an irreversible engine Q1 Q2
I working between the same source (hot reservoir) and sink (cold I Q 1 – W
reservoir). Let us couple the engines I and R in such a way so that I
T1 T2
acts like a heat engine and R acts as a refrigerator. Let I absorb heat
Q1 from the source, deliver work W and release the heat (Q 1 – W) to Q1 R Q1 – W
the sink. We arrange so that R returns the same heat Q1 to the source,
taking heat Q2 from the sink and requiring work W = Q 1 – Q2 to be
done on it. W
Now suppose R < I i.e., if R were to act as an engine it would Fig. 12(b).7. An irreversible engine (I) coupled to a
give less work output than that of I i.e., W < W for a given Q1. With R reversible refrigerator (R). If W  > W, this would
acting like a refrigerator, this would mean Q 2 = Q1 – W > Q1 – W. amount to extraction of heat W  – W from the sink
Thus on the whole, the coupled I-R system extracts heat (Q1 – W) – (Q1 – W) and its full conversion to work, in contradiction
= (W – W) from the cold reservoir and delivers the same amount of with the Second Law of Thermodynamics.
12.52 PHYSICS (Vol. II)—XI
work in one cycle, without any change in the source or anywhere else. This is
clearly against the Kelvin-Planck statement of the Second Law of
Thermodynamics. Hence the assertion I > R is wrong. No engine can have
efficiency greater than that of the Carnot engine.
Aliter. Consider two engines–an irreversible engine A and a reversible SOURCE
(T1)
engine B. Let the two engines be coupled such that A drives B backwards. So, B
is acting as a refrigerator.
A absorbs heat Q1 from the source, performs work W and rejects heat Q 2 Q1 Q’1
to the sink.
A WW B
 Q2 Q’2
Efficiency of A,  

B absorbs heat Q2 from the sink, gets work W performed on it and rejects SINK
(T2)
heat Q1 to the source.

  
Efficiency of B,   Fig. 12(b).8. Carnot Theorem.
 
Suppose A is more efficient than B.
Then, A > B

  or Q1 < Q1

 Q1 – Q1 is positive. So, the net quantity of heat given to the source at
temperature T1 by the compound engine AB is (Q 1 – Q1).
Again, W = Q1 – Q2 = Q1 – Q2 or Q2 – Q2 = Q1 – Q1
 Q  – Q is positive.
2 2 [... Q  – Q is +ve]
1 1

So, the net quantity of heat taken from the sink at temperature T2 by the
compound engine AB is (Q 2 – Q2).
Thus, the compound engine AB is a self-acting device which is transferring
heat from lower temperature to higher temperature without any work being
done by the external agency. This is forbidden by second law of thermodynamics.
So, our assumption that engine A is more efficient than B is wrong.
This proves Carnot’s theorem.

The argument used in the proof of Carnot theorem can be used to show
that a reversible engine with one particular substance cannot be more efficient
than the one using another substance. The maximum efficiency of a Carnot

engine, given by  = 1 – , is independent of the nature of the system performing

the Carnot cycle of operations. Thus we are justified in using an ideal gas as a
system in the calculation of efficiency  of a Carnot engine. The ideal gas has a
simple equation of state, which allows us to readily calculate , but the final
 
result      is true for any Carnot engine.
 
SECOND LAW OF THERMODYNAMICS 12.53

  

    
   

 
 


    

 

  

   
 
    

 
 
    

 

 
   
  

 
    

   
12.54 PHYSICS (Vol. II)—XI

    



 

     

 

  

  

  
 

     

    

 
    
           
 

  
         
  

 


 
 
   


 
  

    


SECOND LAW OF THERMODYNAMICS 12.55



   
  

   

VERY SHORT ANSWER QUESTIONS—Each Question Carries 1 Mark Only


      

 

    
12.56 PHYSICS (Vol. II)—XI

SHORT ANSWER QUESTIONS—Each Question Carries 2 Marks


SECOND LAW OF THERMODYNAMICS 12.57

SHORT ANSWER QUESTIONS—Each Question Carries 3 Marks


 

    

    

    
12.58 PHYSICS (Vol. II)—XI



 

LONG ANSWER QUESTIONS

  
SECOND LAW OF THERMODYNAMICS 12.59
12.60 PHYSICS (Vol. II)—XI

   

 

    


  
  

  
  

   
 
    
 

      
     
     
  

  

    


 
    
   
   
    
   
   

 
   
 
   

     
    

    

 
  
 
      
  
 

SECOND LAW OF THERMODYNAMICS 12.61

P. 1. A heat engine is having a source at tempera-


ture 527°C and sink at temperature 127°C. If the useful
work is required to be done by the engine at the rate of 750 
watt, then find out the amount of heat absorbed by the
engine per second from the source. Also find the efficiency 
of heat engine. [AIPMT Mains 2009]
Solution. T1 = 527°C = (527 + 273) K = 800 K
T2 = 127°C = (127 + 273) K = 400 K
 

= =1–

=1– =

or Q1 = 1500 W

Efficiency, =1– =1– = = 50%.

P. 2. An ideal engine works between temperatures


T1 and T2 . It drives an ideal refrigerator that works
between temperatures T3 and T4 (Fig. 12(b).9). Find the

ratio in terms of T1 , T2 , T3 and T4 .

T1
T2
Q1
  
Q3

 
W

Q4  
Q2

Refrigerator  
  

Engine

Fig. 12(b).9    

Solution. Efficiency of the engine,  =


   


12.62 PHYSICS (Vol. II)—XI

Solution. Temperature of source,


   

 
    
 

F I
 GH  JK

 


    

    

  
  
 

 

F I
 GH  JK


 
    
     


 
   
 

 
   
   
SECOND LAW OF THERMODYNAMICS 12.63

  

  

 
T


 
 
 

P
B C

Solution. Net heat absorbed by the system, D

Fig. 12(b).10

 
 
 
 

 
 
 
 

    
   
   

 

  


 

NCERT EXERCISES (With Solutions)


12.64 PHYSICS (Vol. II)—XI





 


   
    
   
  

 

  
 
 

     

 
  


NCERT EXERCISES (UNIT VIII) 12.65

D
  600

Pressure, P (Nm )
–2
300 E
F

     

2.0 5.0
3
Volume, V (m )

Fig. 12(b).11



 


  
  

12.66 PHYSICS (Vol. II)—XI

NCERT EXEMPLAR PROBLEMS (With Answers)


[Based on Higher Order Thinking Skills].

MCQ I (Only one option is correct.) T T

2 2

1 1

P P

T T

4
P 2 1 1 2
3
2
1
P P
V

Fig. 12(b).12 Fig. 12(b).14

D C
2P

P B
A

14.5  104 V 3V V
580  103 Fig. 12(b).15

P
1
Constant
P=
V
2

V

Fig. 12(b).13
NCERT EXEMPLAR PROBLEMS (UNIT VIII) 12.67

 1

1 
2
 
2 2
 1   1 
   
1     1

  

 
    
  
   


 



T1 + T2 + T3
3

M1T1 + M2T2 + M3T3


M1 + M2 + M3
P
I
M1T1 + M2T2 + M3T3 IV
A
3(M1 + M2 + M3 ) II
B
M1T1s + M2T2s + M3T3s III
M1 + M2 + M3
V

Fig. 12(b).16

M1T1 + M2T2 + M3T3


M1 + M2 + M3

MCQ II (Any number of options may be


correct.)
12.68 PHYSICS (Vol. II)—XI

P 
1 2
VSA
3

Fig. 12(b).17

P
1
Q
2
P

 V
Fig. 12(b).19

T1
Q1

Q2
T2


Fig. 12(b).18
NCERT EXEMPLAR PROBLEMS (UNIT VIII) 12.69

  
     
     

 


 

SA 

 
    



270 1

300 10
1

20

W 1
 20
Q

 

     
 
  

 



 
 
12.70 PHYSICS (Vol. II)—XI

Y
(Pi, Vi) (Pi, Vf)
A
C

P (P 2, Vf)
B

O X
V Vf

Fig. 12(b).20

12.71
12.72 PHYSICS (Vol. II)—XI

 

   
 
 
THERMODYNAMICS (UNIT VIII) 12.73

 
  
 
12.74 PHYSICS (Vol. II)—XI

  

 
  
 

THERMODYNAMICS (UNIT VIII) 12.75

     
   
     

     
   
     






  






  






 
12.76 PHYSICS (Vol. II)—XI

   
  

   
   
   

   
   
   

  

 

 
 
 
THERMODYNAMICS (UNIT VIII) 12.77

 

 



 
  

 

 
 
 
 
12.78 PHYSICS (Vol. II)—XI

     
   
   


THERMODYNAMICS (UNIT VIII) 12.79

 
 

 

 
 
 
12.80 PHYSICS (Vol. II)—XI

O
THERMODYNAMICS (UNIT VIII) 12.81

 

  

   
   
  
  
  
  
   

   

   

   

 

 

 
12.82 PHYSICS (Vol. II)—XI






THERMODYNAMICS (UNIT VIII) 12.83

 


  

  

P (105 Pa)

7
6 B
5
4
3 A
2 C
 1
0 V (10–3 m3)
2 4 6 8

A D
2 × 10 2
P(N m –2)
102 B C

0 1 2 3
V(m 3 )
12.84 PHYSICS (Vol. II)—XI

P C B
3P0

2P0 E

P0 D
A

V0 2V0 V

B
800 K
P
600 K
A C
400 K
V


THERMODYNAMICS (UNIT VIII) 12.85

A
5

P(in kPa) 2 B 

4 6 
3
V(in m ) 


P
B C
6 × 104 Pa

2 × 104 Pa
A

2 × 10–3 m3 4 × 10–3 m3
V
12.86 PHYSICS (Vol. II)—XI







  

  

     
   

 
 

   
      
   

 
    
 
 

  
     

 
  
 


THERMODYNAMICS (UNIT VIII) 12.87

 

 

 
  
  
  
  
 
 
 =

 



   


  
      
   
   

   
 
  

   
  
  

 

 
 
 
  
   
  
  


 

  
  
 
  
   
12.88 PHYSICS (Vol. II)—XI

 
 
 
 

   
 
    
 
  
 
    
 

 


   


      
   
   

   

   
 

 

 

   
   
  

 

   
  
 


 



  
THERMODYNAMICS (UNIT VIII) 12.89

 
 
 

       
   
   



  
   
    
 


     
  
 

 

  






 
   
   
   
   


 
 
  



12.90 PHYSICS (Vol. II)—XI

    
 
=   


 
    

 

 

 
  

 
    

 
  

 

 

  


    

 

   


2 

Isobaric
P Iso 1   
the
rm a
l
Ad 3
ia b  
a ti c

 
V1 V V2

 


 


THERMODYNAMICS (UNIT VIII) 12.91

   


  
    
  
      
  
  
=1×
 
 

 
 


 
  

 


P

B A  
3P

 

1P
C D
0 1V 3V 9V V 

 
  
  
 

 
 





 
 
 
12.92 PHYSICS (Vol. II)—XI
UNIT—IX
BEHAVIOUR OF PERFECT 08 Periods

GASES AND KINETIC


THEORY OF GASES

Chapter–13: Kinetic Theory

SOLVED NUMERICALS 30 PLUS

UNSOLVED NUMERICALS 42 PLUS

VERY SHORT, SHORT & LONG ANSWER QUESTIONS 88 PLUS

QUESTIONS & PROBLEMS BASED ON HOTS 32 PLUS

NCERT EXERCISES, EXAMPLES & EXEMPLAR PROBLEMS 40 PLUS

MULTIPLE CHOICE QUESTIONS 65 PLUS


Boltzmann, Ludwig Eduard (1844–1906)
13.3
13.4 PHYSICS (Vol. II)—XI

T = Constant

O P X

Fig. 13.1. Variation of P with V at


constant T.
KINETIC THEORY 13.5

T = constant

P
  

 1
 V
1
Fig. 13.2. P versus graph.
V


P = constant

 V


 T
Fig. 13.3
13.6 PHYSICS (Vol. II)—XI

 
 

  



V P1
V2 P2
V1 P1
T2
T1 T2

Fig. 13.4. Gas equation.



 

 
 
 
KINETIC THEORY 13.7

 


  

 
13.8 PHYSICS (Vol. II)—XI

P1 V1 P2 V2
N1 T1 N 2 T2

 Ideal gas
1

T1

–1 –1
J mol K
T2
T1> T2> T3
M N

M0 NA pV T3
T

0 200 400 600 800

P (atm)
N R
Fig. 13.5. Real gases approach ideal gas
NA NA behaviour at low pressures and high
temperatures.

1.6

1.4
RT P
M0 1.2

1.0 T1

0.8 T1 > T2 > T3
T1
0.6
0.4 T2 T3 T2

0.2
T3
0
20 60 100 140 160 220
V

 Fig. 13.6. Experimental P-V curves (solid


lines) for steam at three temperatures
compared with Boyle’s law (dotted
lines). P is in units of 22 atm
and V in units of 0.09 litres.
KINETIC THEORY 13.9

1.2
P1>P2>P3
 T 1.0
P1
0.8

 0.6
P2

0.4 P3

  0.2

0
RT RT 0 100 200 300 400 500
 
V V V

 Fig. 13.7. Experimental T-V curves


(soild lines) for CO 2 at three pressures
compared with Charles’ law
(dotted lines). T is in units
of 300 K and V in units of
0.13 litre.





13.10 PHYSICS (Vol. II)—XI


   

 
  

 
    
     

     
  

 
 

  
 
 


 




KINETIC THEORY 13.11

 


 
 
 

 
 
 


     
 
13.12 PHYSICS (Vol. II)—XI

 X
O

  Fig. 13.8. Expression for gas
pressure.

1 
  2 A vx t 
 

  
  
 
KINETIC THEORY 13.13

Q n m A vx2 t
t t

nm A vx2
A

vx2

 vx2

vx2 vx2

1 2 1 2
vx2 v2y vz2 v v2y vz2 v
3 x 3

v2

1
v2
3

N
V

1 N
 v2
3 V

1 M 2
v
3 V

M

V

1
  v2
3
13.14 PHYSICS (Vol. II)—XI

vx2

c2

1 c1
A2 u1 A1
c3 w1 
O X
cn 

Fig. 13.9. Expression for gas pressure.


  
KINETIC THEORY 13.15

   

     


 
     

 

1m 2
  [(u1  u22  u32  .....  uN
2
)
3 l3

   

   

       
  
 
 
  

   

     
         
     
     

     
 
 

   

 

     
13.16 PHYSICS (Vol. II)—XI

     

 
  
 


 

is 8.315 J m ol

 
KINETIC THEORY 13.17

K.E.

Fig. 13.10

 

  
13.18 PHYSICS (Vol. II)—XI

CHECKPOINT
 

( 

  

  
  
KINETIC THEORY 13.19

Fig. 13.11. Random motion of gas


molecules in a box.

speeds between c & c + dc


Number of molecules with
c

Speed
Fig. 13.12. Distribution of speeds in a gas.


 
  
  




13.20 PHYSICS (Vol. II)—XI


 

2.0

dN
dc
1.0
mp rms

dc
0
c1 c2 X
0 200 400 600 1000 1200
–1 800
Speed (m s )

Fig. 13.13. The plotted curve is for


oxygen gas molecules at T = 300 K.
KINETIC THEORY 13.21

Y
4.0 T1

T2 > T 1
3.0

dN T2
dc 2.0

1.0

0 X
0 200 400 600 800 1000 1200
–1
Speed (m s )
Fig. 13.14. A comparison of the Maxwell speed
distribution for oxygen molecules at two
different temperatures
[T1 = 80 K, T2 = 300 K].

  

   
13.22 PHYSICS (Vol. II)—XI

Fig. 13.15 

  

   

 
   
 

   


   

 

 
   
 

  

 
KINETIC THEORY 13.23

 

    
  


 
 

    

  
  


  

   
  
    


 


   
13.24 PHYSICS (Vol. II)—XI




        

    

  
 
 


 

 
KINETIC THEORY 13.25

 

 

 

     
13.26 PHYSICS (Vol. II)—XI

  

 

 

 

  

 
 
 

  



KINETIC THEORY 13.27



13.28 PHYSICS (Vol. II)—XI

   

   

(1)

(2)

Fig. 13.16. The two independent axes


of rotation of a diatomic molecule.

 
1 1 1 1 1
  mvx2 mv2y mvz2 I112 I2 22
2 2 2 2 2
 


2
1  dy  1
  dt 
2   2
   

dy
dt
KINETIC THEORY 13.29

Fig. 13.17. Linear triatomic molecule.


Fig. 13.18. Non-linear triatomic


molecule.

1 1 1
 mvx2 mv2y mvz2
2 2 2


1 1 1
 mvx2 mv2y mvz2
2 2 2

1 1 1 1 1 1
mvx2 kB T mv2y kB T mvz2 kB T
2 2 2 2 2 2


13.30 PHYSICS (Vol. II)—XI

3
RT
2

 
   

 

 

  

  

5 5
2 2

 
  
 

 

 
  
 


KINETIC THEORY 13.31

 

    

 
   
 
 

  

  
          

3 3 
 2 kB T  2 kB T  f kB T 
 

4f

3 f
13.32 PHYSICS (Vol. II)—XI


 

1
2

    
KINETIC THEORY 13.33

Q U
T T

Q U
T T


13.34 PHYSICS (Vol. II)—XI


A
D

E
G
C

Fig. 13.19. Mean free path.

   

    



= 

d
d
 


 Fig. 13.20. Mean free path.

 

KINETIC THEORY 13.35



 
 

   


  

CHECKPOINT

Fig. 13.21. Brownian motion.


13.36 PHYSICS (Vol. II)—XI



 
      




KINETIC THEORY 13.37



VERY SHORT ANSWER QUESTIONS—Each Question Carries 1 Mark Only

B
PV
C

P
Fig. 13.22

 
 
 

 
   
 
13.38 PHYSICS (Vol. II)—XI

  

  


KINETIC THEORY 13.39

 
 
 



SHORT ANSWER QUESTIONS—Each Question Carries 2 Marks

    



13.40 PHYSICS (Vol. II)—XI

  


   




KINETIC THEORY 13.41

SHORT ANSWER QUESTIONS—Each Question Carries 3 Marks

  

 
     
 

  

 
 
 

 
 
 
13.42 PHYSICS (Vol. II)—XI

 
  

 

 
 

 

   

 P, V1
P P, V2

M1  M2

 Fig. 13.23

 
 

 
  
 

 
 
 



KINETIC THEORY 13.43

 

 
 


 
 
 
 

 
13.44 PHYSICS (Vol. II)—XI
KINETIC THEORY 13.45

   
 

   

 

 

 
13.46 PHYSICS (Vol. II)—XI

2
vrms

LONG ANSWER QUESTIONS

 



KINETIC THEORY 13.47

   
      
 
   
   

   


13.48 PHYSICS (Vol. II)—XI



    
    
    



KINETIC THEORY 13.49

 
   

    

   
 



 


  
  


 
 

      
   


    


 


  

 
 
 

    

      
 

   
 

  


13.50 PHYSICS (Vol. II)—XI

  


 

P. 1. How many degrees of freedom are associated


with 2 g of He at NTP ? Calculate the amount of heat energy
required to raise the temperature of this amount from 27°C
to 127°C. Given : Boltzmann’s constant = 1.38 × 10–16 erg
molecule–1 K–1 and Avogadro’s number = 6.02 × 1023.
Solution. Since molecular weight of helium is 4 therefore
the number of molecules in 4 g of He at NTP is 6.02 × 10 23.
P. 2. In an experiment on Brownian motion using a
torsion pendulum (a small mirror of area 1 mm2 mounted
 on a thin torsion fibre with torsion constant = 1.8 × 10 –17
  
J/rad2) the mean value of angular displacement  was
found to be nearly zero, and the fluctuation in that is the
mean square of was found to be 2.6 × 10–4 rad2. Estimate the
value of Boltzmann constant from this data and compare
 it with the correct value. The temperature is 300 K.
Solution. Torsional constant,

 

 


KINETIC THEORY 13.51
P. 5. A cubical box of side 1 metre contains helium
     gas (atomic weight 4) at a pressure of 100 N m –2. During
an observation time of 1 second, an atom travelling with
the root mean square speed parallel to one of the edges of
 
     the cube was found to make 500 hits with a particular
wall, without any collision with other atoms. Take
R = 25/3 J/mol-K and k = 1.38 × 10–23 J K–1

P. 3. Calculate the number of molecules in 1 cm3 of a


perfect gas at 27°C and a pressure of 10 mm of mercury.
Mean kinetic energy of a molecule at 27°C = 4 × 10–4 erg.
Given : density of mercury = 13.6 g cm–3.
Solution. Volume of gas, V = 1 cm3 Solution. (a) Time interval between two successive
collisions with a particular wall,


P. 6. An astronaut takes a cylinder of volume 10
litre containing nitrogen gas at temperature of 27°C and
pressure 50 atmosphere. He makes a hole of area 1 sq. cm
in this cylinder and places it in open space. Estimate the
  time it would take for the cylinder to become empty.
   Boltzmann’s constant = 1.38 × 10–23 J/K.
P. 4. An ideal gas having initial pressure P, volume [Ans. 0.387 second]
V and temperature T is allowed to expand adiabatically
until its volume becomes 5.66 V, while its temperature falls Hint. vrms =  ,
to T/2. (a) How many degrees of freedom do the gas
molecules have ? (b) Obtain the work done by the gas 
during the expansion as a function of the initial pressure
and volume.
Solution. (a) T1V1 – 1 = T2V2 – 1

 
P. 7. A mercury thread of length 10 cm is contained
 in the middle of a narrow horizontal tube of length 100 cm
 and sealed at both the ends. The air in both halves of the
tube is under a pressure of 76 cm of mercury. What distance
will the mercury column move if the tube is placed
 vertically ? [Ans. 2.95 cm]

Hint. When the tube is horizontal, the length of air
column on either side of mercury thread is 45 cm. Let 1 cm 2 be
 the cross-sectional area.

 
  
 
13.52 PHYSICS (Vol. II)—XI

45 cm 45 cm
45 +

10 cm P. 8. One gram mole of oxygen at 27°C and one at-


10 cm
mospheric pressure is enclosed in a vessel. Assuming
45 – the molecules to be moving with vrms , find the number of
collisions per second which the molecules make with one
square metre area of the vessel wall. [Ans. 1.97 × 1027]

Fig. 13.24 Hint. Number of molecules in 1 m 3,

NCERT EXERCISES (With Solutions)

 
  

   
   Y
 
 
T1

 PV (J K –1) T2
    T

O P X

Fig. 13.25



NCERT EXERCISES (UNIT IX) 13.53


   


   
 

 

     



13.54 PHYSICS (Vol. II)—XI

 
 
 



    

  
 

 
  
 















15 cm 76 cm 9 cm

Fig. 13.26
NCERT EXERCISES (UNIT IX) 13.55



24 + h
 

76 – h

Fig. 13.27

    

  

   
 

    
      
  

  
 
 

    
     
   

     
 
  

13.56 PHYSICS (Vol. II)—XI

M = 197 × 10 

  
 
 
      



  
 
 
      



  
 
 
      

 
    
   

  
 
  
      

  
 
 
      
NCERT EXEMPLAR PROBLEMS (UNIT IX) 13.57

NCERT EXEMPLAR PROBLEMS (With Answers)

[Based on Higher Order Thinking SKills]

MCQ I (Only one option is correct.)


M

Fig. 13.29

B D

V
A C (l)
P2
40
G
F 30
P1
20
E H
10
Fig. 13.28
100 200 300 400 500
T(K)

Fig. 13.30


13.58 PHYSICS (Vol. II)—XI

   

MCQ II (Any number of options may be


T = const
correct.) V
P = const
P

T V

P V = const
PV

Fig. 13.31
T T

Fig. 13.32
NCERT EXEMPLAR PROBLEMS (UNIT IX) 13.59




VSA

 
 

  
  
  

SA


  

  Fig. 13.33

   
   
 

1

m
   

 
13.60 PHYSICS (Vol. II)—XI


 
   3

2
mv02

3R



 LA



3
2
3
2
3
2
3kT 3  1.38  10 23  300
m 7.3  10 26

6.023  10 23
22400



13.61
13.62 PHYSICS (Vol. II)—XI




 

 
 
 

      
   
     

      
   
     
KINETIC THEORY OF GASES (UNIT IX) 13.63





13.64 PHYSICS (Vol. II)—XI


 
KINETIC THEORY OF GASES (UNIT IX) 13.65

 
 

   
   
13.66 PHYSICS (Vol. II)—XI

   
 

 
 
 
KINETIC THEORY OF GASES (UNIT IX) 13.67

   
  

  
 

 
 

 


13.68 PHYSICS (Vol. II)—XI

   
     

   
     

= =

= =

=
KINETIC THEORY OF GASES (UNIT IX) 13.69



 
 

  
    
  

 

 
  

   

 
   
 
 

 
 
  

  
 
 


    



13.70 PHYSICS (Vol. II)—XI

 
 



 

 
  
 

 
=


=
 


=


=

 =

=  

 
= 
     
 
KINETIC THEORY OF GASES (UNIT IX) 13.71

 = =


= =

=
 = =
UNIT—X
OSCILLATIONS AND 26 Periods

WAVES

Chapter–14: Oscillations

Chapter–15: Waves

SOLVED NUMERICALS 95 PLUS

UNSOLVED NUMERICALS 180 PLUS

VERY SHORT, SHORT & LONG ANSWER QUESTIONS 215 PLUS

QUESTIONS & PROBLEMS BASED ON HOTS 80 PLUS

NCERT EXERCISES, EXAMPLES & EXEMPLAR PROBLEMS 105 PLUS

MULTIPLE CHOICE QUESTIONS 150 PLUS


Fourier, Jean Baptiste Joseph (1768–1830)
14.01. INTRODUCTION

( )

T
Fig. 14.1. Periodic motion

( )

T
Fig. 14.2. Periodic motion

14.3
14.4 PHYSICS (Vol. II)—XI

Fig. 14.3. Periodic motion


OSCILLATIONS 14.5

y
t

     
Fig. 14.4. Non-harmonic oscillation
  
  

  

F
14.6 PHYSICS (Vol. II)—XI

 
 

   
  
 

   
     
 

   
  
 

   
     
 

  
 
 

  
 
 
OSCILLATIONS 14.7


   
 

 

    
    
 
    
     
 


    
 

 
  




  
  
     
 
     


Fig. 14.5. A block attached to a spring,


the other end of which is fixed to a
rigid wall
14.8 PHYSICS (Vol. II)—XI

Fig. 14.6. An oscillating simple


pendulum





 

 
   
 
 
 

  
   
  



   
   
 

 
   
 
OSCILLATIONS 14.9

  
 

 

 


  



   
14.10 PHYSICS (Vol. II)—XI

   

  

   

     

    

        

  

  

  

   

–A +A

Fig. 14.7. A particle vibrating back and


forth about the origin of x-axis,
  between the limits + A and – A.

 

–A O A –A O A

  t=0 t = T/4

–A O A –A O A

t = T/2 t = 3T/4
 

–A O A –A O A
t=T t = 5T/4
Fig. 14.8. The location of the particle in SHM at the discrete
values t = 0, T/4, T/2, 3T/4, T, 5T/4.
OSCILLATIONS 14.11

Displacement
 
0 t

–A

Fig. 14.9. Displacement as a


continuous function of time for
simple harmonic motion
 
  
2
B
       A
1

Displacement
 0 t

–B
–A

Fig. 14.10. Plots of displacement as a


 function of time for two simple
  
harmonic motions


0 = –
A 4 4
Displacement

      3
0 t

–A 0 = 0


Fig. 14.11. Displacement-time plots of
 two simple harmonic motions having
 same A and  but different phase
angle 0


14.12 PHYSICS (Vol. II)—XI

 


 y
t


Fig. 14.12. Graphical representation of


y = A sin t
 
    
 

 
    
 
t

Fig. 14.13. Graphical representation


of = A cos t

 
   
 

     
         
   
     
           
    

 

 


OSCILLATIONS 14.13

  


    

   


  


   

  Fig. 14.14. Displacement-time plots of


two simple harmonic motions having
 same A and 0 but different 

  
14.14 PHYSICS (Vol. II)—XI


  

 

 

   

 
     
 

  

  



 


   
  
 
    
 
  
   
     
   

 
  
 
 
  
      
     
   

 
 
     
 

 
    
 
OSCILLATIONS 14.15

Y
 P


   X X
O N


Y
Fig. 14.15. SHM is projection of uniform
circular motion

P
A

X’ X
O N
(t)

 Y’
Fig. 14.16. Displacement of projection
point on x-axis

Y

P
N’ 
(t) A
   
X’ X
O
 

 
Y’
 Fig. 14.17. Displacement of projection
  point on Y-axis

   
Y
 P

t

X’ X
O 

    A

  Y’

  Fig. 14.18. SHM with epoch “– 0”


14.16 PHYSICS (Vol. II)—XI

 
Y
 P

 B
t
0
X’ X
O
  
 

Y’
 
Fig. 14.19. SHM with epoch “+ 0”
   

y
y P(t = 0)

P(t = 0) T = 30 s
A B
o
45
T=4s x x
 O O

 
(a)
 (b)
  
Fig. 14.20

 
     
 

  
  
 

  
   
 


  
   
 

  


 


 

  
  
OSCILLATIONS 14.17

        
    
      
   
 
   
  
  


  

Fig. 14.21. (a) Displacement of projection points on x-axis and y-axis (b) Velocity of
projection points (c) Acceleration of projection points.

  

 

 
14.18 PHYSICS (Vol. II)—XI



    

 

  

 

 

  

 

 


 

  
 

 

 
OSCILLATIONS 14.19

 

  


 


 

  2 2
A 0 A –
+
 t
X’ X
N
 

Fig. 14.22. sin (t + 0) in terms of


and A
 

   2 2
A –
Y
 N
  0
y t+ A

t +  0
X’ X
 O

 
Y’

Fig. 14.23. cos (t + 0) in terms of


  y and A

  

  

  
 

 

  
14.20 PHYSICS (Vol. II)—XI

   

  

 
 

 


 

     
 
 
    

+A
Displacement

0
t
–A
 T

     +A
Velocity

0
t

–A

     a
2
+ A
Acceleration

0
  t
        2
  – A

Fig. 14.24. Displacement, velocity and


acceleration of a particle in simple
harmonic motion have the same period
T, but they differ in phase
OSCILLATIONS 14.21

  

 

Displacement
A
3T/4 T
 O
T/4 T/2 t
–A

A

Velocity
 T/2 3T/4
O
T/4 T t
–A


2

Acceleration
A
T/4
O
T/2 3T/4 T t
2
– A

 
Fig. 14.25. Plots of y(t), (t) and a(t) against
   
time t

  

 
  
     
 

  
     
   


 

 
14.22 PHYSICS (Vol. II)—XI




 


 


OSCILLATIONS 14.23

 P

5 cm
  
O

 5 cm 

Q 
 
Fig. 14.26 
  
 
 
 

  


  
 

  





  

  
 

  
14.24 PHYSICS (Vol. II)—XI

 
  




 

 

  
   




    

 
    

  


 

 

  
OSCILLATIONS 14.25




I = 0.082 kg m


 
   
 


 

 

  

  

  

 
    
 


14.26 PHYSICS (Vol. II)—XI

 

  

  

  

  

     

 
    
 

 

 

 


OSCILLATIONS 14.27

 

   

   

  

  

  

K( ) + U( )
E E
U( )
U( )

Energy E
K( )

K( )
–A O +A
Fig. 14.27. Kinetic energy, potential energy and total energy,
as a function of displacement, of a particle in SHM
14.28 PHYSICS (Vol. II)—XI

E = K(t) + U(t)
E
E U(t)

Energy
U(t)
K(t)
K(t)
t
O T/2 T
Fig. 14.28. Kinetic energy, potential energy and total energy,
as a function of time, of a particle in SHM

 
   
 



  

  

   
 
 

    


    

OSCILLATIONS 14.29


       


  

  
  
    


 


   
  
     

     
         
   


 

L
F F + F

m y(< l )
mg m

mg +  F

Fig. 14.29. Mass oscillating on a


vertical spring
14.30 PHYSICS (Vol. II)—XI


 
 



 

1
 n
 
n

 

1
n
OSCILLATIONS 14.31

 
 

k
m

–A x=0 +A
Fig. 14.30. Mass oscillating on a
horizontal spring

 

 
  

 

14.32 PHYSICS (Vol. II)—XI

m2
k1


k
k2

m1 Fig. 14.34

Fig. 14.31



 Fig. 14.32



k1 gradient = =
T2 4 2
k1 k2
m
T2
k2
m Fig. 14.35

(a) (b)

Fig. 14.33



 

OSCILLATIONS 14.33

 
    



  



  


 
 

  
   
 

 
14.34 PHYSICS (Vol. II)—XI

   

  
 


 


  


P   

P
A
P

Fig. 14.36

Fig. 14.37





  


OSCILLATIONS 14.35

 
 
 

 

k1 k2
m

Fig. 14.40

m
k k

Fig. 14.38

F1 F2

O 

Fig. 14.39
14.36 PHYSICS (Vol. II)—XI


T
A

  mg cos 
g s in
O m
mg
Fig. 14.41. Vibrating simple
pendulum

 






  

 

 

 
OSCILLATIONS 14.37

 
 

 
  

   

   

 


 

L sin 
S

  
L



A
mg
  O

Fig. 14.42


 


14.38 PHYSICS (Vol. II)—XI



   

 
   
 


  
  
  


  


 
    

 
    

 



  
æ ö
çè  ÷ø
 



 


OSCILLATIONS 14.39



  


 

 

  

  

æ  ö æ ö
ç  ÷    ç  ÷ø 
è ø è

 æ ö æ ö
çè  ÷ø  ç 
è ÷ø


   

æ ö æ  ö  
 
 ç  ÷ ç  ÷
è ø è ø


  
14.40 PHYSICS (Vol. II)—XI

  

 



  


 

R N    

Mg
sin 
A

Mg cos 
O

Mg
Fig. 14.43

y
2y
a d
y

b c

Fig. 14.44. Oscillations of a liquid


column

 




 


OSCILLATIONS 14.41

  

 



   

 

Fig. 14.45


  
 
 


14.42 PHYSICS (Vol. II)—XI




  Fig. 14.46. Floating cylinder



 

 

 

 
    
 


   

 

 


 


OSCILLATIONS 14.43

 

C
y
B A = 15 cm
l


Fig. 14.47


 

 
  



  

+A

( t)
1 mA2 2
0 2
t
ENERGY

–A

Fig. 14.48. Displacement as a function of Fig. 14.49. Energy-time graph.


time in damped harmonic oscillations.
[Light damping : continuing oscillations
with decreasing amplitude]
14.44 PHYSICS (Vol. II)—XI

Fig. 14.50. Heavily damped system.

Fig. 14.51. Critical damping :


Optimum rate of return to
equilibrium position without
oscillation.

x
  Rigid support

Spring

O Block

Surrounding
medium

  Fig. 14.52. The viscous surrounding


medium exerts a damping force on an
oscillating spring, eventually bringing it
to rest.
OSCILLATIONS 14.45

   

 


 




Fig. 14.53. Displacement as a function of


time in damped harmonic oscillations.
Damping goes on increasing successively
from curve a to d.
14.46 PHYSICS (Vol. II)—XI

 

   

x
Rigid support 

Spring

O Block
 

Surrounding
medium

Fig. 14.54
æ ö
çè  ÷ø


OSCILLATIONS 14.47

Fig. 14.55. Plot for undamped


oscillations

 


 

14.48 PHYSICS (Vol. II)—XI

AMPLITUDE
 = 0

O  X
Fig. 14.56. Variation of amplitude with
frequency of the applied force
 
OSCILLATIONS 14.49

VERY SHORT ANSWER QUESTIONS—Each Question Carries 1 Mark Only

 

æ ö
ç ÷
è ø

æ ö
 ç  ÷÷    
ç
 è ø

14.50 PHYSICS (Vol. II)—XI

  
  




æ ö
 çç  ÷÷
 è ø


 



  

SHORT ANSWER QUESTIONS—Each Question Carries 2 Marks


OSCILLATIONS 14.51

 
         
 


 

  

    

     


 

  

 
 
 

14.52 PHYSICS (Vol. II)—XI

   
        
    
    



Fig. 14.57



1 kg 3 kg

Fig. 14.58

m1m2

m1  m2

13 3
13 4

 k
1

2 

1 300  4
2  3.14 3

SHORT ANSWER QUESTIONS—Each Question Carries 3 Marks

 


  
OSCILLATIONS 14.53



 
   
  
 
  
   d
 
y



R(= mg)

Fig. 14.60

a

 mg

Fig. 14.59
 

2

1 1 1 1
k k1 k2 k3  k4

3 4
1 k2 ( k3  k4 )  k1 ( k3  k4 )  k1k2
k k1k2 ( k3  k4 )

k1k2 ( k3  k4 ) Fig. 14.61
( k1  k2 )( k3  k4 )  k1k2

2

T

T x
A
2  

1.2 1  1.2   
2
2   2 6

1.2 4 1.2 


4
2   2 2


14.54 PHYSICS (Vol. II)—XI

bt

A0 e 2m


bt  b(50 T)
A0 
A0 e 2m A0 e 2m
4

 b(150 T)
e 2m

3
  b(50 t ) 
 2m 
e 
 
3
1 
4
 

 1 
 64 
 

Q. 71. Spring of spring constant 1200 N m –1 is


d2 y
mounted on a smooth frictionless surface and attached 
dt 2
to a block of mass 3 kg. Block is pulled 2 cm to the right
and released. Find angular frequency of oscillation.  

 
Fig. 14.62

[AIPMT Mains 2009]
 
k 1200
Answer.  = = rad s–1 = 20 rad s–1.
m 3

   

 

1
   
2

1 

2
 
dy 1
 
dt 2 

d2 y 1
 
dt 2 2

d2 y 1 
  a sin 2t 
dt 2 2 
OSCILLATIONS 14.55

1 1 1 k 1 k
2 7   2
2 m 4 m
8  

1/7

 
 2  10t     
 


   
 
  
  2  10t   
  2

m2
m1
k


 
Fig. 14.63
 
   
 

 
   
 

 

U 
x x

m 2 
 m2
k 4 2
m1

Fig. 14.64

M 108
NA 6  1026
14.56 PHYSICS (Vol. II)—XI




 


  

  
   
A   
B
C 

900 Fig. 14.65

k 
 
 
400 

k

T 400 2   

3 900 3   

Fl Fl
Ay1 AY

F
k   


 l 1
 AY + k 
 

 l k  AY 
 AYk 
    
Fig. 14.66 
AYk AYk
 lk  AY
lk  AY

m m(lk  AY)
 
K AYk
OSCILLATIONS 14.57

   

 

 y

2y
y
 
    0.3 m
 

Fig. 14.67

 

  

 

   
B

m
45° C
A

Fig. 14.68
14.58 PHYSICS (Vol. II)—XI

 
 
 
  
 

k
    2  
     k

k
m 2

Fig. 14.69 r

  Fig. 14.71

 
 

C 90° B

135°


A

Fig. 14.70

 

m m 
 
K 2k

3
2
Displacement

1
1 2 3 4 5 6 7 8
0
(cm)

–1
–2
–3
Time (s)
 
      Fig. 14.72
 
OSCILLATIONS 14.59

m
k

 
   
Fig. 14.73

  


  

t t +t t t + t

A O B A O B

(a) (b)

t+t t t+t t

A B A O B
O

(c) (d)

Fig. 14.74
14.60 PHYSICS (Vol. II)—XI

Rigid support

k Spring

Mass, m

Vane

l

Damping, b g

Fig. 14.75

 




Frictionless

x
– =0

Fig. 14.76
T

cos 
O O

R
sin 

EARTH

Fig. 14.77
OSCILLATIONS 14.61

x

R

x 

R
  

 
 
Fx g
R   
m  

g m1l12  m2l22


R   
   
 m2 gl2  m1 gl1

Displacement

Acceleration

x R
 
( g / R)x g

6.4  106

9.8
Fig. 14.78

LONG ANSWER QUESTIONS


14.62 PHYSICS (Vol. II)—XI

 

 

 
   
 




OSCILLATIONS 14.63


 
    
 

  
 
  

 
 
 

 


 
 
 


14.64 PHYSICS (Vol. II)—XI

20 cm

8N

Fig. 14.79

 
 
  

   
 
 

 
  
 
Fig. 14.80
  
  
 

  Fig. 14.81

OSCILLATIONS 14.65

Fig. 14.83


–1

–1

Fig. 14.82
k = 20 N m

k = 20 N m

  

  
0.3 kg


 Fig. 14.84
    
14.66 PHYSICS (Vol. II)—XI

–1 –1
k = 20 N m k = 20 N m
0.3 kg

Fig. 14.85

Fig. 14.88

–1
3.0 kg –1
600 N m 600 N m     
   
  

Fig. 14.86

–1
K = 400 N m
200 g

 
Fig. 14.87
OSCILLATIONS 14.67

   
    
    
 

   
   
     
 

   
      
     
  
 
 
   
     
 
 
       
      
   

   


 
14.68 PHYSICS (Vol. II)—XI

  
    


 





 

 

 
 
  

 
   
     


 
  

  


  



 
   


P. 1. In damped oscillations, the amplitude after 50  bt


oscillations is 0.8 a0 , where a0 is the initial amplitude. Solution. at = a0 e 2m , at = a0e–t
Determine the amplitude after 150 oscillations.
5 5
0.8a0 = a0e– (50T) or e(50 T) = or 50T = loge
4 4
OSCILLATIONS 14.69

a0 a 
150 T = loge ; 3 × 50 T = loge 0 ;
at at 

5 a
3 loge = log 0
4 at P. 4. Two identical balls A and B, each of mass 0.1 kg,
3
are attached to two identical massless springs. The
a0 5 125 64 spring-mass system is constrained to move inside a rigid
=   = , at = a0.
at 4 64 125 smooth pipe bent in the form of a circle as shown in
Fig. 14.89. The pipe is fixed in a horizontal plane. The
P. 2. When a force F1 acts on a particle, frequency is centres of the balls can move in a circle of radius 0.06 m.
6 Hz. When a force F2 acts, frequency is 8 Hz. What is the Each spring has a natural length of 0.06  metre and spring
frequency when both the forces act simultaneously in same constant 0.1 N m–1. Initially, both the balls are displaced
direction ? [AIPMT 2004]

Solution. Let m be the mass of the particle and A be its by an angle  = radian w.r.t. the diameter PQ of the
6
amplitude of oscillations. Therefore,
circle and released from rest.
  
  

   
A B
      0 .0
6m 6m
0 .0
/6 /6
P Q
 


4
Fig. 14.89
 3  Solution. Effective force constant,

   
Solution. y1 = 10 sin     
   

 
    
    

   
 
   
   
  

  
  
 

  
    
   
14.70 PHYSICS (Vol. II)—XI

  


 



   

  

 

   

  

  
   
     

 
 
dv dy
 
dt dt


dv
 
dt
    
y
y0 2 2
Fig. 14.90  


  

 
OSCILLATIONS 14.71

 
   
 


 
  
 
   
 

O P
 O
Fg 
C 
 l

  

Fig. 14.91
Fig. 14.93
 




     

 O


 l

 
     
A C
B

 Fig. 14.94
    
   

 
     

   
   
   


Fig. 14.92
14.72 PHYSICS (Vol. II)—XI
m

k
Fig. 14.95

Fig. 14.97

 

   

 
ycos 

y 

  

  

Fig. 14.96  
   
 
 

   
  

   



     
 
   

 
     

NCERT EXERCISES (With Solutions)


NCERT EXERCISES (UNIT X) 14.73

  

 
     
 

  

 
 
 

 
  
X



 

t(s)
(a)

–3 –1 0 1 3 t(s)

(b)

1 4 7 10 13 t(s)
(c)
X

–3 –2 –1 0 1 2 3 t(s)
 
(d)

Fig. 14.98

 


 

  
    
14.74 PHYSICS (Vol. II)—XI

 

 

  
 

  
 

 

  
   
 

 
 

 

 




 
       
 

 
    
 

Fig. 14.99
NCERT EXERCISES (UNIT X) 14.75

 
  

 
    
T=2s P 2m T=4s
3 cm  
( = 0)

P( = 0)
  

Fig. 14.100


    

Fig. 14.102

      
         
   
 
 
   
     

 
  

 

     
       
   
Fig. 14.103
  
    
   
  

Fig. 14.101 Fig. 14.104


14.76 PHYSICS (Vol. II)—XI

  

k m
F


Fig. 14.105
m m 
F k F

Fig. 14.106

 

 

 



  

 




 

   


  




NCERT EXERCISES (UNIT X) 14.77

  

m
   a
V

Air

Fig. 14.107





 
 

 

 
  y
  
 

V


 

Fig. 14.108

 


 
14.78 PHYSICS (Vol. II)—XI

   

 

 

 

  

 








 
 

  
   

  

 
 

  


   


   

  

 

  
NCERT EXERCISES (UNIT X) 14.79

   

  

    
 

    

 

 
   
 
  
  
  
  
    
  
     
 
  
   
    

     



  


    

    
  
 
      
  



 

  
 


 
   

   
    
  
 


 

  


14.80 PHYSICS (Vol. II)—XI

NCERT EXEMPLAR PROBLEMS (With Answers)

[Based on Higher Order Thinking Skills]


MCQ I (Only one option is correct.)

 
 4 – 2 t 
 



l
 ,
g

 
    

 
     

 
 
       


 

 
 
a 2  b2
 
   
 
 
 


   
 
 


  

Fig. 14.109
NCERT EXEMPLAR PROBLEMS (UNIT X) 14.81






 

S1 m S2

Fig. 14.112

  12  22


 
12  22
    

1 k1 1 k2
 , 2 
Fig. 14.110 2 m 2 m

 2t   t  1 k 1 k1  k2 1 k1 k
 30   15      2
    2 m 2 m 2 m m

 t    t  
 15  2   15  2  
       
   

x (t ) MCQ II (Any number of options may be


 P
B
Q correct.)
 t

 x(t)

 t
Fig. 14.111





2


t
14.82 PHYSICS (Vol. II)—XI

Displacement
2

1
0
1 2 3 4 5 6 7 time (s) 2

Fig. 14.113  

 

 

B O C A

AO = OB = 5 cm
BC = 8 cm

Fig. 14.115
Displacement

0
T/4 2T/4 3T/4 T 5T/4 Time (s)

Fig. 14.114 VSA


3T
4
4T
4
T
Displacement

4
T
2 C E G
A
B D F H Time (s)

Fig. 14.116
NCERT EXEMPLAR PROBLEMS (UNIT X) 14.83

A

 +
O P A

Fig. 14.118
K K

Fig. 14.117 

 
     
 

   

  

 
    


 

 SA

Inextensible
string

Pulley

M
Fig. 14.119
14.84 PHYSICS (Vol. II)—XI

 

M
4k

   

     
   
    
    
    
 

  

    

  

   


 
   
 

    LA

 

 
 

 

 

NCERT EXEMPLAR PROBLEMS (UNIT X) 14.85

d
P
y=R–d
O

Fig. 14.120

  
  
 

 








m

Ag


  
14.86 PHYSICS (Vol. II)—XI

Fig. 15.1. Water waves

15.1
15.2 PHYSICS (Vol. II)—XI

NORMAL
LEVEL

Fig. 15.2. Crests and troughs

Fig. 15.3. Waves in strings

Fig. 15.4. Wave-train in string


WAVES 15.3


NORMAL
A C E LEVEL

B D F

Fig. 15.5. Transverse wave


15.4 PHYSICS (Vol. II)—XI

Fig. 15.6. Formation and propagation


of transverse wave


WAVES 15.5

CONDENSATION

RAREFACTION

Fig. 15.7. Positions of different layers of


air when (i ) tuning fork is not vibrating
(ii ) tuning fork is vibrating

A B C D E F

C R C R C


C=COMPRESSION R=RAREFACTION

Fig. 15.8. Formation of compressions and


rarefactions in spring

Fig. 15.9. Formation and propagation of longitudinal wave


15.6 PHYSICS (Vol. II)—XI


WAVES 15.7

  



  
15.8 PHYSICS (Vol. II)—XI

B
K

TO
EXHAUST
PUMP
Fig. 15.10. Sound does not travel
in vacuum


   

 
 
 


   
 
    


 






WAVES 15.9

 
    
 

 
  
  
  

   
  

 






 


 

 
 


15.10 PHYSICS (Vol. II)—XI


 

 

   
  

  


  
  
WAVES 15.11


 

 
    





 

  
  
  




 
 
 

  
  
  


 
 



    

15.12 PHYSICS (Vol. II)—XI


    

 
 

 
    

    

 
     

 
    


    



 


    
   
   

   

WAVES 15.13

 

 

 
 
 
 

 
 
 

   
   
   



cos 

Fig. 15.11. Effect of wind on
velocity of sound

 
15.14 PHYSICS (Vol. II)—XI

  




  


 

  

 

 


   
   
  
 
 





 
 
 

S olu tion .  


 
  

 
   


  
 
WAVES 15.15

Fig. 15.12. Two pulses having equal


and opposite displacements moving
in opposite directions. The
overlapping pulses add up to zero
displacement in (b).

 



15.16 PHYSICS (Vol. II)—XI

 

  
  

   
    
   
 

  
  
 

Y
 Y


O X O X


 (a) (b)

 Fig. 15.13. (a) Two superposing waves with  = 0


(b) Resultant wave due to constructive interference.
 
 
Y Y

O X O X

 

(a) (b)

Fig. 15.14. (a) Two superposing waves with  = 


(b) Resultant waves due to destructive interference.
WAVES 15.17

 Fig. 15.15. Reflection of a pulse


meeting a rigid boundary.


Fig. 15.16. Reflection of pulse at an open


boundary.
15.18 PHYSICS (Vol. II)—XI

 


P
L A

Q
N N
P

M
B
 
Q

Fig. 15.17. Reflection of water


 waves from plane surface
   

 P
S

SP = u
SP =
 
Fig. 15.18. Reflection of water waves
from curved surfaces

N
A B
i r

P Q

WATCH EAR
Fig. 15.19. Reflection of sound
waves from a plane surface
WAVES 15.19

FUNNEL

P Q
WATCH

RUBBER TUBE
EAR

Fig. 15.20. Reflection of sound waves


from a curved surface
15.20 PHYSICS (Vol. II)—XI


A

1  O
O
2 
 
B


 
A

 Fig. 15.21. Refraction of water waves

 
 

 
  
 

CO2

O
u

Fig. 15.22. Sound lens


 

A B C

A B C

A B C
x1 x2

Fig. 15.23. Refraction of waves in


strings
WAVES 15.21

A B C

A B C

Fig. 15.24. Refraction of waves
in strings

B
O
P

 x

Fig. 15.25. Plane progressive wave


 
 

 


 

  
    
 
 



 

15.22 PHYSICS (Vol. II)—XI

   
   
  

 


 
   
 

 
  
  

  
      
 


 
  
    
 

 

   

 

  
     
 
 
   

  

 
 
        

WAVES 15.23

  
  
    
 

 

 
  
  
  

  

    

 

  
  
 

  
  
 

  
 
  

    
15.24 PHYSICS (Vol. II)—XI
 y
X
 a 1

 

(a)
 y X

y (b)
X

(c)
y
X


 (d)
y X

(e)

Fig. 15.26. A harmonic wave progressing along the


positive direction of x-axis at different times.

 


WAVES 15.25
 
  
 

 

 
     
 

 

 A
t

 Fig. 15.27. An element of a string at a


fixed location oscillates in time with
amplitude A and period T, as the wave
 passes over it

15.26 PHYSICS (Vol. II)—XI
 
 


   







  


 

 

 

   
 
 

 
  
  

Fig. 15.28. y-x graph for a harmonic


wave
WAVES 15.27

a b b c c d d
e f f g g h h i

Fig. 15.29. Associated curve of


longitudinal wave

  

   
     
 

   
      
   

 
      
 

 

   
          
    

  
        

15.28 PHYSICS (Vol. II)—XI


  

   

 

   
         
     

  
         
   


   

 
     



    

  

   

  
     

WAVES 15.29

y

t + t


t

 
Fig. 15.30. Progress of a harmonic wave from time
 t to t + t, where t is a small interval. The wave
 pattern as a whole shifts to the right. The crest of
the wave (or a point with any fixed phase) moves
right by the distance  in time t

    
 
 

 

     
    
   

 
  
  

    
 
  

   
       
   
15.30 PHYSICS (Vol. II)—XI

   

  

    
  
 

 

 

 

 

 
 
 

   
WAVES 15.31


   
    
 

   


  
  


15.32 PHYSICS (Vol. II)—XI


 
 





 

 

  

 

  


WAVES 15.33
Open End

A1 A5
(a) t=0
N1 A2 N2 A3 N3 A4 N4

 
 

(b) T
t= 4
A1 N 1 A2 N2 A3 N3 A4 N4 A5
 

 A1
A5
 (c) T
t= N1 N2 N3 N4
2 A2 A3 A4
 
 
  

   
(d) 3T
t=
4 A1 N1 A2 N2 A3 N3 A4 N4 A5

 
A1
 A5
(e) t=T
N1 A2 N2 A3 N3 A4 N4

   

Fig. 15.31. Graphical representation of stationary


wave formed due to reflection of a progressive
 wave from a free boundary

 
15.34 PHYSICS (Vol. II)—XI
Closed End

N1 A1 N3 A3
(a) t=0
N2 A2 N4 A4 N 5

(b) t=T
4 N1 A1 N2 A2 N3 A3 N4 A4 N5


 

N5
(c) t=T
2 N1 A1 N2 A2 N3 A3 N4 A4

3T
(d) t =
4 N1 A1 N2 A2 N3 A3 N4 A4 N 5

N1 A1 N3 A3
(e) t=T
N2 A2 N4 A4 N 5

Fig. 15.32. Graphical representation of stationary


wave formed due to reflection of a progressive
wave from a rigid boundary

t = T/2

t = 3T/8, 5T/8

t = T/4, 3T/4
N1 A1 A2 A3 A4 N5
N2 N3 N4
t = T/8, 7T/8

t = 0, T

Fig. 15.33. Resultant wave at different instants


WAVES 15.35

 
15.36 PHYSICS (Vol. II)—XI

x=0 x=L

Fig. 15.34


  
WAVES 15.37

 

 
 


 




15.38 PHYSICS (Vol. II)—XI
A

Fundamental
 or first harmonic
 
A
1
L=
2

A A

N second harmonic
A A
L = 2

A A A
 third harmonic
N N
 A
A A
33
L=
2
  A A A A
  
  
N N N fourth harmonic
A A A A
 
   A A A A A
  
fifth harmonic
N N N N
A A A A A

A A A A A A

sixth harmonic
N N N N N
A A A A A A

Fig. 15.35. Stationary waves in a stretched string fixed at both ends.


 Various modes of vibration are shown.


   
     
     


  
 
WAVES 15.39






15.40 PHYSICS (Vol. II)—XI



 

 
 


  


  
  





WAVES 15.41







 
 
     
  

    


   

 

80 cm

20 cm

1.6 kg

 
Fig. 15.36

 
 
  

  
 



 


 

  
 
15.42 PHYSICS (Vol. II)—XI

   
 


   

  

  

 


   
   
 
   

 
  

 

  
 
   
   

 
  
 
   
 

   
    
 

 
       


  

l
s

B R

Fig. 15.37. Open organ pipe


WAVES 15.43

 

 
  


 

 
15.44 PHYSICS (Vol. II)—XI

 

1
2

N
 A A
  
L
Fig. 15.38. First mode of vibration
   



 N N
A A A

  L
 Fig. 15.39. Second mode of
vibration

     

3
2

 N
   A A N A N A

  Fig. 15.40. Third mode of vibration



WAVES 15.45

    

   

 




  
 

    
 

  
 
15.46 PHYSICS (Vol. II)—XI


 
   

  

   


   
  

B R

Fig. 15.41. Closed organ pipe

 x=0 x=L

 
 Fig. 15.42

 

 



WAVES 15.47


 



1
4

N
A

L
Fig. 15.43. First mode of vibration


 

 
 
4

N A N A

L
Fig. 15.44. Second mode of vibration


  

  

     
15.48 PHYSICS (Vol. II)—XI

 
   4

N A N A N A
 

L
Fig. 15.45. Third mode of vibration
     

  

 



 

WAVES 15.49

A A
e e

 
L L
4 N 2

N e
A
(a) (b)

Fig. 15.46. End correction


 

 

 

 

  

15.50 PHYSICS (Vol. II)—XI

S T

Fig. 15.47. Resonance tube



   
 A A

1
4
 3
 N 2 N 4


 
 N

Fig. 15.48. First and second


resonant positions
 

 
     

  
WAVES 15.51

 
   


 

Fundamental third fifth seventh ninth eleventh


or harmonic harmonic harmonic harmonic harmonic
first harmonic
Fig. 15.49

 
 


  

     

  


  

 
   


   
15.52 PHYSICS (Vol. II)—XI

  

 
 

   

 

   
   
WAVES 15.53

       
  
 

   
  

       
     
     

  
  
 

    
 A B
   
 

 

        

       

 
       

 

         
   


 
       

 

 
 


15.54 PHYSICS (Vol. II)—XI

1.0
y
0
t(s)
–1.0
(a)

1.0
y
0
t(s)
–1.0
(b)

2.0
1.0
y 0 t(s)
0.5 1.0 1.5 2.0
–1.0
–2.0
(c)

Fig. 15.50. Superposition of two harmonic waves,


one of frequency 11 Hz. (a), and the other of
frequency 9 Hz. (b), giving rise to beats of
frequency 2 Hz, as shown in (c).

  

   



WAVES 15.55


   
 
 

 



 
    
  

 
  
  

  

 
     
    
     

   
  

 

  
 
  


 

 
 
  


  

 

 
  

 
  
 

  

         
 
15.56 PHYSICS (Vol. II)—XI

 waves

O
S

Both source and observer at rest

 O
  S S’
s  s
  waves

Source moving towards the observer


 
Fig. 15.51
   
  

    
 

 

  

 

WAVES 15.57

 

 waves

S O A


  Both source and observer are at rest


    S O O A

Observer moving towards source



    Fig. 15.52


   

  


   

 
       
 


  

 
  
 
15.58 PHYSICS (Vol. II)—XI


 

 
 
w here 

 
 
  
 



 
 


    
       
     


 
 

 
  
 

    

    


    
WAVES 15.59


    


    


    

15.60 PHYSICS (Vol. II)—XI

  




 
  
 
 
 
  
   
 
 

   
 
 

  







  
 
 


WAVES 15.61


    
          
  


     




  


   

  

      

  
  

  
  

 
  
 
  
  
 

 
  

  
  

   
 
15.62 PHYSICS (Vol. II)—XI

Observer
O
   
     
 

Fig. 15.53
    


    
 

 
 
     
 


     
    

    
    
 

   
       
 

  
  
   
 
   
  
  
 

     

WAVES 15.63

  

 
 
 

 
 
 

  

 
 
 

 
 
 


15.64 PHYSICS (Vol. II)—XI

 
 
 

 
 
 

 
    
   
 
 

VERY SHORT ANSWER QUESTIONS—Each Question Carries 1 Mark Only

Fig. 15.54

 
WAVES 15.65


  

Fig. 15.55




 
 

 
    
 

  
15.66 PHYSICS (Vol. II)—XI


  

  

 
   
 

 
WAVES 15.67

SHORT ANSWER QUESTIONS—Each Question Carries 2 Marks

Fig. 15.56





 

A N

 
   
 /4

Fig. 15.57

15.68 PHYSICS (Vol. II)—XI

A A
N


/2
 
Fig. 15.58     

    
  

 
 
WAVES 15.69

 







 


 

  

 
 



   
 

 
    



 
15.70 PHYSICS (Vol. II)—XI

SHORT ANSWER QUESTIONS—Each Question Carries 3 Marks

  

  
 


    
 

 
  


 

   
  

 
 
     
 



 
 



  

 
         

     

 
   

   

   

   

 
 

WAVES 15.71

 
 

 
 


 
 
 

   




 



 

   







   

   
 
 

   

   
S s O o m

   

Fig. 15.59
   
15.72 PHYSICS (Vol. II)—XI

 
 
 
  
 
 

0.1 m

0.08 m

 0.04 m

 
   
0.01 m


 
   
Fig. 15.60



  
 
 
  


  

 


WAVES 15.73


  

 


1 3 
2

Fig. 15.61  
 
 

 
   
   

   

 
 
 

Q. 113. y1 = y0 sin [t – kx], y2 = y0 sin [t + kx]. Two The velocities of two waves are equal and opposite.
waves when they are superimposed , we get progressive or Since the waves are travelling in opposite directions
standing wave ? In terms of given data, state the velocity therefore on superimposition of two waves, standing wave is
of y1 and y2 . [AIPMT Mains 2009] formed.
 Q. 114. A policeman buzzes a whistle of frequency
Answer. Velocity of y1 = in + ve x-direction 400 Hz. A car driver is approaching the policeman. The
speed of car is 54 km h–1. Find out the change in frequency
 experienced by the driver, when driver approaches the
Velocity of y2 = in – ve x-direction. policeman and after he crosses the policeman [Velocity of
sound is 350 m s–1] [AIPMT Mains 2009]
15.74 PHYSICS (Vol. II)—XI

Answer.  = 400 Hz, vo = 54 km h–1 = 15 m s–1 


v = 350 m s–1 or =

 
approach =  or × 100 = × 100 = 2.
Q. 118. Find the minimum stress to produce 1%
 strain, for string of density 4 × 103 kg m–3 and velocity of
departing = 
sound 5000 m s–1. [AIPMT Mains 2005] [Ans. 109 N m–2]


 = = × 400 Hz Hint. v = ; T = v2 ; F = 1% of T.

= 34.3 Hz. Q. 119. A transverse mechanical harmonic wave is
Q. 115. The frequency of a string is 375 Hz. Its mass travelling on a string. Maximum velocity and maximum
per unit length is 4 × 10–3 kg m–1 and tension is 360 N. If its acceleration of a particle on the string are 3 m s–1 and
next frequency is 450 Hz, then find out the mass of the 90 m s–2 respectively. If wave is travelling with a speed
string. [AIPMT Mains 2007] 20 m s–1 on the string, write wave function describing the
wave. [IIT Mains 2005]
Answer. v = =  m s–1 = 300 m s–1 Answer. A = 3, A2 = 90,
 
  = 30 rad s–1 and A = 0.1 m

= 
= 20  k = 1.5

375 = , 450 = The equation of wave will be y = A sin (t ± kx)
or y = (0.1) sin (30t ± 1.5x).
Q. 120. A tuning fork of frequency 480 Hz resonates
450 – 375 = or l= = m=2m
 with a tube, closed at one end, of length 16 cm and diameter
5 cm in fundamental mode. Calculate velocity of sound in
= or m = l = 4 × 10–3 × 2 kg = 8 × 10–3 kg. air. [IIT Mains 2003]

Q. 116. Two waves have equations : y1 = a sin (t +  


Answer.  = v or v=
1); y2 = a sin (t + 2). If the amplitude of the resultant 
wave is equal to the amplitude of each of the superposing
For fundamental mode,
waves, then what will be the phase difference between
them ? [AIPMT Mains 2007] n = 1, leff. = l + 0.3d
Answer. A2 = a2 + a2 + 2a2 cos (1 – 2)
   
or a2 = 2a2 [1 + cos (1 – 2)]  v= m s–1 = 336 m s–1.


or cos (1 – 2) = – ; 1 – 2 = 120° = radian. Q. 121. The sound level at a point 5.0 m away from
a point source is 40 dB. What will be the level at a point
Q. 117. The frequencies of two sonometer wires are 50 m away from the source ? [Ans. 20 dB]
each equal to 500 Hz. What percentage change in tension Hint. Intensity is inversely proportional to square of
should be done in one wire so that 5 beats/s are produced ? distance.
[AIPMT Mains 2007]
Q. 122. An astronaut is approaching the Moon. He
sends out a radio signal of frequency 5 × 109 Hz and finds
Answer.  = or =k that the frequency shift in the echo received is 103 Hz.

Find his speed of approach. [Given : c = 3 × 108 m s–1]
log  = log k + log T

Answer.  =  or v=
  
Differentiating, =

 
 
=
WAVES 15.75

Q. 123. Two loudspeakers, as shown in Fig. 15.62,


Q. 125. A pilot of an aeroplane travelling horizontally
separated by a distance 1.5 m, are in phase. Assume that
at 198 km h–1 fires a gun and hears the echo from the
the amplitudes of the sound from the speakers is
ground after an interval of 3 second. If the speed of sound
approximately same at the position of a listener, who is
in air is 330 m s–1, find the height of the aeroplane from
at a distance 4.0 m in front of one of the speakers.
the ground. [Ans. 488 m]

Hint. = 3 or x = 495 m

Speakers t
M
A B
Listener

h
Fig. 15.62

Fig. 15.63
Answer. The distance of the listener from the second
speaker

Q. 126. What is an echo and how is it produced ? What


should be the minimum distance of an obstacle so that an
echo may be heard of (i) sharp momentary sound (ii) mono-
syllabic sound ? Given : speed of sound is 330 m s–1.
[Ans. (i) 16.5 m (ii) 33 m]
0.27 m = (2 
Hint. (i) = ; (ii) =
 
Q. 127. A pipe 30.0 cm long is open at both ends.
Which harmonic mode of the pipe is resonantly excited by
  
a 1.1 kHz source? Will resonance with the same source be
observed if one end of the pipe is closed? Take the speed of
sound in air as 330 m s– 1.
Answer. First few modes of an open pipe are shown in
figure.

Q. 124. Two identical sinusoidal waves, moving in


the same direction along a stretched string, interfere with A A A A A
N N N
each other. The amplitude of each wave is 10.0 mm and
Fundamental Second harmonic
the phase difference between them is 80°. What is the or
First harmonic
amplitude of the resultant wave and the nature of
interference ?
A A A A A A A A A
N N N N N N N
Answer. Since the two waves are identical, they have
same amplitudes. The amplitude of the resultant wave ym is Third harmonic Fourth harmonic

given by : Fig. 15.64

 

  

15.76 PHYSICS (Vol. II)—XI

 


 
  

  


 

Q. 128. A police siren (S) emitting a sinusoidal wave


with frequency 300 Hz is moving away from the observer
(O) with a speed of 45 m s–1 relative to the air and the 
observer is moving toward the siren with a speed of 15 m s–1
relative to the air (Fig. 15.65). What frequency does the
observer hear ? [Take speed of sound = 340 m s–1]

+
S O to S O

  
Fig. 15.65

     
Answer.  = s   = 300   Hz
     
= 277 Hz

LONG ANSWER QUESTIONS


WAVES 15.77


15.78 PHYSICS (Vol. II)—XI

 
 


 
 

  
     
 

 
  
  


WAVES 15.79

  

  
 
 

 
   
 


 


 
15.80 PHYSICS (Vol. II)—XI

9 kg 9 kg
Fig. 15.66


  

 


 
  
  
WAVES 15.81

Fig. 15.67

 



15.82 PHYSICS (Vol. II)—XI

 
WAVES 15.83

 
 
 
15.84 PHYSICS (Vol. II)—XI


WAVES 15.85

  
  
  
  
    
     
    
 

     
         
   
 
  



  
 
  


 
    
      
  
  
 


  
    
  

  
 
  


 
 

 
 
 N

 

45°
W E
   

 
S
15.86 PHYSICS (Vol. II)—XI

 





     
 

 


  




 

  
 
  

 

 

 






 
 
 
 

 
 

 
     
    
  
WAVES 15.87

 
  
  

 
 


 

=0 =

Fig. 15.68

  

  


  

 

     
   
  
 



 





 

   
 
  

 
15.88 PHYSICS (Vol. II)—XI



   

 

  

  


  

 
     



  

   

 

 

  
     

   

  


 
 

     
 
  
  

NCERT EXERCISES (UNIT X) 15.89

NCERT EXERCISES (With Solutions)


 
 

 
 

 



  

  
 

  
 

  




15.90 PHYSICS (Vol. II)—XI

 
   
   
 
 
 
  
 

 

    
    
    


   
    
   

  
   
 

 



 



NCERT EXERCISES (UNIT X) 15.91





 

 


  


Fig. 15.69


 

  
  
 

 


  
  
 

 

   
     
   


 

 
  


 

 

 

 
   



 
 
15.92 PHYSICS (Vol. II)—XI

 






 
 





  
  
 
  
 
 
 

  
  
 
   

   


  

 


  

NCERT EXERCISES (UNIT X) 15.93


    


 
      
  

Fig. 15.70






  




 

  
15.94 PHYSICS (Vol. II)—XI

 
 

 
   
 
 
 
 
     
 

 
  
 


   
      
    

 


 

   
    
  

    
   
 

 
     
 

  
  
   

  
   
 




 
  
  
 
NCERT EXERCISES (UNIT X) 15.95

    
 
 



 

 
 

  


 

 


 

15.96 PHYSICS (Vol. II)—XI

 
 

 
 

VALUE BASED QUESTION


NCERT EXEMPLAR PROBLEMS (UNIT X) 15.97

NCERT EXEMPLAR PROBLEMS (With Answers)


[Based on Higher Order Thinking Skills].

MCQ I (Only one option is correct.)


 

   
 

  

 

   

 

 
   
 

 
   
 

   

 

é ù
 ê   °ú
ë û

 
   

15.98 PHYSICS (Vol. II)—XI

 
= =


=

 
=


 =

=
 

 

 
   =
 

Fig. 15.71 

 = 
 

= = 
 

MCQ II (Any number of options may be


correct.)


NCERT EXEMPLAR PROBLEMS (UNIT X) 15.99

  

 

 

  =


  = =
 

 
= =

VSA

SA

´
=
15.100 PHYSICS (Vol. II)—XI

 ´
 =

B B D D
A in m
A C C E
´ ´

Displacement
t=0

t=? in m

Fig. 15.73

 
´
 
 

= =
 ´

t=0
0    
10 20 30 50
Displacement

 


´ = =

t = T/4

t = T/2

t = 3T/4

t=T

Fig. 15.72

Fig. 15.74
NCERT EXEMPLAR PROBLEMS (UNIT X) 15.101

= =   

´ 
= =



 

LA


 


´
  


=

 

X X
B A A B
O

Fig. 15.75
15.102 PHYSICS (Vol. II)—XI


 


    



  

 
  = ´ 

   
 
 

15.103
15.104 PHYSICS (Vol. II)—XI

 

  


OSCILLATIONS AND WAVES (UNIT X) 15.105

 
    
 

 

 

 

 


15.106 PHYSICS (Vol. II)—XI

subm erged in a liquid of density 

 
  
 
  
 
  
 
 

 
  
 

 
  
 


 
   

 

 

  

   
OSCILLATIONS AND WAVES (UNIT X) 15.107

 
 

 
  

 
   

  
  
 

   
   
   
15.108 PHYSICS (Vol. II)—XI

 


 












OSCILLATIONS AND WAVES (UNIT X) 15.109

 


 

 

 

 
 
 

 

 
15.110 PHYSICS (Vol. II)—XI


 

 
    

 
    
 

 
       
OSCILLATIONS AND WAVES (UNIT X) 15.111





 

 
 
   
 

   
     
   

 

 


15.112 PHYSICS (Vol. II)—XI

 

 

   
       
   

 
     
   

  
OSCILLATIONS AND WAVES (UNIT X) 15.113

 
     
 

  
  
 


   


   


  
  
 
 

 

 
 

 
 
 

15.114 PHYSICS (Vol. II)—XI



 
 
 


    
   
    

 

 


 
 

  
    
  

 

OSCILLATIONS AND WAVES (UNIT X) 15.115

 

 
15.116 PHYSICS (Vol. II)—XI

(T ake g = 10 m s 

 
 
 



OSCILLATIONS AND WAVES (UNIT X) 15.117

 


 
 

 

 

 


15.118 PHYSICS (Vol. II)—XI

 
   
 

   

 

 
     
 

 
  
 
OSCILLATIONS AND WAVES (UNIT X) 15.119

  

    
   
 

 
  

   
     

 

 
   
 
 

   
   

=  
15.120 PHYSICS (Vol. II)—XI
 

  
  

 

a a
O O
T t T t

a a
O O
T t T t





  =


 





OSCILLATIONS AND WAVES (UNIT X) 15.121

 
 



=  = 

 
  
 
= = 

E KE
E
PE

d KE
d

PE
 
 E KE E PE

PE KE
  d d
 
15.122 PHYSICS (Vol. II)—XI

 

 


 
 
=

 

=  
       
       
 = =          
 

   
  
   
 = =

   
    
     
=  =  = =
   
   
    
=
      
= =    


= = =

OSCILLATIONS AND WAVES (UNIT X) 15.123

 

 


 


 

 


 

   
     
  

  
  
 
  
 
    
      
    

  
    
 
 
  
     
    
 

 
   
 

 
      

 


 

 

 
 
  
 


15.124 PHYSICS (Vol. II)—XI


 
 

  
 


 




 
   
 

   
    
  

 
      
     
 
    
       
 
     
 
 
     
      
 

  
 
  
 

    

  
  

  
  


 

 

 


 
  
 


OSCILLATIONS AND WAVES (UNIT X) 15.125

  
 
 


 

  
 
 
   
 
 
 
 
 
  

 


  

  
 
 
 

   


 
  
 



  

  
15.126 PHYSICS (Vol. II)—XI

 

 
 

 
 

 
    

 =–[

   
   
  
 
  

  

 
 

   

   



 
    

    
     
   

 
  
 


OSCILLATIONS AND WAVES (UNIT X) 15.127

 
 
 
 
   
 
 
 



 

 
    
  

   
      
   

   
    
 


    
       
  
   

  
 
  
    
 
 
    
   
 
 
  
  
 
   =
    
 
 
   
 
 

 = =
 

 
 



= 

15.128 PHYSICS (Vol. II)—XI

 
 
   
 

   
    =
  

 =




    
    
  

    
=   
 

 
  
= 
 =




 
  
=


  

 = 

 

 
= = =

=  

 
   
  


 = = 
 
OSCILLATIONS AND WAVES (UNIT X) 15.129

   

 
 
 

= =

= =

 
 
15.130 PHYSICS (Vol. II)—XI

 
 


OSCILLATIONS AND WAVES (UNIT X) 15.131

 

 
 
 



15.132 PHYSICS (Vol. II)—XI

   
 

 
     
 

 

 
 


OSCILLATIONS AND WAVES (UNIT X) 15.133

 
 


 
 = 
 
 
 = 
 


 

 

 
 


15.134 PHYSICS (Vol. II)—XI
 


 

 

     
 


    
 

 



OSCILLATIONS AND WAVES (UNIT X) 15.135

 

   

   
15.136 PHYSICS (Vol. II)—XI

 
 

 
 

  
OSCILLATIONS AND WAVES (UNIT X) 15.137

 
    
   


  
  
 

  

  

 


15.138 PHYSICS (Vol. II)—XI
 
 


 
  
 
OSCILLATIONS AND WAVES (UNIT X) 15.139

 




  
   
  
15.140 PHYSICS (Vol. II)—XI

 

 
   
 
OSCILLATIONS AND WAVES (UNIT X) 15.141

 
    
 

 
 

   

  
 
   


  


  
 

  

  
  
 

  
 
 

15.142 PHYSICS (Vol. II)—XI

 

   
   
 



 
 

 
OSCILLATIONS AND WAVES (UNIT X) 15.143

 

 

 

  

   

   

   


   
15.144 PHYSICS (Vol. II)—XI




   
   
OSCILLATIONS AND WAVES (UNIT X) 15.145

   
     
   

   
     
   

 
 
  

 

 

  
  
 

15.146 PHYSICS (Vol. II)—XI

   
   
 
 


OSCILLATIONS AND WAVES (UNIT X) 15.147

=  

=  

=  

 =

15.148 PHYSICS (Vol. II)—XI

= = 
= =
 

 =

  
= = =
  

= =

= =

OSCILLATIONS AND WAVES (UNIT X) 15.149

 
   
  

 

 

 


 


   

   
 
 

 
    
 





  
  
  
 
 

 


 
 
  
 
 
 

 
 
   

  
   
15.150 PHYSICS (Vol. II)—XI

 


   
 
 

  

 
  


 
 
 


   

   
  
      
   
  
     


 

 

 
   

 

  
 
   
 
 
 
 
    


 
 
  
OSCILLATIONS AND WAVES (UNIT X) 15.151

 

  

 




 

 
  

 
 
 
 

  

   

  



 

   
  
  
 

  
  

   
 


 


15.152 PHYSICS (Vol. II)—XI

        
         
     
 

 
 
   
 


  
 
     

  
 

 
     
   
   
 

  


  
  

 
 
 





  
 
   
  
  
   
 
 
      
       
      

     
      
    

   
    
         
   
   
   
   
  
 
OSCILLATIONS AND WAVES (UNIT X) 15.153

  

  

  
 
 
  
 




 

 



  
 
 
 
     
  
  
 
 


  

  
 



 
 
 

  
 
  

  

   
   
  

 
  
  
 
   
   
 
15.154 PHYSICS (Vol. II)—XI

  
   
   
  
  
  =
 
 
    
 =
 
  
   
    
  


   
    
  
 = =  
   = =


    
= =
 =  = =
=
   
  

   
 =  


  = =




 
 



 
  
 = 

 

 
= 
=

= = 


 
OSCILLATIONS AND WAVES (UNIT X) 15.155

=


   = 
 


 °=  = 
 


       
=  = 

=
=  
    
 
=
= = =
 
=
 =
 
=
= =
 =  

   =

  
=
 
 =
  = =




<
 
  = 
  
<

15.156 PHYSICS (Vol. II)—XI
LOGARITHMIC
AND
OTHER TABLES
LOGARITHMS
0 1 2 3 4 5 6 7 8 9 1 2 3 4 5 6 7 8 9

10 0000 0043 0086 0128 0170 5 9 13 17 21 26 30 34 38


0212 0253 0294 0334 0374 4 8 12 16 20 24 28 32 36

11 0414 0453 0492 0531 0569 4 8 12 16 20 23 27 31 35


0607 0645 0682 0719 0755 4 7 11 15 18 22 26 29 33

12 0792 0828 0864 0899 0934 3 7 11 14 18 21 25 28 32


0969 1004 1038 1072 1106 3 7 10 14 17 20 24 27 31

13 1139 1173 1206 1239 1271 3 6 10 13 16 19 23 26 29


1303 1335 1367 1399 1430 3 7 10 13 16 19 22 25 29

14 1461 1492 1523 1553 1584 3 6 9 12 15 19 22 25 28


1614 1644 1673 1703 1732 3 6 9 12 14 17 20 23 26

15 1761 1790 1818 1847 1875 3 6 9 11 14 17 20 23 26


1903 1931 1959 1987 2014 3 6 8 11 14 17 19 22 25

16 2041 2068 2095 2122 2148 3 6 8 11 14 16 19 22 24


2175 2201 2227 2253 2279 3 5 8 10 13 16 18 21 23

17 2304 2330 2355 2380 2405 3 5 8 10 13 15 18 20 23


2430 2455 2480 2504 2529 3 5 8 10 12 15 17 20 22

18 2553 2577 2601 2625 2648 2 5 7 9 12 14 17 19 21


2672 2695 2718 2742 2765 2 4 7 9 11 14 16 18 21

19 2788 2810 2833 2856 2878 2 4 7 9 11 13 16 18 20


2900 2923 2945 2967 2989 2 4 6 8 11 13 15 17 19

20 3010 3032 3054 3075 3096 3118 3139 3160 3181 3201 2 4 6 8 11 13 15 17 19
21 3222 3243 3263 3284 3304 3324 3345 3365 3385 3404 2 4 6 8 10 12 14 16 18
22 3424 3444 3464 3483 3502 3522 3541 3560 3579 3598 2 4 6 8 10 12 14 15 17
23 3617 3636 3655 3674 3692 3711 3729 3747 3766 3784 2 4 6 7 9 11 13 15 17
24 3802 3820 3838 3856 3874 3892 3909 3927 3945 3962 2 4 5 7 9 11 12 14 16
25 3979 3997 4014 4031 4048 4065 4082 4099 4116 4133 2 3 5 7 9 10 12 14 15
26 4150 4166 4183 4200 4216 4232 4249 4265 4281 4298 2 3 5 7 8 10 11 13 15
27 4314 4330 4346 4362 4378 4393 4409 4425 4440 4456 2 3 5 6 8 9 11 13 14
28 4472 4487 4502 4518 4533 4548 4564 4579 4594 4609 2 3 5 6 8 9 11 12 14
29 4624 4639 4654 4669 4683 4698 4713 4728 4742 4757 1 3 4 6 7 9 10 12 13
30 4771 4786 4800 4814 4829 4843 4857 4871 4886 4900 1 3 4 6 7 9 10 11 13
31 4914 4928 4942 4955 4969 4983 4997 5011 5024 5038 1 3 4 6 7 8 10 11 12
32 5052 5065 5079 5092 5105 5119 5132 5145 5159 5172 1 3 4 5 7 8 9 11 12
33 5185 5198 5211 5224 5237 5250 5263 5276 5289 5302 1 3 4 5 6 8 9 10 12
34 5315 5328 5340 5353 5366 5378 5391 5403 5416 5428 1 3 4 5 6 8 9 10 11
35 5441 5453 5465 5478 5490 5502 5515 5527 5539 5551 1 2 4 5 6 7 9 10 11
36 5563 5575 5587 5599 5611 5623 5635 5647 5658 5670 1 2 4 5 6 7 8 10 11
37 5682 5694 5705 5717 5729 5740 5752 5763 5775 5786 1 2 3 5 6 7 8 9 10
38 5798 5809 5821 5832 5843 5855 5866 5877 5888 5899 1 2 3 5 6 7 8 9 10
39 5911 5922 5933 5944 5955 5966 5977 5988 5999 6010 1 2 3 4 5 7 8 9 10
40 6021 6031 6042 6053 6064 6075 6085 6096 6107 6117 1 2 3 4 5 6 8 9 10
41 6128 6138 6149 6160 6170 6180 6191 6201 6212 6222 1 2 3 4 5 6 7 8 9
42 6232 6243 6253 6263 6274 6284 6294 6304 6314 6325 1 2 3 4 5 6 7 8 9
43 6335 6345 6355 6365 6375 6385 6395 6405 6415 6425 1 2 3 4 5 6 7 8 9
44 6435 6444 6454 6464 6474 6484 6493 6503 6513 6522 1 2 3 4 5 6 7 8 9
45 6532 6542 6551 6561 6571 6580 6590 6599 6609 6618 1 2 3 4 5 6 7 8 9
46 6628 6637 6646 6656 6665 6675 6684 6693 6702 6712 1 2 3 4 5 6 7 7 8
47 6721 6730 6739 6749 6758 6767 6776 6785 6794 6803 1 2 3 4 5 5 6 7 8
48 6812 6821 6830 6839 6848 6857 6866 6875 6884 6893 1 2 3 4 4 5 6 7 8
49 6902 6911 6920 6928 6937 6946 6955 6964 6972 6981 1 2 3 4 4 5 6 7 8

( iii )
LOGARITHMS
0 1 2 3 4 5 6 7 8 9 1 2 3 4 5 6 7 8 9

50 6990 6998 7007 7016 7024 7033 7042 7050 7059 7067 1 2 3 3 4 5 6 7 8
51 7076 7084 7093 7101 7110 7118 7126 7135 7143 7152 1 2 3 3 4 5 6 7 8
52 7160 7168 7177 7185 7193 7202 7210 7218 7226 7235 1 2 2 3 4 5 6 7 7
53 7243 7251 7259 7267 7275 7284 7292 7300 7308 7316 1 2 2 3 4 5 6 6 7
54 7324 7332 7340 7348 7356 7364 7372 7380 7388 7396 1 2 2 3 4 5 6 6 7
55 7404 7412 7419 7427 7435 7443 7451 7459 7466 7474 1 2 2 3 4 5 5 6 7
56 7482 7490 7497 7505 7513 7520 7528 7536 7543 7551 1 2 2 3 4 5 5 6 7
57 7559 7566 7574 7582 7589 7597 7604 7612 7619 7627 1 2 2 3 4 5 5 6 7
58 7634 7642 7649 7657 7664 7672 7679 7686 7694 7701 1 1 2 3 4 4 5 6 7
59 7709 7716 7723 7731 7738 7745 7752 7760 7767 7774 1 1 2 3 4 4 5 6 7
60 7782 7789 7796 7803 7810 7818 7825 7832 7839 7846 1 1 2 3 4 4 5 6 6
61 7853 7860 7868 7875 7882 7889 7896 7903 7910 7917 1 1 2 3 4 4 5 6 6
62 7924 7931 7938 7945 7952 7959 7966 7973 7980 7987 1 1 2 3 3 4 5 6 6
63 7993 8000 8007 8014 8021 8028 8035 8041 8048 8055 1 1 2 3 3 4 5 5 6
64 8062 8069 8075 8082 8089 8096 8102 8109 9116 8122 1 1 2 3 3 4 5 5 6
65 8129 8136 8142 8149 8156 8162 8169 8176 8182 8189 1 1 2 3 3 4 5 5 6
66 8195 8202 8209 8215 8222 8228 8235 8241 8248 8254 1 1 2 3 3 4 5 5 6
67 8261 8267 8274 8280 8287 8293 8299 8306 8312 8319 1 1 2 3 3 4 5 5 6
68 8325 8331 8338 8344 8351 8357 8363 8370 8376 8382 1 1 2 3 3 4 4 5 6
69 8388 8395 8401 8407 8414 8420 8426 8432 8439 8445 1 1 2 2 3 4 4 5 6
70 8451 8457 8463 8470 8476 8482 8488 8494 8500 8506 1 1 2 2 3 4 4 5 6
71 8513 8519 8525 8531 8537 8543 8549 8555 8561 8567 1 1 2 2 3 4 4 5 5
72 8573 8579 8585 8591 8597 8603 8609 8615 8621 8627 1 1 2 2 3 4 4 5 5
73 8633 8639 8645 8651 8657 8663 8669 8675 8681 8686 1 1 2 2 3 4 4 5 5
74 8692 8698 8704 8710 8716 8722 8727 8733 8739 8745 1 1 2 2 3 4 4 5 5
75 8751 8756 8762 8768 8774 8779 8785 8791 8797 8802 1 1 2 2 3 3 4 5 5
76 8808 8814 8820 8825 8831 8837 8842 8848 8854 8859 1 1 2 2 3 3 4 5 5
77 8865 8871 8876 8882 8887 8893 8899 8904 8910 8915 1 1 2 2 3 3 4 4 5
78 8921 8927 8932 8938 8943 8949 8954 8960 8965 8971 1 1 2 2 3 3 4 4 5
79 8976 8982 8987 8993 8998 9004 9009 9015 9020 9025 1 1 2 2 3 3 4 4 5
80 9031 9036 9042 9047 9053 9058 9063 9069 9074 9079 1 1 2 2 3 3 4 4 5
81 9085 9090 9096 9101 9106 9112 9117 9122 9128 9133 1 1 2 2 3 3 4 4 5
82 9138 9143 9149 9154 9159 9165 9170 9175 9180 9186 1 1 2 2 3 3 4 4 5
83 9191 9196 9201 9206 9212 9217 9222 9227 9232 9238 1 1 2 2 3 3 4 4 5
84 9243 9248 9253 9258 9263 9269 9274 9279 9284 9289 1 1 2 2 3 3 4 4 5
85 9294 9299 9304 9309 9315 9320 9325 9330 9335 9340 1 1 2 2 3 3 4 4 5
86 9345 9350 9355 9360 9365 9370 9375 9380 9385 9390 1 1 2 2 3 3 4 4 5
87 9395 9400 9405 9410 9415 9420 9425 9430 9435 9440 0 1 1 2 2 3 3 4 4
88 9445 9450 9455 9460 9465 9469 9474 9479 9484 9489 0 1 1 2 2 3 3 4 4
89 9494 9499 9504 9509 9513 9518 9523 9528 9533 9538 0 1 1 2 2 3 3 4 4
90 9542 9547 9552 9557 9562 9566 9571 9576 9581 9586 0 1 1 2 2 3 3 4 4
91 9590 9595 9600 9605 9609 9614 9619 9624 9628 9633 0 1 1 2 2 3 3 4 4
92 9638 9643 9647 9652 9657 9661 9666 9671 9675 9680 0 1 1 2 2 3 3 4 4
93 9685 9689 9694 9699 9703 9708 9713 9717 9722 9727 0 1 1 2 2 3 3 4 4
94 9731 9736 9741 9745 9750 9754 9759 9763 9768 9773 0 1 1 2 2 3 3 4 4
95 9777 9782 9786 9791 9795 9800 9805 9809 9814 9818 0 1 1 2 2 3 3 4 4
96 9823 9827 9832 9836 9841 9845 9850 9854 9859 9863 0 1 1 2 2 3 3 4 4
97 9868 9872 9877 9881 9886 9890 9894 9899 9903 9908 0 1 1 2 2 3 3 4 4
98 9912 9917 9921 9926 9930 9934 9939 9943 9948 9952 0 1 1 2 2 3 3 4 4
99 9956 9961 9965 9969 9974 9978 9983 9987 9991 9996 0 1 1 2 2 3 3 3 4

( iv )
ANTILOGARITHMS
0 1 2 3 4 5 6 7 8 9 1 2 3 4 5 6 7 8 9

.00 1000 1002 1005 1007 1009 1012 1014 1016 1019 1021 0 0 1 1 1 1 2 2 2
.01 1023 1026 1028 1030 1033 1035 1038 1040 1042 1045 0 0 1 1 1 1 2 2 2
.02 1047 1050 1052 1054 1057 1059 1062 1064 1067 1069 0 0 1 1 1 1 2 2 2
.03 1072 1074 1076 1079 1081 1084 1086 1089 1091 1094 0 0 1 1 1 1 2 2 2
.04 1096 1099 1102 1104 1107 1109 1112 1114 1117 1119 0 1 1 1 1 2 2 2 2
.05 1122 1125 1127 1130 1132 1135 1138 1140 1143 1146 0 1 1 1 1 2 2 2 2
.06 1148 1151 1153 1156 1159 1161 1164 1167 1169 1172 0 1 1 1 1 2 2 2 2
.07 1175 1178 1180 1183 1186 1189 1191 1194 1197 1199 0 1 1 1 1 2 2 2 2
.08 1202 1205 1208 1211 1213 1216 1219 1222 1225 1227 0 1 1 1 1 2 2 2 3
.09 1230 1233 1236 1239 1242 1245 1247 1250 1253 1256 0 1 1 1 1 2 2 2 3
.10 1259 1262 1265 1268 1271 1274 1276 1279 1282 1285 0 1 1 1 1 2 2 2 3
.11 1288 1291 1294 1297 1300 1303 1306 1309 1312 1315 0 1 1 1 2 2 2 2 3
.12 1318 1321 1324 1327 1330 1334 1337 1340 1343 1346 0 1 1 1 2 2 2 2 3
.13 1349 1352 1355 1358 1361 1365 1368 1371 1374 1377 0 1 1 1 2 2 2 3 3
.14 1380 1384 1387 1390 1393 1396 1400 1403 1406 1409 0 1 1 1 2 2 2 3 3
.15 1413 1416 1419 1422 1426 1429 1432 1435 1439 1442 0 1 1 1 2 2 2 3 3
.16 1445 1449 1452 1455 1459 1462 1466 1469 1472 1476 0 1 1 1 2 2 2 3 3
.17 1479 1483 1486 1489 1493 1496 1500 1503 1507 1510 0 1 1 1 2 2 2 3 3
.18 1514 1517 1521 1524 1528 1531 1535 1538 1542 1545 0 1 1 1 2 2 2 3 3
.19 1549 1552 1556 1560 1563 1567 1570 1574 1578 1581 0 1 1 1 2 2 3 3 3
.20 1585 1589 1592 1596 1600 1603 1607 1611 1614 1618 0 1 1 1 2 2 3 3 3
.21 1622 1626 1629 1633 1637 1641 1644 1648 1652 1656 0 1 1 2 2 2 3 3 3
.22 1660 1663 1667 1671 1675 1679 1683 1687 1690 1694 0 1 1 2 2 2 3 3 3
.23 1698 1702 1706 1710 1714 1718 1722 1726 1730 1734 0 1 1 2 2 2 3 3 4
.24 1738 1742 1746 1750 1754 1758 1762 1766 1770 1774 0 1 1 2 2 2 3 3 4
.25 1778 1782 1786 1791 1795 1799 1803 1807 1811 1816 0 1 1 2 2 2 3 3 4
.26 1820 1824 1828 1832 1837 1841 1845 1849 1854 1858 0 1 1 2 2 3 3 3 4
.27 1862 1866 1871 1875 1879 1884 1888 1892 1897 1901 0 1 1 2 2 3 3 3 4
.28 1905 1910 1914 1919 1923 1928 1932 1936 1941 1945 0 1 1 2 2 3 3 4 4
.29 1950 1954 1959 1963 1968 1972 1977 1982 1986 1991 0 1 1 2 2 3 3 4 4
.30 1995 2000 2004 2009 2014 2018 2023 2028 2032 2037 0 1 1 2 2 3 3 4 4
.31 2042 2046 2051 2056 2061 2065 2070 2075 2080 2084 0 1 1 2 2 3 3 4 4
.32 2089 2094 2099 2104 2109 2113 2118 2123 2128 2133 0 1 1 2 2 3 3 4 4
.33 2138 2143 2148 2153 2158 2163 2168 2173 2178 2183 0 1 1 2 2 3 3 4 4
.34 2188 2193 2198 2203 2208 2213 2218 2223 2228 2234 1 1 2 2 3 3 4 4 5
.35 2239 2244 2249 2254 2259 2265 2270 2275 2280 2286 1 1 2 2 3 3 4 4 5
.36 2291 2296 2301 2307 2312 2317 2323 2328 2333 2339 1 1 2 2 3 3 4 4 5
.37 2344 2350 2355 2360 2366 2371 2377 2382 2388 2393 1 1 2 2 3 3 4 4 5
.38 2399 2404 2410 2415 2421 2427 2432 2438 2443 2449 1 1 2 2 3 3 4 4 5
.39 2455 2460 2466 2472 2477 2483 2489 2495 2500 2506 1 1 2 2 3 3 4 5 5
.40 2512 2518 2523 2529 2535 2541 2547 2553 2559 2564 1 1 2 2 3 4 4 5 5
.41 2570 2576 2582 2588 2594 2600 2606 2612 2618 2624 1 1 2 2 3 4 4 5 5
.42 2630 2636 2642 2649 2655 2661 2667 2673 2679 2685 1 1 2 2 3 4 4 5 6
.43 2692 2698 2704 2710 2716 2723 2729 2735 2742 2748 1 1 2 3 3 4 4 5 6
.44 2754 2761 2767 2773 2780 2786 2793 2799 2805 2812 1 1 2 3 3 4 4 5 6
.45 2818 2825 2831 2838 2844 2851 2858 2864 2871 2877 1 1 2 3 3 4 5 5 6
.46 2884 2891 2897 2904 2911 2917 2924 2931 2938 2944 1 1 2 3 3 4 5 5 6
.47 2951 2958 2965 2972 2979 2985 2992 2999 3006 3013 1 1 2 3 3 4 5 5 6
.48 3020 3027 3034 3041 3048 3055 3062 3069 3076 3083 1 1 2 3 4 4 5 6 6
.49 3090 3097 3105 3112 3119 3126 3133 3141 3148 3155 1 1 2 3 4 4 5 6 6

(v)
ANTILOGARITHMS
0 1 2 3 4 5 6 7 8 9 1 2 3 4 5 6 7 8 9
.50 3162 3170 3177 3184 3192 3199 3206 3214 3221 3228 1 1 2 3 4 4 5 6 7
.51 3236 3243 3251 3258 3266 3273 3281 3289 3296 3304 1 2 2 3 4 5 5 6 7
.52 3311 3319 3327 3334 3342 3350 3357 3365 3373 3381 1 2 2 3 4 5 5 6 7
.53 3388 3396 3404 3412 3420 3428 3436 3443 3451 3459 1 2 2 3 4 5 6 6 7
.54 3467 3475 3483 3491 3499 3508 3516 3524 3532 3540 1 2 2 3 4 5 6 6 7
.55 3548 3556 3565 3573 3581 3589 3597 3606 3614 3622 1 2 2 3 4 5 6 7 7
.56 3631 3639 3648 3656 3664 3673 3681 3690 3698 3707 1 2 3 3 4 5 6 7 8
.57 3715 3724 3733 3741 3750 3758 3767 3776 3784 3793 1 2 3 3 4 5 6 7 8
.58 3802 3811 3819 3828 3837 3846 3855 3864 3873 3882 1 2 3 4 4 5 6 7 8
.59 3890 3899 3908 3917 3926 3936 3945 3954 3963 3972 1 2 3 4 5 5 6 7 8
.60 3981 3990 3999 4009 4018 4027 4036 4046 4055 4064 1 2 3 4 5 6 6 7 8
.61 4074 4083 4093 4102 4111 4121 4130 4140 4150 4159 1 2 3 4 5 6 7 8 9
.62 4169 4178 4188 4198 4207 4217 4227 4236 4246 4256 1 2 3 4 5 6 7 8 9
.63 4266 4276 4285 4295 4305 4315 4325 4335 4345 4355 1 2 3 4 5 6 7 8 9
.64 4365 4375 4385 4395 4406 4416 4426 4436 4446 4457 1 2 3 4 5 6 7 8 9
.65 4467 4477 4487 4498 4508 4519 4529 4539 4550 4560 1 2 3 4 5 6 7 8 9
.66 4571 4581 4592 4603 4613 4624 4634 4645 4656 4667 1 2 3 4 5 6 7 9 10
.67 4677 4688 4699 4710 4721 4732 4742 4753 4764 4775 1 2 3 4 5 7 8 9 10
.68 4786 4797 4808 4819 4831 4842 4853 4864 4875 4887 1 2 3 4 6 7 8 9 10
.69 4898 4909 4920 4932 4943 4955 4966 4977 4989 5000 1 2 3 5 6 7 8 9 10
.70 5012 5023 5035 5047 5058 5070 5082 5093 5105 5117 1 2 4 5 6 7 8 9 11
.71 5129 5140 5152 5164 5176 5188 5200 5212 5224 5236 1 2 4 5 6 7 8 10 11
.72 5248 5260 5272 5284 5297 5309 5321 5333 5346 5358 1 2 4 5 6 7 9 10 11
.73 5370 5383 5395 5408 5420 5433 5445 5458 5470 5483 1 3 4 5 6 8 9 10 11
.74 5495 5508 5521 5534 5546 5559 5572 5585 5598 5610 1 3 4 5 6 8 9 10 12
.75 5623 5636 5649 5662 5675 5689 5702 5715 5728 5741 1 3 4 5 7 8 9 10 12
.76 5754 5768 5781 5794 5808 5821 5834 5848 5861 5875 1 3 4 5 7 8 9 11 12
.77 5888 5902 5916 5929 5943 5957 5970 5984 5998 6012 1 3 4 5 7 8 10 11 12
.78 6026 6039 6053 6067 6081 6095 6109 6124 6138 6152 1 3 4 6 7 8 10 11 13
.79 6166 6180 6194 6209 6223 6237 6252 6266 6281 6295 1 3 4 6 7 9 10 11 13
.80 6310 6324 6339 6353 6368 6383 6397 6412 6427 6442 1 3 4 6 7 9 10 12 13
.81 6457 6471 6486 6501 6516 6531 6546 6561 6577 6592 2 3 5 6 8 9 11 12 14
.82 6607 6622 6637 6653 6668 6683 6699 6714 6730 6745 2 3 5 6 8 9 11 12 14
.83 6761 6776 6792 6808 6823 6839 6855 6871 6887 6902 2 3 5 6 8 9 11 13 14
.84 6918 6934 6950 6966 6982 6998 7015 7031 7047 7063 2 3 5 6 8 10 11 13 15
.85 7079 7096 7112 7129 7145 7161 7178 7194 7211 7228 2 3 5 7 8 10 12 13 15
.86 7244 7261 7278 7295 7311 7328 7345 7362 7379 7396 2 3 5 7 8 10 12 13 15
.87 7413 7430 7447 7464 7482 7499 7516 7534 7551 7568 2 3 5 7 9 10 12 14 16
.88 7586 7603 7621 7638 7656 7674 7691 7709 7727 7745 2 4 5 7 9 11 12 14 16
.89 7762 7780 7798 7816 7834 7852 7870 7889 7907 7925 2 4 5 7 9 11 13 14 16
.90 7943 7962 7980 7998 8017 8035 8054 8072 8091 8110 2 4 6 7 9 11 13 15 17
.91 8128 8147 8166 8185 8204 8222 8241 8260 8279 8299 2 4 6 8 9 11 13 15 17
.92 8318 8337 8356 8375 8395 8414 8433 8453 8472 8492 2 4 6 8 10 12 14 15 17
.93 8511 8531 8551 8570 8590 8610 8630 8650 8670 8690 2 4 6 8 10 12 14 16 18
.94 8710 8730 8750 8770 8790 8810 8831 8851 8872 8892 2 4 6 8 10 12 14 16 18
.95 8913 8933 8954 8974 8995 9016 9036 9057 9078 9099 2 4 6 8 10 12 15 17 19
.96 9120 9141 9162 9183 9204 9226 9247 9268 9290 9311 2 4 6 8 11 13 15 17 20
.97 9333 9354 9376 9397 9419 9441 9462 9484 9506 9528 2 4 7 9 11 13 15 17 20
.98 9550 9572 9594 9616 9638 9661 9683 9705 9727 9750 2 4 7 9 11 13 16 18 20
.99 9772 9795 9817 9840 9863 9886 9908 9931 9954 9977 2 5 7 9 11 14 16 18 20

( vi )
NATURAL SINES
Mean
Degrees

0′ 6′ 12′ 18′ 24′ 30′ 36′ 42′ 48′ 54′ Differences


0°.0 0°.1 0°.2 0°.3 0°.4 0°.5 0°.6 0°.7 0°.8 0°.9
1 2 3 4 5

0 .0000 0017 0035 0052 0070 0087 0105 0122 0140 0157 3 6 9 12 15
1 .0175 0192 0209 0227 0244 0262 0279 0297 0314 0332 3 6 9 12 15
2 .0349 0366 0384 0401 0419 0436 0454 0471 0488 0506 3 6 9 12 15
3 .0523 0541 0558 0576 0593 0610 0628 0645 0663 0680 3 6 9 12 15
4 .0698 0715 0732 0750 0767 0785 0802 0819 0837 0854 3 6 9 12 15
5 .0872 0889 0906 0924 0941 0958 0976 0993 1011 1028 3 6 9 12 14
6 .1045 1063 1080 1097 1115 1132 1149 1167 1184 1201 3 6 9 12 14
7 .1219 1236 1253 1271 1288 1305 1323 1340 1357 1374 3 6 9 12 14
8 .1392 1409 1426 1444 1461 1478 1495 1513 1530 1547 3 6 9 12 14
9 .1564 1582 1599 1616 1633 1650 1668 1685 1702 1719 3 6 9 12 14
10 .1736 1754 1771 1788 1805 1822 1840 1857 1874 1891 3 6 9 12 14
11 .1908 1925 1942 1959 1977 1994 2011 2028 2045 2062 3 6 9 11 14
12 .2079 2096 2113 2130 2147 2164 2181 2198 2215 2232 3 6 9 11 14
13 .2250 2267 2284 2300 2317 2334 2351 2368 2385 2402 3 6 8 11 14
14 .2419 2436 2453 2470 2487 2504 2521 2538 2554 2571 3 6 8 11 14
15 .2588 2605 2622 2639 2656 2672 2689 2706 2723 2740 3 6 8 11 14
16 .2756 2773 2790 2807 2823 2840 2857 2874 2890 2907 3 6 8 11 14
17 .2924 2940 2957 2974 2990 3007 3024 3040 3057 3074 3 6 8 11 14
18 .3090 3107 3123 3140 3156 3173 3190 3206 3223 3239 3 6 8 11 14
19 .3256 3272 3289 3305 3322 3338 3355 3371 3387 3404 3 5 8 11 14
20 .3420 3437 3453 3469 3486 3502 3518 3535 3551 3567 3 5 8 11 14
21 .3584 3600 3616 3633 3649 3665 3681 3697 3714 3730 3 5 8 11 14
22 .3746 3762 3778 3795 3811 3827 3843 3859 3875 3891 3 5 8 11 14
23 .3907 3923 3939 3955 3971 3987 4003 4019 4035 4051 3 5 8 11 14
24 .4067 4083 4099 4115 4131 4147 4163 4179 4195 4210 3 5 8 11 13
25 .4226 4242 4258 4274 4289 4305 4321 4337 4352 4368 3 5 8 11 13
26 .4384 4399 4415 4431 4446 4462 4478 4493 4509 4524 3 5 8 10 13
27 .4540 4555 4571 4586 4602 4617 4633 4648 4664 4679 3 5 8 10 13
28 .4695 4710 4726 4741 4756 4772 4787 4802 4818 4833 3 5 8 10 13
29 .4848 4863 4879 4894 4909 4924 4939 4955 4970 4985 3 5 8 10 13
30 .5000 5015 5030 5045 5060 5075 5090 5105 5120 5135 3 5 8 10 13
31 .5150 5165 5180 5195 5210 5225 5240 5255 5270 5284 2 5 7 10 12
32 .5299 5314 5329 5344 5358 5373 5388 5402 5417 5432 2 5 7 10 12
33 .5446 5461 5476 5490 5505 5519 5534 5548 5563 5577 2 5 7 10 12
34 .5592 5606 5621 5635 5650 5664 5678 5693 5707 5721 2 5 7 10 12
35 .5736 5750 5764 5779 5793 5807 5821 5835 5850 5864 2 5 7 10 12
36 .5878 5892 5906 5920 5934 5948 5962 5976 5990 6004 2 5 7 9 12
37 .6018 6032 6046 6060 6074 6088 6101 6115 6129 6143 2 5 7 9 12
38 .6157 6170 6184 6198 6211 6225 6239 6252 6266 6280 2 5 7 9 11
39 .6293 6307 6320 6334 6347 6361 6374 6388 6401 6414 2 4 7 9 11
40 .6428 6441 6455 6468 6481 6494 6508 6521 6534 6547 2 4 7 9 11
41 .6561 6574 6587 6600 6613 6626 6639 6652 6665 6678 2 4 7 9 11
42 .6691 6704 6717 6730 6743 6756 6769 6782 6794 6807 2 4 6 9 11
43 .6820 6833 6845 6858 6871 6884 6896 6909 6921 6934 2 4 6 8 11
44 .6947 6959 6972 6984 6997 7009 7022 7034 7046 7059 2 4 6 8 10

( vii )
NATURAL SINES
Mean
Degrees

0′ 6′ 12′ 18′ 24′ 30′ 36′ 42′ 48′ 54′ Differences


0°.0 0°.1 0°.2 0°.3 0°.4 0°.5 0°.6 0°.7 0°.8 0°.9
1 2 3 4 5

45 .7071 7083 7096 7108 7120 7133 7145 7157 7169 7181 2 4 6 8 10
46 .7193 7206 7218 7230 7242 7254 7266 7278 7290 7302 2 4 6 8 10
47 .7314 7325 7337 7349 7361 7373 7385 7396 7408 7420 2 4 6 8 10
48 .7431 7443 7455 7466 7478 7490 7501 7513 7524 7536 2 4 6 8 10
49 .7547 7558 7570 7581 7593 7604 7615 7627 7638 7649 2 4 6 8 9
50 .7660 7672 7683 7694 7705 7716 7727 7738 7749 7760 2 4 6 7 9
51 .7771 7782 7793 7804 7815 7826 7837 7848 7859 7869 2 4 5 7 9
52 .7880 7891 7902 7912 7923 7934 7944 7955 7965 7976 2 4 5 7 9
53 .7986 7997 8007 8018 8028 8039 8049 8059 8070 8080 2 3 5 7 9
54 .8090 8100 8111 8121 8131 8141 8151 8161 8171 8181 2 3 5 7 8
55 .8192 8202 8211 8221 8231 8241 8251 8261 8271 8281 2 3 5 7 8
56 .8290 8300 8310 8320 8329 8339 8348 8358 8368 8377 2 3 5 6 8
57 .8387 8396 8406 8415 8425 8434 8443 8453 8462 8471 2 3 5 6 8
58 .8480 8490 8499 8508 8517 8526 8536 8545 8554 8563 2 3 5 6 8
59 .8572 8581 8590 8599 8607 8616 8625 8634 8643 8652 1 3 4 6 7
60 .8660 8669 8678 8686 8695 8704 8712 8721 8729 8738 1 3 4 6 7
61 .8746 8755 8763 8771 8780 8788 8796 8805 8813 8821 1 3 4 6 7
62 .8829 8838 8846 8854 8862 8870 8878 8886 8894 8902 1 3 4 5 7
63 .8910 8918 8926 8934 8942 8949 8957 8965 8973 8980 1 3 4 5 6
64 .8988 8996 9003 9011 9018 9026 9033 9041 9048 9056 1 3 4 5 6
65 .9063 9070 9078 9085 9092 9100 9107 9114 9121 9128 1 2 4 5 6
66 .9135 9143 9150 9157 9164 9171 9178 9184 9191 9198 1 2 3 5 6
67 .9205 9212 9219 9225 9232 9239 9245 9252 9259 9265 1 2 3 4 6
68 .9272 9278 9285 9291 9298 9304 9311 9317 9323 9330 1 2 3 4 5
69 .9336 9342 9348 9354 9361 9367 9373 9379 9385 9391 1 2 3 4 5
70 .9397 9403 9409 9415 9421 9426 9432 9438 9444 9449 1 2 3 4 5
71 .9455 9461 9466 9472 9478 9483 9489 9494 9500 9505 1 2 3 4 5
72 .9511 9516 9521 9527 9532 9537 9542 9548 9553 9558 1 2 3 3 4
73 .9563 9568 9573 9578 9583 9588 9593 9598 9603 9608 1 2 2 3 4
74 .9613 9617 9622 9627 9632 9636 9641 9646 9650 9655 1 2 2 3 4
75 .9659 9664 9668 9673 9677 9681 9685 9690 9694 9699 1 1 2 3 4
76 .9703 9707 9711 9715 9720 9724 9728 9732 9736 9740 1 1 2 3 3
77 .9744 9748 9751 9755 9759 9763 9767 9770 9774 9778 1 1 2 3 3
78 .9781 9785 9789 9792 9796 9799 9803 9806 9810 9813 1 1 2 2 3
79 .9816 9820 9823 9826 9829 9833 9836 9839 9842 9845 1 1 2 2 3
80 .9848 9851 9854 9857 9860 9863 9866 9869 9871 9874 0 1 1 2 2
81 .9877 9880 9882 9885 9888 9890 9893 9895 9898 9900 0 1 1 2 2
82 .9903 9905 9907 9910 9912 9914 9917 9919 9921 9923 0 1 1 2 2
83 .9925 9928 9930 9932 9934 9936 9938 9940 9942 9943 0 1 1 1 2
84 .9945 9947 9949 9951 9952 9954 9956 9957 9959 9960 0 1 1 1 2
85 .9962 9963 9965 9966 9968 9969 9971 9972 9973 9974 0 0 1 1 1
86 .9976 9977 9978 9979 9980 9981 9982 9983 9984 9985 0 0 1 1 1
87 .9986 9987 9988 9989 9990 9990 9991 9992 9993 9993 0 0 0 1 1
88 .9994 9995 9995 9996 9996 9997 9997 9997 9998 9998 0 0 0 0 0
89 .9998 9999 9999 9999 9999 1.000 1.000 1.000 1.000 1.000 0 0 0 0 0
90 1.000

( viii )
NATURAL COSINES
[Numbers in difference columns to be subtracted, not added]

Mean
Degrees

0′ 6′ 12′ 18′ 24′ 30′ 36′ 42′ 48′ 54′ Differences


0°.0 0°.1 0° .2 0°.3 0°.4 0°.5 0°.6 0°.7 0°.8 0°.9
1 2 3 4 5

0 1.000 1.000 1.000 1.000 1.000 1.000 .9999 9999 9999 9999 0 0 0 0 0
1 .9998 9998 9998 9997 9997 9997 9996 9996 9995 9995 0 0 0 0 0
2 .9994 9993 9993 9992 9991 9990 9990 9989 9988 9987 0 0 0 1 1
3 .9986 9985 9984 9983 9982 9981 9980 9979 9978 9977 0 0 1 1 1
4 .9976 9974 9973 9972 9971 9969 9968 9966 9965 9963 0 0 1 1 1
5 .9962 9960 9959 9957 9956 9954 9952 9951 9949 9947 0 1 1 1 2
6 .9945 9943 9942 9940 9938 9936 9934 9932 9930 9928 0 1 1 1 2
7 .9925 9923 9921 9919 9917 9914 9912 9910 9907 9905 0 1 1 2 2
8 .9903 9900 9898 9895 9893 9890 9888 9885 9882 9880 0 1 1 2 2
9 .9877 9874 9871 9869 9866 9863 9860 9857 9854 9851 0 1 1 2 2
10 .9848 9845 9842 9839 9836 9833 9829 9826 9823 9820 1 1 2 2 3
11 .9816 9813 9810 9806 9803 9799 9796 9792 9789 9785 1 1 2 2 3
12 .9781 9778 9774 9770 9767 9763 9759 9755 9751 9748 1 1 2 3 3
13 .9744 9740 9736 9732 9728 9724 9720 9715 9711 9707 1 1 2 3 3
14 .9703 9699 9694 9690 9686 9681 9677 9673 9668 9664 1 1 2 3 4
15 .9659 9655 9650 9646 9641 9636 9632 9627 9622 9617 1 2 2 3 4
16 .9613 9608 9603 9598 9593 9588 9583 9578 9573 9568 1 2 2 3 4
17 .9563 9558 9553 9548 9542 9537 9532 9527 9521 9516 1 2 3 3 4
18 .9511 9505 9500 9494 9489 9483 9478 9472 9466 9461 1 2 3 4 5
19 .9455 9449 9444 9438 9432 9426 9421 9415 9409 9403 1 2 3 4 5
20 .9397 9391 9385 9379 9373 9367 9361 9354 9348 9342 1 2 3 4 5
21 .9336 9330 9323 9317 9311 9304 9298 9291 9285 9278 1 2 3 4 5
22 .9272 9265 9259 9252 9245 9239 9232 9225 9219 9212 1 2 3 4 6
23 .9205 9198 9191 9184 9178 9171 9164 9157 9150 9143 1 2 3 5 6
24 .9135 9128 9121 9114 9107 9100 9092 9085 9078 9070 1 2 4 5 6
25 .9063 9056 9048 9041 9033 9026 9018 9011 9003 8996 1 3 4 5 6
26 .8988 8980 8973 8965 8957 8949 8942 8934 8926 8918 1 3 4 5 6
27 .8910 8902 8894 8886 8878 8870 8862 8854 8846 8838 1 3 4 5 7
28 .8829 8821 8813 8805 8796 8788 8780 8771 8763 8755 1 3 4 6 7
29 .8746 8738 8729 8721 8712 8704 8695 8686 8678 8669 1 3 4 6 7
30 .8660 8652 8643 8634 8625 8616 8607 8599 8590 8581 1 3 4 6 7
31 .8572 8563 8554 8545 8536 8526 8517 8508 8499 8490 2 3 5 6 8
32 .8480 8471 8462 8453 8443 8434 8425 8415 8406 8396 2 3 5 6 8
33 .8387 8377 8368 8358 8348 8339 8329 8320 8310 8300 2 3 5 6 8
34 .8290 8281 8271 8261 8251 8241 8231 8221 8211 8202 2 3 5 7 8
35 .8192 8181 8171 8161 8151 8141 8131 8121 8111 8100 2 3 5 7 8
36 .8090 8080 8070 8059 8049 8039 8028 8018 8007 7997 2 3 5 7 9
37 .7986 7976 7965 7955 7944 7934 7923 7912 7902 7891 2 4 5 7 9
38 .7880 7869 7859 7848 7837 7826 7815 7804 7793 7782 2 4 5 7 9
39 .7771 7760 7749 7738 7727 7716 7705 7694 7683 7672 2 4 6 7 9
40 .7660 7649 7638 7627 7615 7604 7593 7581 7570 7559 2 4 6 8 9
41 .7547 7536 7524 7513 7501 7490 7478 7466 7455 7443 2 4 6 8 10
42 .7431 7420 7408 7396 7385 7373 7361 7349 7337 7325 2 4 6 8 10
43 .7314 7302 7290 7278 7266 7254 7242 7230 7218 7206 2 4 6 8 10
44 .7193 7181 7169 7157 7145 7133 7120 7108 7096 7083 2 4 6 8 10

( ix )
NATURAL COSINES
[Numbers in difference columns to be subtracted, not added]
Degrees

0′ 6′ 12′ 18′ 24′ 30′ 36′ 42′ 48′ 54′ Mean Differences
0°.0 0°.1 0°.2 0°.3 0°.4 0°.5 0°.6 0°.7 0°.8 0°.9
1 2 3 4 5

45 .7071 7059 7046 7034 7022 7009 6997 6984 6972 6959 2 4 6 8 10
46 .6947 6934 6921 6909 6896 6884 6871 6858 6845 6833 2 4 6 8 11
47 .6820 6807 6794 6782 6769 6756 6743 6730 6717 6704 2 4 6 9 11
48 .6691 6678 6665 6652 6639 6626 6613 6600 6587 6574 2 4 7 9 11
49 .6561 6547 6534 6521 6508 6494 6481 6468 6455 6441 2 4 7 9 11
50 .6428 6414 6401 6388 6374 6361 6347 6334 6320 6307 2 4 7 9 11
51 .6293 6280 6266 6252 6239 6225 6211 6198 6184 6170 2 5 7 9 11
52 .6157 6143 6129 6115 6101 6088 6074 6060 6046 6032 2 5 7 9 12
53 .6018 6004 5990 5976 5962 5948 5934 5920 5906 5892 2 5 7 9 12
54 .5878 5864 5850 5835 5821 5807 5793 5779 5764 5750 2 5 7 9 12
55 .5736 5721 5707 5693 5678 5664 5650 5635 5621 5606 2 5 7 10 12
56 .5592 5577 5563 5548 5534 5519 5505 5490 5476 5461 2 5 7 10 12
57 .5446 5432 5417 5402 5388 5373 5358 5344 5329 5314 2 5 7 10 12
58 .5299 5284 5270 5255 5240 5225 5210 5195 5180 5165 2 5 7 10 12
59 .5150 5135 5120 5105 5090 5075 5060 5045 5030 5015 3 5 8 10 13
60 .5000 4985 4970 4955 4939 4924 4909 4894 4879 4863 3 5 8 10 13
61 .4848 4833 4818 4802 4787 4772 4756 4741 4726 4710 3 5 8 10 13
62 .4695 4679 4664 4648 4633 4617 4602 4586 4571 4555 3 5 8 10 13
63 .4540 4524 4509 4493 4478 4462 4446 4431 4415 4399 3 5 8 10 13
64 .4384 4368 4352 4337 4321 4305 4289 4274 4258 4242 3 5 8 11 13
65 .4226 4210 4195 4179 4163 4147 4131 4115 4099 4083 3 5 8 11 13
66 .4067 4051 4035 4019 4003 3987 3971 3955 3939 3923 3 5 8 11 14
67 .3907 3891 3875 3859 3843 3827 3811 3795 3778 3762 3 5 8 11 14
68 .3746 3730 3714 3697 3681 3665 3649 3633 3616 3600 3 5 8 11 14
69 .3584 3567 3551 3535 3518 3502 3486 3469 3453 3437 3 5 8 11 14
70 .3420 3404 3387 3371 3355 3338 3322 3305 3289 3272 3 5 8 11 14
71 .3256 3239 3223 3206 3190 3173 3156 3140 3123 3107 3 6 8 11 14
72 .3090 3074 3057 3040 3024 3007 2990 2974 2957 2940 3 6 8 11 14
73 .2924 2907 2890 2874 2857 2840 2823 2807 2790 2773 3 6 8 11 14
74 .2756 2740 2723 2706 2689 2672 2656 2639 2622 2605 3 6 8 11 14
75 .2588 2571 2554 2538 2521 2504 2487 2470 2453 2436 3 6 8 11 14
76 .2419 2402 2385 2368 2351 2334 2317 2300 2284 2267 3 6 8 11 14
77 .2250 2233 2215 2198 2181 2164 2147 2130 2113 2096 3 6 9 11 14
78 .2079 2062 2045 2028 2011 1994 1977 1959 1942 1925 3 6 9 11 14
79 .1908 1891 1874 1857 1840 1822 1805 1788 1771 1754 3 6 9 11 14
80 .1736 1719 1702 1685 1668 1650 1633 1616 1599 1582 3 6 9 12 14
81 .1564 1547 1530 1513 1495 1478 1461 1444 1426 1409 3 6 9 12 14
82 .1392 1374 1357 1340 1323 1305 1286 1271 1253 1236 3 6 9 12 14
83 .1219 1201 1184 1167 1149 1132 1115 1097 1080 1063 3 6 9 12 14
84 .1045 1028 1011 0993 0976 0958 0941 0924 0906 0889 3 6 9 12 14
85 .0872 0854 0837 0819 0802 0785 0767 0750 0732 0715 3 6 9 12 15
86 .0698 0680 0663 0645 0628 0610 0593 0576 0558 0541 3 6 9 12 15
87 .0523 0506 0488 0471 0454 0436 0419 0401 0384 0360 3 6 9 12 15
88 .0349 0332 0314 0297 0279 0262 0244 0227 0209 0192 3 6 9 12 15
89 .0175 0157 0140 0122 0105 0087 0070 0052 0035 0017 3 6 9 12 15
90 .0000
(x)
NATURAL TANGENTS
Degrees

0′ 6′ 12′ 18′ 24′ 30′ 36′ 42′ 48′ 54′ Mean Differences
0°.0 0°.1 0°.2 0°.3 0°.4 0°.5 0°.6 0°.7 0°.8 0°.9
1 2 3 4 5

0 .0000 0017 0035 0052 0070 0087 0105 0122 0140 0157 3 6 9 12 15
1 .0175 0192 0209 0227 0244 0262 0279 0297 0314 0332 3 6 9 12 15
2 .0349 0367 0384 0402 0419 0437 0454 0472 0489 0507 3 6 9 12 15
3 .0524 0542 0559 0577 0594 0612 0629 0647 0664 0682 3 6 9 12 15
4 .0699 0717 0734 0752 0769 0787 0805 0822 0840 0857 3 6 9 12 15

5 .0875 0892 0910 0928 0945 0963 0981 0998 1016 1033 3 6 9 12 15
6 .1051 1069 1086 1104 1122 1139 1157 1175 1192 1210 3 6 9 12 15
7 .1228 1246 1263 1281 1299 1317 1334 1352 1370 1388 3 6 9 12 15
8 .1405 1423 1441 1459 1477 1495 1512 1530 1548 1566 3 6 9 12 15
9 .1584 1602 1620 1638 1655 1673 1691 1709 1727 1745 3 6 9 12 15
10 .1763 1781 1799 1817 1835 1853 1871 1890 1908 1926 3 6 9 12 15
11 .1944 1962 1980 1998 2016 2035 2053 2071 2089 2107 3 6 9 12 15
12 .2126 2144 2162 2180 2199 2217 2235 2254 2272 2290 3 6 9 12 15
13 .2309 2327 2345 2364 2382 2401 2419 2438 2456 2475 3 6 9 12 15
14 .2493 2512 2530 2549 2568 2586 2605 2623 2642 2661 3 6 9 12 16
15 .2679 2698 2717 2736 2754 2773 2792 2811 2830 2849 3 6 9 13 16
16 .2867 2886 2905 2924 2943 2962 2981 3000 3019 3038 3 6 9 13 16
17 .3057 3076 3096 3115 3134 3153 3172 3191 3211 3230 3 6 10 13 16
18 .3249 3269 3288 3307 3327 3346 3365 3385 3404 3424 3 6 10 13 16
19 .3443 3463 3482 3502 3522 3541 3561 3581 3600 3620 3 7 10 13 16
20 .3640 3659 3679 3699 3719 3739 3759 3779 3799 3819 3 7 10 13 17
21 .3839 3859 3879 3899 3919 3939 3959 3979 4000 4020 3 7 10 13 17
22 .4040 4061 4081 4101 4122 4142 4163 4183 4204 4224 3 7 10 14 17
23 .4245 4265 4286 4307 4327 4348 4369 4390 4411 4431 3 7 10 14 17
24 .4452 4473 4494 4515 4536 4557 4578 4599 4621 4642 4 7 11 14 18

25 .4663 4684 4706 4727 4748 4770 4791 4813 4834 4856 4 7 11 14 18
26 .4877 4899 4921 4942 4964 4986 5008 5029 5051 5073 4 7 11 15 18
27 .5095 5117 5139 5161 5184 5206 5228 5250 5272 5295 4 7 11 15 18
28 .5317 5340 5362 5384 5407 5430 5452 5475 5498 5520 4 8 11 15 19
29 .5543 5566 5589 5612 5635 5658 5681 5704 5727 5750 4 8 12 15 19

30 .5774 5797 5820 5844 5867 5890 5914 5938 5961 5985 4 8 12 16 20
31 .6009 6032 6056 6080 6104 6128 6152 6176 6200 6224 4 8 12 16 20
32 .6249 6273 6297 6322 6346 6371 6395 6420 6445 6469 4 8 12 16 20
33 .6494 6519 6544 6569 6594 6619 6644 6669 6694 6720 4 8 13 17 21
34 .6745 6771 6796 6822 6847 6873 6899 6924 6950 6976 4 9 13 17 21
35 .7002 7028 7054 7080 7107 7133 7159 7186 7212 7239 4 9 13 18 22
36 .7265 7292 7319 7346 7373 7400 7427 7454 7481 7508 5 9 14 18 23
37 .7536 7563 7590 7618 7646 7673 7701 7729 7757 7785 5 9 14 18 23
38 .7813 7841 7869 7898 7926 7954 7983 8012 8040 8069 5 9 14 19 24
39 .8098 8127 8156 8185 8214 8243 8273 8302 8332 8361 5 10 15 20 24
40 .8391 8421 8451 8481 8511 8541 8571 8601 8632 8662 5 10 15 20 25
41 .8693 8724 8754 8785 8816 8847 8878 8910 8941 8972 5 10 16 21 26
42 .9004 9036 9067 9099 9131 9163 9195 9228 9260 9293 5 11 16 21 27
43 .9325 9358 9391 9424 9457 9490 9523 9556 9590 9623 6 11 17 22 28
44 .9657 9691 9725 9759 9793 9827 9861 9896 9930 9965 6 11 17 23 29

( xi )
NATURAL TANGENTS
Degrees

0′ 6′ 12′ 18′ 24′ 30′ 36′ 42′ 48′ 54′ Mean Differences
0°.0 0°.1 0°.2 0°.3 0°.4 0°.5 0°.6 0°.7 0°.8 0°.9
1 2 3 4 5

45 1.0000 0035 0070 0105 0141 0176 0212 0247 0283 0319 6 12 18 24 30
46 1.0355 0392 0428 0464 0501 0538 0575 0612 0649 0686 6 12 18 25 31
47 1.0724 0761 0799 0837 0875 0913 0951 0990 1028 1067 6 13 19 25 32
48 1.1106 1145 1184 1224 1263 1303 1343 1383 1423 1463 7 13 20 27 33
49 1.1504 1544 1585 1626 1667 1708 1750 1792 1833 1875 7 14 21 28 34

50 1.1918 1960 2002 2045 2088 2131 2174 2218 2261 2305 7 14 22 29 36
51 1.2349 2393 2437 2482 2527 2572 2617 2662 2708 2753 8 15 23 30 38
52 1.2799 2846 2892 2938 2985 3032 3079 3127 3175 3222 8 16 24 31 39
53 1.3270 3319 3367 3416 3465 3514 3564 3613 3663 3713 8 16 25 33 41
54 1.3764 3814 3865 3916 3968 4019 4071 4124 4176 4229 9 17 26 34 43

55 1.4281 4335 4388 4442 4496 4550 4605 4659 4715 4770 9 18 27 36 45
56 1.4826 4882 4938 4994 5051 5108 5166 5224 5282 5340 10 19 29 38 48
57 1.5399 5458 5517 5577 5637 5697 5757 5818 5880 5941 10 20 30 40 50
58 1.6003 6066 6128 6191 6255 6319 6383 6447 6512 6577 11 21 32 43 53
59 1.6643 6709 6775 6842 6909 6977 7045 7113 7182 7251 11 23 34 45 56
60 1.7321 7391 7461 7532 7603 7675 7747 7820 7893 7966 12 24 36 48 60
61 1.8040 8115 8190 8265 8341 8418 8495 8572 8650 8728 13 26 38 51 64
62 1.8807 8887 8967 9047 9128 9210 9292 9375 9458 9542 14 27 41 55 68
63 1.9626 9711 9797 9883 9970 2.0057 2.0145 2.0233 2.0323 2.0413 15 29 44 58 73
64 2.0503 0594 0686 0778 0872 0965 1060 1155 1251 1348 16 31 47 63 78
65 2.1445 1543 1642 1742 1842 1943 2045 2148 2251 2355 17 34 51 68 85
66 2.2460 2566 2673 2781 2889 2998 3109 3220 3332 3445 18 37 55 73 92
67 2.3559 3673 3789 3906 4023 4142 4262 4383 4504 4627 20 40 60 79 99
68 2.4751 4876 5002 5129 5257 5386 5517 5649 5782 5916 22 43 65 87 108
69 2.6051 6187 6325 6464 6605 6746 6889 7034 7179 7326 24 47 71 95 119

70 2.7475 7625 7776 7929 8083 8239 8397 8556 8716 8878 26 52 78 104 131
71 2.9042 9208 9375 9544 9714 9887 3.0061 3.0237 3.0415 3.0595 29 58 87 116 145
72 3.0777 0961 1146 1334 1524 1716 1910 2106 2305 2506 32 64 96 129 161
73 3.2709 2914 3122 3332 3544 3759 3977 4197 4420 4646 36 72 108 144 180
74 3.4874 5105 5339 5576 5816 6059 6305 6554 6806 7062 41 81 122 163 204

75 3.7321 7583 7848 8118 8391 8667 8947 9232 9520 9812 46 93 139 186 232
76 4.0108 0408 0713 1022 1335 1653 1976 2303 2635 2972 53 107 160 213 267
77 4.3315 3662 4015 4374 4737 5107 5483 5864 6252 6646
78 4.7046 7453 7867 8288 8716 9152 9594 5.0045 5.0504 5.0970 Mean differences
79 5.1446 1929 2422 2924 3435 3955 4486 5026 5578 6140 cease to be
sufficiently
80 5.6713 7297 7894 8502 9124 9758 6.0405 6.1066 6.1742 6.2432 accurate.
81 6.3138 3859 4596 5350 6122 6912 7720 8548 9395 7.0264
82 7.1154 2066 3002 3962 4947 5958 6996 8062 9158 8.0285
83 8.1443 2636 3863 5126 6427 7769 9152 9.0579 9.2052 9.3572
84 9.5144 9.677 9.845 10.02 10.20 10.39 10.58 10.78 10.99 11.20
85 11.43 11.66 11.91 12.16 12.43 12.71 13.00 13.30 13.62 13.95
86 14.30 14.67 15.06 15.46 15.89 16.35 16.83 17.34 17.89 18.46
87 19.08 19.74 20.45 21.20 22.02 22.90 23.86 24.90 26.03 27.27
88 28.64 30.14 31.82 33.69 35.80 38.19 40.92 44.07 47.74 52.08
89 57.29 63.66 71.62 81.85 95.49 114.6 143.2 191.0 286.5 573.0
90 ∞
( xii )
LOGARITHMS OF SINES
Degrees

0′ 6′ 12′ 18′ 24′ 30′ 36′ 42′ 48′ 54′ Mean Differences
0°.0 0°.1 0°.2 0°.3 0°.4 0°.5 0°.6 0°.7 0°.8 0°.9
1 2 3 4 5

0 – ∞ 3 .2419 3 .5429 7190 8439 9408 2 .0200 2 .0870 2 .1450 2 .1961


1 2 .2419 2832 3210 3558 3880 4179 4459 4723 4971 5206
2 2 .5428 5640 5842 6035 6220 6397 6567 6731 6889 7041
3 2 .7188 7330 7468 7602 7731 7857 7979 8098 8213 8326
4 2 .8436 8543 8647 8749 8849 8946 9042 9135 9226 9315 16 32 48 64 80

5 2 .9403 9489 9573 9655 9736 9816 9894 9970 1 .0046 1 .0120 13 26 39 52 65
6 1 .0192 0264 0334 0403 0472 0539 0605 0670 0734 0797 11 22 33 44 55
7 1 .0859 0920 0981 1040 1099 1157 1214 1271 1326 1381 10 19 29 38 48
8 1 .1436 1489 1542 1594 1646 1697 1747 1797 1847 1895 8 17 25 34 42
9 1 .1943 1991 2038 2085 2131 2176 2221 2266 2310 2353 8 15 23 30 38

10 1 .2397 2439 2482 2524 2565 2606 2647 2687 2727 2767 7 14 20 27 34
11 1 .2806 2845 2883 2921 2959 2997 3034 3070 3107 3143 6 12 19 25 31
12 1 .3179 3214 3250 3284 3319 3353 3387 3421 3455 3488 6 11 17 23 28
13 1 .3521 3554 3586 3618 3650 3682 3713 3745 3775 3806 5 11 16 21 26
14 1 .3837 3867 3897 3927 3957 3986 4015 4044 4073 4102 5 10 15 20 24

15 1 .4130 4158 4186 4214 4242 4269 4296 4323 4350 4377 5 9 14 18 23
16 1 .4403 4430 4456 4482 4508 4533 4559 4584 4609 4634 4 9 13 17 21
17 1 .4659 4684 4709 4733 4757 4781 4805 4829 4853 4876 4 8 12 16 20
18 1 .4900 4923 4946 4969 4992 5015 5037 5060 5082 5104 4 8 11 15 19
19 1 .5126 5148 5170 5192 5213 5235 5256 5278 5299 5320 4 7 11 14 18

20 1 .5341 5361 5382 5402 5423 5443 5463 5484 5504 5523 3 7 10 14 17
21 1 .5543 5563 5583 5602 5621 5641 5660 5679 5698 5717 3 6 10 13 16
22 1 .5736 5754 5773 5792 5810 5828 5847 5865 5883 5901 3 6 9 12 15
23 1 .5919 5937 5954 5972 5990 6007 6024 6042 6059 6076 3 6 9 12 15
24 1 .6093 6110 6127 6144 6161 6177 6194 6210 6227 6243 3 6 8 11 14

25 1 .6259 6276 6292 6308 6324 6340 6356 6371 6387 6403 3 5 8 11 13
26 1 .6418 6434 6449 6465 6480 6495 6510 6526 6541 6556 3 5 8 10 13
27 1 .6570 6585 6600 6615 6629 6644 6659 6673 6687 6702 2 5 7 10 12
28 1 .6716 6730 6744 6759 6773 6787 6801 6814 6828 6842 2 5 7 9 12
29 1 .6856 6869 6883 6896 6910 6923 6937 6950 6963 6977 2 4 7 9 11

30 1 .6990 7003 7016 7029 7042 7055 7068 7080 7093 7106 2 4 6 9 11
31 1 .7118 7131 7144 7156 7168 7181 7193 7205 7218 7230 2 4 6 8 10
32 1 .7242 7254 7266 7278 7290 7302 7314 7326 7338 7349 2 4 6 8 10
33 1 .7361 7373 7384 7396 7407 7419 7430 7442 7453 7464 2 4 6 8 10
34 1 .7476 7487 7498 7509 7520 7531 7542 7553 7564 7575 2 4 6 7 9

35 1 .7586 7597 7607 7618 7629 7640 7650 7661 7671 7682 2 4 5 7 9
36 1 .7692 7703 7713 7723 7734 7744 7754 7764 7774 7785 2 3 5 7 9
37 1 .7795 7805 7815 7825 7835 7844 7854 7864 7874 7884 2 3 5 7 8
38 1 .7893 7903 7913 7922 7932 7941 7951 7960 7970 7979 2 3 5 6 8
39 1 .7989 7998 8007 8017 8026 8035 8044 8053 8063 8072 2 3 5 6 8

40 1 .8081 8090 8099 8108 8117 8125 8134 8143 8152 8161 1 3 4 6 7
41 1 .8169 8178 8187 8195 8204 8213 8221 8230 8238 8247 1 3 4 6 7
42 1 .8255 8264 8272 8280 8289 8297 8305 8313 8322 8330 1 3 4 6 7
43 1 .8338 8346 8354 8362 8370 8378 8396 8394 8402 8410 1 3 4 5 7
44 1 .8418 8426 8433 8441 8449 8457 8464 8472 8480 8487 1 3 4 5 6

( xiii )
LOGARITHMS OF SINES
Degrees

0′ 6′ 12′ 18′ 24′ 30′ 36′ 42′ 48′ 54′ Mean Differences
0°.0 0°.1 0°.2 0°.3 0°.4 0°.5 0°.6 0°.7 0°.8 0°.9
1 2 3 4 5

45 1 .8495 8502 8510 8517 8525 8532 8540 8547 8555 8562 1 2 4 5 6
46 1 .8569 8577 8584 8591 8598 8606 8613 8620 8627 8634 1 2 4 5 6
47 1 .8641 8648 8655 8662 8669 8676 8683 8690 8697 8704 1 2 3 5 6
48 1 .8711 8718 8724 8731 8738 8745 8751 8758 8765 8771 1 2 3 4 6
49 1 .8778 8784 8791 8797 8804 8810 8817 8823 8830 8836 1 2 3 4 5
50 1 .8843 8849 8855 8862 8868 8874 8880 8887 8893 8899 1 2 3 4 5
51 1 .8905 8911 8917 8923 8929 8935 8941 8947 8953 8959 1 2 3 4 5
52 1 .8965 8971 8977 8983 8989 8995 9000 9006 9012 9018 1 2 3 4 5
53 1 .9023 9029 9035 9041 9046 9052 9057 9063 9069 9074 1 2 3 4 5
54 1 .9080 9085 9091 9096 9101 9107 9112 9118 9123 9128 1 2 3 4 5
55 1 .9134 9139 9144 9149 9155 9160 9165 9170 9175 9181 1 2 3 3 4
56 1 .9186 9191 9196 9201 9206 9211 9216 9221 9226 9231 1 2 3 3 4
57 1 .9236 9241 9246 9251 9255 9260 9265 9270 9275 9279 1 2 2 3 4
58 1 .9284 9289 9294 9298 9303 9308 9312 9317 9322 9326 1 2 2 3 4
59 1 .9331 9335 9340 9344 9349 9353 9358 9362 9367 9371 1 1 2 3 4
60 1 .9375 9380 9384 9388 9393 9397 9401 9406 9410 9414 1 1 2 3 4
61 1 .9418 9422 9427 9431 9435 9439 9443 9447 9451 9455 1 1 2 3 3
62 1 .9459 9463 9467 9471 9475 9479 9483 9487 9491 9495 1 1 2 3 3
63 1 .9499 9503 9506 9510 9514 9518 9522 9525 9529 9533 1 1 2 3 3
64 1 .9537 9540 9544 9548 9551 9555 9558 9562 9566 9569 1 1 2 2 3
65 1 .9573 9576 9580 9583 9587 9590 9594 9597 9601 9604 1 1 2 2 3
66 1 .9607 9611 9614 9617 9621 9624 9627 9631 9634 9637 1 1 2 2 3
67 1 .9640 9643 9647 9650 9653 9656 9659 9662 9666 9669 1 1 2 2 3
68 1 .9672 9675 9678 9681 9684 9687 9690 9693 9696 9699 0 1 1 2 2
69 1 .9702 9704 9707 9710 9713 9716 9719 9722 9724 9727 0 1 1 2 2
70 1 .9730 9733 9735 9738 9741 9743 9746 9749 9751 9754 0 1 1 2 2
71 1 .9757 9759 9762 9764 9767 9770 9772 9775 9777 9780 0 1 1 2 2
72 1 .9782 9785 9787 9789 9792 9794 9797 9799 9801 9804 0 1 1 2 2
73 1 .9806 9808 9811 9813 9815 9817 9820 9822 9824 9826 0 1 1 2 2
74 1 .9828 9831 9833 9835 9837 9839 9841 9843 9845 9847 0 1 1 1 2
75 1 .9849 9851 9853 9855 9857 9859 9861 9863 9865 9867 0 1 1 1 2
76 1 .9869 9871 9873 9875 9876 9878 9880 9882 9884 9885 0 1 1 1 2
77 1 .9887 9889 9891 9892 9894 9896 9897 9899 9901 9902 0 1 1 1 1
78 1 .9904 9906 9907 9909 9910 9912 9913 9915 9916 9918 0 1 1 1 1
79 1 .9919 9921 9922 9924 9925 9927 9928 9929 9931 9932 0 0 1 1 1
80 1 .9934 9935 9936 9937 9939 9940 9941 9943 9944 9945 0 0 1 1 1
81 1 .9946 9947 9949 9950 9951 9952 9953 9954 9955 9956 0 0 1 1 1
82 1 .9958 9959 9960 9961 9962 9963 9964 9965 9966 9967 0 0 1 1 1
83 1 .9968 9968 9969 9970 9971 9972 9973 9974 9975 9975 0 0 0 1 1
84 1 .9976 9977 9978 9978 9979 9980 9981 9981 9982 9983 0 0 0 0 1
85 1 .9983 9984 9985 9985 9986 9987 9987 9988 9988 9989 0 0 0 0 0
86 1 .9989 9990 9990 9991 9991 9992 9992 9993 9993 9994 0 0 0 0 0
87 1 .9994 9994 9995 9995 9996 9996 9996 9997 9997 9997 0 0 0 0 0
88 1 .9997 9998 9998 9998 9998 9999 9999 9999 9999 9999 0 0 0 0 0
89 1 .9999 9999 0.0000 0000 0000 0000 0000 0000 0000 0000
90 0.0000
( xiv )
LOGARITHMS OF COSINES
[Numbers in difference columns to be subtracted, not added]
Degrees

0′ 6′ 12′ 18′ 24′ 30′ 36′ 42′ 48′ 54′ Mean Differences
0°.0 0°.1 0°.2 0°.3 0°.4 0°.5 0°.6 0°.7 0°.8 0°.9
1 2 3 4 5

0 0.0000 0000 0000 0000 0000 0000 0000 0000 0000 1 .9999 0 0 0 0 0
1 1 .9999 9999 9999 9999 9999 9999 9998 9998 9998 9998 0 0 0 0 0
2 1 .9997 9997 9997 9997 9996 9996 9996 9995 9995 9994 0 0 0 0 0
3 1 .9994 9994 9993 9993 9992 9992 9991 9991 9990 9990 0 0 0 0 0
4 1 .9989 9989 9988 9988 9987 9987 9986 9985 9985 9984 0 0 0 0 0
5 1 .9983 9983 9982 9981 9981 9980 9979 9978 9978 9977 0 0 0 0 1
6 1 .9976 9975 9975 9974 9973 9972 9971 9970 9969 9968 0 0 0 1 1
7 1 .9968 9967 9966 9965 9964 9963 9962 9961 9960 9959 0 0 1 1 1
8 1 .9958 9956 9955 9954 9953 9952 9951 9950 9949 9947 0 0 1 1 1
9 1 .9946 9945 9944 9943 9941 9940 9939 9937 9936 9935 0 0 1 1 1
10 1 .9934 9932 9931 9929 9928 9927 9925 9924 9922 9921 0 0 1 1 1
11 1 .9919 9918 9916 9915 9913 9912 9910 9909 9907 9906 0 1 1 1 1
12 1 .9904 9902 9901 9899 9897 9896 9894 9892 9891 9889 0 1 1 1 1
13 1 .9887 9885 9884 9882 9880 9878 9876 9875 9873 9871 0 1 1 1 2
14 1 .9869 9867 9865 9863 9861 9859 9857 9855 9853 9851 0 1 1 1 2
15 1 .9849 9847 9845 9843 9841 9839 9837 9835 9833 9831 0 1 1 1 2
16 1 .9828 9826 9824 9822 9820 9817 9815 9813 9811 9808 0 1 1 2 2
17 1 .9806 9804 9801 9799 9797 9794 9792 9789 9787 9785 0 1 1 2 2
18 1 .9782 9780 9777 9775 9772 9770 9767 9764 9762 9759 0 1 1 2 2
19 1 .9757 9754 9751 9749 9746 9743 9741 9738 9735 9733 0 1 1 2 2
20 1 .9730 9727 9724 9722 9719 9716 9713 9710 9707 9704 0 1 1 2 2
21 1 .9702 9699 9696 9693 9690 9687 9684 9681 9678 9675 0 1 1 2 2
22 1 .9672 9669 9666 9662 9659 9656 9653 9650 9647 9643 1 1 2 2 3
23 1 .9640 9637 9634 9631 9627 9624 9621 9617 9614 9611 1 1 2 2 3
24 1 .9607 9604 9601 9597 9594 9590 9587 9583 9580 9576 1 1 2 2 3
25 1 .9573 9569 9566 9562 9558 9555 9551 9548 9544 9540 1 1 2 2 3
26 1 .9537 9533 9529 9525 9522 9518 9514 9510 9507 9503 1 1 2 3 3
27 1 .9499 9495 9491 9487 9483 9479 9475 9471 9467 9463 1 1 2 3 3
28 1 .9459 9455 9451 9447 9443 9439 9435 9431 9427 9422 1 1 2 3 3
29 1 .9418 9414 9410 9406 9401 9397 9393 9388 9384 9380 1 1 2 3 4
30 1 .9375 9371 9367 9362 9358 9353 9349 9344 9340 9335 1 1 2 3 4
31 1 .9331 9326 9322 9317 9312 9308 9303 9298 9294 9289 1 2 2 3 4
32 1 .9284 9279 9275 9270 9265 9260 9255 9251 9246 9241 1 2 2 3 4
33 1 .9236 9231 9226 9221 9216 9211 9206 9201 9196 9191 1 2 3 3 4
34 1 .9186 9181 9175 9170 9165 9160 9155 9149 9144 9139 1 2 3 3 4

35 1 .9134 9128 9123 9118 9112 9107 9101 9096 9091 9085 1 2 3 4 5
36 1 .9080 9074 9069 9063 9057 9052 9046 9041 9035 9029 1 2 3 4 5
37 1 .9023 9018 9012 9006 9000 8995 8989 8983 8977 8971 1 2 3 4 5
38 1 .8965 8959 8953 8947 8941 8935 8929 8923 8917 8911 1 2 3 4 5
39 1 .8905 8899 8893 8887 8880 8874 8868 8862 8855 8849 1 2 3 4 5
40 1 .8843 8836 8830 8823 8817 8810 8804 8797 8791 8784 1 2 3 4 5
41 1 .8778 8771 8765 8758 8751 8745 8738 8731 8724 8718 1 2 3 5 6
42 1 .8711 8704 8697 8690 8683 8676 8669 8662 8655 8648 1 2 3 5 6
43 1 .8641 8634 8627 8620 8613 8606 8598 8591 8584 8577 1 2 4 5 6
44 1 .8569 8562 8555 8547 8540 8532 8525 8517 8510 8502 1 2 4 5 6

( xv )
LOGARITHMS OF COSINES
[Numbers in difference columns to be subtracted, not added]
Degrees

0′ 6′ 12′ 18′ 24′ 30′ 36′ 42′ 48′ 54′ Mean Differences
0°.0 0°.1 0°.2 0°.3 0°.4 0°.5 0°.6 0°.7 0°.8 0°.9
1 2 3 4 5

45 1 .8495 8487 8480 8472 8464 8457 8449 8441 8433 8426 1 3 4 5 6
46 1 .8418 8410 8402 8394 8386 8378 8370 8362 8354 8346 1 3 4 5 7
47 1 .8338 8330 8322 8313 8305 8297 8289 8280 8272 8264 1 3 4 6 7
48 1 .8255 8247 8238 8230 8221 8213 8204 8195 8187 8178 1 3 4 6 7
49 1 .8169 8161 8152 8143 8134 8125 8117 8108 8099 8090 1 3 4 6 7
50 1 .8081 8072 8063 8053 8044 8035 8026 8017 8007 7998 2 3 5 6 8
51 1 .7989 7979 7970 7960 7951 7941 7932 7922 7913 7903 2 3 5 6 8
52 1 .7893 7884 7874 7864 7854 7844 7835 7825 7815 7805 2 3 5 7 8
53 1 .7795 7785 7774 7764 7754 7744 7734 7723 7713 7703 2 3 5 7 9
54 1 .7692 7682 7671 7661 7650 7640 7629 7618 7607 7597 2 4 5 7 9
55 1 .7586 7575 7564 7553 7542 7531 7520 7509 7498 7487 2 4 6 7 9
56 1 .7476 7464 7453 7442 7430 7419 7407 7396 7384 7373 2 4 6 8 10
57 1 .7361 7349 7338 7326 7314 7302 7290 7278 7266 7254 2 4 6 8 10
58 1 .7242 7230 7218 7205 7193 7181 7168 7156 7144 7131 2 4 6 8 10
59 1 .7118 7106 7093 7080 7068 7055 7042 7029 7016 7003 2 4 6 9 11
60 1 .6990 6977 6963 6950 6937 6923 6910 6896 6883 6869 2 4 7 9 11
61 1 .6856 6842 6828 6814 6801 6787 6773 6759 6744 6730 2 5 7 9 12
62 1 .6716 6702 6687 6673 6659 6644 6629 6615 6600 6585 2 5 7 10 12
63 1 .6570 6556 6541 6526 6510 6495 6480 6465 6449 6434 3 5 8 10 13
64 1 .6418 6403 6387 6371 6356 6340 6324 6308 6292 6276 3 5 8 11 13
65 1 .6259 6243 6227 6210 6194 6177 6161 6144 6127 6110 3 6 8 11 14
66 1 .6093 6076 6059 6042 6024 6007 5990 5972 5954 5937 3 6 9 12 15
67 1 .5919 5901 5883 5865 5847 5828 5810 5792 5773 5754 3 6 9 12 15
68 1 .5736 5717 5698 5679 5660 5641 5621 5602 5583 5563 3 6 10 13 16
69 1 .5543 5523 5504 5484 5463 5443 5423 5402 5382 5361 3 7 10 14 17
70 1 .5341 5320 5299 5278 5256 5235 5213 5192 5170 5148 4 7 11 14 18
71 1 .5126 5104 5082 5060 5037 5015 4992 4969 4946 4923 4 8 11 15 19
72 1 .4900 4876 4853 4829 4805 4781 4757 4733 4709 4684 4 8 12 16 20
73 1 .4659 4634 4609 4584 4559 4533 4508 4482 4456 4430 4 9 13 17 21
74 1 .4403 4377 4350 4323 4296 4269 4242 4214 4186 4158 5 9 14 18 23
75 1 .4130 4102 4073 4044 4015 3986 3957 3927 3897 3867 5 10 15 20 24
76 1 .3837 3806 3775 3745 3713 3682 3650 3618 3586 3554 5 11 16 21 26
77 1 .3521 3488 3455 3421 3387 3353 3319 3284 3250 3214 6 11 17 23 28
78 1 .3179 3143 3107 3070 3034 2997 2959 2921 2883 2845 6 12 19 25 31
79 1 .2806 2767 2727 2687 2647 2606 2565 2524 2482 2439 7 14 20 27 34
80 1 .2397 2353 2310 2266 2221 2176 2131 2085 2038 1991 8 15 23 30 38
81 1 .1943 1895 1847 1797 1747 1697 1646 1594 1542 1489 8 17 25 34 42
82 1 .1436 1381 1326 1271 1214 1157 1093 1040 0981 0920 10 19 29 38 48
83 1 .0859 0797 0734 0670 0605 0539 0472 0403 0334 0264 11 22 33 44 55
84 1 .0192 0120 0046 2 .9970 2 .9894 2 .9816 2 .9736 2 .9655 2 .9573 2 .9489 13 26 39 52 65
85 2 .9403 9315 9226 9135 9042 8946 8849 8749 8647 8543 16 32 48 64 80
86 2 .8436 8326 8213 8098 7979 7857 7731 7602 7468 7330
87 2 .7188 7041 6889 6731 6567 6397 6220 6035 5842 5640
88 2 .5428 5206 4971 4723 4459 4179 3880 3558 3210 2832
89 2 .2419 1961 1450 0870 0200 3 .9408 3 .8439 3 .7190 3 .5429 3 .2419
90 –∞

( xvi )
LOGARITHMS OF TANGENTS
Degrees

0′ 6′ 12′ 18′ 24′ 30′ 36′ 42′ 48′ 54′ Mean Differences
0°.0 0°.1 0°.2 0°.3 0°.4 0°.5 0°.6 0°.7 0°.8 0°.9
1 2 3 4 5

0 –∞ 3 .2419 3 .5429 3 .7190 3 .8439 3 .9409 2 .0200 2 .0870 2 .1450 2 .1962


1 2 .2419 2833 3211 3559 3881 4181 4461 4725 4973 5208
2 2 .5431 5643 5845 6038 6223 6401 6571 6736 6894 7046
3 2 .7194 7337 7475 7609 7739 7865 7988 8107 8223 8336
4 2 .8446 8554 8659 8762 8862 8960 9056 9150 9241 9331 16 32 48 64 81

5 2 .9420 9506 9591 9674 9756 9836 9915 9992 1 .0068 1 .0143 13 26 40 53 66
6 1 .0216 0289 0360 0430 0499 0567 0633 0699 0764 0828 11 22 34 45 56
7 1 .0891 0954 1015 1076 1135 1194 1252 1310 1367 1423 10 20 29 39 49
8 1 .1478 1533 1587 1640 1693 1745 1797 1848 1898 1948 9 17 26 35 43
9 1 .1997 2046 2094 2142 2189 2236 2282 2328 2374 2419 8 16 23 31 39

10 1 .2463 2507 2551 2594 2637 2680 2722 2764 2805 2846 7 14 21 28 35
11 1 .2887 2927 2967 3006 3046 3085 3123 3162 3200 3237 6 13 19 26 32
12 1 .3275 3312 3349 3385 3422 3458 3493 3529 3564 3599 6 12 18 24 30
13 1 .3634 3668 3702 3736 3770 3804 3837 3870 3903 3935 6 11 17 22 28
14 1 .3968 4000 4032 4064 4095 4127 4158 4189 4220 4250 5 10 16 21 26

15 1 .4281 4311 4341 4371 4400 4430 4459 4488 4517 4546 5 10 15 20 25
16 1 .4575 4603 4632 4660 4688 4716 4744 4771 4799 4826 5 9 14 19 23
17 1 .4853 4880 4907 4934 4961 4987 5014 5040 5066 5092 4 9 13 18 22
18 1 .5118 5143 5169 5195 5220 5245 5270 5295 5320 5345 4 8 13 17 21
19 1 .5370 5394 5419 5443 5467 5491 5516 5539 5563 5587 4 8 12 16 20

20 1 .5611 5634 5658 5681 5704 5727 5750 5773 5796 5819 4 8 12 15 19
21 1 .5842 5864 5887 5909 5932 5954 5976 5998 6020 6042 4 7 11 15 19
22 1 .6064 6086 6108 6129 6151 6172 6194 6215 6236 6257 4 7 11 14 18
23 1 .6279 6300 6321 6341 6362 6383 6404 6424 6445 6465 3 7 10 14 17
24 1 .6486 6506 6527 6547 6567 6587 6607 6627 6647 6667 3 7 10 13 17

25 1 .6687 6706 6726 6746 6765 6785 6804 6824 6843 6863 3 7 10 13 16
26 1 .6882 6901 6920 6939 6958 6977 6996 7015 7034 7053 3 6 9 13 16
27 1 .7072 7090 7109 7128 7146 7165 7183 7202 7220 7238 3 6 9 12 15
28 1 .7257 7275 7293 7311 7330 7348 7366 7384 7402 7420 3 6 9 12 15
29 1 .7438 7455 7473 7491 7509 7526 7544 7562 7579 7597 3 6 9 12 15

30 1 .7614 7632 7649 7667 7684 7701 7719 7736 7753 7771 3 6 9 12 14
31 1 .7788 7805 7822 7839 7856 7873 7890 7907 7924 7941 3 6 9 11 14
32 1 .7958 7975 7992 8008 8025 8042 8059 8075 8092 8109 3 6 8 11 14
33 1 .8125 8142 8158 8175 8191 8208 8224 8241 8257 8274 3 5 8 11 14
34 1 .8290 8306 8323 8339 8355 8371 8388 8404 8420 8436 3 5 8 11 14

35 1 .8452 8468 8484 8501 8517 8533 8549 8565 8581 8597 3 5 8 11 13
36 1 .8613 8629 8644 8660 8676 8692 8708 8724 8740 8755 3 5 8 11 13
37 1 .8771 8787 8803 8818 8834 8850 8865 8881 8897 8912 3 5 8 10 13
38 1 .8928 8944 8959 8975 8990 9006 9022 9037 9053 9068 3 5 8 10 13
39 1 .9084 9099 9115 9130 9146 9161 9176 9192 9207 9223 3 5 8 10 13

40 1 .9238 9254 9269 9284 9300 9315 9330 9346 9361 9376 3 5 8 10 13
41 1 .9392 9407 9422 9438 9453 9468 9483 9499 9514 9529 3 5 8 10 13
42 1 .9544 9560 9575 9590 9605 9621 9636 9651 9666 9681 3 5 8 10 13
43 1 .9697 9712 9727 9742 9757 9772 9788 9803 9818 9833 3 5 8 10 13
44 1 .9848 9864 9879 9894 9909 9924 9939 9955 9970 9985 3 5 8 10 13

( xvii )
LOGARITHMS OF TANGENTS
Degrees

0′ 6′ 12′ 18′ 24′ 30′ 36′ 42′ 48′ 54′ Mean Differences
0°.0 0°.1 0°.2 0°.3 0°.4 0°.5 0°.6 0°.7 0°.8 0°.9
1 2 3 4 5

45 .0000 0015 0030 0045 0061 0076 0091 0106 0121 0136 3 5 8 10 13
46 .0152 0167 0182 0197 0212 0228 0243 0258 0273 0288 3 5 8 10 13
47 .0303 0319 0334 0349 0364 0379 0395 0410 0425 0440 3 5 8 10 13
48 .0456 0471 0486 0501 0517 0532 0547 0562 0578 0593 3 5 8 10 13
49 .0608 0624 0639 0654 0670 0685 0700 0716 0731 0746 3 5 8 10 13
50 .0762 0777 0793 0808 0824 0839 0854 0870 0885 0901 3 5 8 10 13
51 .0916 0932 0947 0963 0978 0994 1010 1025 1041 1056 3 5 8 10 13
52 .1072 1088 1103 1119 1135 1150 1166 1182 1197 1213 3 5 8 10 13
53 .1229 1245 1260 1276 1292 1308 1324 1340 1356 1371 3 5 8 11 13
54 .1387 1403 1419 1435 1451 1467 1483 1499 1516 1532 3 5 8 11 13
55 .1548 1564 1580 1596 1612 1629 1645 1661 1677 1694 3 5 8 11 14
56 .1710 1726 1743 1759 1776 1792 1809 1825 1842 1858 3 5 8 11 14
57 .1875 1891 1908 1925 1941 1958 1975 1992 2008 2025 3 6 8 11 14
58 .2042 2059 2076 2093 2110 2127 2144 2161 2178 2195 3 6 9 11 14
59 .2212 2229 2247 2264 2281 2299 2316 2333 2351 2368 3 6 9 12 14
60 .2386 2403 2421 2438 2456 2474 2491 2509 2527 2545 3 6 9 12 15
61 .2562 2580 2598 2616 2634 2652 2670 2689 2707 2725 3 6 9 12 15
62 .2743 2762 2780 2798 2817 2835 2854 2872 2891 2910 3 6 9 12 15
63 .2928 2947 2966 2985 3004 3023 3042 3061 3080 3099 3 6 9 13 16
64 .3118 3137 3157 3176 3196 3215 3235 3254 3274 3294 3 6 10 13 16
65 .3313 3333 3353 3373 3393 3413 3433 3453 3473 3494 3 7 10 13 17
66 .3514 3535 3555 3576 3596 3617 3638 3659 3679 3700 3 7 10 14 17
67 .3721 3743 3764 3785 3806 3828 3849 3871 3892 3914 4 7 11 14 18
68 .3936 3958 3980 4002 4024 4046 4068 4091 4113 4136 4 7 11 15 19
69 .4158 4181 4204 4227 4250 4273 4296 4319 4342 4366 4 8 12 15 19
70 .4389 4413 4437 4461 4484 4509 4533 4557 4581 4606 4 8 12 16 20
71 .4630 4655 4680 4705 4730 4755 4780 4805 4831 4857 4 8 13 17 21
72 .4882 4908 4934 4960 4986 5013 5039 5066 5093 5120 4 9 13 18 22
73 .5147 5174 5201 5229 5256 5284 5312 5340 5368 5397 5 9 14 19 23
74 .5425 5454 5483 5512 5541 5570 5600 5629 5659 5689 5 10 15 20 25
75 .5719 5750 5780 5811 5842 5873 5905 5936 5968 6000 5 10 16 21 26
76 .6032 6065 5097 6130 6163 6196 6230 6264 6298 6332 6 11 17 22 28
77 .6366 6401 6436 6471 6507 6542 6578 6615 6651 6688 6 12 18 24 30
78 .6725 6763 6800 6838 6877 6915 6954 6994 7033 7073 6 13 19 26 32
79 .7113 7154 7195 7236 7278 7320 7363 7406 7449 7493 7 14 21 28 35
80 .7537 7581 7626 7672 7718 7764 7811 7858 7906 7954 8 16 23 31 39
81 .8003 8052 8102 8152 8203 8255 8307 8360 8413 8467 9 17 26 35 43
82 .8522 8577 8633 8690 8748 8806 8865 8924 8985 9046 10 20 29 39 49
83 .9109 9172 9236 9301 9367 9433 9501 9570 9640 9711 11 22 34 45 56
84 .9784 9857 9932 1.0008 1.0085 1.0164 1.0244 1.0326 1.0409 1.0494 13 26 40 53 66
85 1.0580 0669 0759 0850 0944 1040 1138 1238 1341 1446 16 32 48 64 81
86 1.1554 1664 1777 1893 2012 2135 2261 2391 2525 2663
87 1.2806 2954 3106 3264 3429 3599 3777 3962 4155 4357
88 1.4569 4792 5027 5275 5539 5819 6119 6441 6789 7167
89 1.7581 8038 8550 9130 9800 2.0591 2.1561 2.2810 2.4571 2.7581

( xviii )

You might also like